You are on page 1of 452

TR NG THPT CHUYÊN LÝ T TR NG

TOÁN − TIN H C

CHUYÊN Đ
B T NG TH C
Th c hi n: Võ Qu c Bá C n
i nói u
----oOo----

t ng th c là m t trong nh ng v n hay và khó nh t c a ch ng trình toán


ph thông b i nó có m t trên h u kh p các l nh v c c a toán h c và nó òi h i
chúng ta ph i có m t v n ki n th c t ng i v ng vàng trên t t c các l nh v c.
i ng i chúng ta, c bi t là các b n yêu toán, dù ít dù nhi u thì c ng ã t ng
au u tr cm tb t ng th c khó và c ng ã t ng có c m t c m giác t hào
khi mà mình ch ng minh cb t ng th c ó. Nh m “kích ho t” ni m say mê
t ng th c trong các b n, tôi xin gi i thi u v i v i các b n cu n sách “chuyên
t ng th c”.
Sách g m các ph ng pháp ch ng minh b t ng th c m i mà hi n nay ch a c
ph bi n cho l m. Ngoài ra, trong sách g m m t s l ng l n b t ng th c do tôi
sáng tác, còn l i là do tôi l y toán trên internet nh ng ch a có l i gi i ho c có
i gi i nh ng là l i gi i hay, l , p m t. Ph n l n các bài t p trong sách u do tôi
gi i nên không th nào tránh kh i nh ng ng nh n, sai l m, mong các b n thông
m.
Hy v ng r ng cu n sách s giúp cho các b n m t cái nhìn khác v b t ng th c và
mong r ng qua vi c gi i các bài toán trong sách s giúp các b n có th tìm ra
ph ng pháp c a riêng mình, nâng cao c t duy sáng t o. Tôi không bi t các
n ngh sao nh ng theo quan m c a b n thân tôi thì n u ta h c t t v b t ng
th c thì c ng có th h c t t các l nh v c khác c a toán h c vì nh ã nói trên b t
ng th c òi h i chúng ta ph i có m t ki n th c t ng h p t ng i v ng vàng.
Tôi không nói suông âu, ch c h n b n c ng bi t n anh Ph m Kim Hùng, sinh
viên h CNTN khoa toán, tr ng HKHTN, HQG Hà N i, ng i ã c tham
hai k thi IMO và u t k t qu cao nh t trong i tuy n VN. B n bi t
không? Trong th i h c ph thông, anh y ch chuyên tâm rèn luy n b t ng th c
thôi. (Các b n l u ý là tôi không khuy n khích b n làm nh tôi và anh y âu nhé!)

1
c dù ã c g ng biên so n m t cách th t c n th n, nh ng do trình có h n nên
không th tránh kh i nh ng sai sót, mong các b n thông c m và góp ý cho tôi
cu n sách ngày càng c hoàn thi n h n. Chân thành c m n.
i óng góp xin g i v m t trong các a ch sau:
+ Võ Qu c Bá C n, C65 khu dân c Phú An, ph ng Phú Th , qu n
Cái R ng, thành ph C n Th .
(071.916044
+ Email. babylearnmath@yahoo.com
Kính t ng các th y ng B o Hòa, Phan i Nh n, Tr n Di u Minh, Hu nh B u
Tính, cô T Thanh Th y Tiên và toàn th các th y cô giáo trong t Toán Tin, thân
ng các b n cùng l p.

2
TS B T NG TH C THÔNG D NG

1. B t ng th c AM-GM.
u a1 , a2 ,..., an là các s th c không âm thì
1 n
.∑ ai ≥ n a1a2 ...an
n i=1
ng th c x y ra khi và ch khi a1 = a2 = ... = an .
2. B t ng th c AM-HM.
u a1 , a2 ,..., an là các s th c d ng thì
1 n 1
.∑ ai ≥
n i=1 1 n 1
.∑
n i=1 ai
ng th c x y ra khi và ch khi a1 = a2 = ... = an .
3. B t ng th c Bunhiacopxki.
Cho 2n s th c a1 , a2 ,..., an và b1 , b2 ,..., bn . Khi ó, ta có
(a12 + a22 + ... + an2 )(b12 + b22 + ... + bn2 ) ≥ (a1b1 + a2b2 + ... + an bn ) 2
a a a
ng th c x y ra khi và ch khi 1 = 2 = ... = n .
b1 b2 bn
4. B t ng th c Minkowski.
Cho 2n s th c d ng a1 , a2 ,..., an và b1 , b2 ,..., bn . Khi ó v i m i r ≥ 1, ta có
1 1 1
 n r   n r r  n r r
 ∑ i i   ∑ ai  +  ∑ bi 
r
( a + b ) ≤
 i=1   i =1   i=1 
5. B t ng th c AM-GM m r ng.
u a1 , a2 ,..., an là các s th c không âm và β1 , β 2 ,..., β n là các s th c không âm
có t ng b ng 1 thì
β1a1 + β 2 a2 + ... + β n an ≥ a1β1 a2β2 ..anβn
6. B t ng th c Chebyshev.
Cho 2n s th c a1 ≤ a2 ≤ ... ≤ an và b1 , b2 ,..., bn . Khi ó
a) N u b1 ≤ b2 ≤ ... ≤ bn thì
n
 n  n 
n.∑ ai bi ≥  ∑ ai   ∑ bi 
i =1  i=1   i=1 
a) N u b1 ≥ b2 ≥ ... ≥ bn thì
n
 n  n 
n.∑ ai bi ≤  ∑ ai   ∑ bi 
i =1  i=1   i=1 

3
 a1 = a2 = ... = an
ng th c x y ra khi và ch khi 
b1 = b2 = ... = bn
7. B t ng th c Holder.
Cho 2n s th c không âm a1 , a2 ,..., an và b1 , b2 ,..., bn . Khi ó v i m i p, q > 1 th a
1 1
+ = 1, ta có
p q
1 1
n
 n
p  n
q
∑ aibi ≤  ∑ aip   ∑ biq 
i =1  i=1   i=1 
8. B t ng th c Schur.
i m i b ba s không âm a, b, c và r ≥ 0, ta luôn có b t ng th c
r
a ( a − b)( a − c ) + b (b − c)(b − a ) + c (c − a )(c − b) ≥ 0
r r

ng th c x y ra khi và ch khi a = b = c ho c a = b, c = 0 và các hoán v .


9. B t ng th c Jensen.
Gi s f ( x) là m t hàm l i trên [a, b] . Khi ó, v i m i x1 , x2 ,..., xn ∈ [ a, b] và
α1 , α 2 ,..., α n ≥ 0 th a α1 + α 2 + ... + α n = 1 ta có b t ng th c
 n  n
f  ∑ α i xi  ≥ ∑ α i f ( xi )
 i =1  i=1
10. B t ng th c s p x p l i.
Cho 2 dãy n u cùng t ng a1 ≤ a2 ≤ ... ≤ an và b1 ≤ b2 ≤ ... ≤ bn . Khi ó, v i
i1 , i2 ,..., in là m t hoán v b t kì c a 1, 2,..., n ta có
a1b1 + a2b2 + ... + anbn ≥ ai1 bi1 + ai2 bi2 + ... + ain bin ≥ a1bn + a2bn−1 + ... + anb1
11. B t ng th c Bernulli.
i x > −1 , ta có
+ u r ≥ 1 ∨ r ≤ 0 thì (1 + x ) r ≥ 1 + rx
+ u 1 > r > 0 thì (1 + x ) r ≤ 1 + rx

4
T NG TH C THU N NH T

1. M u.
u h t các b t ng th c c n (AM-GM, Bunhiacopxki, Holder, Minkowsky,
Chebyshev ...) u là các b t ng th c thu n nh t. u này hoàn toàn không ng u
nhiên. V logíc, có th nói r ng, ch có các il ng cùng b c m i có th so sánh
i nhau m t cách toàn c c c.
Chính vì th , b t ng th c thu n nh t chi m m t t l r t cao trong các bài toán b t
ng th c, c bi t là b t ng th c i s (khi các hàm s là hàm i s , có b c
u h n). i v i các hàm gi i tích (m , l ng giác, logarith), các b t ng th c
ng c coi là thu n nh t vì các hàm s có b c ∞ (theo công th c Taylor).
Trong bài này, chúng ta s c p t i các ph ng pháp c b n ch ng minh b t
ng th c thu n nh t, c ng nh cách chuy n t m t b t ng th c không thu n nh t
m tb t ng th c thu n nh t. N m v ng và v n d ng nhu n nhuy n các ph ng
pháp này, chúng ta có th ch ng minh c h u h t các b t ng th c s c p.
2. B t ng th c thu n nh t.
Hàm s f ( x1 , x2 ,..., xn ) c a các bi n s th c x1 , x2 ,..., xn c là hàm thu n nh t b c
α n u v i m i s th c t ta có
f (tx1 , tx2 ,..., txn ) = t α f ( x1 , x2 ,..., xn )
t ng th c d ng
f ( x1 , x2 ,..., xn ) ≥ 0
i f là m t hàm thu n nh t c g i là b t ng th c thu n nh t (b c α ).
Ví d các b t ng th c AM-GM, b t ng th c Bunhiacopxki, b t ng th c
Chebyshev là các b t ng th c thu n nh t. B t ng th c Bernoulli, b t ng th c
sin x < x v i x > 0 là các b t ng th c không thu n nh t.

5
3. Ch ng minh b t ng th c thu n nh t.
3.1. Ph ng pháp d n bi n.
c m c a nhi u b t ng th c, c bi t là các b t ng th c i s là d u b ng
y ra khi t t c ho c m t vài bi n s b ng nhau (xu t phát t b t ng th c c b n
x 2 ≥ 0 !). Ph ng pháp d n bi n d a vào c m này làm gi m s bi n s c a
t ng th c, ab t ng th c v d ng n gi n h n có th ch ng minh tr c ti p
ng cách kh o sát hàm m t bi n ho c ch ng minh b ng quy n p.
ch ng minh b t ng th c
f ( x1 , x2 ,..., xn ) ≥ 0 (1)
Ta có th th ch ng minh
x +x x +x 
f ( x1 , x2 ,..., xn ) ≥ f  1 2 , 1 2 ,..., xn  (2)
 2 2 
ho c

f ( x1 , x2 ,..., xn ) ≥ f ( x1 x2 , x1 x2 ,..., xn ) (3)

Sau ó chuy n vi c ch ng minh (1) v vi c ch ng minh b t ng th c


f ( x1 , x1 , x3 ,..., xn ) = g ( x1 , x3 ,..., xn ) ≥ 0 (4)
c là m t b t ng th c có s bi n ít h n. D nhiên, các b t ng th c (2), (3) có
th không úng ho c ch úng trong m t s u ki n nào ó. Vì ta ch thay i2
bi n s nên thông th ng thì tính úng nc ab t ng th c này có th ki m tra
c d dàng.
Ví d 1.
Cho a, b, c > 0 . Ch ng minh b t ng th c

a3 + b3 + c 3 + 3abc ≥ a 2b + b 2c + c 2 a + ab 2 + bc 2 + ca 2
Ch ng minh.
Xét hàm s f (a, b, c) = a 3 + b3 + c3 + 3abc − ( a 2b + b 2c + c 2 a + ab 2 + bc 2 + ca 2 )
Ta có
 b+c b+c  5a 
f (a, b, c) − f  a, ,  =  b + c −  (b − c )
2

 2 2   4 

6
Do ó, n u a = min{a, b, c} ( u này luôn có th gi s ) thì ta có

 b+c b+c
f ( a , b, c ) ≥ f  a , , 
 2 2 
Nh v y, ch ng minh b t ng th c u bài, ta ch c n ch ng minh
f ( a , b, b ) ≥ 0
Nh ng b t ng th c này t ng ng v i
a3 + 2b3 + 3ab 2 − (a 2b + a 2b + b 2 a + b3 + b 2 a + b 3 ) ≥ 0
⇔ a 3 + ab 2 − 2a 2b ≥ 0
⇔ a ( a − b) 2 ≥ 0
Ví d 2. (Vietnam TST 1996)
Cho a, b, c là các s th c b t k . Ch ng minh r ng
4
F ( a, b, c ) = ( a + b)4 + (b + c )4 + (c + a ) 4 − .( a 4 + b4 + c 4 ) ≥ 0
7
i gi i.
Ta có
 b+c b+c
F ( a , b, c ) − F  a, , =
 2 2 
4
= ( a + b)4 + (b + c )4 + (c + a ) 4 − .( a 4 + b4 + c 4 ) −
7
4  4 b+c 
4 4
 b+c
− 2 a +  − (b + c ) + . a + 2 
4
 
 2  7   2  
b + c  4  (b + c) 4 
4

= ( a + b ) + (c + a ) − 2  a +
4 4
 + . − b4 − c4 
 2  7  8 
3  4 4 (b + c) 4 
= a (4b + 4c − (b + c) ) + 3a (2b + 2c − (b + c) ) +  b + c −
3 3 3 2 2 2 2

7 8 
3
= 3a (b + c )(b − c) 2 + 3a 2 (b − c )2 + (b − c ) 2 (7b 2 + 7c 2 + 10bc)
56
3
= 3a (a + b + c)(b − c )2 + (b − c) 2 (7b 2 + 7c 2 + 10bc)
56

7
3
h ng (b − c )2 (7b 2 + 7c 2 + 10bc) luôn không âm. N u a, b, c cùng d u thì b t
56
ng th c c n ch ng minh là hi n nhiên. N u a, b, c không cùng d u thì ph i có ít
nh t 1 trong ba s a, b, c cùng d u v i a + b + c . Không m t tính t ng quát, gi s
ó là a .
 b+c b+c
ng th c trên suy ra F ( a, b, c ) ≥ F  a, ,  . Nh v y ta ch còn c n
 2 2 
ch ng minh
F ( a, b, b) ≥ 0 ∀a, b ∈ R
4
⇔ 2( a + b)4 + (2b)4 − .(a 4 + 2b 4 ) ≥ 0 ∀a, b ∈ R
7
u b = 0 thì b t ng th c là hi n nhiên. N u b ≠ 0 , chia hai v c a b t ng th c
a
cho b 4 r i t x= thì ta cb t ng th c t ng ng
b
4
2( x + 1)4 + 16 − .( x 4 + 2) ≥ 0
7
t ng th c cu i cùng có th ch ng minh nh sau
4
Xét f ( x) = 2( x + 1)4 + 16 − .( x 4 + 2)
7
Ta có
16 3
f / ( x) = 8( x + 1)3 − .x
7
2
f / ( x) = 0 ⇔ x + 1 = .x ⇔ x = −2.9294
3
7
f min = f (−2.9294) = 0.4924 > 0
(Các ph n tính toán cu i c tính v i chính xác t i 4 ch s sau d u ph y. Do
f min tính c là 0.4924 nên n u tính c sai s tuy t i thì giá tr chính xác c a
f min v n là m t s d ng. Vì ây là m t b t ng th c r t ch t nên không th tránh

8
4 16
c các tính toán v i s l trên ây. Ch ng h n n u thay b ng xmin = −3
7 27
4
*
thì f min có giá tr âm! ây f * ( x) = 2( x + 1) 4 + 16 − .( x 4 + 2) .)
7
3.2. Ph ng pháp chu n hóa.
ng th ng g p c a b t ng th c thu n nh t là
f ( x1 , x2 ,..., xn ) ≥ g ( x1 , x2 ,..., xn )
trong ó f và g là hai hàm thu n nh t cùng b c.
Do tính ch t c a hàm thu n nh t, ta có th chuy n vi c ch ng minh b t ng th c
trên v vi c ch ng minh b t ng th c f ( x1 , x2 ,..., xn ) ≥ A v i m i x1 , x2 ,..., xn th a
mãn u ki n g ( x1 , x2 ,..., xn ) = A . Chu n hóa m t cách thích h p, ta có th làm n
gi n các bi u th c c a b t ng th c c n ch ng minh, t n d ng c m t s tính
ch t c bi t c a các h ng s .
Ví d 3. (B t ng th c v trung bình l y th a)
Cho b n s th c d ng ( x) = ( x1 , x2 ,..., xn ) . V i m i s th c r ta t
1
 x1r + x2r + ... + xnr  r
M r ( x) =  
 n 
Ch ng minh r ng v i m i r > s > 0 ta có M r ( x ) ≥ M s ( x ).
i gi i.
Vì M r (tx ) = tM r ( x) v i m i t > 0 nên ta ch c n ch ng minh b t ng th c úng
cho các s th c d ng x1 , x2 ,..., xn tho mãn u ki n M s ( x) = 1 , t c là c n ch ng
minh M r ( x ) ≥ 1 v i m i x1 , x2 ,..., xn tho mãn u ki n M s ( x) = 1 . u này có th
vi t n gi n l i là
Ch ng minh x1r + x2r + ... + xnr ≥ n v i x1s + x2s + ... + xns = n .
ch ng minh b t ng th c cu i cùng, ta áp d ng b t ng th c Bernoulli
r r
r
xir = ( xis ) s = (1 + ( xis − 1)) s ≥ 1 + .( xis − 1) ∀i = 1, n
s
ng các b t ng th c trên l i, ta c u ph i ch ng minh.
9
Ví d 4. (VMO 2002)
Ch ng minh r ng v i x, y , z là các s th c b t k ta có b t ng th c
3
6( x + y + z )( x 2 + y 2 + z 2 ) ≤ 27 xyz + 10( x 2 + y 2 + z 2 ) 2
i gi i.
t ng th c này r t c ng k nh. N u th c hi n phép bi n i tr c ti p s r t khó
kh n (ví d th bình ph ng kh c n). Ta th c hi n phép chu n hóa n gi n
hóa b t ng th c ã cho. N u x 2 + y 2 + z 2 = 0 , thì x = y = z = 0 , b t ng th c tr

thành ng th c. N u x 2 + y 2 + z 2 > 0 , do b t ng th c ã cho là thu n nh t, ta có

th gi s x 2 + y 2 + z 2 = 9 . Ta c n ch ng minh 2( x + y + z ) ≤ xyz + 10 v i u ki n

x2 + y2 + z 2 = 9 . ch ng minh u này, ta ch c n ch ng minh

[2( x + y + z ) − xyz ]2 ≤ 100


Không m t tính t ng quát, có th gi s x ≤ y ≤ z . Áp d ng b t ng th c

Bunhiacopxky, ta có
[2 ( x + y + z ) − xyz ]2 = [2( x + y ) + z (2 − xy )]2
≤ [( x + y )2 + z 2 ][4 + (2 − xy )2 ]
= (9 + 2 xy )(8 − 4 xy + x 2 y 2 )
= 72 − 20 xy + x 2 y 2 + 2 x 3 y 3
= 100 + ( xy + 2)2 (2 xy − 7)

x ≤ y ≤ z ⇒ z 2 ≥ 3 ⇒ 2 xy ≤ x 2 + y 2 ≤ 6, t c là ( xy + 2) 2 (2 xy − 7) ≤ 0 . T ây,

t h p v i ánh giá trên ây ta c u c n ch ng minh.


x+ y z
 =
u b ng x y ra khi và ch khi  2 2 − xy .
 xy + 2 = 0

ây gi i ra c x = −1, y = 2, z = 2 .
thu t chu n hóa cho phép chúng ta bi n m t b t ng th c ph c t p thành m t
t ng th c có d ng n gi n h n. u này giúp ta có th áp d ng các bi n i
i s m t cách d dàng h n, thay vì ph i làm vi c v i các bi u th c c ng k nh ban
10
u. c bi t, sau khi chu n hóa xong, ta v n có th áp d ng ph ng pháp d n bi n
gi i. Ta a ra l i gi i th hai cho bài toán trên
t f ( x, y, z ) = 2( x + y + z ) − xyz .

Ta c n ch ng minh f ( x, y, z ) ≤ 10 v i x 2 + y 2 + z 2 = 9 .
Xét
 
f  x,


y2 + z 2 y2 + z 2
2
,
2
 − f ( x, y , z ) = 2


( )
2( y 2 + z 2 ) − y − z −
x( y − z)2
2

 2 x
= ( y − z )2  − 
 2( y 2 + z 2 ) + y + z 2 
 
+ N u x, y , z > 0 , ta xét hai tr ng h p
* 1 ≤ x ≤ y ≤ z . Khi ó

2( x + y + z ) − xyz ≤ 2 3( x 2 + y 2 + z 2 ) − 1 = 6 3 − 1 < 10
* 0 < x ≤ 1 . Khi ó

2( x + y + z ) − xyz ≤ 2 x + 2 2( y 2 + z 2 ) = 2 x + 2 2(9 − x 2 ) = g ( x)

Ta có g ( x) =
/
2 ( 9 − x2 − x 2 ) > 0 , suy ra g ( x) ≤ g (1) = 10 .
9 − x2
+ N u trong 3 s x, y , z có m t s âm, không m t tính t ng quát, ta có th gi s là

 y2 + z 2 y 2 + z 2 
x < 0 . Khi ó f  x, ,  ≥ f ( x, y , z ) , nên ta ch c n ch ng minh
 2 2 
 
 y2 + z 2 y2 + z 2 
f  x, ,  ≤ 10
 2 2 
 
x (9 − x 2 )
⇔ 2 x + 2 2(9 − x 2 ) − ≤ 10
2
⇔ h( x ) = x3 − 5 x + 4 2(9 − x 2 ) ≤ 20

4x 2
Ta có h / ( x) = 3 x 2 − 5 − .
9−x 2

11
Gi i ph ng trình h / ( x) = 0 (v i x < 0 ), ta c x = −1 . ây là mc c ic a
h , do ó h( x) ≤ h(−1) = 20 .
ng cách chu n hóa, ta có th a m t bài toán b t ng th c v bài toán tìm giá
tr l n nh t hay nh nh t c a m t hàm s trên m t mi n (ch ng h n trên hình c u
x 2 + y 2 + z 2 = 9 nh ví d 4). u này cho phép chúng ta v n d ng cm ts
thu t tìm giá tr l n nh t, giá tr nh nh t (ví d nh b t ng th c Jensen, hàm
i,...).
Ví d 5.
Cho a, b, c là các s th c d ng. Ch ng minh r ng

(b + c − a ) 2 (c + a − b) 2 (a + b − c)2 3
+ + ≥
a 2 + (b + c) 2 b 2 + (c + a ) 2 c 2 + (a + b) 2 5
i gi i.
Ta ch c n ch ng minh b t ng th c cho các s d ng a, b, c tho a + b + c = 1 .
Khi ó b t ng th c có th vi t l i thành
(1 − 2a )2 (1 − 2b)2 (1 − 2c ) 2 3
+ + ≥
2a 2 − 2a + 1 2b 2 − 2b + 1 2c 2 − 2c + 1 5
1 1 1 27
⇔ 2 + 2 + 2 ≤
2a − 2a + 1 2b − 2b + 1 2c − 2c + 1 5
27
⇔ f ( a ) + f (b) + f (c) ≤ (5.1)
5
1
Trong ó f ( x) =
2x − 2x + 1
2

27 1
ý r ng = 3 f   , ta th y (5.1) có d ng b t ng th c Jensen. Tuy nhiên, tính
5 3
o hàm c p hai c a f ( x) , ta có

4(6 x 2 − 6 x + 1)
f // ( x) =
(2 x 2 − 2 x + 1)3

12
 3− 3 3+ 3 
hàm ch l i trên kho ng  ,  nên không th áp d ng b t ng th c
 6 6 

27
Jensen m t cách tr c ti p. Ta ch ng minh f (a ) + f (b) + f (c ) ≤ b ng các nh n
5
xét b sung sau
1
f max = f   = 2
 2
 1 1 
f ( x) t ng trên  0,  và gi m trên  ,1
 2 2 
 3− 3   3 + 3  12
f = f  =
 6   6  7
 3− 3 3+ 3 
u có ít nh t 2 trong 3 s a, b, c n m trong kho ng  ,  , ch ng h n là
 6 6 

a, b thì áp d ng b t ng th c Jensen ta có
 a+b 1− c  4
f (a ) + f (b) ≤ 2 f  =2f  = 2
 2   2  c +1
Nh v y trong tr ng h p này, ta ch c n ch ng minh
1 4 27
+ 2 ≤
2c − 2c + 1 c + 1 5
2

Quy ng m u s và rút g n ta cb t ng th c t ng ng
27c 4 − 27c3 + 18c 2 − 7c + 1 ≥ 0
⇔ (3c − 1) 2 (3c 2 − c + 1) ≥ 0 (ñuùng)
Nh v y, ta ch còn c n xét tr ng h p có ít nh t hai s n m ngoài kho ng
 3− 3 3+ 3  3+ 3 3− 3
 ,  . N u ch ng h n a ≥ thì rõ ràng b, c ≤ và nh v y,
 6 6  6 6

36 27
do nh n xét trên f (a ) + f (b) + f (c) ≤ < .
7 5
3− 3
Ta ch còn duy nh t m t tr ng h p c n xét là có hai s , ch ng h n a, b ≤ .
6

13
3 3 1
Lúc này, do a + b ≤ 1 − nên c ≥ > .
3 3 2
Theo các nh n xét trên, ta có
3− 3   3  24 15 + 6 3 27
f ( a ) + f (b ) + f ( c ) ≤ 2 f  + f  = + < .
 6   3  7 13 5

Ghi chú.
Bài toán trên có m t cách gi i ng n g n và c áo h n nh sau
t ng th c có th vi t l i thành
a (b + c ) b(c + a ) c( a + b) 6
+ 2 + 2 ≤
a + (b + c) b + (c + a )
2 2 2
c + (a + b) 2
5
Không m t tính t ng quát, có th gi s a + b + c = 1 . Khi ó, b t ng th c vi t l i
thành
a(1 − a) b(1 − b) c (1 − c) 6
+ 2 + 2 ≤
2a − 2a + 1 2b − 2b + 1 2c − 2c + 1 5
2

( a + 1) 2 ( a + 1) 2 (1 − a )(3 + a )
Ta có 2a(1 − a) ≤ . Do ó 1 − 2a + 2a ≥ 1 −
2
= .T ó
4 4 4
a(1 − a ) a (1 − a ) 4a
≤ =
2a − 2a + 1 (1 − a )(3 + a) 3 + a
2

4
ng t
b(1 − b ) 4b

2b − 2b + 1 3 + b
2

c(1 − c ) 4c
≤ .
2c − 2c + 1 3 + c
2

Và ch ng minh b t ng th c u bài, ta ch c n ch ng minh


4a 4b 4c 6
+ + ≤
3+ a 3+b 3+c 5
1 1 1 9
t ng th c cu i cùng này t ng ng v i + + ≥ là hi n
3 + a 3 + b 3 + c 10
nhiên (Áp d ng B T AM-GM).

14
Chu n hóa là m t k thu t c b n. Tuy nhiên, k thu t ó c ng òi h i nh ng kinh
nghi m và tinh t nh t nh. Trong ví d trên, t i sao ta l i chu n hóa
x 2 + y 2 + z 2 = 9 mà không ph i là x 2 + y 2 + z 2 = 1 (t nhiên h n)? Và ta có t
c nh ng hi u qu mong mu n không n u nh chu n hóa x + y + z = 1 ? ó là
nh ng v n mà chúng ta ph i suy ngh tr c khi th c hi n b c chu n hóa.
3.3. Ph ng pháp tr ng s .
t ng th c AM-GM và b t ng th c Bunhiacopxki là nh ng b t ng th c
thu n nh t. Vì th , chúng r t h u hi u trong vi c ch ng minh các b t ng th c
thu n nh t. Tuy nhiên, do u ki n x y ra d u b ng c a các b t ng th c này r t
nghiêm ng t nên vi c áp d ng m t cách tr c ti p và máy móc ôi khi khó em l i
t qu . áp d ng t t các b t ng th c này, chúng ta ph i nghiên c u k u
ki n x y ra d u b ng và áp d ng ph ng pháp tr ng s .
Ví d 6.
Ch ng minh r ng n u x, y , z là các s th c không âm thì
3
6(− x + y + z )( x 2 + y 2 + z 2 ) + 27 xyz ≤ 10( x 2 + y 2 + z 2 ) 2
i gi i.
d ng nguyên lý c b n « u b ng x y ra khi m t c p bi n s nào ó b ng nhau»,
ta có th tìm ta c d u b ng c a b t ng th c trên x y ra khi y = z = 2 x . u
này cho phép chúng ta m nh d n ánh giá nh sau
3
10( x + y +
2 2 2 2
z ) − 6(− x + y + z )( x 2 + y 2 + z 2 ) =
 1

= ( x + y + z ) 10( x + y + z ) − 6( − x + y + z ) 
2 2 2 2 2 2 2
 
 
 10 2 1 1

= ( x + y + z )  .( x + y + z ) (1 + 2 + 2 ) − 6(− x + y + z ) 
2 2 2 2 2 2 2 2 2 2
 3 
 
 10 
≥ ( x 2 + y 2 + z 2 )  .( x + 2 y + 2 z ) − 6(− x + y + z ) 
 3 
( x 2 + y 2 + z 2 )(28 x + 2 y + 2 z )
= (6.1)
3

15
Áp d ng b t ng th c AM-GM, ta có
4 4
 y2   z2  2 y   z 
2 2
x 2 y8 z8
x + y + z = x + 4  + 4  ≥ 9 x     = 9
2 2 2 29 9

 4   4  4   4 48

28 x + 2 y + 2 z = 7.4 x + 2 y + 2 z ≥ 9 9 (4 x )7 (2 y )(2 z ) = 9 9 48 x 7 yz
Nhân hai b t ng th c trên v theo v , ta c

x2 y8 z8 9 8 7
( x + y + z )(28 x + 2 y + 2 z ) ≥
2 2 2
99 .9 4 x yz = 81xyz (6.2)
48
(6.1) và (6.2) ta suy ra b t ng th c c n ch ng minh.
Trong ví d trên, chúng ta ã s d ng c b t ng th c Bunhiacopxki và b t ng
th c AM-GM có tr ng s . L i gi i r t hi u qu và n t ng. Tuy nhiên, s thành
«
công c a l i gi i trên n m hai dòng ng n ng i u. Không có c oán »
ó, khó có th thu c k t qu mong mu n. D i ây ta s xét m t ví d v vi c
ch n các tr ng s thích h p b ng ph ng pháp h s b t nh các u ki n x y
ra d u b ng c tho mãn.
Ví d 7.
Ch ng minh r ng n u 0 ≤ x ≤ y thì ta có b t ng th c
1 1
13x ( y −
2 2 2
x ) + 9 x( y + 2 2 2
x ) ≤ 16 y 2
i gi i.
Ta s áp d ng b t ng th c AM-GM cho các tích v trái. Tuy nhiên, n u áp d ng
t cách tr c ti p thì ta c
13( x 2 + y 2 − x 2 ) 9( x 2 + y 2 + x 2 )
VT ≤ + = 9 x 2 + 11 y 2 (7.1)
2 2
ây không ph i là u mà ta c n (T ây ch có th suy ra VT ≤ 20 y 2 ). S d ta
không thu c ánh giá c n thi t là vì d u b ng không th ng th i x y ra hai
n áp d ng b t ng th c AM-GM. u ch nh, ta a vào các h s d ng a, b
nh sau

16
1 1
13( ax)( y −
2
x )9(by )( y +
2 2 2 2 2
x )
VT = +
a b
13( a x + y − x ) 9(b x + y 2 + x 2 )
2 2 2 2 2 2
≤ + (7.2)
2a 2b
ánh giá trên úng v i m i a, b > 0 (ch ng h n v i a = b = 1 ta c (7.1)) và ta s
ph i ch n a, b sao cho
a) V ph i không ph thu c vào x
b) D u b ng có th ng th i x y ra hai b t ng th c
Yêu c u này t ng ng v i h
13(a 2 − 1) 9(b 2 + 1)
 + =0
 2 a 2b
  2 2
∃x, y : a x = y − x
2 2

 b x = y + x
2 2 2 2

13(a 2 − 1) 9(b 2 + 1)
 + =0
c là có h  2a 2b .
a 2 + 1 = b 2 − 1

 1
a = 2
Gi i h ra, ta c . Thay hai giá tr này vào (7.2) ta c
b = 3
 2
 x2 2  9x2 
VT ≤ 13  + y − x  + 3 
2
+ y 2 + x 2  = 16 y 2
 4   4 
Ghi chú.
Trong ví d trên, th c ch t ta ã c nh y và tìm giá tr l n nh t c a v trái khi x
thay i trong n [0, y ] .
4. B t ng th c thu n nh t i x ng.
Khi g p các b t ng th c d ng a th c thu n nh t i x ng, ngoài các ph ng
pháp trên, ta còn có th s d ng ph ng pháp khai tri n tr c ti p và d ng nh lý v
nhóm các s h ng. Ph ng pháp này c ng k nh, không th t p nh ng ôi lúc t ra

17
khá hi u qu . Khi s d ng b ng ph ng pháp này, chúng ta th ng dùng các ký
hi u quy c sau n gi n hóa cách vi t

∑ Q( x1 , x2 ,..., xn ) = ∑ Q( xσ (1) , xσ (2) ,..., xσ ( n) )


sym σ

trong ó, σ ch y qua t t c các hoán v c a {1, 2,..., n} .


Ví d v i n = 3 và ba bi n s x, y , z thì

∑ x 3 = 2 x3 + 2 y 3 + 2 z 3
sym

∑ x 2 y = x2 y + y 2 z + z 2 x + x 2 z + z 2 y + y 2 x
sym

∑ xyz = 6 xyz
sym

i v i các bi u th c không hoàn toàn i x ng, ta có th s d ng ký hi u hoán v


vòng quanh nh sau

∑ x 2 y =x 2 y + y 2 z + z 2 x
cyc

Ph ng pháp này c xây d ng d a trên tính so sánh c c a m t s t ng i


ng cùng b c - nh lý v nhóm các s h ng (h qu c a b t ng th c Karamata)
mà chúng ta s phát bi u và ch ng minh d i ây. Trong tr ng h p 3 bi n, ta còn
có ng th c Schur.
u s = ( s1 , s2 ,..., sn ) và t = (t1 , t2 ,..., tn ) là hai dãy s không t ng. Ta nói r ng s là

 s1 + s2 + ... + sn = t1 + t2 + ... + tn
tr i c a t n u  .
 s1 + s2 + ... + si ≥ t1 + t2 + ... + ti ∀i = 1, n
nh lý Muirhead. («Nhóm»)
u s và t là các dãy s th c không âm sao cho s là tr i c a t thì

∑ x1s x2s ...xns ≥ ∑ x1t x2t ...xnt


1 2 n 1 2 n

sym sym

Ch ng minh.

18
u tiên ta ch ng minh r ng n u s là tr i c a t thì t n t i các h ng s không âm
kσ , v i σ ch y qua t p h p t t c các hoán v c a {1, 2,..., n} , có t ng b ng 1 sao
cho

∑ kσ (sσ (1) , sσ (2) ,..., sσ ( n) ) = (t1 , t2 ,..., tn )


σ

Sau ó, áp d ng b t ng th c AM-GM nh sau

∑ x1 x2σ (2 ) ...xnσ ( n ) = ∑ kτ x1σ (τ (1)) x2σ (τ (2 )) ...xnσ (τ ( n )) ≥ ∑ x1σ (1) x2σ (2 ) ...xnσ ( n )
sσ (1) s s s s s t t t

σ σ ,τ σ

Ví d , v i s = (5,2,1) và t = (3,3,2) , ta có
3 3 1 1
(3,3,2) = .(5,2,1) + . + .(2,1,5) + .(1, 2,5)
8 8 8 8
Và ta có ánh giá
3 x 5 y 2 z + 3 x 2 y 5 z + x 2 yz 5 + xy 2 z 5
≥ x3 y 3 z 2
8
ng b t ng th c trên và các b t ng th c t ng t , ta thu cb t ng th c

∑ x5 y 2 z ≥ ∑ x3 y3 z 2
sym sym

Ví d 8.
Ch ng minh r ng v i m i s th c d ng a, b, c ta có
1 1 1 1
+ 3 3 + 3 ≤
a + b + abc b + c + abc c + a + abc abc
3 3 3

i gi i.
Quy ng m u s và nhân hai v cho 2, ta có

∑ (a3 + b3 + abc)(b3 + c3 + abc)abc ≤


sym

≤ 2(a 3 + b3 + abc )(b3 + c3 + abc)(c 3 + a 3 + abc )


⇔ ∑ ( a 7 bc + 3a 4b 4c + 4a 5b 2c 2 + a 3b3c3 ) ≤
sym

≤ ∑ (a 3b3c 3 + 2a 6b3 + 3a 4b 4c + 25 b 2c 2 + a 7bc)


sym

⇔ ∑ (2a b − 2a5b2c 2 ) ≥ 0
6 3

sym

t ng th c này úng theo nh lý nhóm.


19
Trong ví d trên, chúng ta ã g p may vì sau khi th c hi n các phép bi n i is ,
ta thu cm tb t ng th c t ng i n gi n, có th áp d ng tr c ti p nh lý
nhóm. Tuy nhiên, không ph i tr ng h p nào nh lý này c ng gi i quy t v n
. Trong tr ng h p 3 bi n s , ta có m t k t qu r t p khác là nh lý Schur.
nh lý. (Schur)
Cho x, y , z là các s th c không âm. Khi ó v i m i r > 0

x r ( x − y )( x − z ) + y r ( y − z )( y − x) + z r ( z − x)( z − y ) ≥ 0
u b ng x y ra khi và ch khi x = y = z hay khi hai trong ba s x, y , z b ng nhau
còn s th ba b ng 0.
Ch ng minh.
Vì b t ng th c hoàn toàn i x ng i v i ba bi n s , không m t tính t ng quát,
ta có th gi s x ≥ y ≥ z . Khi ó b t ng th c có th vi t l i d i d ng

( x − y )( x r ( x − z ) − y r ( y − z )) + z r ( x − z )( y − z ) ≥ 0
và m i m t th a s v trái u hi n nhiên không âm.
Tr ng h p hay c s d ng nh t c a b t ng th c Schur là khi r = 1 . B t ng
th c này có th vi t l i d i d ng

∑ ( x2 − 2 x 2 y + xyz) ≥ 0
sym

ây chính là b t ng th c ví d 1.
Ví d 9.
Cho a, b, c là các s d ng. Ch ng minh r ng

 1 1 1  9
(ab + bc + ca )  + + 2 

 ( a + b ) 2
(b + c ) 2
( c + a )  4
i gi i.
Quy ng m u s , khai tri n và rút g n, ta c

∑ (4a5b − a 4b2 − 3a3b3 + a 4bc − 2a3b2c + a 2b2c 2 ) ≥ 0 (9.1)


sym

Dùng b t ng th c Schur
x( x − y )( x − z ) + y ( y − z )( y − x) + z ( z − x)( z − y ) ≥ 0

20
Nhân hai v v i 2xyz r i c ng l i, ta c

∑ (a 4bc − 2a3b 2c + a 2b2c 2 ) ≥ 0 (9.2)


sym

Ngoài ra, áp d ng nh lý nhóm (hay nói cách khác − b t ng th c AM-GM có


tr ng s ) ta có

∑ (4a5b − a 4b 2 − 3a3b3 ) ≥ 0 (9.3)


sym

(9.2), (9.3) suy ra (9.1) và ó chính là u ph i ch ng minh.


Nói nb t ng th c thu n nh t i x ng, không th không nói n các hàm s
n
i x ng c b n. ó là các bi u th c S1 = ∑ xi , S 2 = ∑ xi x j ,..., Sn = x1 x2 ...xn .
i =1 1≤i < j ≤ n

i các b t ng th c liên quan n các hàm i x ng này, có m t th thu t r t h u


hi u c g i là «th thu t gi m bi n s b ng nh lý Rolle». Chúng ta trình bày ý
ng c a th thu t này thông qua ví d sau
Ví d 10.
Cho a, b, c, d là các s th c d ng. Ch ng minh r ng
1 1
 ab + ac + ad + bc + bd + cd  2  abc + abd + acd + bcd  3
  ≥ 
 6   4 
i gi i.
t S 2 = ab + ac + ad + bc + bd + cd , S3 = abc + abd + acd + bcd . Xét a th c

P ( x) = ( x − a )( x − b)( x − c)( x − d ) = x 4 − ( a + b + c + d ) x 3 + S2 x 2 − S3 x + abcd


P ( x ) có 4 nghi m th c a, b, c, d (n u có các nghi m trùng nhau thì ó là nghi m

i). Theo nh lý Rolle, P / ( x) c ng có 3 nghi m ( u d ng) u , v, w . Do P / ( x)


có h s cao nh t b ng 4 nên
P / ( x ) = 4( x − u )( x − v)( x − w) = 4 x 3 − 4(u + v + w) x 2 + 4(uv + vw + wu ) x − 4uvw
t khác
P / ( x ) = 4 x 3 − 3(a + b + c + d ) x 2 + S 2 x − S3

21
suy ra S 2 = 2(uv + vw + wu ), S3 = 4uvw và b t ng th c c n ch ng minh u bài
có th vi t l i theo ngôn ng u , v, w là
1
 uv + vw + wu
1
2
  ≥ (uvw)
3
 3 
t ng th c này hi n nhiên úng theo b t ng th c AM-GM.
5. Thu n nh t hóa b t ng th c không thu n nh t.
Trong các ph n trên, chúng ta ã trình bày các ph ng pháp c b n ch ng minh
tb t ng th c thu n nh t. ó không ph i là t t c các ph ng pháp (và d nhiên
không bao gi có th tìm c t t c !), tuy v y có th giúp chúng ta nh h ng t t
khi g p các b t ng th c thu n nh t. Nh ng n u g p b t ng th c không thu n
nh t thì sao nh ? Có th b ng cách nào ó a các b t ng th c không thu n
nh t v các b t ng th c thu n nh t và áp d ng các ph ng pháp nói trên c
không? Câu tr l i là có. Trong h u h t các tr ng h p, các b t ng th c không
thu n nh t có th av b t ng th c thu n nh t b ng m t quá trình mà ta g i là
thu n nh t hóa. Chúng ta không th “ch ng minh” m t “ nh lý” c phát bi u
ki u nh th , nh ng có hai lý do tin vào nó: th nh t, th c ra ch có các i
ng cùng b c m i có th so sánh c, còn các il ng khác b c ch so sánh
c trong các ràng bu c nào ó. Th hai, nhi u b t ng th c không thu n nh t ã
c “t o ra” b ng cách chu n hóa ho c thay các bi n s b ng các h ng s . Ch c n
chúng ta i ng c l i quá trình trên là s tìm c nguyên d ng ban u.
t ví d r t n gi n cho lý lu n nêu trên là t b t ng th c thu n nh t
x 3 + y 3 + z 3 ≥ x 2 y + y 2 z + z 2 x , b ng cách cho z = 1 , ta cb t ng th c không
thu n nh t
x3 + y3 + 1 ≥ x2 y + y 2 + x
Ví d 11. (England 1999)
Cho p, q, r là các s th c d ng tho u ki n p + q + r = 1 . Ch ng minh
7( p + q + r ) ≤ 2 + 9 pqr

22
Ví d 12. (IMO 2000)
Cho a, b, c là các s th c d ng tho mãn u ki n abc = 1 . Ch ng minh

 1  1  1
 a − 1 +  b − 1 +  c − 1 +  ≤ 1
 b  c  a
ng d n.
x y z
t a = ,b = ,c = !
y z x
Ví d 13. (IMO, 1983)
Ch ng minh r ng n u a, b, c là ba c nh c a m t tam giác thì

a 2b ( a − b ) + b 2 c ( b − c ) + c 2 a ( c − a ) ≥ 0
ng d n.
t a = y + z, b = z + x, c = x + y !

23
Bài t p

Bài 1.
Cho x, y , z > 0 . Ch ng minh r ng

x 3 y 3 z 3 x3 z 3 y 3 x 2 y 2 z 2 yz zx xy
+ + + + + ≥ + + + + +
y 3 z 3 x3 z 3 y 3 x 3 yz zx xy x 2 y 2 z 2
Bài 2.
Ch ng minh b t ng th c sau v i m i s th c d ng x, y , z
9 x y z 2
≥ + + ≥
4( x + y + z ) ( x + y )( x + z ) ( y + z )( y + x) ( z + x)( z + y ) x + y + z
Bài 3.
Cho x, y , z là các s th c d ng tho mãn u ki n 2 x + 4 y + 7 z = 2 xyz . Tìm giá
tr nh nh t c a bi u th c
P= x+y+z
Bài 4.
Cho a, b, c là các s th c d ng tho a 2 + b2 + c 2 + abc = 4 . Ch ng minh r ng
a+b+c ≤3
Bài 5. (IMO 1984)
Cho x, y , z là các s th c không âm tho mãn u ki n x + y + z = 1 . Ch ng minh
ng
7
0 ≤ xy + yz + zx − 2 xyz ≤
27
Bài 6. (Iran, 1996)
Cho a, b, c > 0 . Ch ng minh r ng

 1 1 1  9
(ab + bc + ca )  + + 2 

 ( a + b ) 2
(b + c ) 2
( c + a )  4

24
Bài 7. (VMO 1996)
Cho a, b, c, d là các s th c không âm tho mãn u ki n
2(ab + ac + ad + bc + bd + cd ) + abc + abd + acd + bcd = 16
Ch ng minh r ng
3( a + b + c + d ) ≥ 2( ab + ac + ad + bc + bd + cd )
Bài 8. (Poland 1996)
Cho a, b, c là các s th c tho mãn u ki n a + b + c = 1 . Ch ng minh r ng
a b c 9
+ 2 + 2 ≤
a + 1 b + 1 c + 1 10
2

Bài 9. (Poland 1991)


Cho x, y , z là các s th c tho mãn u ki n x 2 + y 2 + z 2 = 2 . Ch ng minh r ng
x + y + z ≤ 2 + xyz
Bài 10. (IMO 2001)
Cho a, b, c > 0 . Ch ng minh r ng
a b c
+ + ≥1
a 2 + 8bc b2 + 8ca c 2 + 8ab

25
PH NG PHÁP D N BI N

I. M u.
c m chung c a nhi u b t ng th c, c bi t là các b t ng th c i s là d u
ng x y ra khi t t c ho c m t vài bi n s b ng nhau. Có m t ph ng pháp ánh
giá trung gian cho phép ta gi m bi n s c a b t ng th c c n ch ng minh. Ph ng
pháp d n bi n d a vào c m này làm gi m s bi n s c a b t ng th c, a
t ng th c v d ng n gi n h n có th ch ng minh tr c ti p b ng cách kh o sát
hàm m t bi n.
ch ng minh b t ng th c d ng f ( x1 , x2 ,..., xn ) ≥ 0, ta ch ng minh
f ( x1 , x2 ,..., xn ) ≥ f (t , t ,..., xn )
Trong ó t là l ng trung bình c a x1 , x2 ,... ch ng h n nh trung bình nhân ho c
trung bình c ng. N u c nh v y thì ti p t c sang b c th hai c a phép ch ng
minh là ch ra r ng
f (t , t ,..., xn ) ≥ 0
t nhiên, b t ng th c này ã gi m s bi n s i m t và th ng là d ch ng minh
nb t ng th c ban u. Vi c l a ch n l ng trung bình nào d n bi n tùy
thu c vào c thù c a bài toán, và ôi khi l ng t khá c bi t.
Th ng thì, b c th nh t trong 2 b c chính trên là khó h n c vì th c ch t ta
n ph i làm vi c v i các cl ng có ít nh t là ba bi n s . Sau ây là m t vài
ng d n bi n th ng g p.
II. Ph ng pháp d n bi n trong is .
1. D n bi n ba bi n s .
ây là ph n n gi n nh t c a ph ng pháp d n bi n. Và ng c l i c ng có th nói
ph ng pháp d n bi n hi u qu nh t trong tr ng h p này.

26
Ví d 1.1.
Cho a, b, c ≥ 0 th a mãn a 2 + b2 + c 2 = 3 . Ch ng minh r ng

a + b + c ≥ a 2b2 + b 2c 2 + c 2 a 2
i gi i.
t f (a, b, c) = a + b + c − a 2b 2 − b 2c 2 − c 2 a 2

Gi s a = min{a, b, c} thì d th y a ≤ 1, b 2 + c 2 ≥ 2 ⇒ b + c ≥ 2
Xét hi u

 b2 + c 2 b 2 + c 2   (b + c )2 1 
f (a, b, c) − f  a, ,  = (b − c) 2  − 
 2 2   4 2 
b + c + 2(b + c ) 
2
  
2 1 
≥ (b − c ) 2  − ≥0
4 2 +2
Do ó
 b2 + c2 b2 + c 2 
f (a, b, c) ≥ f  a, , 
 2 2 
 
(b 2 + c 2 ) 2
= a + 2(b + c ) − a (b + c ) −
2 2 2 2 2
4
(3 − a )2 2
= a + 2(3 − a 2 ) − a 2 (3 − a 2 ) −
4
 3(a + 1) 2 3 
= (a − 1)2  − 
 4 − 2
+ − 
 2(3 a ) 3 a 
3 3
≥ (a − 1)2  −  = 0
4 4
⇒ f ( a, b, c) ≥ 0
ng th c x y ra khi và ch khi a = b = c = 1.
Ví d 1.2.
Cho a, b, c ≥ 0 th a mãn a + b + c = 3 . Ch ng minh r ng

f (a, b, c) = (a 2 + a + 1)(b 2 + b + 1)(c 2 + c + 1) ≤ 27


i gi i.
Gi s a ≤ b, c ⇒ a ≤ 1, b + c ≥ 2. Xét hi u
27
 b+c b+c
f ( a , b, c ) − f  a , , =
 2 2 
(a 2 + a + 1)(b − c) 2 (4 − (b + c) 2 − (b + c) − 4bc)
= ≤0
16
 b+c b+c
⇒ f ( a, b, c) ≤ f  a, , 
 2 2 
2
  b + c 2 b + c 
= ( a + a + 1)  
2
+ + 1
  2  2 
 
( a − 1) (a ( a − 1)(a − 12a + 48) − 37 a − 71)
2 2
= + 27
16
≤ 27
⇒ f ( a, b, c) ≤ 27
ng th c x y ra khi và ch khi a = b = c = 1.
Ví d 1.3.
Cho a, b, c ∈ R . Ch ng minh r ng

f (a, b, c) = a 2 + b 2 + c 2 − ab − bc − ca ≥ 0
i gi i.
Xét hi u
 b+c b+c 3
f ( a , b, c ) − f  a , ,  = .(b − c ) ≥ 0
2

 2 2  4
 b+c b+c b+c  b+c
2 2
⇒ f ( a, b, c) ≥ f  a, ,  = a − a (b + c) + 
2
 = a−  ≥0
 2 2   2   2 
⇒ f ( a, b, c) ≥ 0
Nh n xét.
Ch c ai c ng c m th y ây là m t b t ng th c quá d , quá c b n và tôi ngh ch c
ng có ng i không hi u n i t i sao tôi l i a ví d này vào. Nh ng hãy chú ý
ng nh ng cái hay trong nh ng bài toán n gi n không ph i là không có và bây
gi tôi s trình bày ý t ng mà tôi c m th y thích thú nh t trong bài này mà mình
phát hi n c (có th không ch mình tôi).
Vì f (a, b, c) là hàm i x ng v i các bi n a, b, c nên theo trên, ta có

28
 b+c b+c
f ( a, b, c ) ≥ f  a, , 
 2 2 
b+c b+c
= f , a, 
 2 2 
 b + c 2a + b + c 2a + b + c 
≥ f , , 
 2 4 4 
= ... ≥ ...
Và ý t ng dãy s b t u xu t hi n.
Xét các dãy s (an ), (bn ),(cn ) c xác nh b i

a0 = a, b0 = b, c0 = c
b2 n + c2 n
a2 n+1 = a2 n , b2 n +1 = c2 n+1 = , ∀n ∈ N
2
a +c
a2 n+ 2 = b2 n +1 , b2 n+ 2 = c2 n +2 = 2 n +1 2 n +1 , ∀n ∈ N
2
th y
a+b+c
lim an = lim bn = lim cn = =t
n→+∞ n→+∞ n→+∞ 3

f (a, b, c) ≥ f (an , bn , cn ), ∀n ∈ N
Do hàm f (a, b, c) liên t c nên
f (a, b, c) ≥ f ( lim an , lim bn , lim cn ) = f (t , t , t ) = 0
n→+∞ n→+∞ n→+∞

⇒ f ( a, b, c) ≥ 0
ng th c x y ra khi và ch khi a = b = c.
Cách là trên là m t ý t ng có th nói là khá c áo và là c s hình thành nên
cách th c d n bi n b n bi n s mà chúng ta s xét ngay bây gi .
2. D n bi n b n bi n s .
Khác v i ba bi n s d n bi n b n bi n s khó kh n và ph c t p h n nhi u. Trong
tr ng h p này ki u d n bi n thông th ng mà chúng ta v n làm v i ba bi n vô tác
ng. Và ví d 1.3 chính là ti n xây d ng nên ng l i t ng quát gi i
quy t các bài b t ng th c có th gi i b ng d n bi n k t h p dãy s .

29
Ví d 2.1. (D tuy n IMO 1993)
Cho a, b, c, d ≥ 0 th a mãn a + b + c + d = 1 . Ch ng minh r ng
1 176
abc + abd + acd + bcd ≤ + .abcd
27 27
i gi i.
t
176
f (a, b, c, d ) = abc + abd + acd + bcd − .abcd
27
 176 
= bc (a + d ) + ad  b + c − .bc 
 27 
 176 
= ad (b + c ) + bc  a + d − .ad 
 27 
V im ib b ns (a, b, c, d ) th a mãn a + b + c + d = 1 , n u t n t i hai s trong
176
n s này, ch ng h n b, c th a mãn b + c − .bc ≤ 0 thì
27
 176 
f (a, b, c, d ) = bc( a + d ) + ad  b + c − .bc 
 27 
≤ bc( a + d )
3
b+c+a+d 
≤ 
 3 
1
=
27
Do ó, không m t tính t ng quát có th gi s v i m i b b n s (a, b, c, d ) th a
mãn a + b + c + d = 1 thì hai s b t k trong b b n s này, ch ng h n a, d , u th a
176
mãn a + d − .ad ≥ 0
27
Khi ó, ta có
 176 
f (a, b, c, d ) = ad (b + c ) + bc  a + d − .ad 
 27 
2
b+c  176 
≤ ad (b + c ) +   a + d − .ad 
 2   27 

30
 b+c b+c 
= f  a, , ,d 
 2 2 
Xét các dãy (bn ),(cn ), (d n ) c xác nh b i

b0 = b, c0 = c, d 0 = d
b2 n + c2 n
b2 n +1 = d 2 n , c2 n +1 = d 2 n +1 = , ∀n ∈ N
2
b +c
b2 n + 2 = c2 n+1 , c2 n +2 = d 2 n+ 2 = 2 n+1 2 n+1 , ∀n ∈ N
2
a + bn + cn + d n = 1 ∀n ∈ N

Khi ó, d th y  1− a
n→+∞
lim bn = lim c n = lim d n =
n→+∞ n→+∞ 3
cách t, ta có f (a, b, c, d ) ≤ f (a, bn , cn , d n ), ∀n ∈ N
Do f liên t c nên
f (a, b, c, d ) ≤ f (a, lim bn , lim cn , lim d n )
n→+∞ n→+∞ n→+∞

 1− a 1− a 1− a 
= f  a, , , 
 3 3 3 
2 3 3
 1 − a   1 − a  176  1 − a 
= 3a   +  − .a  
 3   3  27  3 
a (4a − 1)2 (11a − 14) 1
= +
729 27
1

27
⇒ pcm.
1 1 1 1 1 1 1 
ng th c x y ra khi và ch khi (a, b, c, d ) =  , , ,  ,  , , , 0  .
4 4 4 4 3 3 3 
Ngoài cách trên ta có th làm n gi n nh sau
a+d a+d 
Ta có th gi s f (a, b, c, d ) ≤ f  , b, c,  v i m i a, b, c, d ≥ 0 th a mãn
 2 2 
u ki n a + b + c + d = 1 (vì trong tr ng h p ng c l i bài toán c gi i quy t).
Vì tính i x ng c a hàm f (a, b, c, d ) ta có

31
a+d a+d  a+d b+c b+c a+d 
f (a, b, c, d ) ≤ f  , b, c, ≤ f  , , , 
 2 2   2 2 2 2 
a+d b+c 1 1
≤ f , , , 
 2 2 4 4
1 1 1 1 1
≤ f , , , =
 4 4 4 4  27
Cách làm trên khá hay nh ng ch có th áp d ng c v i m t s ít bài toán d ng
này.
Ví d 2.2.
Cho a, b, c, d ≥ 0 th a mãn a + b + c + d = 1 . Ch ng minh r ng
148 1
f (a, b, c, d ) = a 4 + b 4 + c 4 + d 4 + abcd ≥
27 27
i gi i.
Xét hi u
 a +b a +b  7 37 
D = f (a, b, c, d ) − f  , , c, d  = ( a − b)2  .(a − b) 2 + 3ab − .cd 
 2 2  8 27 

 a +b a +b 
ó, n u có ab ≥ cd ⇒ D ≥ 0 ⇒ f (a, b, c, d ) ≥ f  , , c, d 
 2 2 
Gi s a ≥ b ≥ c ≥ d .
Xét các dãy s (an ), (bn ),(cn ) c xác nh b i

a0 = a, b0 = b, c0 = c
a2 n−1 + c2 n−1
a2 n = b2 n−1 , b2 n = c2 n = ∀n ∈ N*
2
a2 n + b2 n
a2 n+1 = b2 n+1 = , c2 n+1 = c2 n ∀n ∈ N
2

an + bn + cn + d = 1 ∀n ∈ N

th y anbn ≥ cn d ∀n ∈ N
 a + b + c 1− d
 lim an = lim bn = lim cn = =
n→+∞ n→+∞ n→+∞ 3 3

32
f (a, b, c, d ) ≥ f (an , bn , cn , d ) ∀n ∈ N
Do f liên t c nên
f (a, b, c, d ) ≤ f ( lim an , lim bn , lim cn , d )
n→+∞ n→+∞ n→+∞

 1− d 1− d 1− d 
= f  d, , , 
 3 3 3 
4 3
1− d  148  1 − d 
= 3  +d +
4
d 
 3  27  3 
d (4d − 1) 2 (19d + 20) 1
= +
729 27
1

27
⇒ pcm.
1 1 1 1 1 1 1 
ng th c x y ra khi và ch khi (a, b, c, d ) =  , , ,  ,  , , , 0  .
4 4 4 4 3 3 3 
Ví d 2.3.
Cho a, b, c, d ≥ 0 th a mãn a + b + c + d = 4 . Ch ng minh r ng
16 + 2abcd ≥ 3( ab + ac + ad + bc + bd + cd )
i gi i.
Ta có
16 + 2abcd ≥ 3( ab + ac + ad + bc + bd + cd )
⇔ 3( a 2 + b 2 + c 2 + d 2 ) + 4abcd ≥ 16

t f (a, b, c, d ) = 3( a 2 + b 2 + c 2 + d 2 ) + 4abcd
Xét hi u
 c+d c+d  23 
D = f (a, b, c, d ) − f  a, b, ,  = (c − d )  − ab 
 2 2  2 
 c+d c+d 
ó nh n th y n u 3 ≥ 2ab ⇒ D ≥ 0 ⇒ f (a, b, c, d ) ≥ f  a, b, , 
 2 2 
n ây có th s d ng dãy s nh bài tr c ho c có th làm nh sau
Gi s a ≤ b ≤ c ≤ d ⇒ ab ≤ 1

33
 c+d c+d 
⇒ f ( a, b, c, d ) ≥ f  a, b, , 
 2 2 
3
= 3( a 2 + b 2 ) + .(c + d ) 2 + ab(c + d ) 2
2
3
= ((4 − a − b)2 − 6) ab + 3( a + b) 2 + (4 − a − b) 2
2
9
= ( x 2 − 8 x + 10) y + .x 2 − 12 x + 24
2
= g ( x, y )
Trong ó x = a + b, y = ab .

Ta có 2 y ≤ x ≤ 2. Xét các tr ng h p
2
9 9  4
+ N u x − 8 x + 10 ≥ 0 ⇒ g ( x, y ) ≥ .x 2 − 12 x + 24 = . x −  + 16 ≥ 16
2
2 2  3

+ N u x 2 − 8 x + 10 < 0
x2 9 2 ( x − 2) 2 ( x 2 − 4 x + 8)
⇒ g ( x, y ) ≥ ( x − 8 x + 10). + .x − 12 x + 24 =
2
+ 16 ≥ 16
4 2 4
⇒ pcm.
4 4 4 
ng th c x y ra khi và ch khi (a, b, c, d ) = (1,1,1,1),  , , , 0  .
3 3 3 
Ví du 2.4. (Vasile Cirtoaje)
Cho a, b, c, d ≥ 0 th a mãn a 2 + b 2 + c2 + d 2 = 1 . Ch ng minh r ng
(1 − a )(1 − b)(1 − c)(1 − d ) ≥ abcd
i gi i.
Ta có B sau
. (China TST 2004)
Cho a, b, c, d ≥ 0 th a mãn abcd = 1 . Khi ó, ta có
1 1 1 1
f (a, b, c, d ) = + + + ≥1
(1 + a ) 2 (1 + b) 2 (1 + c) 2 (1 + d ) 2
Ch ng minh.
th y, n u x, y > 0 th a mãn xy ≥ 1 thì

34
1 1 2
+ ≥
(1 + x ) (1 + y )
( )
2 2 2
1 + xy

ó ta có n u ab ≥ 1 thì f (a, b, c, d ) ≥ f ( ab , ab , c, d )
Gi s a ≥ b ≥ c ≥ d và xét các dãy s (an ), (bn ),(cn ) c xác nh b i

a0 = a, b0 = b, c0 = c
a2 n+1 = b2 n+1 = a2 nb2 n , c2 n +1 = c2 n , ∀n ∈ N
a2 n+ 2 = b2 n +2 = a2 n+1c2 n+1 , c2 n + 2 = b2 n +1 , ∀n ∈ N


a b c d = 1 ∀n ∈ N
 n n n
th y anbn ≥ 1 ∀n ∈ N
 1
 lim an = lim bn = lim cn = 3 abc =
n→+∞ n→+∞ n→+∞ 3
d
ó
f (a, b, c, d ) ≥ f (an , bn , cn , d ), ∀n ∈ N
⇒ f ( a, b, c, d ) ≥ f ( lim an , lim bn , lim cn , d )
n→+∞ n→+∞ n→+∞

 1 1 1 
= f  3 , 3 , 3 ,d 
 d d d 
33 d 2 1
= +
( ) (1 + d )2
2
3
d +1

=
3
d2 ( 3
d −1 ) ( 2 d + 2d + d
2
3 4 3 2
+ 43 d + 3 ) +1
( d + 1) (1 + d )
2
3 2

≥1
⇒ f ( a, b, c, d ) ≥ 1
yB c ch ng minh.
Tr l i bài toán, ta có a 2 + b 2 + c 2 + d 2 = 1 ⇒ a, b, c, d ∈ [0,1]
u abcd = 0 thì (1 − a )(1 − b)(1 − c)(1 − d ) ≥ abcd .
u abcd > 0 .

35
1− a 1− b 1− c 1− d
t x= ,y = ,z = ,t = ⇒ x, y, z , t > 0
a b c d
1 1 1 1
Gi thi t a 2 + b2 + c2 + d 2 = 1 ⇔ + + + =1
(1 + x ) (1 + y ) (1 + z ) (1 + t ) 2
2 2 2

Và b t ng th c c n ch ng minh t ng ng v i
xyzt ≥ 1
1
Gi s ng c l i xyzt < 1 . Khi ó, t t/ = thì xyzt / = 1 và t < t / .
xyz
Áp d ng B , ta c
1 1 1 1
1≤ + + +
(1 + x ) 2
(1 + y ) 2
(1 + z ) 2
(1 + t / ) 2
1 1 1 1
< + + + =1
(1 + x ) 2
(1 + y ) 2
(1 + z ) 2
(1 + t ) 2
y u gi s sai.
⇒ xyzt ≥ 1
⇒ pcm.
1
ng th c x y ra khi và ch khi a = b = c = d = .
2
Nh n xét.
ây là m t bài toán hay và l i gi i v a r i ã s d ng hai công c là i bi n và
n bi n (v i các bi n m i). Ngoài ra có th d n bi n tr c ti p v i các bi n ban u
(dành cho m i ng i).
3. D n bi n v i nhi u bi n s h n.
Ví d 3.1.
Cho a, b, c, d , e ≥ 0 th a mãn a + b + c + d + e = 5 . Ch ng minh r ng

f (a, b, c, d , e) = 4( a 2 + b 2 + c 2 + d 2 + e 2 ) + 5abcde ≥ 25
i gi i.
Xét hi u
 d +e d +e 2 5 
D = f (a, b, c, d , e) − f  a, b, c, ,  = (d − e)  2 − .abc 
 2 2   4 
36
8  d +e d +e
ó, ta có n u abc ≤ ⇒ D ≥ 0 ⇒ f (a, b, c, d , e) ≥ f  a, b, c, , .
5  2 2 
Gi s a ≤ b ≤ c ≤ d ≤ e và xét các dãy s (cn ),( dn ), (en ) c xác nh b i

c0 = c, d 0 = d , e0 = e
d 2 n −2 + e2 n− 2
c2 n −1 = c2 n −2 , d 2 n −1 = e2 n −1 = , ∀n ∈ N*
2
c +e
c2 n = d 2 n −1 , d 2 n = e2 n = 2 n −1 2 n−1 , ∀n ∈ N*
2
th y
a + b + cn + d n + en = 1 ∀n ∈
8
a ≤ b ≤ min{cn , d n , en } ∀n ∈ ⇒ abcn ≤ ∀n ∈
5

c+ d +e 5−a −b
lim cn = lim d n = lim en = =
n→+∞ n→+∞ n→+∞ 3 3
ó, ta có
f (a, b, c, d , e) ≥ f ( a, b, cn , d n , en ) ∀n ∈ N
Suy ra
f (a, b, c, d , e) ≥ f ( a, b, lim cn , lim d n , lim en )
n→+∞ n→+∞ n→+∞

 5−a −b 5− a −b 5−a −b 
= f  a, b, , , 
 3 3 3 
4 5ab(5 − a − b)3
= 4( a 2 + b 2 ) + .(5 − a − b) 2 +
3 27
4 5ab(5 − a − b)3
= 4( a + b) 2 − 8ab + .(5 − a − b)2 +
3 27
5 y (5 − x)
2 3
16 x − 40 x + 100
2
= − 8y +
27 3
= g ( y)
Trong ó x = a + b, y = ab .
Ta có
10 y (5 − x)3
g / ( y) = −8
27

37
10 y (5 − x)3
+N u − 8 ≥ 0 thì
27
2
16 x 2 − 40 x + 100 16  5
g ( y ) ≥ g (0) = = .  x −  + 25 ≥ 25
3 3  4

10 y (5 − x)3
+N u − 8 < 0 thì
27
 x2 
g ( y) ≥ g  
 4 
  x2  
 5   (5 − x)  2
3

x 16 x 2 − 40 x + 100
=  
4
− 8. +
 27  4 3
 
 
( x − 2) 2 (−5 x 3 + 55 x 2 − 135 x + 225)
= + 25
108
dàng ch ng minh
−5 x3 + 55 x 2 − 135x + 225 ≥ 0 ∀x ∈ [0, 2]
Do ó
g ( y ) ≥ 25
⇒ pcm.
5 5 5 5 
ng th c x y ra khi và ch khi (a, b, c, d , e) = (1,1,1,1,1),  , , , , 0  .
4 4 4 4 
Ví d 3.2.
Cho x1 , x2 ,..., xn ≥ 0 th a mãn x1 + x2 + ... + xn = 1 . Tìm giá tr l n nh t c a bi u th c

f ( x1 , x2 ,..., xn ) = ∑ xi x j ( xi + x j )
1≤i < j ≤ n

i gi i.
n   n 2
Ta có f ( x1 , x2 ,..., xn ) = ∑ x i2 x j + ∑ i j ∑ i  ∑ j  = ∑ xi .(1 − xi )
x x 2
= x 2
. x
1≤i < j ≤ n 1≤ i < j ≤ n i =1  j ≠i  i=1
Xét hi u

38
f ( x1 ,..., xi + x j ,...,0, xn ) − f ( x1 ,..., xi ,..., x j ,..., xn ) = 2 xi x j (2 − 3( xi + x j ))

Do ó, n u 3( xi + x j ) ≤ 2 , thì f ( x1 ,..., xi ,..., x j ,..., xn ) ≤ f ( x1 ,..., xi + x j ,...,0, xn ) .

Xét t t c các b s ( x1 , x2 ,..., xn ) sao cho f ( x1 , x2 ,..., xn ) t max f .


Trong ó, ch n ra b s (a1 , a2 ,..., an ) sao cho s ph n t d ng trong b s ó là ít
nh t (luôn có th ch n c vì s s d ng là h u h n).
Gi s a1 ≥ a2 ≥ ... ≥ ak > 0 = ak +1 = ak + 2 = ... = an .
u k ≥ 3 thì ta có
a2 + a3 3
1 = a1 + a2 + ... + an ≥ + a2 + a3 = .(a2 + a3 ) ⇒ 3( a2 + a3 ) ≤ 2
2 2
Do ó
f (a1 , a2 ,..., an ) ≤ f ( a1 , a2 + a3 , 0,..., an ) ⇒ f (a1 , a2 + a3 ,0,..., an ) = max f
u này vô lý do b s (a1 , a2 + a3 , 0,..., an ) có s s d ng ít h n b s (a1 , a2 ,..., an ) .
y k ≤ 2 . Do ó
1
f (a1 , a2 ,..., an ) = a1a2 (a1 + a2 ) = a1 (1 − a1 ) ≤
4
Do ó
1
f ( x1 , x2 ,..., xn ) ≤
4
1
ng th c x y ra ch ng h n khi x1 = x2 = , x3 = x4 = ... = xn .
2
4. Các ki u d n bi n khác.
Trong môt s tr ng h p, các ki u d n bi n thông th ng ( ã nói ph n m u)
vô tác d ng (th ng do d u b ng không ph i x y ra khi t t c các bi n b ng nhau).
Vì v y, xu t hi n m t s ki u d n bi n khác.
Ví d 4.1.
Cho x, y , z ≥ 0 th a mãn xy + yz + zx = 1 . Tìm min c a
1 1 1
f ( x, y, z ) = + +
x+ y y+z z+x
i gi i.
39
Khác v i nh ng ví d tr c, ví d này có hai u khi n vi c d n bi n khó kh n
n là c c tr t c không ph i khi c ba bi n b ng nhau và bi u th c u ki n
a bi n h t s c khó ch u. Sau ây là m t trong nh ng l i gi i cho bài này.
Gi s x ≥ y , z và t a = y + z thì ax ≤ 1 và 2x ≥ a . Xét hi u

 1  (1 − ax)(2 x − a + a 2 x)
f ( x, y, z ) − f  0, a,  = ≥0
 a (1 + x 2 )(1 + a 2 )
 1  ( a − 1) 2 (2a 2 − a + 2) 5 5
⇒ f ( x, y , z ) ≥ f  0, a,  = + ≥
 a 2a(1 + a 2 ) 2 2
ng th c x y ra khi và ch khi ( x, y, z ) = (1,1,0).
y
5
min f ( x, y , z ) =
2
Ví d 4.2.
Cho a, b, c ≥ 0 th a mãn a + b + c = 1 . Tìm giá tr l n nh t c a bi u th c

f (a, b, c) = (a 3 + a + 7)(b 3 + b + 7)(c 3 + c + 7)


i gi i.
ng tính toán tr c ti p (ho c gi s có b = c ), ta d oán c max f = 441 t
c ch ng h n khi a = 1, b = c = 0. T ó, d n n l i gi i nh sau
2
Gi s a ≤ b, c ⇒ b + c ≥ .
3
t khác, do 0 ≤ a, b, c ≤ 1 ⇒ b 2 + c 2 ≤ b + c ≤ 1, bc ≤ 1 .
Xét hi u
f (a, b, c) − f (a, b + c, 0) = (a 3 + a + 7)bc(b 2c 2 + 7(b 2 + c 2 ) + 1 − 21(b + c ))
 2
≤ ( a 3 + a + 7)bc 1 + 7 + 1 − 21. 
 3
≤0
⇒ f ( a, b, c) ≤ f (a, b + c, 0)
= 7(a 3 + a + 7)((1 − a)3 + 1 − a + 7)
= 7 a (a − 1)((1 − a )(2 − a 2 + a 3 ) + 19) + 441 ≤ 441

40
ng th c x y ra khi và ch khi (a, b, c) = (1,0,0).
y max f = 441 .
III. D n bi n trong tam giác.
1. D n bi n l ng giác trong tam giác.
Trong tam giác ph ng pháp d n bi n ab t ng th c ã cho tr ng h p tam
giác th ng v tr ng h p tam giác cân.
Ví d 5.1.
Cho tam giác ABC không tù. Cg ng minh r ng
sin B.sin C sin C.sin A sin A.sin B 5
f ( A, B, C ) = + + ≥
sin A sin B sin C 2
i gi i.
π π
Gi s A ≥ B, C ⇒ ≥ A≥ .
2 3
Xét hi u
B−C  2 A 
sin 2  4sin 2
A.sin
 B+C B+C 
f ( A, B, C ) − f  A, = 2 . 2 − 1
 
2 
,
 2 sin A  sin B.sin C 
 
B −C
sin 2
≥ 2 . 4sin 2 A − 1
 
sin A  2 
≥0
2 B+C
sin
 B+C B+C  2 = 2sin A + 1 cotg A
⇒ f ( A, B, C ) ≥ f  A, ,  = 2sin A +
 2 2  sin A 2 2
A
t t = cotg ⇒ t ≥ 1 .
2

1 A 4t 1 (t − 1)(t 2 − 4t + 5) 5 5
2sin A + .cotg = 2 + .t = + ≥
2 2 t +1 2 2(t 2 + 1) 2 2

5
⇒ f ( A, B, C ) ≥
2

41
⇒ pcm.
π π
ng th c x y ra khi và ch khi A = , B = C = và các hoán v t ng ng.
2 4
Nh n xét.
ây là d ng l ng giác c a ví d 4.1. D th y r ng d n bi n bài này d ch u và d
ngh h n bài kia r t nhi u.
Ví d 5.2. (VMO 1993)
Cho tam giác ABC . Tìm min c a
f ( A, B, C ) = (1 + cos 2 A)(1 + cos 2 B )(1 + cos 2 C )
i gi i.
+ Cách 1.
π 1
Gi s A ≤ B, C ⇒ A ≤ ⇒ cos A ≥
3 2
Xét hi u
 B +C B +C 
f ( A, B, C ) − f  A, , =
 2 2 
B − C 6cos A − cos( B − C ) − 1
= (1 + cos 2 A).sin 2 .
2 2
B − C 3 −1 − 1
≥ (1 + cos 2 A).sin 2 .
2 2
≥0
 B +C B +C 
⇒ f ( A, B, C ) ≥ f  A, , 
 2 2 
2
 B+C 
= (1 + cos A) 1 + cos 2
2

 2 
(1 + cos 2 A) ( 3 − cos A )
2

=
4
(2cos A − 1) 2 (4(1 − cos A)(4 − cos A) + 3) 125
= +
64 64
125

64

42
π
ng th c x y ra khi và ch khi A = B = C = .
3
125
y min f ( A, B, C ) = .
64
+ Cách 2.
π C 3
Gi s A≥ B ≥C ⇒C ≤ ⇒ cos ≥
3 2 2
Ta có
(1 + cos 2 A)(1 + cos 2 B ) = (cos A + cos B ) 2 + (1 − cos A cos B ) 2
2
C A− B  2 A− B C
= 4sin 2
.cos 2 +  cos − 1 − cos 2 
2 2  2 2
 A− B 
= f  cos 2 
 2 
Ta có
 A− B  2 C  A− B C
f /  cos 2  = 4sin + 2  cos 2 − 1 − cos 2 
 2  2  2 2
 A−B C
= 2  cos 2 + 1 − 3cos 2 
 2 2
≤0
Do ó

A−B
2
  2 C 
f  cos 2  ≥ f (1) = 1 + sin 
 2   2
 A+ B A+ B 
⇒ f ( A, B, C ) ≥ f  , ,C 
 2 2 
125
n ây, l p lu n hoàn toàn t ng t nh cách 1, ta có min f ( A, B, C ) = .
64
Ví d 5.3.
Cho tam giác ABC . Ch ng minh r ng
A− B B −C C−A 2
cos + cos + cos ≥ .(sin A + sin B + sin C )
2 2 2 3
i gi i.
43
π
Gi s A ≤ B, C ⇒ A ≤ .B t ng th c c n ch ng minh t ng ng
3
B −C B −C π − 3A 2 4 A B−C
cos + 2cos .cos − .sin A − .cos .cos ≥0
2 4 4 3 3 2 2

 4 A  B−C  B −C π − 3A 2
⇔ 1 − .cos  .  2cos2 − 1 + 2cos .cos − .sin A ≥ 0
 3 2  4  4 4 3
 4 A π − 3A 2
Xét hàm s f ( x) = 1 − .cos  .(2 x 2 − 1) + 2 x.cos − sin A
 3 2 4 3

B −C  2 
i x = cos ⇒ x ∈ ,1
4  2 
Ta có
 4 A π − 3A
f / ( x) = 4 x 1 − .cos  + 2cos
 3 2 4
 4 π π − 3A
≤ 4 x 1 − .cos  + 2cos
 3 6 4
π − 3A
= −4 x + 2cos
4
2 π − 3A
< −4. + 2cos
2 4
<0
4 A π − 3A 2
⇒ f ( x ) ≥ f (1) = 1 − .cos + 2cos − .sin A = g ( A)
3 2 4 3
Ta có
2 2 A 3 π − 3A
g / ( A) = − .cos A + .sin + .sin
3 3 2 2 4
 A A  A  2  A A 3 2 
=  sin + cos  2sin − 1  . sin + cos  − 
 4 4  2  3  4 4 4 
π
≤ 0(do 0 < A ≤ )
3
π 
⇒ g ( A) ≥ g   = 0
3
⇒ f (1) ≥ 0

44
⇒ f ( x) ≥ 0
⇒ ñpcm.
Nh n xét. Vi c s d ng công c o hàm trong ph ng pháp d n bi n r t có l i khi
vi c bi n it ng ng ph c t p.
2. D n bi n theo các c nh.
Ví d .
Cho tam giác ABC th a mãn a ≥ b, c . Ch ng minh r ng

3
la + mb + mc ≤ .( a + b + c )
2
i gi i.
Ta coù

la + mb + mc =
bc
b+c
1
. (b + c) 2 − a 2 + .
2 ( 2a 2 + 2b 2 − c 2 + 2a 2 − b 2 + 2c 2 )
= f ( a , b, c)

Tröôùc heát, ta chöùng minh

b+c
2
2a + 2b − c + 2a − b + 2c ≤ 2 2a + 
2 2 2 2 2 2

2
(1)
 2 
Thaät vaäy

(1) ⇔ (b − c)2 (a + b + c)(b + c − a) ≥ 0 (hieån nhieân ñuùng)


Maët khaùc, ta laïi coù

bc 1
. (b + c ) 2 − a 2 ≤ . (b + c ) 2 − a 2 (2)
b+c 2
Töø (1) vaø (2), ta coù
2
1 b+c
f (a, b, c) ≤ . (b + c ) 2 − a 2 + 2a 2 +  
2  2 
 b+c b+c
= f  a, ,  (3)
 2 2 
Ta seõ chöùng minh

45
 b+c b+c 3
f  a, , ≤ .( a + b + c ) (4)
 2 2  2
Thaät vaäy

b+c
2
1 3
(4) ⇔ . (b + c ) 2 − a 2 + 2a 2 +   ≤ .(a + b + c)
2  2  2
2 2
b+c b+c  b+c
⇔   −1 + 8 +   ≤ 3 1 + 
 a   a   a 
b+c
⇔ x 2 − 1 + 8 + x 2 ≤ 3(1 + x) ( trong ñoù x = ⇒ x ∈ (1, 2])
a
⇔ ( x − 2)3 ( x + 2) ≤ 0 (hieån nhieân ñuùng)
Keát hôïp (3) vaø (4), ta suy ra ñpcm.
Ñaúng thöùc xaûy ra khi vaø chæ khi a = b = c.
Tuy ã r t c g ng nh ng bài vi t này c ng không th vét h t các ki u và d ng bài
p d n bi n c ng nh nói v t duy và cách th c hình thành ph ng pháp. Nh ng
tôi ngh nó c ng ã các b n hình thành nên ph ng pháp này trong u, t ó
các b n s t c m nh n c cái hay c a ph ng pháp này c ng nh các ki u d n
bi n khác mà bài vi t này ch a c p n. Chú ý r ng các l i gi i trên là phù
p v i bài vi t này nên c ng có th có nh ng cách khác hay h n.
VI. Bài t p.
Bài 1. (Vietnam TST 1996)
Cho a, b, c ∈ R.Tìm giaù trò nhoû nhaát cuûa bieåu thöùc

4
P = ( a + b) 4 + (b + c) 4 + (c + a ) 4 − .( a 4 + b4 + c 4 )
7
Bài 2. (China TST 2004)
Cho a, b, c, d > 0 thoûa maõn abcd = 1. Chöùng minh raèng
1 1 1 1
P= + + + ≥1
(1 + a ) 2
(1 + b) 2
(1 + c) 2
(1 + d ) 2

46
Bài 3.
Cho a, b, c ≥ 0 thoûa maõn a + b + c = 1. Chöùng minh raèng

15 1
a3 + b3 + c 3 + .abc ≥
4 4
Bài 4.
Cho a, b, c, d ≥ 0 thoûa maõn a + b + c + d = 4. Chöùng minh raèng

abc + abd + acd + bcd + a 2b 2c 2 + a 2b 2 d 2 + a 2 c 2 d 2 + b 2 c 2 d 2 ≤ 8


Bài 5. (Ph m Kim Hùng)
Cho a, b, c ≥ 0 thoûa maõn a + b + c = 3. Chöùng minh raèng

(a 2 + b)(b 2 + c)(c 2 + a) ≤ 13 + abc


Bài 6.
Cho a, b, c, d ≥ 0 thoûa maõn a + b + c + d = 4.
a) Ch ng minh r ng

2 ( )
a + b + c + d ≥ abc + abd + acd + bcd + 4

b) Tìm min c a

P=7 ( )
a + b + c + d − abc + abd + acd + bcd

Bài 7.
Cho tam giác nh n ABC . Ch ng minh r ng
2 2 2
 sin B.sin C   sin C.sin A   sin A.sin B  9
  +  +  ≥
 sin A   sin B   sin C  4
Bài 8.
Cho tam giác ABC . Ch ng minh r ng

(
p ≤ 2R + 3 3 − 4 r )
Bài 9.
Cho a, b, c, d , e ≥ 0 th a mãn a + b + c + d + e = 1 . Ch ng minh r ng
1845 1
a5 + b5 + c 5 + d 5 + e5 + .abcde ≥
256 256

47
Bài 10.
Cho a, b, c ≥ 0 thoûa maõn a + b + c = 3. Tìm giá tr l n nh t và giá tr nh nh t c a
bi u th c

( )(
P = a2 + a + 3 b 2 + b + 3 c2 + c + 3)( )
Bài 11. (Ph m Kim Hùng)
Cho a, b, c > 0 th a mãn abc = 1 . Ch ng minh r ng
a+3 b+3 c+3
+ + ≥3
(a + 1) (b + 1)
2 2
(c + 1) 2
Bài 12.
Cho x, y , z ≥ 0 th a mãn xy + yz + zx = 1 . Ch ng minh r ng
1 1 1 1
+ + ≥2+
y+z z+x x+ y 2
Bài 13.
Cho a, b, c ≥ 0 thoûa maõn a + b + c = 1. . Ch ng minh r ng

1− a 1− b 1− c 2
+ + ≤ 1+
1+ a 1+ b 1+ c 3
Bài 14.
Cho a, b, c, d ≥ 0 . Ch ng minh r ng

3( a 4 + b 4 + c 4 + d 4 ) + 4abcd ≥ ( a + b + c + d )(a 3 + b3 + c 3 + d 3 )
Bài 15. (Ph m Kim Hùng)
Cho a, b, c ≥ 0 thoûa maõn a + b + c = 3. Ch ng minh r ng

36( ab + bc + ca) ≥ ( a 3b3 + b3c 3 + c 3a 3 )( a 3 + b3 + c3 )


Bài 16. (Võ Qu c Bá C n)
Cho a, b, c ≥ 0 thoûa maõn a + b + c = 1. Tìm min

ab + bc + ca
P=
( a b + b c + c 4 a 4 )(a 4 + b 4 + c 4 )
4 4 4 4

48
N BI N KHÔNG XÁC NH

I. D n bi n không xác nh.


Cái tên nghe có v l nh ? tìm hi u ph ng pháp m i m này chúng ta hãy cùng
bàn n hai bài toán quen thu c sau
Bài toán 1.
Cho n là s nguyên d ng và x1 , x2 ..., xn là các s th c thu c n [ p, q ] v i p , q
là hai s th c cho tr c. Tìm giá tr l n nh t c a bi u th c f ( x1 , x2 ..., xn )
Bài toán 2.
Cho n là s nguyên d ng và là x1 , x2 ..., xn các s th c không âm có t ng b ng n .
Tìm giá tr nh nh t c a bi u th c f ( x1 , x2 ..., xn )
c hai bài trên thì f ( x1 , x2 ..., xn ) u là các bi u th c i x ng c a x1 , x2 ..., xn )
Thông th ng i v i các Bài toán 1 chúng ta th ng s p th t các bi n và d n giá
tr c a bi n v hai biên so sánh tr c ti p chúng. Ch ng h n so sánh
f ( x1 , x2 ..., xn ) v i f ( p, x2 ..., xn ) v i m c ích là a bài toán v tr ng h p n
gi n v i s l ng bi n ít h n. Còn v i Bài toán 2 ch c ch n các b n s ngh ngay
x +x x +x 
n ánh giá f ( x1 , x2 ..., xn ) ≥ f  1 2 , 1 2 ,..., xn  ho c hi h u l m thì chúng
 2 2 
ta có ánh giá f ( x1 , x2 ..., xn ) ≥ f (0, x1 + x2 ,..., xn ) . Có th th y nh ng suy ngh nh
trên là vô cùng t nhiên nh ng nói chung là khó th c hi n vì nh ng bài có th gi i
tr c ti p là t ng i n gi n. Vì v y chúng ta c n m t b c phát tri n h n cho
ph ng pháp này ó là d n bi n không xác nh. V y d n bi n không xác nh là
gì? Tôi có th gi i thi u luôn t t ng chính c a ph ng pháp này là “D n các bi n
do v m t trong nh ng m c bi t mà ta ch a th xác nh rõ s d n c th v
m c bi t nào”. Có v h i khó hi u ph i không? Chúng ta s cùng quay tr l i
i 2 bài toán trên

49
(i) V i Bài toán 1, thay vì ch ng minh f ( x1 , x2 ..., xn ) ≤ f ( p, x2 ,..., xn ) chúng ta s
ch ng minh
f ( x1 , x2 ..., xn ) ≤ max{ f ( p, x2 ,..., xn ), f (q, x2 ,..., xn )}
(ii) V i Bài toán 2, thay vì ánh giá ã nói trên chúng ta s ch ra c
  x1 + x2 x1 + x2  
f ( x1 , x2 ..., xn ) ≥ min  f  , ,..., xn  , f (0, x1 + x2 ,..., xn ) 
  2 2  
c n ây b n ng v i c i vì nó ch ti n b h n ph ng pháp ban um t
chút khi u ki n d n bi n c n i l ng mà trông l i có v ph c t p v i max, min
ng nh ng! B n hãy xem th s c m nh c a t t ng này thông qua ví d quen
thu c sau ây nh ng tr c h t chúng ta hãy nv iB c b n
1.
Cho a, b, c là các s th c th a mãn b ≥ c . Khi ó ít nh t m t trong hai b t ng th c
sau úng
(i) a ≥ c
(ii) a ≤ b
Ch ng minh.
Gi s c hai b t ng th c trên u sai ta suy ra c > a > b ≥ c (Mâu thu n).
qu 1.
Cho a, b là các s th c. Khi ó ít nh t m t trong hai b t ng th c sau úng
(i) a ≥ b
(ii) a ≤ b
Các b n ng nên xem th ng B 1, tuy ây là m t B n gi n theo úng
ngh a c a nó nh ng l i là m t B c c k hi u qu y. Sau ây là m t ví d cho
th y u ó
Ví d 1.
Cho p, q là hai s th c d ng, n là s nguyên d ng và x1 , x2 ,..., xn là các s th c
thu c n [ p, q ] v i p, q là hai s th c d ng cho tr c. Tìm giá tr l n nh t c a
bi u th c

50
1 1 1
f ( x1 , x2 ,..., xn ) = ( x1 + x2 + ... + xn )  + + ... + 
 x1 x2 xn 

i gi i.
1 1 1
t S = x2 + x3 + ... + xn , T = + + ... +
x2 x3 xn

f ( x1 , x2 ,..., xn ) ≤ f ( p, x2 ,..., xn )
1  1 
⇔ ( x1 + S )  + T  ≤ ( p + S )  + T 
 x1  p 
 S 
⇔ ( x1 − p )  T − ≤0
 px1 
S
⇔T ≤ (1)
px1
f ( x1 , x2 ,..., xn ) ≤ f ( q, x2 ,..., xn )
1  1 
⇔ ( x1 + S )  + T  ≤ (q + S )  + T 
 x1  q 
 S 
⇔ ( x1 − q )  T − ≤0
 qx1 
S
⇔T ≥ (2)
qx1

S S
Do ≥ nên theo B 1 s có ít nh t m t trong hai b t ng th c (1), (2)
px1 qx1
úng.
Suy ra f ( x1 , x2 ..., xn ) ≤ max{ f ( p, x2 ,..., xn ), f (q, x2 ,..., xn )}
Hoàn toàn t ng t ta nh n c k t qu sau
n t i y1 , y2 ,..., yn ∈ { p, q} sao cho f ( x1 , x2 ,..., xn ) ≤ f ( y1 , y2 ,..., yn )
Bài toán a v tìm giá tr l n nh t c a f ( y1 , y2 ,..., yn ) v i y1 , y2 ,..., yn ∈ { p, q} .
Không quá khó kh n chúng ta tìm c

51
n 2 ( p + q) 2 n
+ max f ( y1 , y2 ,..., yn ) = v i n ch n khi trong t p { y1 , y2 ,..., yn } có s
4 pq 2
n
ng p và s còn l i b ng q .
2
( n 2 − 1)( p + q ) 2
+ max f ( y1 , y2 ,..., yn ) = 1 + v i n l khi trong t p { y1 , y2 ,..., yn } có
4 pq
n −1 n +1
s b ng p và s còn l i b ng q ho c ng c l i.
2 2
ây chúng ta i n k t lu n cho bài toán.
Ch c h n các b n ã t ng gi i quy t bài toán này b ng cách s d ng ph ng pháp
hàm l i c ng r t nhanh g n nh ng có l chúng ta ph i công nh n v i nhau r ng
cách gi i b ng t t ng d n bi n không xác nh trên r t p và phù h p v i trình
c a c các b n Trung h c c s . B ng phép ch ng minh t ng t , chúng ta có
th gi i c bài toán sau
Ví d 2.
Cho p, q là hai s th c d ng, n là s nguyên d ng và x1 , x2 ,..., xn là các s th c
thu c n [ p, q ] . Tìm giá tr l n nh t c a bi u th c

x1n + x2n + ... + xnn


f ( x1 , x2 ,..., xn ) =
x1 x2 ...xn
hai ví d trên u ã có trên t p chí Toán H c Và Tu i Tr cùng v i tr ng h p
n = 3, p = 1, q = 2 tuy nhiên cách ch ng minh theo tôi c bi t r t thi u t nhiên và
khó có kh n ng gi i t ng quát.
Nh v y là i v i các bài toán b t ng th c có biên rõ ràng nh Bài toán 1 thì
chúng ta ã có m t l i gi i h p lý còn v i Bài toán 2 thì sao? Dù các bi n không
m trong m t gi i h n rõ ràng nh ng chúng ta có th t m hi u c r ng v i hai
bi n x1 , x2 thì chúng luôn n m trong [0, x1 + x2 ] và có nh ng c p m c bi t c n

x +x x +x 
chú ý là (0, x1 + x2 ) và  1 2 , 1 2  . gi i quy t tri t Bài toán 2 chúng ta
 2 2 
c th hóa t t ng d n bi n không xác nh b ng nh lý sau

52
II. nh lý d n bi n không xác nh U.M.V (Undefined Mixing Variables).
nh lý U.M.V. Cho x1 , x2 ,..., xn là các s th c không âm có t ng là m t h ng s
ng cho tr c. f ( x1 , x2 ,..., xn ) là m t hàm liên t c, i x ng c a ( x1 , x2 ,..., xn )
th a mãn u ki n
  x1 + x2 x1 + x2  
f ( x1 , x2 ,..., xn ) ≥ min  f  , ,..., xn  , f (0, x1 + x2 ..., xn ) 
  2 2  
i m i ( x1 , x2 ,..., xn ) th a mãn u ki n ã cho.
Khi ó, giá tr nh nh t c a f ( x1 , x2 ,..., xn ) là giá tr nh nh t c a
Ct (t = 0,1, 2,..., n − 1) trong ó Ct (t = 0,1, 2,..., n − 1) là giá tr c a f ( x1 , x2 ,..., xn )
khi trong ( x1 , x2 ,..., xn ) có t s b ng 0 và n − t s còn l i b ng nhau.
Ch ng minh.
Tr c h t, ta ch ng minh B sau
2. Cho m t b s th c không âm ( x1 , x2 ,..., xn ) (n ≥ 2) th c hi n phép bi n
i ∆ nh sau
Ch n xi = max ( x1 , x2 ,..., xn ) và x j = min ( x1 , x2 ,..., xn ) .

xi + x j
Gán xi , x j b i nh ng v n gi nguyên v trí c a chúng trong ( x1 , x2 ,..., xn ) .
2
x1 + x2 + ... + xn
Khi ó sau vô h n l n th c hi n ta c x1 = x2 = ... = xn = .
n
Ch ng minh.
Ký hi u dãy ban u là ( x11 , x12 ,..., x1n ) .
Ta ch ng minh b ng quy n p.
i n = 2 thì B hi n nhiên úng.
Gi s b úng v i n := n − 1 ta ch ng minh nó úng v i n := n .
Th t v y, gi s l n th k nào ó th c hi n phép bi n i ∆ ta s nh n cb
( x1k , x2k ,..., xnk ) .

i mk = min{x1k , x2k ,..., xnk }, M k = max{x1k , x2k ,..., xnk } .

53
th y {mk } là dãy không gi m b ch n trên b i M 1 nên ∃ lim mk = m , còn {M k }
k →∞

là dãy không t ng b ch n d i b i m1 nên ∃ lim M k = M .


k →∞

u b c th k nào ó th c hi n phép bi n i ∆ mà x1k = mk ho c x1k = M k thì


x1 c g i là có tham gia vào phép bi n i ∆ b c th k .
i u1 < u2 < ... < u s là t t c nh ng l n x1 tham gia phép bi n id i vai trò s
nh nh t, còn v1 < v2 < ... < vt là t t c nh ng l n x1 tham gia phép bi n id i vai
trò s l n nh t.
*) N u s + t < ∞ , t k0 = max{s, t} suy ra t b c k0 tr i thì x1 s không tham
gia vào phép bi n i ∆ n a. Nh th ta ch áp d ng phép bi n i này cho b
( x2k0 , x3k0 ,..., xnk0 ) .
Áp d ng gi thi t quy n p, ta nh n cb
x2k0 + x3k0 + ... + xnk0
x2k0 = x3k0 = ... = xnk0 = .
n −1
Do x1 không tham gia vào phép bi n i ∆ nào n a nên

x2k0 + x3k0 + ... + xnk0


x1k0 = x2k0 = ... = xnk0 =
n −1
ây ta có pcm.
**) N u s + t = ∞ . Không gi m t ng quát, gi s s = ∞ suy ra lim x1uk = m .
k →∞

+ Tr ng h p 1. t < ∞
Do lim mk = m, lim M k = M nên theo nh ngh a gi i h n thì v i m i ε > 0 nh
k →∞ k →∞

thì
∃n1 sao cho v i m i N > n1 thì mN − m < ε

∃n2 sao cho v i m i N > n2 thì M N − M < ε

Ch n n3 = max{vt , n1 , n2 } , suy ra v i m i ui − 1 > n3 thì

mui −1 − m < ε , M ui −1 − M < ε

54
mui −1 + M ui −1 M +m
mà x1ui = nên x1ui − <ε
2 2
M +m
⇒ lim x1ui =
i →∞ 2
M +m
⇒ lim x1k =
k →∞ 2
+ Tr ng h p 2. t = ∞ .
Hoàn toàn t ng t ta suy ra
M +m
lim x1ui =
i →∞ 2
M +m
lim x1vi =
i →∞ 2
M +m
Vì v y lim x1k =
k →∞ 2
Do ó trong m i tr ng h p ta u có
M +m
lim x1k =
k →∞ 2
M +m
Hoàn toàn t ng t ta nh n c k t qu sau lim xik = v i m i i = 1,2,..., n
k →∞ 2
nên ta có pcm.
Ch ng minh nh lý.
Th c hi n thu t toán βt v i t ∈ {0,1, 2,..., n − 1} cho tr ng h p t p ( x1 , x2 ,..., xn ) ã
có t s x1 = x2 = ... = xt = 0 nh sau

cho g n ta quy c f ( x1 , x2 ,..., xn ) = f ( xi , x j ) trong ó

xi = max{x1 , x2 ,..., xn }, x j = min{x1 , x2 ,..., xn } th a mãn x j > 0 .

 x + x j xi + x j 
Ti n hành so sánh f ( xi , x j ) v i f  i ,  và f (0, xi + x j ) .
 2 2 
 xi + x j xi + x j 
*) N u f ( xi , x j ) < f (0, xi + x j ) thì f ( xi , x j ) ≥ f  ,  . Khi ó áp d ng
 2 2 
thu t toán ∆ cho {xt +1 , xt + 2 ,..., xn } . N u trong m t b c nào ó l i có

55
f ( xi , x j ) ≥ f (0, xi + x j ) thì chuy n sang thu t toán β t+1 . N u không có thì phép

bi n i ∆ s c th c hi n vô h n l n nên xt∞+1 = xt∞+ 2 = ... = xn∞ .


**) N u f ( xi , x j ) ≥ f (0, xi + x j ) ta chuy n tr c ti p sang thu t toán β t+1 .

Rõ ràng thu t toán β n−1 ã là thu t toán h ng và ó là k t qu c nh.


Vì v y nh lý ã c ch ng minh hoàn ch nh.
Trong nh lý U.M.V ta có th thay th u ki n t ng các bi n b ng các u ki n
khác nh t ng bình ph ng, t ng l p ph ng...và có cách d n bi n t ng ng thì
nh lý v n úng và cách ch ng minh không có gì khác.
qu . Cho x1 , x2 ,..., xn là các s th c không âm có t ng là m t h ng s d ng
cho tr c. f ( x1 , x2 ,..., xn ) là m t hàm liên t c, i x ng c a ( x1 , x2 ,..., xn ) th a mãn
u ki n
   x1 + x2 x1 + x2  
 f ( x1 , x2 ..., xn ) ≥ min  f  , ,..., xn  , f (0, x1 + x2 ..., xn ) 
   2 2  

 f (0, x2 , x3 ,..., xn ) ≥ 0

 f  x1 + x2 ... + xn , x1 + x2 ... + xn ,..., x1 + x2 ... + xn  ≥ 0
  n n n 
v i m i ( x1 , x2 ,..., xn ) th a mãn u ki n ã cho thì f ( x1 , x2 ,..., xn ) ≥ 0 .
III. M t s ng d ng c a ph ng pháp d n bi n không xác nh.
s d ng ph ng pháp d n bi n không xác nh rõ ràng ta ph i th c hi n theo
trình t hai b c
c 1. Xác l p u ki n d n bi n.
c 2. Gi i quy t bài toán v i u ki n ã xác l p bên trên.
n nhiên B c 2 chính là n i dung c a nh lý U.M.V và ã c gi i quy t m t
cách hoàn toàn tri t . Do ó, ph n quan tr ng nh t c a chúng ta c n ph i làm ó
là th c hi n cB c 1. M t u kì l là b c này th ng c x lý r t g n nh
ng cách s d ng B 1, m t b g n nh hi n nhiên d a trên quan h th t
a các s trên tr c s th c. Chúng ta hãy tìm hi u rõ h n qua các ví d c tr ng
sau

56
Ví d 3. (Phát tri n t m t bài IMO)
Cho n là s nguyên d ng và x1 , x2 ,..., xn là các s th c không âm có t ng b ng n .
Tìm s th c d ng kn t t nh t b t ng th c sau luôn úng

(1 + x1 )(1 + x2 )...(1 + xn ) ≤ 2 n + kn .( x1 x2 ...xn − 1)


i gi i.
t f ( x1 , x2 ,..., xn ) = (1 + x1 )(1 + x2 )...(1 + xn ) − 2n − kn .( x1 x2 ...xn − 1)
S = (1 + x3 )(1 + x4 )...(1 + xn )
T = x3 x4 ...xn

x +x x +x 
f ( x1 , x2 ,..., xn ) ≤ f  1 2 , 1 2 ,..., xn  (3.1)
 2 2 
 x +x  x +x 
2 2

⇔ (1 + x1 )(1 + x2 ) S − kn x1 x2T − 1 + 1 2  S + kn  1 2  T ≤ 0
 2   2 
x −x 
2

⇔  1 2  ( k nT − S ) ≤ 0
 2 
⇔ k nT ≤ S (3.2)

f ( ( x1 , x2 ,..., xn ) ) ≤ f ( (0, x1 + x2 ,..., xn ) ) (3.3)


⇔ (1 + x1 )(1 + x2 ) S − kn x1 x2T − (1 + x1 + x2 ) S ≤ 0
⇔ knT ≥ S (3.4)
(3.2), (3.4) ta có ngay ít nh t m t trong hai b t ng th c (3.1), (3.3) úng suy ra
 x +x x +x  
f ( x1 , x2 ..., xn ) ≤ max  f  1 2 , 1 2 ,..., xn  , f (0, x1 + x2 ..., xn ) 
  2 2  
Theo nh lý U.M.V ta có
max f ( x1 , x2 ,..., xn ) = max Ct (t = 0,1,..., n − 1)
= max{C0 , C1}

  2n − 1 n−1 
= max 0,   − 2 n
+ k n
  n − 1  
Vì v y b t ng th c bài th a mãn thì

57
n−1
 2n − 1 
  − 2 n + kn ≤ 0
 n −1 
n−1
 2n − 1 
⇔ kn ≤ 2 −  n

 n −1 
n−1
 2n − 1 
Do ó giá tr t t nh t c a kn th a mãn bài là kn = 2 −  
n

 n −1 
Ví d 3 th c s là m t bài toán r t khó ã t ng có m t d ng này hay d ng khác
trong các thi vô ch. Ch c ch n các b n ã t ng c m nh n c bi u th c t
giá tr t t nh t ngoài tr ng h p n bi n b ng nhau thì còn m t tr ng h p m t bi n
ng 0 nh ng v n vô cùng t c t i vì không có cách nào ép nó v c 0. Gi ây
U.M.V ã cho b n m t h ng i khá sáng s a.
Ví d 4. ( inh Ng c An)
Cho p ≤ n là các s nguyên d ng và x1 , x2 ,..., xn là các s th c không âm có t ng
ng n . Tìm giá tr l n nh t c a bi u th c
f ( x1 , x2 ,..., xn ) = ∑ ( xi1 xi2 ...xi p ) k
1≤i1 ≤i2 ≤...≤i p ≤ n

Trong ó k là s th c không nh h n 2.
i gi i.
t
A= ∑ ( xi1 xi2 ...xip − 2 ) k
3≤i1 ≤ i2 ≤...≤ i p − 2 ≤ n

B= ∑ ( xi1 xi2 ...xi p −1 ) k


3≤i1 ≤ i2 ≤...≤ i p −1 ≤ n

C= ∑ ( xi1 xi2 ...xi p ) k


3≤ i1 ≤i2 ≤...≤i p ≤ n

Ta s ch ng minh
 x +x x +x  
f ( x1 , x2 ..., xn ) ≤ max  f  1 2 , 1 2 ,..., xn  , f (0, x1 + x2 ..., xn ) 
  2 2  
Th t v y

58
x +x x +x 
f ( x1 , x2 ..., xn ) ≤ f  1 2 , 1 2 ,..., xn  (4.1)
 2 2 
2k k
x +x  x +x 
⇔ x1k x2k A + ( x1k + x2k ) B − 1 2  A − 2 1 2  B ≤ 0
 2   2 

 x1 + x2 
2k

 2  − x1k x2k
⇔   B
k

x +x  A
x1k + x2k − 2 1 2 
 2 
f ( x1 , x2 ..., xn ) ≤ f (0, x1 + x2 ..., xn ) (4.2)
⇔ x1k x2k A + ( x1k + x2k ) B − ( x1 + x2 ) k B ≤ 0
B x1k x2k
⇔ ≥
A ( x1 + x2 )k − x1k − x2k
ít nh t m t trong hai b t ng th c (4.1), (4.2) ch c ch n úng thì
2k
 x1 + x2 
 2  − x1k x2k
  x1k x2k

 x + x 
k
( x1 + x2 ) k − x1k − x2k
x1 + x2 − 2 
k k 1 2

 2 
2k k
 x1 + x2   x1 + x2 
 2  − x1
k k
x2 x1
k
+ x2
k
− 2  2 
⇔  ≥  
k k
x1 x2 ( x1 + x2 ) − x1 − x2
k k k

2k k
 x1 + x2  x +x 
 2  ( x1 + x2 ) − 2  1 2 
k

⇔  ≥  2 
k k
x1 x2 ( x1 + x2 ) k − x1k − x2k
( x1 + x2 ) 2 k (2 k −1 − 1)( x1 + x2 ) k
⇔ 2 k k k ≥ k −1
2 x1 x2 2 (( x1 + x2 ) k − x1k − x2k )
⇔ ( x1 + x2 ) k (( x1 + x2 ) k − x1k − x2k ) ≥ (22 k − 2k +1 ) x1k x2k
u này hi n nhiên do
k
( x1 + x2 ) ≥ 2 ( x1 x2
k k
)2 (theo b t AM-GM)
k
( x1 + x2 ) − k
x1k − x2k ≥ (2 − 2)( x1 x2
k
)2 v i k ≥2

59
y ta có
 x +x x +x  
f ( x1 , x2 ..., xn ) ≤ max  f  1 2 , 1 2 ,..., xn  , f (0, x1 + x2 ..., xn ) 
  2 2  
Vì th theo nh lý U.M.V ta có
max f ( x1 , x2 ,..., xn ) = max Ct (t = 0,1,..., n − 1)
kp
 n 
= max Cnp−t .   (t = 0,1,..., n − 1)
 n −t 
 p p  n kp 
= max Cn , Cn−1.   
  n − 1  

i n = 3 ta có bài toán quen thu c


Cho a, b, c, k ≥ 0 th a mãn a + b + c = 3 . Ch ng minh r ng

  3 2 k 
(ab) + (bc) + (ca ) ≤ max 3,   
k k k

  2  
n th y có u gì kì l không? Hình nh U.M.V này ch ng thèm quan tâm ns
bi n n = 3 hay n b t kì thì c ng th .
Ví d 5. (t ng quát t b t Turkervici)
Cho n là s nguyên d ng và x1 , x2 ,..., x2 n là các s th c không âm. Ch ng minh
ng
(n − 1)( x12 n + x22 n + ... + x22nn ) + nx1 x2 ... x2 n ≥ ∑ xin x nj
1≤i< j ≤ 2 n

i gi i.
t ng th c ã cho t ng ng v i
2
 2n 
(2n − 1)( x12 n + x22 n + ... + x22nn ) + 2nx1 x2 ... x2 n ≥  ∑ xin 
 i=1 
t
2
 2n 
f ( x1 , x2 ,..., x2 n ) = (2n − 1)( x12 n + x22 n + ... + x22nn ) + 2nx1 x2 ...x2 n −  ∑ xin 
 i=1 

60
s = x1 x2

x1n + x2n
t= n ≥ x1 x2 = s
2
Ta có
 x n + xn xn + x n 
f ( x1 , x2 ,..., x2 n ) ≥ f  n 1 2 n 1
, 2
, x3 ,..., x2 n  (5.1)
 2 2 
 
⇔ (2n − 1)( x12 n + x22 n + ... + x22nn ) + 2nx1 x2 ...x2 n −
2
  x n + x n 2   xn + x n 

− (2n − 1) 2  1 2 2n 2n

 + x3 + ... + x2 n − 2n  n
1 2
 x3 ...x2 n ≥ 0
  
 
2   2 

( x1n − x2n ) 2  nx ...x 
⇔  (2n − 1) − n−1 n−23 2 n n−1 
≥0
2  t + t s + ... + s 
2n − 1 n−1 n−2
⇔ .(t + t s + ... + s n−1 ) ≥ x3 ...x2 n
n

(
f ( x1 , x2 ,..., x2 n ) ≥ f 0, n x1n + x2n , x3 ,..., x2 n ) (5.2)

⇔ (2n − 1)( x12 n + x22 n + ... + x22nn ) + 2nx1 x2 ...x2 n − (2n − 1)(( x1n + x2n )2 + ... + x22nn ) ≥ 0
 2n − 1 n−1 n−1 
⇔ x1 x2  x3 ...x2 n − .x1 x2  ≥ 0
 n 
2n − 1 n−1 n−1
⇔ x3 ...x2 n ≥ .x1 x2
n
2n − 1 n−1 n−2 2n − 1 n−1 2n − 1 n−1 n−1
Vì .(t + t s + ... + s n−1 ) ≥ .s = x1 x2 nên theo B 1 thì
n n n
có ít nh t m t trong hai b t ng th c (5.1), (5.2) úng.
y
  x n + x n x n + x n  
f ( x1 , x2 ,..., x2 n ) ≥ min  f  n 1
  2
2 n 1
,
2
2


(
, x3 ,..., x2 n  , f 0, n x1n + x2n , x3 ,..., x2 n ) 

Theo B t ng th c Bunhiacopxki thì
(2n − 1)( x22 n + x32 n + ... + x22nn ) ≥ ( x2n + x3n + ... + x2nn ) 2
nên f (0, x2 , x3 ,..., x2 n ) ≥ 0 .

61
, t ,..., t ) = 0 nên theo H qu c a
t khác f (t123 nh lý U.M.V, ta có u ph i ch ng
2 n soá t

minh.
ng th c x y ra khi và ch khi x1 = x2 = ... = x2 n ho c x1 = 0, x2 = x3 = ... = x2 n và
các hoán v .
Ví d 5 là bài toán t ng quát c a B t ng th c Turkervici (n = 4) . Trên th c t v i
tr ng h p riêng này, bài toán ã r t khó và v i tr ng h p t ng quát nó ã th hi n
c g n nh toàn b v p c a ph ng pháp này... B n th y không? Nó c ng “d
th ng” y ch ?

62
Bài t p ng d ng

Bài 1. ( inh Ng c An)


Cho n là s nguyên d ng và x1 , x2 ,..., xn là các s th c thu c [1, 2] . Tìm giá tr l n
nh t c a bi u th c
2
1 1 1
f ( x1 , x2 ,..., xn ) = ( x1 + x2 + ... + xn )  + + ... + 
 x1 x2 xn 

Bài 2. ( inh Ng c An)


Cho a, b, c là các s th c không âm có t ng b ng 3, k , m là các s th c th a mãn
k ≥ 1, m ≥ 0 . Tìm giá tr l n nh t c a bi u th c

f ( a, b, c) = a 2 k + b2 k + c 2 k + m[(ab) k + (bc) k + (ca ) k ]


Bài 3. ( inh Ng c An)
Cho p ≤ n là các s nguyên d ng và x1 , x2 ,..., xn là các s th c không âm có t ng
ng n . Tìm giá tr l n nh t c a bi u th c
f ( x1 , x2 ,..., xn ) = ∑ ( xi1 xi2 ...xi p ) k
1≤i1 ≤i2 ≤...≤i p ≤ n

Trong ó k là s th c b t kì.
Bài 5. (Ph m Kim Hùng)
Cho a, b, c, d là các s th c không âm th a mãn a + b + c + d = 4 . Tìm giá tr l n
nh t c a bi u th c
f ( a, b, c, d ) = (2 + a 2 )(2 + b 2 )(2 + c 2 )(2 + d 2 )
Bài 6. ( inh Ng c An)
Cho n là s nguyên d ng và x1 , x2 ,..., xn là các s th c không âm có t ng b ng n .
Tìm s th c m t t nh t sao cho b t ng th c sau úng v i m i b ( x1 , x2 ,..., xn )
th a mãn bài
x1m + x2m + ... + xnm + x1 x2 ...xn ≥ n + 1

63
Bài 7. (IMO Shortlist 1993)
Cho a, b, c, d ≥ 0 th a mãn a + b + c + d = 1 . Ch ng minh r ng
1 176
abc + abd + acd + bcd ≤ + .abcd
27 27
Bài 8. (Crux mathematicorum)
Cho a, b, c ≥ 0 . Ch ng minh r ng

48a 48b 48c


1+ + 1+ + 1+ ≥ 15
b+c c+a a+b
Bài 9. ( inh Ng c An)
Tìm th c k t t nh t b t ng th c sau úng v i m i a, b, c ≥ 0
k ( ab + bc + ca )
2(a 3 + b3 + c 3 ) + + 1 ≥ 3( a 2 + b2 + c 2 )
a+b+c
Bài 10. (Ph m Kim Hùng)
Cho n là s nguyên d ng và x1 , x2 ,..., xn là các s th c không âm có t ng b ng n .
Tìm giá tr nh nh t c a bi u th c
1 1 1 
f ( x1 , x2 ,..., xn ) = x12 + x22 + ... + xn2 + x1 x2 ...xn  + + ... + 
 x1 x2 xn 

Bài 11. (V ình Quý)


Cho n là s nguyên d ng và x1 , x2 ,..., xn là các s th c không âm có t ng b ng n .
Tìm giá tr t t nh t c a s th c k sao cho b t ng th c sau luôn úng
1 1 1
+ + ... + + kx1 x2 ...xn ≤ 1 + k
n − 1 + x1 n − 1 + x2 n − 1 + xn

64
PHÖÔNG PHAÙP THAM SOÁ HOÙA

1. tv n .
i v i ph n l n các b t ng th c i s không i x ng v i các bi n thì d u b ng
trong các b t ng th c này x y ra khi các giá tr các bi n không b ng nhau. Trong
ch g trình ph thông thì các b t ng th c c n nh Cauchy, Bunhiacopski l i
c phát bi u d i d ng i x ng, d u ng th c x y ra khi các bi n b ng nhau
ho c t l . Vi c áp d ng các b t ng th c c n trên gi i các bài toán c c tr
không i x ng c n c quan tâm m t cách thích áng. Qua bài vi t này, tôi mu n
nêu m t ph ng pháp gi i bài toán c c tr không i x ng b ng cách s d ng các
t ng th c c n thông d ng g i là ph ng pháp tham s hóa.
i dung ch y u c a ph ng pháp này nh sau: t vi c phân tích tính không i
ng c a các bi n có trong bài toán c c tr , th ng c cho d i các d ng:
ng 1. H s các bi n trong bi u th c c n tìm c c tr là không b ng nhau.
ng 2. Các bi n thu c các mi n khác nhau c a t p s th c.
ng 3. u ki n ràng bu c c a các bi n trong gi thi t bài toán là không
i x ng v i các bi n.
Ta a thêm bào các tham s ph c n thi t th ng là các h s ho c l y th a c a
các bi n có trong các ánh giá trung gian, sau ó ch n các tham s ph t t c các
u ng th c x y ra, t ó nh n c 1 h ph ng trình mà n là các bi n và các
tham s ph , tham s ph c ch n h p lí ch khi h ph ng trình t ng ng có
nghi m. Trong bài vi t này tôi nêu m t l p bài toán c c tr không i x ng th ng
p, tác gi ngh r ng nh ng mô hình c th này th t có ý ngh a vì v i k t qu c a
các bài toán này s cho ta m t l p bài toán c c tr không i x ng c th mi n là
xây d ng c b bi n th a mãn u ki n ràng bu c t ng ng.
2. M t s bài toán n hình.
Bài toán 1.
Cho x, y , z là các s th c d ng thay i th a mãn u ki n xy + yz + zx = 1 và
cho a là s th c d ng không i. Tìm giá tr nh nh t c a bi u th c
65
P = a( x2 + y 2 ) + z 2 .
i gi i.
Phân tích. u ki n ràng bu c i x ng v i x, y , z .
Bi u th c P i x ng v i x, y , vai trò c a z trong bi u th c P là không i x ng
i x, y .

z2
Do v y, ta có th ngh r ng m c c tr s t c khi x = y , và = α x2 = α y 2 .
2
phân tích trên, ta có th trình bày l i gi i c a bài toán nh sau
i α > 0 (ch n sau), áp d ng b t ng th c AM-GM cho 2 s d ng, ta có

z2 α
αx + ≥ 2
2
xz
2 2
z2 α
α y2 + ≥2 yz
2 2
α 2 α
2
(
x + y2 ≥ 2
2
)xy

ng v các b t ng th c trên ta nh n c
 α 2 α α
α +  (x + y ) + z ≥ 2 ( xy + yz + zx ) = 2
2 2

 2 2 2

α
Ch n α sao cho α + = a.
2
hay
α −1 + 1 + 8a
=
2 4
 1
 z2  x = y =
 = α x2 = α y2 
4
1 + 8a
u ng th c x y ra khi và ch khi  2 hay 
 xy + yz + zx = 1  z = −1 + 1 + 8a
  2 4 1 + 8a
t lu n
−1 + 1 + 8a
minP = .
2
66
Bài toán 2.
Cho a, b, c là các s th c d ng th a mãn u ki n ab + bc + ca = 1 và u , v là các
d ng c nh. Tìm giá tr nh nh t c a bi u th c
P = ua 2 + vb 2 + c 2 .

i gi i.
t cách t nhiên t l i gi i c a Bài toán 1, ta phân tích
u = x + y, v = z + t ,1 = m + n
trong ó x, y , z , t , m, n là các s d ng s ch n sau.
Áp d ng b t ng AM-GM cho 2 s d ng, ta có

xa 2 + tb 2 ≥ 2 xtab,
ya 2 + nc 2 ≥ 2 ynca,
zb 2 + mc 2 ≥ 2 zmbc.
ng v các b t ng th c trên, ta nh n c
P ≥ 2 xtab + 2 ynca + 2 zmbc.

 xa 2 = tb 2

u ng th c x y ra khi và ch khi  ya 2 = nc 2
 2
 zb = mc
2

hay
 x b2
 = 2
t a
 n a2
 = 2 ⇒ xzn = ytm. (1)
y c
 z c2
 = 2
m b

Ch n x, y , z , t , m, n sao cho xt = yn = zm = k 2 th a mãn (1).


Ta có
(1) ⇔ ( x + y )( z + t )( m + n) = uv

67
⇔ ( xz + xt + yz + yt )( m + n) = uv
⇔ xzm + xtm + yzm + ytm + xzn + xtn + yzn + ytn = uv
⇔ ( x + y + m + n + z + t )k 2 + 2 xzn = uv
⇔ ( u + v + 1) k 2 + 2 xzn = uv

Mà ( xzn)(utm) = k 6 nên xzn = k 3 .


Do ó
2k 3 + (u + v + 1)k 2 − uv = 0 (2)
Rõ ràng (2) có nghi m d ng duy nh t k0 .
y minP = 2k0 v i k0 là nghi m d ng duy nh t c a ph ng trình (2).
Nh n xét.
Bài toán 1 và Bài toán 2 th c s có ý ngh a khi ta ch n x, y , z ho c a, b, c là các
bi n c bi t, mi n là u ki n ràng bu c c a các bi n c th a mãn. Ch ng h n,
khi ta ch n mô hình là tam giác ABC .
A B C
u t x = tg , y = tg , z = tg , ta s có xy + yz + zx = 1 , áp d ng vào mô hình
2 2 2
Bài toán 1 ho c Bài toán 2 ta s thu c m t l p các bài toán c c tr d ng không
i x ng trong tam giác.
Ho c là x = cotgA, y = cotgB, z = cotgC , ta c ng s có ràng bu c xy + yz + zx = 1 ,
ng t ta c ng s có m t l p các bài toán c c tr không i x ng khác i v i tam
giác.
Nói chung, t t ng chính c a Bài toán 1 và Bài toán 2 là mu n xây d ng m t l p
các bài toán m i ta ch c n xây d ng m t l p các bi n i s , ho c l ng giác th a
mãn u ki n ràng bu c t ng ng. Thi t ngh r ng t t t ng này có th xây
ng c r t nhi u l p bài toán nh th .
Bài toán 3.
Cho x1 , x2 ,..., xn là các s th c th a mãn u ki n x1 + x2 + ... + xn = 0 và

x1 + x2 + ... + xn = 1 . Tìm giá tr l n nh t c a bi u th c

P= ∏ xi − x j .
1≤ i < j ≤ n

68
i gi i.
+ Tr ng h p 1. n = 2 là tr ng h p t m th ng vì lúc này P = 1 không i,
+ Tr ng h p 2. n = 3 , không m t tính t ng quát ta gi s x1 ≤ x2 ≤ x3 .
Áp d ng b t ngth c AM-GM cho 3 s không âm, ta có
P x −x 
= ( x2 − x1 )  3 1  ( x3 − x2 )
2  2 
3
  x3 − x1  
 ( x2 − x1 ) +  2  + ( x3 − x2 ) 
≤   
 3 
 
 
3
x −x 
= 3 1
 2 
1

8
1
Do ó P ≤
4
  1
 x1 + x2 + x3 = 0  x1 = − 2
 
ng th c x y ra khi và ch khi  x3 − x1 = x1 + x2 + x3 = 1 ⇔  x2 = 0 .
 
 x2 − x1 = x3 − x1 = x3 − x2 ≥ 0
1
 x3 =
 2  2
y
1
maxP = .
4
+ Tr ng h p 3. n = 4 , m t cách t nhiên ta d oán r ng max P t c khi
 x1 = − x4

 x2 = − x3
i gi thi t x1 ≤ x2 ≤ x3 ≤ x4 .
Nh v y thì x2 − x1 = x4 − x3 .
u xem hi u x2 − x1 = x4 − x3 là n v và t x3 − x2 = a , thì ta s có b bi n mà
bi u th c P t max c n th a mãn u ki n

69
x3 − x1 x3 − x2 x4 − x2 x4 − x1
x2 − x1 = x4 − x3 = = = =
a +1 a a +1 a+2
cách phân tích trên, l i gi i c a bài toán trong tr ng h p n = 4 s nh sau
i gi thi t x1 ≤ x2 ≤ x3 ≤ x4 , ta có
P = ( x2 − x1 )( x3 − x1 )( x4 − x1 )( x3 − x2 )( x4 − x2 )( x4 − x3 )
Do ó
P
=
a (a + 2)( a + 1) 2
(x − x ) (x − x ) (x − x ) (x − x )
= ( x2 − x1 ). 3 1 . 4 1 . 3 2 . 4 2 .( x4 − x3 )
a +1 a+2 a a +1
6
 ( x3 − x1 ) ( x4 − x1 ) ( x3 − x2 ) ( x4 − x2 ) 
 ( x − x ) + + + + + ( x4 − x3 ) 
a +1 a+2 a +1
2 1
≤ a

 6 
 
6
  1 1   1 1  
 ( x4 − x1 )  1 + a + 1 + a + 2  +  −1 + a + a + 1  ( x3 − x2 ) 
=     
 6 
 
 
Ta ch n a > 0 sao cho
1 1 1 1
1+ + = −1 + +
a +1 a + 2 a a +1
hay a = 2 − 1. Khi ó,
1 1 1 1 3 2
1+ + = −1 + + =
a +1 a + 2 a a +1 2
và ta thu c
6
3 2 
P  .( − x1 − x2 + x3 + x4 )  1
≤ 2  ≤ 9
( )( )( 2 )
2
2 −1 2 +1  6  2
 
 
1
hay P ≤
28
ng th c x y ra khi và ch khi
70

x + x + x + x = 0
 1 2 3 4
 − x1 − x2 + x3 + x4 = x1 + x2 + x3 + x4 = 1

 x2 − x1 = x4 − x3 = x3 − x2 = x3 − x1 = x4 − x2 = x4 − x1 ≥ 0
 2 −1 2 2 2 +1

 2− 2
 x4 = − x1 =
 4
Gi i h ph ng trình này ta nh n c
x = − x = 2
 3 2
4
t lu n
1
maxP = .
28
+ Tr ng h p 4. n = 5 .
Phân tích.
i gi thi t x1 ≤ x2 ≤ x3 ≤ x4 ≤ x5 , t l i gi i c a các tr ng h p 2 và 3, m t cách
nhiên, ta ngh ngay r ng b s P t max là x5 = − x1 , x4 = − x2 , x3 = 0 .
Do v y x5 − x4 = x2 − x1 , x3 − x2 = x4 − x3 , t ó ta có th oán nh n r ng n u xem
hi u x2 − x1 b ng n v và x3 − x2 b ng a thì b s P t max c n ph i th a
u ki n
x2 − x1 x4 − x3 x3 − x2 x3 − x1 x5 − x3
= = = = =
1 a a a +1 a +1
x −x x −x x −x x −x x −x
= 4 2 = 5 2 = 4 1= 5 1= 5 4
2a 2a + 1 2a + 1 2a + 2 1
cách phân tích trên, l i gi i c a bài toán trong tr ng h p n = 5 s nh sau
Không m t tính t ng quát, ta gi s x1 ≤ x2 ≤ x3 ≤ x4 ≤ x5 , t ó suy ra
P = ( x2 − x1 )( x3 − x1 )( x4 − x1 )( x5 − x1 )( x3 − x2 )x
x( x4 − x2 )( x5 − x2 )( x4 − x3 )( x5 − x3 )( x5 − x4 )

Xét bi u th c
P
Q=
4a 2 ( a + 1)3 (2a + 1) 2

71
Vi t Q d i d ng
( x2 − x1 ) ( x3 − x1 ) ( x4 − x1 ) ( x5 − x1 ) ( x3 − x2 )
Q= . . . . x
1 a +1 2a + 1 2 a + 2 a
(x − x ) (x − x ) (x − x ) (x − x ) (x − x )
x 4 2 . 5 2 . 4 3 . 5 3 . 5 4
2a 2a + 1 a a +1 1
Áp d ng b t ngth c AM−GM cho 10 s không âm, ta có
1  ( x2 − x1 ) ( x3 − x1 ) ( x4 − x1 ) ( x5 − x1 ) ( x3 − x2 )
Q≤  + + + + +
1010  1 a +1 2a + 1 2a + 2 a
( x − x ) (x − x ) (x − x ) ( x − x ) (x − x ) 
10
+ 4 2 + 5 2 + 4 3 + 5 3 + 5 4 
2a 2a + 1 a a +1 1 
10
1   1 3   1 3 
= 10  ( x5 − x1 ) 1 + +  + ( x4 − x2 )  −1 + + 
10   2a + 1 2(a + 1)   2a + 1 2a  
Ch n a > 0 sao cho
1 3 1 3
1+ + = −1 + +
2a + 1 2(a + 1) 2a + 1 2a
1
hay a = . Khi ó,
2
 1 3 1 3 5
1 + 2a + 1 + 2(a + 1) = −1 + 2a + 1 + 2a = 2

Q = 4 P
 27
ây, ta thu c
10
1 5  1
Q ≤ 10 . .( − x1 − x2 + x4 + x5 )  ≤ 20
10  2  2
27
Do ó P ≤
222
ng th c x y ra khi và ch khi

72
 x1 + x2 + x3 + x4 + x5 = 0

 − x1 − x2 + x4 + x5 = x1 + x2 + x3 + x4 + x5 = 1

 x2 − x1 = 2( x3 − x1 ) = x4 − x1 = x5 − x1 = 2( x3 − x2 ) =
 3 2 3
 x −x 2( x5 − x3 )
= x4 − x2 = 5 2 = 2( x4 − x3 ) = = x5 − x4 ≥ 0
 2 3
 3
 x1 = − x5 = −
8

 1
Gi i h ph ng trình này ta nh n c  x2 = − x4 = − .
 8
 x3 = 0


t lu n
27
maxP = .
222
Nh n xét.
ng ph ng pháp t ng t s tìm c l i gi i c a bài toán v i n 6.
Bài toán 4. (Võ Qu c Bá C n)
Cho m, n, p là dài ba c nh c a m t tam giác cho tr c và tam giác ABC nh n.
Tìm giá tr nh nh t c a bi u th c
P = tg m A.tg n B.tg pC .
i gi i.
Xét bi u th c
1
Q= = cotg m A.cotg n B.cotg p C .
P
Bài toán ã cho t ng ng v i tìm max c a Q .
Khi nhìn th y bi u th c Q , ít nhi u ta c ng ngh n ng th c quen thu c
cotgA.cotgB + cotgB.cotgC + cotgC.cotgA = 1
Và t ây, ta ngh ngay r ng bài này có th dùng b t ng AM-GM suy r ng, do ó
ta a vào các tham s d ng x, y , z (ch n sau) sao cho

73
Q = (cotgA.cotgB ) x .(cotgB.cotgC ) y .(cotgC.cotgA) z
= (cotgA) x+ z .(cotgB ) x + y .(cotgC ) y + z .
Ta ph i ch n x, y , z sao cho

 1
 x = 2 .(m + n − p )
x + z = m 
  1
 x + y = n ⇔  y = .(−m + n + p ) .
y + z = p  2
  1
 z = 2 .(m − n + p)

ây, ta có
y
 cotgB.cotgC 
x z
Q  cotgA.cotgB   cotgC.cotgA 
=  .  . 
   
x y z
x y z x  y  z
Áp d ng b t ng th c Cauchy suy r ng, ta có
Q

xx y y z z
x+ y+ z
1   cotgA.cotgB   cotgB.cotgC   cotgC.cotgA  
≤ .  x   + y   + z 
( x + y + z ) x+ y+ z   x   y   z 
1
=
( x + y + z ) x+ y+ z
xx y y zz
Do ó Q ≤
( x + y + z ) x+ y + z
Suy ra

( x + y + z ) x+ y + z ( m + n + p ) m+n + p
P≥ =
xx y y z z (− m + n + p ) − m + n+ p (m − n + p ) m −n+ p ( m + n − p ) m + n− p
ng th c x y ra khi và ch khi
cotgA.cotgB cotgB.cotgC cotgC.cotgA
= = .
x y z
Hay

74
 xz (m − n + p )(m + n − p )
cotgA = =
 y( x + y + z) ( −m + n + p )( m + n + p)
 xy (− m + n + p )(m + n − p )
cotgB = =
 z( x + y + z) (m − n + p )( m + n + p)
 yz (m − n + p )( −m + n + p )
cotgC = =
 x( x + y + z ) ( m + n − p)( m + n + p )

t lu n

( m + n + p) m + n+ p
minP = .
(− m + n + p) − m + n + p ( m − n + p) m − n + p ( m + n − p) m + n− p

Bài toán 5. (Vietnam TST 2001)


Cho a, b, c > 0 và 21ab + 2bc + 8ca ≤ 12 . Tìm giá tr nh nh t c a bi u th c
1 2 3
P= + + .
a b c
i gi i.
1 2 3
Phân tích. n gi n, ta s t x= , y = , z = thì ta nh n c m t bài toán
a b c
ng ng nh sau
“ x, y , z > 0 và 6 x + 12 y + 21z ≤ 6 xyz . Tìm giá tr nh nh t c a bi u th c
P = x + y + z. ”
Nh n th y t gi thi t 6 x + 12 y + 21z ≤ 6 xyz , ta có th suy ra c

x m y n z p ≥ k (m, n, p > 0)
Do ó ta ngh ngay r ng bài này có th s d ng b t ng AM-GM suy r ng c.
Th t v y
x y z
P = m. + n + p.
m n p
1
 x  m
 y
n
z
p
 n+ p
m +
≥ (m + n + p)    .  .  
 m  p  
  n

75
1
 k  m+n + p
≥ (m + n + p)  m n p 
m n p 
Nh v y, nhi m v c a ta bây gi ch là ph i tìm m, n, p n a thôi.
Rõ ràng, ta ch c n xét m + n + p = 1 là . Khi ó, ta có
6 xyz ≥ 6 x + 12 y + 21z
x y z
= 6m. + 12n. + 21 p.
m n p
1
  x 6 m  y 12 n  z  21 p  6 m+12 n + 21 p
≥ (6m + 12n + 21 p )    .  .  
 m   n   p  
 
 6m
1 − 6m + 12n + 21 p = km

 12n
1 − = kn
tìm m, n, p ta c n ph i gi i h sau  6m + 12n + 21 p
 21 p
1 − = kp
 6m + 12n + 21 p
m + n + p = 1

Hay
 6m
1 − 6m + 12n + 21 p = 2 m

 12n m = 1 − n − p
1 − = 2 n 
 6m + 12n + 21 p ⇔  4n 2 + 10np + 6n − 5 p = 2
  2
1 −
21 p
= 2p 5 p + 2np − n + 3 p = 1
 6m + 12n + 21 p
m + n + p = 1

4n 2 + 10np + 6n − 5 p = 2
Xét h (*)  2
5 p + 2np − n + 3 p = 1

(2t + 5) p 2 + (3 − t ) p = 1 (1)
t n = tp (t > 0) , h (*) tr thành  2
(4t + 10t ) p + (6t − 5) p = 2
2
(2)

y (2) − 2x (1) , ta c

76
p ((4t 2 + 6t − 10) p + 8t − 11) = 0
u t = 1 thì h (*) vô nghi m, do ó t ≠ 1 .
11 − 8t
⇒ p= (3)
4t 2 + 6t − 10
11
Do p > 0, t > 0 nên 1 < t < . Thay (3) vào (1) và thu g n, ta c
8
16t 4 − 12t 3 − 146t 2 + 30t + 175 = 0
⇔ (4t − 5)(2t + 5)(2t 2 − 4t − 7) = 0
5 11
⇔t= (do 1 < t < )
4 8
 2
 m =
5

 1
ó, ta có n = . Th l i, ta th y th a.
 3
 4
 p = 15

ng th c trên x y ra khi và ch khi
x y z
 = =
m n p
6 x + 12 y + 21z = 5 xyz

 5x 15 z
 = 3y =
⇔ 2 4
6 x + 12 y + 21z = 5 xyz
x = 3

 5
⇔ y =
 2
 z = 2
 1
a = 3

 4
⇒ b =
 5
 3
c = 2

77
các phân tích và ch n tham s trên, ta i n m t l i gi i c c k n gi n nh
sau
1 4 3
t a= ,b = ,c = , bài toán chuy n v
3x 5y 2z
“ x, y , z > 0 và 3 x + 5 y + 7 z ≤ 15 xyz . Tìm giá tr nh nh t c a bi u th c
1
P = .(6 x + 5 y + 4 z ). ”
2
Áp d ng b t ng AM-GM cho 15 s d ng, ta có

15 xyz ≥ 3 x + 5 y + 7 z ≥ 1515 x 3 y 5 z 7
⇒ 15 x12 y10 z 8 ≥ 1
⇔ 15 x 6 y 5 z 4 ≥ 1
i áp d ng b t ng th c AM-GM cho 15 s d ng, ta có
1 15 15
P = .(6 x + 5 y + 4 z ) ≥ .15 x 6 y 5 z 4 ≥
2 2 2
 1
a = 3

x = y = z  4
ng th c x y ra khi và ch khi  ⇔ x = y = z = 1 ⇔ b = .
15 xyz = 3 x + 5 y + 7 z  5
 3
 c =
 2
t lu n
15
minP = .
2
Bài toán 6.
Cho x, y , z ≥ 0 và x + y + z = 3. Tìm giá tr nh nh t c a bi u th c

P = x 4 + 2 y 4 + 3z 4
i gi i.
im is d ng a, b, c , theo b t ng th c Holder, ta có

P (a 4 + 2b 4 + 3c 4 )3 ≥ (a 3 x + 2b3 y + 3c 3 z ) 4

78
Ch n a, b, c sao cho a 3 = 2b 3 = 3c3 = k 3 , khi ó, ta có

k 12 ( x + y + z ) 4 (3k 3 ) 4
P≥ =
( a 4 + 2b 4 + 3c 4 )3 (a 4 + 2b 4 + 3c 4 )3
ng th c x y ra thì ta ph i có
x y z x+ y+ z
= = = =1
a b c a+b+c
Do v y, ta có
a + b + c = 3
 3
a = 2b = 3c = k
3 3 3

a = k

b = 2 k
3

⇔ c = 3 3k

k = 3
 1+ 3 2 + 3 3
ây, ta d dàng suy ra k t qu c a bài toán.
Bài toán 7.
Ch ng minh r ng v i m i s d ng a1 , a2 ,..., an ta luôn có

1 2 n 1 1 1 
+ + ... + < 4  + + ... + 
a1 a1 + a2 a1 + a2 + ... + an  a1 a2 an 

Ch ng minh.
Áp d ng b t ng th c Bunhiacopxki, ta có
 x12 x22 xk2 
(a1 + a2 + ... + ak )  + + ... +  ≥ ( x1 + x2 + ... + xk )2
 a1 a2 ak 
k k  x12 x22 xk2 
⇒ ≤ .  + + ... + 
a1 + a2 + ... + ak ( x1 + x2 + ... + xk )2  a1 a2 ak 

nh các s x1 , x2 ,..., xn và cho k ch y t 1 n n , r i l y t ng, ta c

1 2 n c c c
+ + ... + ≤ 1 + 2 + ... + n
a1 a1 + a2 a1 + a2 + ... + an a1 a2 an
Trong ó
79
kxk2 (k + 1) xk2 nxk2
ck = + + ... +
( x1 + x2 + ... + xk ) 2 ( x1 + x2 + ... + xk +1 ) 2 ( x1 + x2 + ... + xn )2

Ta có th ch n xk = k ∀k = 1, n, khi ó

 k k +1 n 
ck = k 2  + + ... + 2 
 (1 + 2 + ... + k ) (1 + 2 + ... + ( k + 1)) (1 + 2 + ... + n) 
2 2

 k k +1 n 
= 4k 2  2 + + ... + 
 k (k + 1) ( k + 1) 2 (k + 2)2 n 2 ( n + 1) 2 
2

 1 1 1 
= 4k 2  + + ... + 2 
 k (k + 1) ( k + 1)( k + 2) n( n + 1) 
2 2

 1 1 1  1  1 1  1 1 1 
= 4k 2  . − + . −  + ... + . − 
 k +1  k k +1 k + 2  k +1 k + 2  n + 1  n n + 1  
1 1 1 1 1 
= 4k 2  − − − − ... − 
 k n + 1 ( k + 1) (k + 2) ( n + 1) 2 
2 2

1 1 1 1 1 
< 4k 2  − − − − ... − 
 k n + 1 ( k + 1)(k + 2) (k + 2)( k + 3) (n + 1)( n + 2) 
1 1 1 
= 4k 2  − − 
 k k +1 n +1 
1 1 
< 4k 2  − 
 k k +1
4k
<
k +1
<4
t ng th c c ch ng minh hoàn toàn.
Bài toán 8.
Ch ng minh b t ng th c sau v i m i s th c x1 , x2 ,..., xn
2 2
x +x   x + x + ... + xn 
x12 +  1 2  + ... +  1 2  ≤ 4( x1 + x2 + ... + xn )
2 2 2

 2   n 
Ch ng minh.
im is d ng c1 , c2 ,..., cn tùy ý, ta có

80
 x12 x22 xk2 
 + + ... +  (c1 + c2 + ... + ck ) ≥ ( x1 + x2 + ... + xk )
2

 c1 c2 ck 

Do ó
2
 x1 + x2 + ... + xk  c1 + c2 + ... + ck 2 c1 + c2 + ... + ck 2
  ≤ .x1 + .x2 +
 k  k 2c1 k 2ck
c + c + ... + ck 2
+ ... + 1 2 2 .xk
k ck
Cho k ch y t 1 n n , r i l y t ng, ta c
2 2
x +x   x + x + ... + xn 
x12 +  1 2  + ... +  1 2  ≤ α1 x1 + α 2 x2 + ... + α n xn
2 2 2

 2   n 
Trong ó
c1 + c2 + ... + ck c1 + c2 + ... + ck +1 c1 + c2 + ... + cn
αk = + + ... + ∀k = 1, n
k 2 ck ( k + 1) 2 ck n 2ck

Ta ch n ck = k − k − 1 ⇒ c1 + c2 + ... + ck = k

1  1 1 1 
⇒ αk = . 3 2 + + 32
ck  k (k + 1)32
n 

Chú ý r ng
1 1
− k− k+
1 1 2 2
− =
k−
1
k+
1  1  1
 k −  k + 
2 2  2  2
1
=
 1 1  1  1
 k + + k −   k −  k + 
 2 2  2  2
1
≥ 32
2k
1 1 1 1 1 1
⇒ ≥ − ≥ 32 + + ... + 3 2
1 1 1 2k 2(k + 1)32
2n
k− k− n+
2 2 2

81
⇒ αk ≤
2
=
2 ( k + k −1 )≤4
1 1
ck k − k−
2 2
t ng th c c ch ng minh hoàn toàn.
3. Bài t p ngh .
Bài 1. (Vietnam TST 1994)
1
Cho a, b, c, d là các s th c th a mãn u ki n ≤ a 2 + b 2 + c 2 + d 2 ≤ 1 . Tìm giá tr
2
nh nh t và giá tr l n nh t c a bi u th c
P = (a − 2b + c) 2 + (b − 2c + d ) 2 + (b − 2a )2 + (c − 2d )2

Bài 2.
Cho x, y > 0 và x + y ≥ 4 . Tìm giá tr nh nh t c a bi u th c
6 10
P = 2x + 3 y + +
x y
Bài 3.
Cho a, b, c > 0 và a + 2b + 3c ≥ 20 . Tìm giá tr nh nh t c a bi u th c
3 9 4
P = a+b+c+ + +
a 2b c
Bài 4.
Cho a, b, c > 0 và a + b + c = 3 . Tìm giá tr l n nh t c a bi u th c
P = 2ab + 4bc + 3ca
Bài 5. (Toán H c Tu i Tr 2005)
a) Cho tam giác ABC . Tìm giá tr l n nh t c a bi u th c
P = sin A.sin 2 B.sin 3 C
b) Cho tam giác ABC , m, n, p là các s th c d ng cho tr c. Tìm giá tr
n nh t c a bi u th c
P = sin m A.sin n B.sin p C

82
Bài 6. (VMEO 2004)
Cho tam giác nh n ABC. Tìm giá tr nh nh t c a bi u th c
P = tgA + 2tgB + 5tgC
Bài 7. (VMEO 2005)
Cho a, b, c là các s th c d ng cho tr c và x, y , z là các s th c d ng th a mãn
ax + by + cz = xyz . Tìm giá tr nh nh t c a bi u th c
P= x+y+z
Bài 8.
Cho a1 , a2 ,..., an là n s th c d ng cho tr c và x1 , x2 ,..., xn là n s th c d ng
n n
th a mãn ∑ ai xi = ∏ xi . Tìm giá tr nh nh t c a bi u th c
i =1 i =1

n
P = ∑ xi
i =1

Bài 9. ch n i tuy n HSP Hà N i 2005)


Cho x, y , z là các s th c d ng th a mãn xy + yz + zx = 7 xyz . Tìm giá tr nh nh t
a bi u th c
8 x 4 + 1 108 y 5 + 1 16 z 6 + 1
P= + +
x2 y2 z2
Bài 10. (Toán H c Tu i Tr 2005)
Cho x, y , z ∈ [0,1] . Tìm giá tr l n nh t c a bi u th c
P = ( x − y )( y − z )( z − x)( x + y + z )
Bài 11.
Ch ng minh r ng v i m i dãy s d ng a1 , a2 ,..., an ta có

1 1 1 1 1 1 
+ + ... + < 2  + + ... + 
a1 a1 + a2 a1 + a2 + ... + an  a1 a2 an 

83
PHÖÔNG PHAÙP HEÄ SOÁ BAÁT ÑÒNH

Trong thôøi caáp 2, khi ñoïc lôøi giaûi cuûa khaù nhieàu baøi toaùn ñaëc bieät laø baát ñaúng
thöùc, toâi khoâng theå hieåu noåi taïi sao ngöôøi ta laïi nghó ra ñöôïc lôøi giaûi ñoù vaø toâi
caûm thaáy ñoù laø moät lôøi giaûi thieáu töï nhieân nhöng toâi cuõng caûm thaáy voâ cuøng thaùn
phuïc ngöôøi ñaõ nghó ra lôøi giaûi ñoù. Nhöng baây giôø khi ñaõ ñöôïc laøm quen vôùi taát caû
caùc kieán thöùc toaùn sô caáp, toâi môùi hieåu ñöôïc ñaáy khoâng phaûi laø moät caùi gì môùi laï
caû maø noù ñaõ coù moät phöông phaùp haún hoi. Trong baøi naøy, toâi xin giôùi thieäu vôùi
caùc baïn moät trong nhöõng phöông phaùp ñoù: “Phöông phaùp heä soá baát ñònh”. Phöông
phaùp naøy tuy coù moät soá haïn cheá nhöng noù vaãn laø moät phöông phaùp hay vaø khaù
maïnh. Caùc baïn neân chuù yù ñeán noù vì ngoaøi vieäc giuùp ta chöùng minh moät baát ñaúng
thöùc khoù thì noù coøn laø 1 “lieàu thuoác boå “ cho moät phöông phaùp chöùng minh baát
ñaúng thöùc cöïc maïnh: “Phöông phaùp phaân tích bình phöông S.O.S” vì noù giuùp ta
ñöa moät baát ñaúng thöùc veà daïng S.O.S nhanh choùng hôn caùc kieåu bieán ñoåi thoâng
thöôøng.
Sau ñaây laø moät soá ví duï

Ví duï 1. (USAMO 2003)


Cho a, b, c > 0. Chöùng minh raèng

(2a + b + c) 2 (2b + c + a ) 2 (2c + a + b) 2


+ + ≤8
2a 2 + (b + c ) 2 2b 2 + (c + a ) 2 2c 2 + (a + b) 2

Nhaùp.
Nhaän xeùt raèng daáu baèng xaûy ra khi vaø chæ khi a = b = c.
Do caû hai veá cuûa baát ñaúng thöùc ñaõ cho ñoàng baäc neân ta coù theå chuaån hoùa cho
a + b + c = 3 . Khi ñoù, baát ñaúng thöùc caàn chöùng minh trôû thaønh

84
( a + 3) 2 (b + 3) 2 (c + 3) 2
+ + ≤8
2a 2 + (3 − a) 2 2b 2 + (3 − b) 2 2c 2 + (3 − c )2

Ta seõ tìm soá thöïc α sao cho baát ñaúng thöùc cho moïi a ∈ (0,3)

( a + 3) 2 8
≤ α ( a − 1) +
2a + (3 − a )
2 2
3
⇔ f ( a ) = 3α a 3 + (7 − 9α )a 2 + (15α − 22) a + 15 − 9α ≥ 0

Ta caàn tìm α sao cho f (a ) ≥ 0 ∀a ∈ (0,3) vaø f ( a) = 0 ⇔ a = 1. Ñeå coù ñöôïc ñieàu

naøy, ta caàn coù


4
f / (1) = 0 ⇔ 9α + 2(7 − 9α ) + 15α − 22 = 0 ⇔ α =
3
Vaäy nhieäm vuï cuûa ta baây giôø laø xeùt xem baát ñaúng thöùc sau coù ñuùng hay khoâng

( a + 3) 2 4 4
≤ .a +
2a + (3 − a)
2 2
3 3

Vôùi nhöõng laäp luaän nhö treân, ta ñi ñeán moät lôøi giaûi khoâng maáy töï nhieân nhö sau
Lôøi giaûi.
Khoâng maát tính toång quaùt, coù theå giaû söû a + b + c = 3 . Khi ñoù, baát ñaúng thöùc caàn
chöùng minh trôû thaønh

( a + 3) 2 (b + 3) 2 (c + 3) 2
+ + ≤8
2a 2 + (3 − a) 2 2b 2 + (3 − b) 2 2c 2 + (3 − c )2

Ta seõ chöùng minh

( a + 3) 2 4 4
≤ .a + (*)
2a + (3 − a)
2 2
3 3

Thaät vaäy
(*) ⇔ ( a − 1) 2 (4a + 3) ≥ 0 (ñuùng)

Vaäy (*) ñuùng.


Töông töï, ta coù

85
(b + 3) 2 4 4
≤ .b +
2b 2 + (3 − b)2 3 3
(c + 3) 2 4 4
≤ .c +
2c + (3 − c)
2 2
3 3

Do ñoù
( a + 3) 2 (b + 3) 2 (c + 3) 2 4
+ + ≤ .(a + b + c) + 4 = 8
2a + (3 − a)
2 2
2b + (3 − b)
2 2
2c + (3 − c )
2 2
3
⇒ ñpcm.

Rieâng ñoái vôùi baøi toaùn treân coøn coù moät caùch tìm α cöïc nhanh laø

( a + 3) 2 1 8a + 6 1 8a + 6 4 4
= + ≤ + = .a +
2a + (3 − a)
2 2
3 3(a − 1) + 6 3
2
6 3 3

Nhöng vôùi ñöôøng loái naøy thì ta khoù maø laøm maïnh baát ñaúng thöùc hôn ñöôïc. Thaät
vaäy, toâi ñaõ coá gaéng raát nhieàu ñeå duøng ñöôøng loái naøy ñeå chöùng minh baát ñaúng thöùc
sau nhöng vaãn baát löïc

( a + 3) 2 (b + 3) 2 (c + 3) 2
+ + ≤6
4a 2 + (3 − a) 2 4b 2 + (3 − b) 2 4c 2 + (3 − c) 2

Vôùi a, b, c > 0 thoûa a + b + c = 3.

Coù theå thaáy caùch tìm α ban ñaàu laø caùch tìm hay nhaát, nhöng noù ñoøi hoûi khaù nhieàu
tính toaùn raát baát lôïi cho nhöõng baïn tính toaùn khoâng ñöôïc toát cho laém, vaø ñoâi khi
bieåu thöùc ñeà baøi cho quaù phöùc taïp (chaúng haïn nhö quaù nhieàu caên thöùc). Vì nhöõng
lí do ñoù, toâi xin ñöôïc giôùi thieäu vôùi caùc baïn moät caùch tìm α khaù hieäu quaû döïa
treân baát ñaúng thöùc AM-GM, cuï theå laø ñoái vôùi baøi toaùn treân
Ta seõ tìm α , β sao cho baát ñaúng thöùc sau ñuùng cho moïi soá döông a, b, c

(2a + b + c) 2 α a + β b + β c

2a 2 + (b + c ) 2 a+b+c

AÙp duïng baát ñaúng thöùc AM-GM (xin ñöôïc löu yù vôùi caùc baïn laø trong caùch tìm
naøy, ta khoâng caàn ñeå yù ñeán chieàu baát ñaúng thöùc, toâi xin ñöôïc kyù hieäu → ñeå thay

86
cho daáu baát ñaúng thöùc vaø ta cuõng khoâng caàn ñeå yù ñeán α , β aâm hay döông vì ñaây

chæ laø nhaùp thoâi), ta coù

(2a + b + c) 2
1 1
8 3 −
→ .a .(bc ) 6
2a 2 + (b + c ) 2 3
α 1 β 1
αa + βb + βc α + 2β α + 2 β − 3 −
→ .a .(bc ) α + 2 β 3
a+b+c 3

α + 2 β = 8
  16
 α =
 α 1 1 3
Ta choïn α , β sao cho  − = . Giaûi heä naøy, ta ñöôïc  .
α + 2 β 3 3 β = 4
 b 1 1  3
 − =−
α + 2 β 3 6

Vaäy nhieäm vuï cuûa chuùng ta baây giôø laø xeùt tính ñuùng ñaén cuûa baát ñaúng thöùc

(2a + b + c) 2 16a + 4b + 4c

2a 2 + (b + c ) 2 3( a + b + c)

Ta coù theå chuaån hoùa cho a + b + c = 3 roài chöùng minh töông töï nhö treân, hoaëc bieán
ñoåi töông ñöông.

Ví duï 2.
Cho a, b, c > 0. Chöùng minh raèng

a3 b3 c3
+ + ≥1
a 3 + (b + c)3 b3 + (c + a )3 c 3 + ( a + b)3

Nhaùp.
Ñaây laø moät baøi toaùn hay, töông ñoái khoù. Ta coù theå giaûi baèng caùch laøm töông töï
nhö treân, xin daønh cho caùc baïn. ÔÛ ñaây, toâi xin giôùi thieäu moät caùch giaûi khaùc nhö
sau
Nhaän xeùt raèng daáu baèng xaûy ra khi vaø chæ khi a = b = c .
Ta seõ tìm p sao cho baát ñaúng thöùc sau ñuùng

87
a3 ap

a 3 + (b + c)3 a p + b p + c p

Chuùng ta coù 2 caùch choïn p söû duïng ñaïo haøm hoaëc döïa vaøo baát ñaúng thöùc AM-

GM, veà phía toâi, toâi raát ngaïi tính toaùn neân chæ xin ñöôïc trình baøy caùch döïa vaøo baát
ñaúng thöùc AM-GM, mong caùc baïn thoâng caûm.
AÙp duïng baát ñaúng thöùc AM-GM, ta coù
4 2
a3 1 −
→ .a 3 .(bc ) 3
a + (b + c)
3 3
3
2p p
ap 1 3 −
→ .a .(bc ) 3
ap + bp + cp 3
Töø ñaây, baèng caùch ñoàng nhaát heä soá, ta suy ra ñöôïc p = 2.

Vaäy nhieäm vuï cuûa chuùng ta baây giôø laø kieåm tra tính ñuùng ñaén cuûa baát ñaúng thöùc

a3 a2

a 3 + (b + c)3 a 2 + b2 + c 2

Vôùi nhöõng laäp luaän nhö treân, ta ñi ñeán lôøi giaûi nhö sau
Lôøi giaûi.
Ta seõ chöùng minh

a3 a2
≥ (*)
a 3 + (b + c)3 a 2 + b 2 + c 2

Thaät vaäy:

1 a
(*) ⇔ ≥
a 3 + (b + c )3 a + b2 + c 2
2

⇔ ( a 2 + b 2 + c 2 ) 2 ≥ a ( a 3 + (b + c ) 3 )
⇔ 2a 2 (b 2 + c 2 ) + (b 2 + c 2 ) 2 ≥ a (b + c)3
⇔ (b 2 + c 2 )((a 2 + b 2 ) + (a 2 + c 2 )) ≥ a (b + c)3
⇔ (b 2 + c 2 )((a − b) 2 + (a − c) 2 ) + 2(b 2 + c 2 ) a(b + c) ≥ a (b + c )3
⇔ (b 2 + c 2 )((a − b) 2 + (a − c) 2 ) + a (b + c )(b − c ) 2 ≥ 0 (ñuùng)

88
Vaäy (*) ñuùng.
Töông töï, ta coù

b3 b2

b3 + (c + a )3 a 2 + b 2 + c 2
c3 c2

c3 + ( a + b)3 a 2 + b 2 + c 2

Do ñoù

a3 b3 c3 a2 + b2 + c2
+ + ≥ =1
a 3 + (b + c)3 b3 + (c + a)3 c3 + ( a + b)3 a 2 + b 2 + c 2
⇒ ñpcm.

Ñaúng thöùc xaûy ra khi vaø chæ khi a = b = c.

* Nhaän xeùt 1.
Caû hai ví duï treân ñeàu söû duïng ñaúng thöùc

ap + bp + cp 1 α (a + b + c ) + β ( a + b + c)
1= = .
a +b +c
p p p
α +β a +b +c

Moät caâu hoûi ñaët ra cho ta laø khi naøo thì ta phaûi tìm p vaø khi naøo thì ta phaûi tìm

α , β ? Coù leõ caùc baïn seõ hôi luùng tuùng ôû choã naøy nhöng thaät ra thì ta chæ caàn nhìn

bieåu thöùc ôû ñeà baøi laø bieát ngay thoâi, chaúng haïn nhö ôû ví duï 1, xeùt baát ñaúng thöùc

(2a + b + c) 2 8a p

2a 2 + (b + c ) 2 a p + b p + c p

Khi cho a → 0, b = c = 1 thì ta coù VT → 1,VP → 0 neân baát ñaúng thöùc naøy khoâng

theå ñuùng vôùi moïi soá döông a, b, c.

Ví duï 3.
Cho a, b, c > 0. Chöùng minh raèng

a3 b3 c3 a+b+c
+ + ≥
a 2 + b2 b2 + c 2 c 2 + a 2 2
Nhaùp.
89
Nhaän xeùt raèng daáu baèng xaûy ra khi vaø chæ khi a = b = c.
Ta seõ tìm α sao cho baát ñaúng thöùc sau ñuùng

2a 3
≥ α a + (1 − α )b
a 2 + b2
AÙp duïng baát ñaúng thöùc AM-GM, ta coù

2a 3
→ a 2b −1
a +b
2 2

α a + (1 − α )b → aα b1−α

Töø ñaây, baèng caùch ñoàng nhaát heä soá, ta coù α = 2 .


Vaäy nhieäm vuï cuûa ta baây giôø laø kieåm chöùng tính ñuùng ñaén cuûa baát ñaúng thöùc
2a 3
≥ 2a − b
a2 + b2
Ta ñi ñeán lôøi giaûi nhö sau
Lôøi giaûi.
Ta coù

2a 3
≥ 2a − b (*)
a 2 + b2
Thaät vaäy
(*) ⇔ b( a − b)2 ≥ 0 (ñuùng)

Vaäy (*) ñuùng.

2b3 2c 3
Töông töï, ta coù ≥ 2b − c, ≥ 2c − a
b2 + c2 c2 + a2
Do ñoù

2a 3 2b3 2c 3
+ + ≥ a + b + c (ñpcm)
a 2 + b2 b2 + c 2 c 2 + a 2
Ñaúng thöùc xaûy ra khi vaø chæ khi a = b = c.

* Nhaän xeùt 2.

90
Baèng kinh nghieäm baûn thaân, toâi cho raèng ñieàu kieän caàn ñeå söû duïng phöông phaùp
naøy vôùi caùc baát ñaúng thöùc thuaàn nhaát laø
1) Daáu ñaúng thöùc xaûy ra khi vaø chæ khi caùc bieán soá baèng caùc giaù trò trong moät
taäp höõu haïn naøo ñoù (thöôøng taäp naøy chæ goàm coù 1 giaù trò, toái ña laø 2 giaù
trò).
2) Baát ñaúng thöùc ñeà baøi cho laø toång cuûa moät daõy caùc bieåu thöùc ñoãi xöùng
nhau vaø toàn taïi moät caùch chuaån hoùa ñeå moãi bieåu thöùc chæ coøn phuï thuoäc
vaøo moät bieán soá hoaëc caùc bieåu thöùc laø hoaùn vò lieân tieáp cuûa nhau.
Baây giôø ta seõ xeùt moät soá ví duï veà baát ñaúng thöùc coù ñieàu kieän

Ví duï 4.
Cho a, b, c > 0 thoûa a 2 + b 2 + c 2 = 3. Chöùng minh raèng

1 1 1 4
+ + + .(a + b + c) ≥ 7
a b c 3
Nhaùp.
Nhaän xeùt raèng ñaúng thöùc xaûy ra khi vaø chæ khi a = b = c = 1.

(
Ta coù a 2 + b 2 + c 2 = 3 ⇒ a, b, c ∈ 0, 3 . )
Ta seõ tìm α sao cho baát ñaúng thöùc sau ñuùng vôùi moïi a ∈ 0, 3 ( )
1 4 7
+ .a ≥ α (a 2 − 1) + (*)
a 3 3
Ta coù
(*) ⇔ f ( a ) = 3α a 3 − 4a 2 + (7 − 3α ) a − 3 ≤ 0

( )
Ta caàn tìm α sao cho f (a ) ≤ 0 ∀a ∈ 0, 3 vaø f ( a) = 0 ⇔ a = 1. Ñeå coù ñöôïc

ñieàu naøy ta caàn coù


1
f / (1) = 0 ⇔ 9α − 8 + 7 − 3α = 0 ⇔ α =
6
Baây giôø ta chæ coøn phaûi xeùt tính ñuùng ñaén cuûa baát ñaúng thöùc
91
1 4 1 7
+ .a ≥ .( a 2 − 1) +
a 3 6 3
Ta ñi ñeán lôøi giaûi nhö sau
Lôøi giaûi.

(
Ta coù a 2 + b 2 + c 2 = 3 ⇒ a, b, c ∈ 0, 3 . )
Ta seõ chöùng minh
1 4 1 7
+ .a ≥ .(a 2 − 1) + (**)
a 3 6 3
Thaät vaäy

(**) ⇔ ( a − 1) 2 (6 − a) ≥ 0 (ñuùng do 3 > a > 0)

Vaäy (**) ñuùng.


Töông töï, ta coù
1 4 1 7
+ .b ≥ .(b 2 − 1) +
b 3 6 3
1 4 1 7
+ .c ≥ .(c 2 − 1) +
c 3 6 3
Do ñoù
1 1 1 4 1
+ + + .(a + b + c) ≥ .(a 2 + b 2 + c 2 − 3)+ = 7
a b c 3 6
⇒ ñpcm.

Ñaúng thöùc xaûy ra khi vaø chæ khi a = b = c = 1.


Xin ñöôïc löu yù vôùi caùc baïn raèng khoâng phaûi luùc naøo ta cuõng löïa choïn haøm laø
nhöõng haøm tuyeán tính hoaëc haøm luõy thöøa khoâng thoâi, maø ñoâi luùc ta caàn phaûi löïa
choïn haøm phaân thöùc, haøm caên, ... Ví duï sau seõ cho chuùng ta thaáy roõ ñieàu ñoù
Ví duï 5.(APMO 2005)
Cho a, b, c > 0 thoûa abc = 8. Chöùng minh raèng

a2 b2 c2 4
+ + ≥
(1 + a )(1 + b )
3 3
(1 + b )(1 + c )
3 3
(1 + c )(1 + a )
3 3 3
92
Lôøi giaûi.
Ta coù
1 2
≥ ∀x > 0 (*)
1 + x3 2 + x2

Thaät vaäy, ta coù


(*) ⇔ (2 + x 2 ) 2 ≥ 4(1 + x 3 )
⇔ x 2 ( x − 2) 2 ≥ 0 (ñuùng)

Vaäy (*) ñuùng.


Do ñoù

a2 4a 2 2S (a, b, c) 2
∑ ≥∑ = =
(2 + a )(2 + b ) 36 + S (a, b, c) 1 +
2 2 36
cyc (1 + a 3 )(1 + b3 ) cyc
S (a, b, c)

trong ñoù S (a, b, c) = 2(a 2 + b 2 + c 2 ) + a 2b 2 + b 2c 2 + c 2 a 2

Theo baát ñaúng thöùc AM−GM, ta coù

a 2 + b 2 + c 2 ≥ 3 3 ( abc) 2 = 12
a 2b 2 + b 2c 2 + c 2 a 2 ≥ 3 3 (abc) 4 = 48
⇒ S ( a, b, c ) ≥ 72

Do ñoù

a2 2 2 4
∑ ≥
36
≥ = ⇒ ñpcm.
36 3
cyc (1 + a 3 )(1 + b3 ) 1+ 1+
S ( a , b, c ) 72

Ñaúng thöùc xaûy ra khi vaø chæ khi a = b = c = 2.

93
BAØI TAÄP.

Baøi 1. (IMO 2001)


Cho a, b, c > 0. Chöùng minh raèng

a2 b2 c2
+ + ≥1
a 2 + 8bc b2 + 8ca c 2 + 8ab

Baøi 2.
Cho a, b, c > 0. Chöùng minh raèng

a3 b3 c3 a+b+c
+ 2 + 2 ≥
a + ab + b b + bc + c
2 2 2
c + ca + a 2
3
Baøi 3.
Cho a, b, c, d > 0. Chöùng minh raèng

a4 b4 c4 d4 a+b+c+d
+ + + ≥
(a + b )(a + b) (b + c )(b + c) (c + d )(c + d ) ( d + a )(d + a )
2 2 2 2 2 2 2 2
4
Baøi 4.
Cho a, b, c, d > 0 thoûa a + b + c + d = 1. Chöùng minh raèng

1
6(a 3 + b 3 + c 3 + d 3 ) ≥ a 2 + b 2 + c 2 + d 2 +
8
Baøi 5. (Voõ Quoác Baù Caån)
Cho a, b, c > 0. Chöùng minh raèng

(b + c − a ) 2 (c + a − b) 2 (a + b − c) 2 3( a 2 + b2 + c 2 )
+ + ≥
2a 2 + (b + c) 2 2b 2 + (c + a ) 2 2c 2 + ( a + b)2 2(a + b + c) 2

Baøi 6.
Cho a, b, c > 0 thoûa a + b + c = 2. Tìm giaù trò lôùn nhaát cuûa bieåu thöùc

11b 3 − a 3 11c3 − b3 11a 3 − c 3


P= + +
ab + 4b 2 bc + 4c 2 ca + 4a 2

94
Baøi 7.
Cho a, b, c, d > 0. Chöùng minh raèng

a b c c
+ + + ≥1
3
a 3 + 63bcd 3
b 3 + 63cda 3
c3 + 63dab 3
c 3 + 63abc

Baøi 8.
Cho a, b, c > 0 thoûa a 3 + b 3 + c 3 = 3. Chöùng minh raèng

1 1 1 5 2 27
+ + + .(a + b 2 + c 2 ) ≥
a b c 4 4
Baøi 9.
Cho a, b, c > 0. Chöùng minh raèng

(a + b − 3c ) 2 (b + c − 3a ) 2 (c + a − 3b) 2 1
+ + ≥
(a + b) + 2c
2 2
(b + c ) + 2 a
2 2
(c + a) + 2b
2 2
2

Baøi 10.
Cho a, b, c > 0. Chöùng minh raèng

(3a + b + c )3 (3b + c + a )3 (3c + a + b)3 375


+ + ≤
(b + c )3 + 3a 3 (c + a )3 + 3b3 (a + b)3 + 3c 3 11

Baøi 11.
Cho a, b, c laø ñoä daøi ba caïnh cuûa moät tam giaùc. Chöùng minh raèng

1 1 1 9  1 1 1 
+ + + ≥ 4 + + 
a b c a+b+c  a+b b+c c+a
Baøi 12.
Cho x, y , z > 0 thoûa x + y + z = 1. Tìm giaù trò lôùn nhaát cuûa bieåu thöùc

x y z
P= + 2 + 2
x +1 y +1 z +1
2

95
Baøi 13.
Cho a, b, c > 0. Chöùng minh raèng
3 3 3
 a   b   c  3
  +  +  ≥
b+c c+a  a+b 8
Baøi 14. (Moldova 2005)
Cho a, b, c > 0 thoûa a 4 + b 4 + c 4 = 3. Chöùng minh raèng

1 1 1
+ + ≤1
4 − ab 4 − bc 4 − ca
Baøi 15.
Cho a, b, c > 0. Chöùng minh raèng

(a + b − c)2 (b + c − a )2 (c + a − b) 2 9(a 2 + b 2 + c 2 )
+ + ≥
(a + b) 2 + 3c 2 (b + c )2 + 3a 2 (c + a ) 2 + 3b 2 7(a + b + c) 2

Baøi 16.
Cho a, b, c > 0. Chöùng minh raèng

(a + b − c) 2 (b + c − a ) 2 (c + a − b )2 a 2 + b2 + c 2
+ + ≥
7( a + b) 2 + 17c 2 7(b + c) 2 + 17a 2 7(c + a ) 2 + 17b 2 5( a + b + c ) 2

Baøi 17.
Cho a, b, c > 0 thoûa abc = 1. Chöùng minh raèng

a2 + 1 b2 + 1 c2 + 1
+ + ≤ a+b+c
2 2 2
Baøi 18.
Cho a, b, c > 0 thoûa a + b + c = 3. Chöùng minh raèng

1 1 1 3
+ + ≥
3
a7 + 7 3
b7 + 7 3
c7 + 7 2

96
Baøi 19. (Vasile Cirtoaje)
Cho a, b, c, d ≥ 0 thoûa a + b + c + d = 4. Chöùng minh raèng

1 1 1 1
+ 2 + 2 + 2 ≥2
a +1 b +1 c +1 d +1
2

Baøi 20.
Cho a, b, c, d ≥ 0 thoûa a + b + c + d = 4. Chöùng minh raèng

a b c d 1
+ + + ≤
5 + 3a 2
5 + 3b 5 + 3c 5 + 3d
2 2 2
2
Baøi 21. (Olympic 30 - 4 - 2006)
Cho a, b, c > 0. Chöùng minh raèng

a4 b4 c4
+ + ≤1
a 4 + 3 ( a 6 + b6 )( a 3 + c3 ) 2 b 4 + 3 (b 6 + c 6 )(b3 + a 3 ) 2 c 4 + 3 (c 6 + a 6 )(c3 + b 3 ) 2
Baøi 22. (Japan 1997)
Cho a, b, c > 0. Chöùng minh raèng

a2 b2 c2 3
+ 2 + 2 ≥
a + (b + c ) b + (c + a ) c + (a + b)
2 2 2 2
5

Baøi 23. (Phaïm Vaên Thuaän)


Cho a, b, c > 0. Chöùng minh raèng

a3 b3 c3 1
+ 3 + 3 ≥
a + (b + c ) b + (c + a) c + ( a + b)
3 3 3 3
3

Baøi 22.
Cho a, b, c > 0. Chöùng minh raèng

a b c
+ + ≥1
a + 2(b + c)
2 2
b + 2(c + a)
2 2
c + 2( a + b) 2
2

Baøi 23. (Phaïm Kim Huøng, Voõ Quoác Baù Caån)


Cho a, b, c, d ≥ 0 thoûa a + b + c + d = 4 vaø k ≥ 2. Chöùng minh raèng

(a k +1 + 1)(b k +1 + 1)(c k +1 + 1)(d k +1 + 1) ≥ ( a k + 1)(b k + 1)(c k + 1)(d k + 1)


97
Baøi 24.
Cho a, b, c > 0 thoûa a + b + c = 1. Chöùng minh raèng

a +1 b +1 c +1 36
+ + ≥
c(2 − b) a(2 − c) b(2 − a) 5

Baøi 25. (Romania 2005)


Cho a, b, c > 0 thoûa a + b + c = 3. Chöùng minh raèng

1 1 1
2
+ 2
+ 2 ≥ a2 + b2 + c2
a b c
Baøi 26. (Phaïm Vaên Thuaän)
Cho a, b, c ≥ 0 thoûa a + b + c ≥ 3. Chöùng minh raèng

1 1 1
+ 2 + 2 ≤1
a +b +c b +c+ a c + a+b
2

98
PH NG PHÁP PHÂN TÍCH BÌNH PH NG S.O.S

A. N I DUNG PH NG PHÁP.
I. Bài toán m u và nh lý.
Thông th ng, khi ng tr c m t bài toán quen bi t, cách chúng ta th ng b t u
gi i quy t không ph i là th mò m m các b t ng th c ã bi t, không ph i là
tìm ngay m t cách d n bi n nào ó mà thông th ng nh t là a v các d ng bình
ph ng. u này d a trên tính ch t c b n nh t c a s th c “ x 2 ≥ 0, ∀x ∈ R ”. Có
t nhi u bài toán, cho dù b n ch ng hay vô tình, u ã s d ng ph ng pháp
này trong ch ng minh. Tuy nhiên, r t có th nh ng ub ns p c c trong
c này s làm b n th c s ng c nhiên…
Chúng ta s m uv ib t ng th c AM-GM, ây có th coi là b t ng th c c
n nh t trong nh ng b t ng th c c b n. Nh ng chúng ta ch tìm hi u b t ng
th c này trong tr ng h p n r t nh . V i n = 2 ch ng h n, ta có b t ng th c
Ví d 1. i m i a, b ≥ 0 , ta có b t ng th c a 2 + b 2 ≥ 2ab.
không có nhi u u c n ph i bàn t i b t ng th c trên, ngay khi các b n h c
s th c thì vi c ch ng minh b t ng th c ó quá d . B t ng th c t ng ng
i (a − b) 2 ≥ 0 m t u quá hi n nhiên. Bây gi , chúng ta xét ti p khi n = 3 và b t
ng th c sau ây
Ví d 2. i m i a, b, c ≥ 0 , ta có b t ng th c a3 + b3 + c 3 ≥ 3abc.
Khi h i v m t cách ch ng minh th t c th cho b t ng th c này, chúng ta s c m
th y có m t chút b i r i! T t nhiên, b t ng th c trên không khó, l i gi i ch trong
duy nh t m t dòng…
1
VT − VP = .(a + b + c)((a − b) 2 + (b − c) 2 + (c − a ) 2 )
2
Và ch c ch n ây là cách làm thông minh nh t, vì chúng ta không ph i qua m t
c trung gian nào c . C hai ví d trên u c ch ng minh b ng ph ng pháp

99
phân tích bình ph ng nh ng theo m t ngh a t ng i h p. Thu n l i r t l n trong
i gi i bài toán b ng cách này là vi c s d ng r t ít ki n th c “cao c p”, th m chí
n không c n bi t b t k m t nh lý nào v b t ng th c c . Ngoài ra, nó còn là
t ph ng pháp r t t nhiên theo suy ngh c a chúng ta.
u c k các bài toán ch ng tr c, các b n ã g p không ít nh ng bài toán s
ng ph ng pháp này trong ch ng minh. Còn bây gi , chúng ta s khái quát hóa
cách s d ng và i tìm b n ch t c a m t ph ng pháp c c k hi u qu .
Bài toán quan tr ng mà chúng ta ph i xét n trong m c này là m t b t ng th c
i ti ng ã c gi i thi u ch ng tr c, b t ng th c Iran 96.
Bài toán 1. (Iran 96)
i m i s th c a, b, c không âm, ta có

 1 1 1  9
(ab + bc + ca )  + + ≥
 ( a + b) (b + c) (c + a ) 2  4
2 2

ây c ng là bài toán có hình th c phát bi u r t n gi n và p m t. Ngoài ra, nó


còn là m t b t ng th c r t khó khi b n ch a c ti p c n tr c ó. Nh ng tr c
tiên, chúng ta hãy xem l i b t ng th c trong k thi IMO 2005 và tìm m t ch ng
minh th t t nhiên cho nó.
Ví d 3. (IMO 2005)
x, y , z là các s th c d ng th a xyz ≥ 1 . Ch ng minh

x5 − x 2 y5 − y 2 z5 − z 2
+ + ≥0
x5 + y 2 + z 2 y 5 + z 2 + x2 z 5 + x 2 + y 2
Ch ng minh.
Không m t tính t ng quát ta ch c n xét tr ng h p xyz = 1 là (các b n hãy t
tìm hi u lý do t i sao nhé!). Khi ó, ta có
x5 − x 2 x 5 − x 2 .xyz x 4 − x 2 yz 2 x4 − x2 ( y 2 + z 2 )
= = ≥
( )
x 5 + y 2 + z 2 x 5 + y 2 + z 2 xyz x 4 + ( y 2 + z 2 ) yz 2 x 4 + ( y 2 + z 2 )2

t a = x 2 , b = y 2 , c = z 2 . Khi ó, ta ch c n ch ng minh

100
2a 2 − a (b + c)
∑ 2 a 2 + (b + c ) 2 ≥ 0
cyc

 a b 
⇔ ∑ ( a − b)  2 − 2 2 
≥0
cyc  2 a + (b + c ) 2
2b + ( c + a ) 
c 2 + ac + bc + a 2 + b 2 − ab
⇔ ∑ ( a − b) . 2
≥ 0 (ñuùng)
cyc (2a 2 + (b + c) 2 )(2b 2 + (c + a ) 2 )

⇒ pcm.
ng th c x y ra khi và ch khi a = b = c ⇔ x = y = z = 1 .
Ch ng minh trên không ph i là cách duy nh t, có th còn nhi u ch ng minh c
áo h n. Nh ng n u xem xét khách quan thì ch ng minh trên hoàn toàn r t t nhiên
và c b n. Nói khái quát, khi ng tr cm tb t ng th c b t k ba bi n a, b, c ta
tìm cách a chúng v d ng t ng các bình ph ng ký hi u
S a (b − c ) 2 + Sb (c − a ) 2 + Sc (a − b) 2 ≥ 0
Ph n a v d ng chính t c trên là b c u tiên trong cách s d ng ph ng pháp
S.O.S. N u b n ã khá quen v i b t ng th c thì vi c l p công th c trên là t ng
i n gi n, ch c n bi t qua m t s phép bi n i và h ng ng th c, còn n u b n
ch a quen, thì các th c m c s c gi i quy t trong m c “Bi u di n c s c a
ph ng pháp S.O.S và m t s k thu t phân tích”.
t nhiên, n u trong bi u di n c s ó, các h s S a , Sb , S c u không âm thì bài
toán c ch ng minh. T tr c t i nay, ây v n là cách b n th ng làm nh ng
ây ch là tr ng h p n gi n nh t trong k thu t ch ng minh c a ph ng pháp
S.O.S. u quan tr ng h n, S.O.S giúp chúng ta gi i quy t các tr ng h p mà theo
quan ni m c là không th áp d ng c “có m t h s trong S a , Sb , S c không
ng”.
Thông th ng, trong các bài toán i x ng ta có th gi s a ≥ b ≥ c . V i các bài
toán hoán v thì ph i xét thêm tr ng h p a ≤ b ≤ c . Trong tr ng h p a ≥ b ≥ c , ta
có các nh n xét sau
1. N u Sb ≥ 0 , do (a − c) 2 ≥ (a − b) 2 + (b − c) 2 nên

101
S a (b − c ) 2 + Sb (c − a ) 2 + Sc (a − b) 2 ≥ ( Sa + Sb )(b − c) 2 + ( Sc + Sb )( a − b)2
và ph n còn l i c a bài toán là ch ng minh S a + Sb ≥ 0, Sb + Sc ≥ 0. Nh ng hai b t
ng th c này luôn có th ch ng minh khá n gi n, vì chúng không còn ph i nhân
thêm v i các bình ph ng (a − b) 2 ,(b − c )2 , (c − a ) 2 .

2. N u Sb ≤ 0 , do (a − c) 2 ≤ 2(a − b) 2 + 2(b − c) 2 nên

S a (b − c) 2 + Sb (c − a ) 2 + Sc ( a − b) 2 ≥ ( Sa + 2Sb )(b − c) 2 + ( S c + 2Sb )( a − b) 2


ng v y, vi c ch ng minh còn l i S a + 2 Sb ≥ 0, Sc + 2 Sb ≥ 0. s n gi n h n r t
nhi u.
Trong nhi u tr ng h p, ta c n thêm m t s cl ng m nh h n, ch ng h n c
ng hay dùng n là
a
a − c ≥ .(b − c) (a ≥ b ≥ c)
b
Ch ng h n khi ta có Sb , Sc ≥ 0 thì

a2 (b − c) 2 2
S a (b − c ) + Sb (c − a ) ≥ S a (b − c ) + Sb . 2 .(b − c) =
2 2 2 2
2
.(a Sb + b 2 S a )
b b
và nh v y bài toán s c ch ng minh n u a 2 Sb + b 2 Sa ≥ 0 .
Ta có th tóm t t các k t qu trên thành nh lý nh sau
nh lý S.O.S.
Xét bi u th c
S = f (a, b, c) = Sa (b − c) 2 + Sb (c − a ) 2 + Sc (a − b) 2
trong ó S a , Sb , S c là các hàm s theo a, b, c .
1. N u S a , Sb , Sc ≥ 0 thì S ≥ 0 .
2. N u a ≥ b ≥ c và Sb , Sa + Sb ≥ 0, Sb + Sc ≥ 0 thì S ≥ 0 .
3. N u a ≥ b ≥ c và S a , Sc , Sa + 2Sb ≥ 0, Sc + 2 Sb ≥ 0 thì S ≥ 0 .

4. N u a ≥ b ≥ c và Sb , Sc , a 2 Sb + b 2 Sa ≥ 0 thì S ≥ 0 .
5. N u S a + Sb + Sc ≥ 0 và S a Sb + Sb S c + Sc Sa ≥ 0 thì S ≥ 0 .
Ngoài ra, S ≥ 0 v i m i a, b, c thì ta ph i có
102
S a + Sb a =b ≥ 0, Sb + Sc b=c ≥ 0, Sc + Sa c =a ≥ 0.

Trong ó, S a + Sb a =b có ngh a là ta xét bi u th c S a + Sb khi a = b . V i các bài

toán i x ng, ta có ngay S a = Sb khi a = b . Nh n xét này r t quan tr ng trong các


bài toán tìm h ng s t t nh t.
ng nh nh lý này còn có v quá n gi n và nêú nói r ng nó có ng d ng v i
u h t các b t ng th c 3 bi n thì th t khó mà t ng t ng c. Nh ng th c t
S.O.S ã làm c u này và ây là m t u r t ng c nhiên.
t câu h i n a t ra là v i nh ng bi u th c nào thì ta có th chuy n v d ng
chính t c S.O.S nh v y? Câu tr l i là m i hàm s i x ng f (a, b, c) th a mãn
u ki n f (a, a, a ) = 0 và f có th ch a c n th c, phân th c c a a, b, c luôn luôn
có bi u di n y. Ch ng minh u này b n xem trong ph n ti p theo.
Bây gi là m t s ví d c th ch ng minh tính hi u qu c a ph ng pháp này,
và n u có th thì tr c tiên b n hãy th ch ng minh chúng theo cách khác.
Ví d 4.
Ch ng minh r ng v i m i s th c không âm a, b, c ta luôn có

a2 + b2 + c 2 8abc
+ ≥2
ab + bc + ca ( a + b)(b + c)(c + a )
Ch ng minh.
Ta chú ý n hai ng th c sau ây
1
a 2 + b 2 + c 2 − ab − bc − ca = .((a − b) 2 + (b − c) 2 + (c − a )2 )
2
( a + b )( b + c )( c + a ) − 8abc = c(a − b)2 + a(b − c)2 + b(c − a )2
Nh th sau khi thêm b t 1 m i s h ng v trái, ta có b t ng th c t ng ng
(a − b) 2 + (b − c) 2 + (c − a ) 2 2c (a − b) 2 + 2a (b − c) 2 + 2b(c − a) 2

ab + bc + ca ( a + b)(b + c)(c + a )
Ta tìm c
( a + b)(b + c )(c + a ) abc
Sa = − 2a = b + c − a −
ab + bc + ca ab + bc + ca

103
( a + b)(b + c)(c + a) abc
Sb = − 2b = c + a − b −
ab + bc + ca ab + bc + ca
( a + b)(b + c)(c + a) abc
Sc = − 2c = a + b − c −
ab + bc + ca ab + bc + ca
Do tính i x ng nên có th gi s a ≥ b ≥ c , khi ó d th y Sb , Sc ≥ 0 . D a vào
tiêu chu n th nh t, ta ch c n ch ng minh r ng S a + Sb ≥ 0 là xong. Nh ng u
này r t hi n nhiên vì
2abc 2c 2 (a + b)
S a + Sb = 2c − = ≥0
ab + bc + ca ab + bc + ca
yb t ng th c ã c ch ng minh.
ng th c x y ra khi và ch khi a = b = c ho c a = b, c = 0 ho c các hoán v t ng
ng.
Chúng ta hãy tr l i v i b t ng th c Iran 96.
Ví d 5. (Iran TST 1996)
i m i s th c x, y , z không âm, ta có

 1 1 1  9
( xy + yz + zx)  + + 2 

 ( x + y ) 2
( y + z ) 2
( z + x )  4
Ch ng minh.
t a = x + y, b = y + z, c = z + x . Ta ph i ch ng minh

 1 1 1  9
(2ab + 2bc + 2ca − a 2 − b2 − c 2 )  2 + 2 + 2  ≥
a b c  4
ng bi n i n gi n, ta có th chuy n b t ng th c v d ng
 2 1
∑ (a − b)2  ab − c2  ≥ 0
cyc

2 1
Sa = −
bc a 2
2 1
Sb = −
ca b 2
2 1
Sc = − 2
ab c

104
Gi s r ng a ≥ b ≥ c thì S a , Sb ≥ 0 . S d ng tiêu chu n 4, ta ch c n ch ng minh

b 2 Sb + c 2 S c ≥ 0
⇔ b3 + c 3 ≥ abc
nh ng b t ng th c này hi n nhiên úng vì a ≤ b + c ⇒ b3 + c 3 ≥ bc(b + c) ≥ abc.
ng th c x y ra khi và ch khi a = b = c ho c a = b, c = 0 ho c các hoán v t ng
ng.
Có m t vài ch ng minh khác cho b t ng th c Iran 96, cách thông th ng chúng ta
bi t là khai tri n và s d ng b t ng th c Schur (ho c dùng nh lý Muirhead),
ho c dùng a th c i x ng. Tuy nhiên, b n cs ng ý v i tôi r ng các ph ng
pháp ó ch có ý ngh a là ch ng minh b t ng th c úng v m t toán h c, ch
không l i nhi u n t ng. Vi c bi t s d ng ph ng pháp S.O.S ã làm cho bài
toán tr nên n gi n h n r t nhi u, ây th c s là m t l i gi i p và ng n g n,
th a mãn c m quan toán h c c a nhi u ng i.
Ph ng pháp phân tích bình ph ng ã t ng xu t hi n theo cách này hay cách khác
trong m t s b t ng th c, vì nó là m t h ng suy ngh r t t nhiên iv ib t
ng th c. Nh ng ch c ch n ây s là l n u tiên mà ph ng pháp này ch
th ng và c coi là ph ng pháp chính th ng cho chúng ta. Nó em l i cho chúng
ta m t cách nhìn ch ng và vô cùng hi u qu i v i các bài toán mà ch m t th i
gian ng n tr c còn là nh ng bài toán vô cùng khó kh n. B t ng th c Iran 96
c coi là bài toán c b n ng d ng ph ng pháp này (m c dù tác gi ngh n
S.O.S t m t b t ng th c c h n). S.O.S là tên l y t ch cái u tiên c a c m t
Sum of Square.
II. Bi u di n c s ph ng pháp S.O.S.
1. M u.
Trong các bài toán c d n ra các m c tr c h n các b n ã nh n th y s l p i
p l i c a bi u th c d ng F ( a, b, c) = S a (b − c) 2 + Sb (c − a )2 + S c ( a − b)2 . Các nh
lý sau ây s cho th y s t n t i c a bi u di n ó. Chúng tôi t gi i h n mình trong
các l p b t ng th c 3 bi n i x ng, tuy nhiên u ó s không làm h n ch t m

105
ng d ng c a ph ng pháp này. Các b n có th s d ng các ví d ki m ch ng
ng v i cùng t
ng d i ây, h u h t các b t ng th c hoán v ba bi n c ng có nh ng bi u di n
ng t . Chúc các b n may m n!
2. Các khái ni m c b n.
2.1. T p xác nh (TX ).
ây tr i n u không có gì thay i, cho bài toán rõ ràng và tránh nh ng
phi n ph c không áng có, TX c a t t c các hàm s và b t ng th c s gi i h n
trong t p s th c R3+ , h n n a, ôi khi h p lý chúng ta s b i m (0,0,0) .
2.2. nh ngh a 1: Hàm i x ng ba bi n.
t hàm phân th c ba bi n F ( a, b, c) c g i là i x ng n u và ch n u ng
nh t th c sau F ( a, b, c ) = F ( x, y , z) úng v i m i hoán v ( x, y, z ) c a (a, b, c) .H n
a n u v i m i s th c d ng x mà F ( x, x, x ) = 0 thì F ( a, b, c) c g i là hàm
i x ng ba bi n chu n.
2.3. nh ngh a 2: Hàm n a i x ng ba bi n.
t hàm phân th c ba bi n G ( a, b, c ) c g i là n a i x ng n u và ch n u ng
nh t th c sau G ( a, b, c) = G ( a, c, b) úng v i m i b ba s th c d ng (a, b, c) . H n
a n u v i m i c p hai s th c d ng x, y mà G ( x, y, y ) = 0 thì G ( a, b, c ) c
i là hàm n a i x ng ba bi n chu n.
3. Các nh lý c s .
3.1. nh lý 1: C s c a ph ng pháp S.O.S.
Gi s F ( a, b, c) là m t a th c i x ng ba bi n chu n, thì t n t i m t a th c n a
i x ng G ( a, b, c ) sao cho ng nh t th c sau úng

F ( a, b, c) = G ( a, b, c)(b − c) 2 + G (b, c, a )(c − a ) 2 + G (c, a , b)( a − b) 2


Tr c khi a ra m t ch ng minh c a nh lý này d a trên m t s hi u bi t n
gi n v không gian vect chúng tôi mu n nh n m nh v i các b n r ng nh lý trên
là áp d ng i v i t t c các hàm phân th c i x ng ba bi n. B i vì nh lý 1
n ch trong các l p a th c ba bi n nên có th nói t i b c c a a th c. Trong a

106
th c ba bi n a, b, c s ch a (và ch ch a!) các h ng t tm,n, p a mb n c p trong ó m, n, p

là các s nguyên không âm.


Ch ng minh nh lý 1.
Ta ch ng minh nh lý 1 cho l p các a th c b c n . Ký hi u S ( F ) là t p h p t t c
các a th c ba bi n F ( a, b, c) i x ng chu n b c n , S (Q ) là t p h p t t c các a
th c G ( a, b, c ) i x ng ba bi n chu n b c n d ng

G ( a, b, c) = G ( a, b, c)(b − c )2 + G (b , c , a )(c − a ) 2 + G (c, a , b)( a − b)2


ây G ( a, b, c ) là a th c n a i x ng ba bi n b c n − 2 (ta xét n ≥ 2 vì v i n = 1
thì nh lý hi n nhiên úng).
Rõ ràng S (Q ) là không gian vect con c a không gian vect F ( a, b, c) . Và do ó,
chi u c a S (Q ) không v t quá s chi u c a S ( F ) . (*)
i các s nguyên không âm α , β , γ xét các a th c c bi t sau ây

(i) Fα ,β ,γ ( a , b, c) = ∑ aα b β cγ
sym

(ii) Gα , β ,γ ( a, b, c ) = aα b β cγ + aα bγ c β

(iii) Qα ,β ,γ (a, b, c) = Gα ,β ,γ (a, b, c)(b − c) 2 +

1 + Gα ,β ,γ (b, c, a )(c − a ) 2 + Gα ,β ,γ (c, a, b)( a − b) 2

Ký hi u f n là t p h p t t c các b s (α , β , γ ) th a mãn các u ki n


α + β + γ = n,α ≥ β ≥ γ .
Rõ ràng t p h p t t c các a th c Fα ,β ,γ (a, b, c) v i (α , β , γ ) ∈ f n là h sinh cl p

tuy n tính c a S ( F ) do ó s chi u c a S ( F ) b ng s ph n t c a f n .(1)


Ký hi u qn là t p h p t t c các b s (α , β , γ ) th a mãn các u ki n
α + β + γ = n − 2, α + 2 ≥ β ≥ γ .
Rõ ràng t p h p t t c các a th c Gα ,β ,γ ( a, b, c) v i (α , β , γ ) ∈ qn là h vect c

p tuy n tính c a S (Q ) do ó s chi u c a S (Q ) không nh h n s ph n t c a


qn . (2)

107
các k t qu (1), (2) v i chú ý là f n và qn có cùng s ph n t ta suy ra s chi u
a S (Q ) không nh h n s chi u c a S ( F ) . (**)
y t các k t qu (*), (**) suy ra s chi u c a hai không gian S (Q ) , S ( F ) là b ng
nhau, t ó suy ra m i ph n t c a không gian S ( F ) u có th bi u di n qua các
ph n t c a không gian S (Q ) . ây là k t qu c n ph i ch ng minh.
nh lý này có th nh n th y m t thu t toán tìm bi u di n c s , ó là tìm ma
tr n chuy n gi a hai không gian vect S (Q ) và S ( F ) . D i ây là m t thu t toán
c p h n.
3.2 nh lý 2: Thu t toán tìm bi u di n c s .
Gi s M (a, b, c), N ( a, b, c) là hai a th c n a i x ng ba bi n, h n n a v i m i s
M ( x , x, x )
th c d ng x thì phân s là m t h ng s t . Khi ó t n t i hàm s n a
N ( x , x, x )
i x ng ba bi n G ( a, b, c ) sao cho ng nh t th c sau úng
M ( a, b, c) M (b, c, a) M (c, a, b)
F ( a , b, c ) = + + − 3t
N ( a, b, c) N (b, c, a ) N (c, a, b)
= G ( a, b, c )(b − c )2 + G (b, c, a )(c − a ) 2 + G (c, a, b)( a − b)2
Ch ng minh nh lý 2.
i v i hàm n a i x ng G ( a, b, c ) chúng ta ti n hành ghép c p các h ng t n a

i x ng a mb n c p + a mb p c n . Sau ó, nhóm t t c các h ng t có cùng b c vào m t


nhóm. B s (n1 , n2 ,..., nk ) v i n1 > n2 > ... > nk g m t t c các giá tr b c c a a
th c ó s p theo th t gi m d n g i là b ch th cho a th c ó. Khi ó, ta có th
vi t
k
G ( a, b, c) = ∑ ∑ g m,n, p .a m (b n c p + b p c n )
i =1 m + n + p = ni , n≥ p

M ( x , x, x )
Rõ ràng u ki n là m t h ng s v i m i s th c d ng x t ng ng
N ( x , x, x )
i s ki n b ch th c a các a th c M (a, b, c), N ( a, b, c) là gi ng nhau. Và do ó
ta xét hi u

108
k

M (a, b, c)
∑ ∑ α m,n, p .a m (b n c p + b p c n )
i =1 m + n + p = ni ,n≥ p
−t =
N (a, b, c) N (a, b, c)

trong ó, α m,n, p = mm,n , p − tnm,n , p và do ó ∑ α m,n, p = 0, ∀i = 1, 2,..., n.


m + n + p = ni , n≥ p

Bây gi i v i m i t ng bên trong t ng ng v i m i giá tr ni c a t s chúng ta


ti n hành s p x p l i th t các h ng t trong t s c a phân s trên sau ó s dùng
t bi n i nh làm xu t hi n các nhân t a − b, b − c, c − a.
Tr c h t ta chia các nghi m nguyên không âm (m, n, p ) th a mãn n ≥ p c a
ph ng trình m + n + p = ni thành ni nhóm theo các giá tr m . S p x p l i th t
các nhóm theo gi m d n c a m . Trong m i nhóm thì giá tr c a m là c nh, ta
p x p l i các nghi m nguyên không âm c a ph ng trình n + p = ni − m theo
gi m d n c a n n u ni − m l và theo t ng d n c a n n u ni − m ch n. Sau khi
ã s p th t xong, chúng ta có m t th t m i c a các t p nghi m ban u, mà ta

{
ký hi u là ( m j , n j , p j ) j = 1, 2,..., l , } ây l là m t hàm s ph thu c ni .

n gi n ta ký hi u

a j = a j (b j c + b j c j ), b j = am j ,n j , p j
m n pj p n

Khi ó m u s có th vi t l i m t cách n gi n là
a1b1 + a2b2 + ... + anbn =
= ( a1 − a2 )b1 + (a2 − a3 )(b1 + b2 ) + ... + ( al −1 − al )(b1 + b2 + ... + bl ).
d ng u ki n b1 + b2 + ... + bl = 0 và chia các hi u a1 − a2 , a2 − a3 ,..., al −1 − al
vào ba lo i sau
b n− p − c n− p
(i) a m (b n +1c p + b p c n +1 ) − a m (b n c p +1 + b p +1c n ) = a mb n c p . .(b − c )2
b−c
(ii) a m+1 (b n c n + b n c n ) − a m (b n+1c n + b n c n+1 ) = a m b n c n [( a − b) − (c − a)]
Xét bi u th c
a mb n c n [(a − b) − (c − a )] b m c n a n [(b − c ) − ( a − b)] c m a n b n [(c − b) − (b − c )]
+ +
N ( a , b, c ) N (b, c, a) N ( c, a, b)
109
Ti n hành ghép t ng ph n trong ba h ng t trong bi u th c này thành ba c p theo
các nhân t a − b, b − c, c − a. M t trong ba h ng t m i s là

 a m−n b m−n 
(a − b) a n b n c n  −  = ( a − b) .G (c, a, b)
2

 N ( a, b, c) N (b, c, a) 
trong ó
c n a nb n a m−n .N (b, c, a) − b m−n .N (a, b, c)
G (c, a , b) = .
N (a, b, c).N (b, c, a ) a −b
ây, ta ã s d ng N (b, c, a ) = N (b, a, c ). Do c t s và m u s c a phân s trên
u là nh ng a th c n a i x ng ba bi n a, b, c và i x ng hai bi n a, b nên
G (c, a, b) là hàm n a i x ng ba bi n.

(iii) a m+1 (b n+1c n + b n c n +1 ) − a m (b n+1c n+1 + b n+1c n +1 ) = a mbn c n [c (a − b) − b(c − a )]


Xét bi u th c
a mb n c n [c( a − b) − b(c − a )] b m c n a n [a (b − c) − c(a − b)]
+ +
N (a, b, c) N (b, c, a )
c m a nb n [b(c − a) − a(b − c )]
+
N (c, a, b)
Ti n hành ghép t ng ph n trong ba h ng t trong bi u th c này thành ba c p theo
các nhân t a − b, b − c, c − a. M t trong ba h ng t m i s là

n n n +1  a m−n b m−n 
(a − b) a b c  −  = ( a − b) .G (c, a, b)
2

 N (a, b, c) N (b, c, a ) 
trong ó
c n+1a nb n a m−n .N (b, c, a) − b m−n .N (a, b, c)
G (c, a , b) = .
N (a, b, c).N (b, c, a ) a −b
ây ta ã s d ng N (b, c, a ) = N (b, a, c ). Do c t s và m u s c a phân s trên
u là nh ng a th c n a i x ng ba bi n a, b, c và i x ng hai bi n a, b nên
G (c, a, b) là hàm n a i x ng ba bi n.

110
y trong c ba tr ng h p ta u ch ra cách bi n i thích h p a bi u th c
d ng bi u di n c n thi t. u này hoàn thành vi c ch ng minh nh lý 2. Ni m
tin v s t n t i bi u di n c s ã c kh ng nh.
B. CÁC BÀI T P ÁP D NG.
I. Bài t p có l i gi i.
Bài 1.
1
Cho a, b, c > 0 th a min{a, b, c} ≥ .max{a, b, c} . Ch ng minh r ng
4

 1 1 1  9 1  ( a − b) 2 
(ab + bc + ca )  + +  ≥ + . ∑ 
 ( a + b ) (b + c )
2 2
(c + a )2  4 16  cyc (a + b) 2 

Ch ng minh.
1
Không m t tính t ng quát gi s c ≥ b ≥ a ≥ .c > 0.
4
b + c = 2 x a = − x + y + z
 
t c + a = 2 y ⇔ b = x − y + z ⇒ x, y , z là dài ba c nh c a m t tam giác
a + b = 2 z c = x + y − z
 
1
Do c ≥ b ≥ a ≥ .c nên x ≥ y ≥ z > 0 và 4(− x + y + z ) ≥ x + y − z ⇒ 3 y + 5 z ≥ 5 x
4
t ng th c c n ch ng minh tr thành
 1 1 1  9 1  ( x − y)2 
(2 xy + 2 yz + 2 zx − x 2 − y 2 − z 2 )  2 + 2 + 2  ≥ + . ∑ 
 4x 4y 4 z  4 16  cyc z 2 
 1 1 1  1  ( x − y )2 
⇔ (2 xy + 2 yz + 2 zx − x − y − z )  2 + 2 + 2  ≥ 9 + .  ∑
2 2 2

x y z  4  cyc z 2 
 2 5 
⇔ ∑ ( x − y )2  − 2  ≥ 0
cyc  xy 4 z 
2 5 2 5 2 5
t Sx = − 2 , Sy = − 2 , Sz = − 2
yz 4 x zx 4 y xy 4 z
t ng th c c n ch ng minh t ng ng v i
S x ( y − z ) 2 + S y ( z − x ) 2 + S z ( x − y )2 ≥ 0

Do x ≥ y ≥ z > 0 và 3 y + 5 z ≥ 5 x nên S x > 0 và 8 y ≥ 5 x ⇒ S y ≥ 0

Ta ch ng minh
111
y2S y + z 2Sz ≥ 0
2 y2 2z2 5
⇔ + ≥
xz xy 2
⇔ 4( y 3 + z 3 ) ≥ 5 xyz
Mà 3 y + 5 z ≥ 5 x nên ta ch c n ch ng minh

4( y 3 + z 3 ) ≥ (3 y + 5 z ) yz
⇔ ( y − z )(4 y 2 + yz − 4 z 2 ) ≥ 0 (ñuùng)
y
Ta có x − z ≥ .( x − y ) ≥ 0 .
z
Do ó
S x ( y − z ) 2 + S y ( z − x) 2 + S z ( x − y ) 2 ≥ S y ( z − x )2 + S z ( x − y )2
y2
≥ Sy. 2
.( x − y ) 2 + S z ( x − y ) 2
z
( x − y)2 ( y 2 S y + z 2 S z )
=
z2
≥0
⇒ pcm.
5
ng th c x y ra khi và ch khi x = y = z ho c y = z = .x .
8
Bài 2. (Vasile Cirtoaje)
a, b, c > 0 th a a 2 + b2 + c 2 = 1 . Ch ng minh r ng
1 1 1 9
+ + ≤
1 − ab 1 − bc 1 − ca 2
Ch ng minh.
t ng th c c n ch ng minh t ng ng v i
 2 
∑  3 − 1 − ab  ≥ 0
cyc

1 − 3ab
⇔∑ ≥0
cyc 1 − ab

2 − 6ab
⇔∑ ≥0
cyc 1 − ab

112
2(a 2 + b 2 + c 2 ) − 6ab
⇔∑ ≥0
cyc 1 − ab
3( a − b) 2 2c 2 − a 2 − b 2
⇔∑ +∑ ≥0
cyc 1 − ab cyc 1 − ab
3( a − b) 2 c2 − a2 b2 − c 2
⇔∑ +∑ −∑ ≥0
cyc 1 − ab cyc 1 − ab cyc 1 − ab

3( a − b) 2 a 2 − b2 a 2 − b2
⇔∑ +∑ −∑ ≥0
cyc 1 − ab cyc 1 − bc cyc 1 − ca

3( a − b) 2 (a − b) 2 ( a + b)c
⇔∑ −∑ ≥0
cyc 1 − ab cyc (1 − bc)(1 − ca )

⇔ ∑ ( a − b) 2 (3 − 4ac − 4bc + a 2bc + ab 2c + 3abc 2 ) ≥ 0


cyc
t
S a = 3 − 4ab − 4ac + ab 2c + abc 2 + 3a 2bc
Sb = 3 − 4ab − 4bc + a 2bc + abc 2 + 3ab 2c
Sc = 3 − 4bc − 4ac + ab 2c + a 2bc + 3abc 2
t ng th c c n ch ng minh t ng ng v i
S a (b − c ) 2 + Sb (c − a ) 2 + Sc (a − b) 2 ≥ 0
Ta có
S a > 3 − 4ab − 4ac
= 3(a 2 + b 2 + c 2 ) − 4ab − 4ac
 a2   a2 
= 3  + b 2  + 3  + c 2  − 4ab − 4ac
 2   2 
≥ 3 2ab + 3 2ab − 4ab − 4ac
>0
Do ó S a > 0
ng t Sb > 0, Sc > 0

⇒ S a (b − c) 2 + Sb (c − a) 2 + Sc ( a − b) 2 ≥ 0
⇒ pcm.
ng th c x y ra khi và ch khi a = b = c .

113
Bài 3. (Vietnam Team Selection Test 2006)
Cho x, y , z ∈ [1,2] . Ch ng minh

1 1 1  x 
( x + y + z)  + +  ≥ 6  ∑
x y z  cyc y + z 
 
Ch ng minh.
Ta có, b t ng th c c n ch ng minh t ng ng
( x + y + z )( xy + yz + zx)  2x 
− 9 ≥ 3∑  − 1
xyz cyc  y + z 
z ( x − y)2 ( x − y )2
⇔∑ ≥ 3∑
cyc xyz cyc ( x + z )( y + z )

z 2 + xz + yz − 2 xy
⇔∑ .( x − y ) 2 ≥ 0
cyc xy ( x + z )( y + z )

( z 2 + xz + yz − 2 xy ) z ( x + y )
⇔∑ .( x − y )2 ≥ 0
cyc xyz ( x + y )( y + z )( z + x)
⇔ ∑ ( z 3 x + z 3 y + 2 xyz 2 + x 2 z 2 + y 2 z 2 − 2 x 2 yz − 2 xy 2 z )( x − y )2 ≥ 0
cyc

t
S x = x3 y + x3 z + 2 x 2 yz + x 2 y 2 + x 2 z 2 − 2 xy 2 z − 2 xyz 2
S y = y 3 x + y 3 z + 2 xy 2 z + x 2 y 2 + y 2 z 2 − 2 x 2 yz − 2 xyz 2
S z = z 3 x + z 3 y + 2 xyz 2 + x 2 z 2 + y 2 z 2 − 2 x 2 yz − 2 xy 2 z
Khi ó b t ng th c c n ch ng tr thành
S x ( y − z ) 2 + S y ( z − x ) 2 + S z ( x − y )2 ≥ 0

Không m t tính t ng quát gi s x ≥ y ≥ z .


x
Do x, y , z ∈ [1,2] nên y + z ≥ x ≥ y ≥ z ≥ .
2
Ta có
S x = x3 y + x3 z + 2 x 2 yz + x 2 y 2 + x 2 z 2 − 2 xy 2 z − 2 xyz 2
= x 3 y + x3 z + x( y + z )( xy + xz − 2 yz )
>0

114
S y = y 3 x + y 3 z + 2 xy 2 z + x 2 y 2 + y 2 z 2 − 2 x 2 yz − 2 xyz 2
= y ( z + x)( y 2 + xy + yz − 2 zx )
≥ y ( z + x )( z 2 + xz + z 2 − 2 zx )
= yz ( z + x )(2 z − x )
≥0
S y + S z = x( y 3 + z 3 ) + yz ( y + z ) 2 + x 2 ( y − z ) 2 − 2 x 2 yz
≥ xyz ( y + z ) + yz ( y + z ) 2 − 2 x 2 yz
≥ x 2 yz + x 2 yz − 2 x 2 yz
=0
Do ó theo tiêu chu n 2, ta có pcm.
ng th c x y ra khi và ch khi ( x, y, z ) = (t , t , t ),(2,1,1) (t ∈ [1, 2]) .
Bài 4.
a, b, c > 0 . Ch ng minh r ng
ab + bc + ca
∑ 8a 2 + bc
≥1
cyc

Ch ng minh.
Ta có b t ng th c c n ch ng minh t ng ng v i
 ab + bc + ca bc 
∑  8a + bc
2
− ≥0
ab + bc + ca 
cyc

(ab + bc + ca ) 2 − bc(8a 2 + bc)


⇔∑ ≥0
cyc 8a 2 + bc
a 2b 2 + a 2c 2 − 6a 2bc + 2ab 2c + 2abc 2
⇔∑ ≥0
cyc 8a 2 + bc
a 2 (b − c) 2 b + c − 2a
⇔∑ + 2abc ∑ 2 ≥0
cyc 8a + bc
2
cyc 8a + bc

a 2 (b − c) 2
Rõ ràng ta có ∑ 8a 2 + bc ≥ 0 .
cyc

Do ó ta ch c n ch ng minh

115
b + c − 2a
∑ 8a 2 + bc ≥0
cyc

c−a a −b
⇔∑ − ∑ ≥0
cyc 8a + bc cyc 8a + bc
2 2

a −b a −b
⇔∑ −∑ 2 ≥0
cyc 8b 2
+ ca cyc 8 a + bc
(a − b) 2 (8a + 8b − c)
⇔∑ ≥0
cyc (8a + bc)(8b + ca )
2 2

⇔ ∑ ( a − b) 2 (8a + 8b − c )(8c 2 + ab) ≥ 0


cyc

Không m t tính t ng quát gi s a ≥ b ≥ c > 0 .


t
S a = (8b + 8c − a )(8a 2 + bc)
Sb = (8c + 8a − b)(8b 2 + ca )
Sc = (8a + 8b − c )(8c 2 + ab)
Th thì ta có Sb , Sc > 0 .
t ng th c c n ch ng minh t ng ng v i
S a (b − c ) 2 + Sb (c − a ) 2 + Sc (a − b) 2 ≥ 0

Ta có a 2 (8b 2 + ca ) ≥ b 2 (8a 2 + bc )
Do ó
a 2 Sb + b 2 S a = a 2 (8c + 8a − b)(8b 2 + ca ) + b 2 (8b + 8c − a )(8a 2 + bc )
≥ b 2 (8c + 8a − b)(8a 2 + bc) + b 2 (8b + 8c − a )(8a 2 + bc)
= b 2 (8a 2 + bc)(7 a + 7b + 16c)
>0
Do ó áp d ng tiêu chu n 4, ta có pcm.

116
Bài 5.
a, b, c > 0 . Ch ng minh r ng

(b + c ) 2
∑ a 2 + bc ≥ 0
cyc

Ch ng minh.
Không m t tính t ng quát gi s a ≥ b ≥ c > 0 .
Ta có b t ng th c c n ch ng minh t ng ng v i
b 2 + c 2 − 2a 2
∑ a 2 + bc ≥ 0
cyc

c2 − a2 a 2 − b2
⇔∑ − ∑ ≥0
cyc a 2 + bc cyc a 2 + bc
a2 − b2 a 2 − b2
⇔∑ 2 −∑ 2 ≥0
cyc b + ca cyc a + bc

⇔ ∑ ( a − b) 2 ( a + b)( a + b − c )(c 2 + ab) ≥ 0


cyc
t
S a = (b + c )(b + c − a )(a 2 + bc)
Sb = (c + a )(c + a − b)(b 2 + ca )
S c = ( a + b)( a + b − c )(c 2 + ab)
Th thì ta có Sb , Sc ≥ 0
t ng th c c n ch ng minhh t ng ng v i
S a (b − c ) 2 + Sb (c − a ) 2 + Sc (a − b) 2 ≥ 0
Ta ch ng minh
b 2 S a + a 2 Sb ≥ 0 (*)
⇔ a 2 (c + a − b)(c + a)(b 2 + ca ) ≥ b 2 (a − b − c)(b + c )( a 2 + bc)
* N u a ≤b+c b t ng th c (*) hi n nhiên úng.
* N u a >b+c
c + a − b > a − b − c > 0

Ta có c + a ≥ b + c > 0 nên (*) úng.
 2 2
a (b + ca) ≥ b (c + ab) > 0
2 2

117
ây, áp d ng tiêu chu n 4, ta suy ra c pcm.
ng th c x y ra khi và ch khi (a, b, c) = (t , t , t ), (t , t , 0) (t > 0). .
Bài 6. (Crux Mathematicorum)
a, b, c là dài ba c nh c a m t tam giác. Ch ng minh

( ) +( b − c) +( )
2 2 2
a− b c− a 4 R 
≤ . − 2 
( a + b + c) 9 r 
2

Ch ng minh.
Ta có
R pabc
−2 = −2
r 4S 2
2abc
= −2
(a + b − c)(c + a − b)(b + c − a )
(a − b) 2
=∑
(c + a − b)(b + c − a )
Do ó b t ng th c c n ch ng minh t ng ng v i

( )
2
a− b 4 ( a − b)2
∑ ≤ .∑
( ) 9 cyc (b + c − a )(c + a − b)
2
cyc a+ b+ c

( ) ( )( )
2 
⇔∑ a − b  4  − 9(b + c − a )(c + a − b)   ≥ 0
2 2
a+ b a+ b+ c
cyc   
Do a, b, c là dài ba c nh c a m t tam giác nên a , b , c c ng là dài ba
nh c a m t tam giác. Do ó

( )( )
2 2
4 a+ b a+ b+ c − 9(b + c − a )(c + a − b) > 16c 2 − 9c 2 = 7c 2 > 0

ng t

( )( )
2 2
4 b+ c a+ b+ c − 9(a + b − c)(c + a − b) > 16a 2 − 9a 2 = 7 a 2 > 0

4( a) ( c)
2 2
c+ a+ b+ − 9(b + c − a )(a + b − c) > 16b 2 − 9b 2 = 7b 2 > 0

ây, ta suy ra pcm.


ng th c x y ra khi và ch khi a = b = c .
118
Baøi 7. (Vasile Cirtoaje)
a, b, c laø ñoä daøi ba caïnh cuûa moät tam giaùc. Khi ñoù, ta coù
a b c a c b
3 + +  ≥ 2  + +  + 3
b c a c b a
Chöùng minh.
Ta coù
a b c a c b
3 + +  ≥ 2  + +  + 3
b c a c b a
a b c  a c b 
⇔ 3 + + − 3  ≥ 2  + + − 3 
b c a  c b a 
⇔ ∑ (5a − 5b + 3c )( a − b)2 ≥ 0
cyc

Ñaët S a = 5b − 5c + 3a, Sb = 5c − 5a + 3b, Sc = 5a − 5b + 3c .


Baát ñaúng thöùc caàn chöùng minh töông ñöông vôùi
S a (b − c ) 2 + Sb (c − a ) 2 + Sc (a − b) 2 ≥ 0
+ Tröôøng hôïp 1. a ≤ b ≤ c . Khi ñoù, ta coù Sb ≥ 0 vaø
S a + Sb = 8b − 2a > 0 (do b ≥ a )
S c + Sb = 8c − 2b > 0
Do ñoù
S a (b − c )2 + Sa (c − a) 2 + Sc (a − b) 2 ≥ ( S a + Sb )(b − c) 2 + ( Sc + Sb )( a − b)2 ≥ 0
⇒ ñpcm.
+ Tröôøng hôïp 2. a ≥ b ≥ c . Khi ñoù ta coù S a , S c ≥ 0 . Do ñoù neáu Sb ≥ 0 thì ta coù
ngay ñpcm, vì vaäy ta chæ caàn xeùt tröôøng hôïp Sb ≤ 0 laø ñuû.
+ Tröôøng hôïp 2.1. a + ( )
3 − 1 c ≤ 3b ⇔ a − c ≤ 3(b − c )
Ta coù
S a + 3Sb = 14b + 10c − 12a ≥ 12(b + c − a ) > 0
Do ñoù
S a (b − c ) 2 + Sa (c − a) 2 + Sc (a − b) 2 ≥ ( S a + 3Sb )(b − c) 2 ≥ 0
⇒ ñpcm.
+ Tröôøng hôïp 2.2. a + ( )
3 − 1 c ≥ 3b ⇔ a − b ≥ ( )
3 − 1 (b − c )
3b
+ Tröôøng hôïp 2.2.1. a ≥
2
Ta coù
S a + 2 Sb = 11b + 5c − 7 a ≥ 8(b + c ) − 7 a > 0

119
 3b 
Sc + 2Sb = b + 13c − 5a > b + 13( a − b) − 5a = 8  a −  ≥ 0
 2
Do ñoù
S a (b − c)2 + S a (c − a) 2 + Sc (a − b) 2 ≥ ( S a + 2 Sb )(b − c) 2 + ( Sc + 2 Sb )( a − b) 2 ≥ 0
⇒ ñpcm.
3b
+ Tröôøng hôïp 2.2.2. a ≤
2
a
+ Tröôøng hôïp 2.2.2.1. a + c ≥ 2b ⇒ c ≥
3
Ta coù
S a + 2 Sb = 11b + 5c − 7 a ≥ 8(b + c ) − 7 a > 0
2a a
Sc + 2Sb = b + 13c − 5a ≥ + 13. − 5a = 0
3 3
Do ñoù
S a (b − c)2 + S a (c − a) 2 + Sc (a − b) 2 ≥ ( S a + 2 Sb )(b − c) 2 + ( Sc + 2 Sb )( a − b) 2 ≥ 0
⇒ ñpcm.
+ Tröôøng hôïp 2.2.2.2. a + c ≤ 2b ⇔ a − c ≤ 2(b − c )
Ta coù
( ) ( 3 − 1)
2 2
S a + 4 Sb + 3 − 1 Sc = (5a − 5b + 3c) + 17b + 15c − 17a

( 3 − 1) c
Do a + ( )
3 − 1 c ≥ 3b neân b ≤
a
3
+
3
Suy ra

5a − 5b + 3c ≥ 5a −
5a 5

( 3 −1 c) + 3c
3 3

=
5 ( 3 −1 a ) + (5 − 2 3 ) c
3 3

>
5 ( 3 −1 a )
3
Do ñoù
( )
3
3 −1 a
( )
2 5
S a + 4 Sb + 3 − 1 Sc > + 17b + 15c − 17 a
3
( )
3
5 3 −1 a
≥ + 16 ( b + c ) − 17 a
3

120
( )
3
5 3 −1 a
> −a>0
3
Do ñoù
S a (b − c ) 2 + Sb (c − a ) 2 + Sc ( a − b) 2 ≥  Sa + 4Sb + ( )
3 − 1 Sc  (b − c ) 2 ≥ 0
2

 
⇒ ñpcm.
Bài 8.
x, y , z > 0 . Ch ng minh r ng

(
x 2 + y 2 + z 2 + xy + yz + zx ≥ 2 x y 2 + z 2 + y z 2 + x 2 + z x 2 + y 2 )
Ch ng minh.
Ta có, b t ng th c c n ch ng minh t ng ng v i
2( x 2 + y 2 + z 2 ) − 2( xy + yz + zx) ≥

(
≥ 2 2 x y 2 + z 2 + y z 2 + x 2 + z x 2 + y 2 − 4( xy + yz + zx) )
(
⇔ ∑ ( x − y ) 2 ≥ 2∑ x 2( y 2 + z 2 ) − x( y + z )
cyc cyc
)
x( y − z ) 2
⇔ ∑ ( x − y ) ≥ 2∑ 2

cyc cyc 2( y 2 + z 2 ) + y + z
z( x − y )2
⇔ ∑ ( x − y ) ≥ 2∑ 2

cyc cyc 2( x 2 + y 2 ) + x + y
Ta l i có
z ( x − y)2 z ( x − y)2
2∑ ≤∑ (theo bñt Bunhiacopxki)
cyc 2( x 2 + y 2 ) + x + y cyc x + y
Do ó ta ch c n ch ng minh
z ( x − y )2
∑ ( x − y)2 ≥ ∑ x+ y
cyc cyc

 z 
⇔ ∑ 1 −  ( x − y) ≥ 0
2

cyc  x + y 
x y z
t Sx = 1− , Sy = 1− , Sz = 1 −
y+z z+x x+ y

121
t ng th c c n ch ng minh t ng ng v i
S x ( y − z ) 2 + S y ( z − x ) 2 + S z ( x − y )2 ≥ 0

Không m t tính t ng quát gi s x ≥ y ≥ z > 0 . Khi ó S y , S z > 0

Ta có
x2 y xy 2
x 2 S y + y 2 S x = x2 + y 2 − − ≥ x 2 + y 2 − 2 xy ≥ 0
x+z y+z
Do ó, áp d ng tiêu chu n 4, ta có pcm.
ng th c x y ra khi và ch khi x = y = z .
Bài 9. (Hojoo Lee)
a, b, c > 0 . Ch ng minh r ng

2( a3 + b3 + c3 ) 9( a + b + c) 2
+ 2 ≥ 33
abc a + b2 + c 2
Ch ng minh.
Ta có
 2  2

2(a 3 + b3 + c 3 ) 9(a + b + c ) 2
 ∑
 cyc ( a − b )  ∑
 cyc c ( a − b ) 
+ 2 − 33 =   ≥0
abc a +b +c2 2
abc( a + b + c )
2 2 2

⇒ pcm.
ng th c x y ra khi và ch khi a = b = c .
Bài 10. (IMO 2005)
x, y , z là các s th c d ng th a xyz ≥ 1 . Ch ng minh

x5 − x 2 y5 − y 2 z5 − z 2
+ + ≥0
x5 + y 2 + z 2 y 5 + z 2 + x2 z 5 + x 2 + y 2
Ch ng minh.
Không m t tính t ng quát ta ch c n xét tr ng h p xyz = 1 là . Khi ó, ta có

x5 − x 2 x 5 − x 2 .xyz x 4 − x 2 yz 2 x4 − x2 ( y 2 + z 2 )
= = ≥
( )
x 5 + y 2 + z 2 x 5 + y 2 + z 2 xyz x 4 + ( y 2 + z 2 ) yz 2 x 4 + ( y 2 + z 2 )2

t a = x 2 , b = y 2 , c = z 2 . Khi ó, ta ch c n ch ng minh
122
2a 2 − a (b + c)
∑ 2 a 2 + (b + c ) 2 ≥ 0
cyc

 a b 
⇔ ∑ ( a − b)  2 − 2 2 
≥0
cyc  2 a + (b + c ) 2
2b + ( c + a ) 
c 2 + ac + bc + a 2 + b 2 − ab
⇔ ∑ ( a − b) . 2
≥ 0 (ñuùng)
cyc (2a 2 + (b + c) 2 )(2b 2 + (c + a ) 2 )

⇒ pcm.
ng th c x y ra khi và ch khi a = b = c ⇔ x = y = z = 1 .
Bài 11. (Moldova 2006)
a, b, c là dài ba c nh c a m t tam giác. Ch ng minh r ng

b  c  a 
a 2  − 1 + b 2  − 1 + c 2  − 1 ≥ 0
c  a  b 
Ch ng minh.
+ Cách 1.
Ta có b t ng th c c n ch ng minh t ng ng v i

∑ a3b(b − c) ≥ 0
cyc

1
⇔ ∑
2 cyc
c 2 (b + c − a )( a − b) 2 ≥ 0 (ñuùng)

⇒ pcm.
ng th c x y ra khi và ch khi a = b = c .
+ Cách 2.
Ta có b t ng th c c n ch ng minh t ng ng v i

123
a 2b
∑ c ≥ ∑ a2
cyc cyc

 a 2b  1
⇔ ∑ + bc − 2ab  ≥ ∑ (a − b) 2
cyc  c  2 cyc
b(c − a) 2 1
⇔∑ ≥ ∑ ( a − b) 2
cyc c 2 cyc
c 1
⇔ ∑ ( a − b) 2  −  ≥ 0
cyc a 2
a 1 b 1 c 1
t Sa = − , Sb = − , Sc = −
b 2 c 2 a 2
t ng th c c n ch ng minh t ng ng v i
S a (b − c ) 2 + Sb (c − a ) 2 + Sc (a − b) 2 ≥ 0
+ Tr ng h p 1. b + c > a ≥ b ≥ c. Th thì ta có S a , Sb > 0 .
Ta có
b c
Sb + S c = + − 1 > 0(do b ≥ c > 0)
c a
Do ó, áp d ng tiêu chu n 2, ta có pcm.
+ Tr ng h p 2. a ≤ b ≤ c < a + b. Th thì ta có S c , Sb > 0 .
Ta có
b a b c −b b c (b − c ) 2
Sb + Sa = + − 1 > + −1 = + − 2 = ≥0
c b c b c b bc
Do ó, áp d ng tiêu chu n 2, ta có pcm.
Tóm l i, trong m i tr ng h p, ta luôn có
S a (b − c ) 2 + Sb (c − a ) 2 + Sc (a − b) 2 ≥ 0 ( pcm)

ng th c x y ra khi và ch khi a = b = c .

124
Bài 12.
x, y , z là các s th c d ng. Ch ng minh r ng

x 2 + yz y 2 + zx z 2 + xy 3
+ + ≥
( y + z ) 2 ( z + x) 2 ( x + y ) 2 2
Ch ng minh.
t a = y + z, b = z + x, c = x + y . Khi ó, ta có b t ng th c c n ch ng minh t ng
ng v i
 2bc − ca − ab + a 2 
∑ 2
− 1 ≥ 0
cyc  a 
 b(c − a ) c (a − b) 
⇔ ∑ − ≥0
cyc  a2 a2 
b(c − a ) c( a − b)
⇔∑ − ∑ ≥0
cyc a2 cyc a 2

c( a − b) c( a − b)
⇔∑ 2
−∑ 2
≥0
cyc b cyc a
c( a − b)2 ( a + b)
⇔∑ ≥ 0 (ñuùng)
cyc a 2b 2
⇒ pcm.
ng th c x y ra khi và ch khi a = b = c .
Bài 13. (Gabriel Dospinescu)
a, b, c > 0 . Ch ng minh r ng

(a 2 + b 2 )(b 2 + c 2 )(c 2 + a 2 )( ab + bc + ca) 2 ≥ 8a 2b 2 c 2 (a 2 + b 2 + c 2 )2


Ch ng minh.
Ta có
(a 2 + b 2 )(b 2 + c 2 )(c 2 + a 2 )( ab + bc + ca) 2 ≥ 8a 2b 2 c 2 (a 2 + b 2 + c 2 )2
 
⇔ ∑ a 6 (b − c) 2 (b 2 + bc + c 2 ) + 2abc (a 2b 2 + b 2c 2 + c 2 a 2 )  ∑ c( a − b) 2  ≥ 0 (ñuùng)
 cyc 
cyc  
⇒ pcm.
ng th c x y ra khi và ch khi a = b = c .

125
Bài 14. (Old And New Inequalities)
a, b, c > 0 . Ch ng minh r ng

a 3 − b3 1
∑ a + b ≤ 4 .∑ (a − b)2
cyc cyc

Ch ng minh.
b + c = 2 x a = − x + y + z
 
t c + a = 2 y ⇔ b = x − y + z ⇒ x, y , z là dài ba c nh c a m t tam giác.
a + b = 2 z c = x + y − z
 
Khi ó, b t ng th c c n ch ng minh t ng ng v i

( x − y )3
∑ z ≤ ∑ ( x − y)2
cyc cyc

Ta có
( x − y )3 ( x − y ) 2 ( y + z − x)
∑ ( x − y) − ∑ z = ∑
2
z
≥0
cyc cyc cyc

( x − y )3
⇒ ∑ ( x − y)2 ≥ ∑ (1)
cyc cyc z

Ta l i có
( x − y )3 ( x − y)2 ( z + x − y )
∑ ( x − y) + ∑ z = ∑
2
z
≥0
cyc cyc cyc

( x − y )3
⇒ ∑ ( x − y ) 2 ≥ −∑ (2)
cyc cyc z

(1) và (2) ta suy ra pcm.


ng th c x y ra khi và ch khi a = b = c .

126
Bài 15. (USA Team Selection Test 2004)
a, b, c > 0 . Ch ng minh r ng

a + b + c − 3 3 abc ≤ 3max {( a− b , ) (
2
b− c , ) (
2
c− a )}
2

Ch ng minh.
t a = x 6 , b = y 6 , c = z 6 ( x, y, z > 0) . Khi ó b t ng th c c n ch ng minh t ng
ng v i
x 6 + y 6 + z 6 − 3 x 2 y 2 z 2 ≤ 3max{( x3 − y 3 ) 2 , ( y 3 − z 3 ) 2 ( z 3 − x 3 ) 2 }
Do ó, ta ch c n ch ng minh
x 6 + y 6 + z 6 − 3 x 2 y 2 z 2 ≤ ( x 3 − y 3 ) 2 + ( y 3 − z 3 )2 + ( z 3 − x 3 )2
⇔ ∑ ( x − y ) 2 (2( x 2 + y 2 + xy ) 2 − ( x 2 + y 2 + z 2 )( x + y )2 ) ≥ 0
cyc

t
S x = 2( y 2 + z 2 + yz ) 2 − ( x 2 + y 2 + z 2 )( y + z )2
= y 4 + z 4 + 4 y 2 z 2 + 2 y 3 z + 2 yz 3 − x 2 y 2 − x 2 z 2 − 2 x 2 yz
S y = 2( z 2 + x 2 + zx ) 2 − ( x 2 + y 2 + z 2 )( z + x )2
= z 4 + x 4 + 4 z 2 x 2 + 2 x 3 z + 2 xz 3 − x 2 y 2 − y 2 z 2 − 2 xy 2 z
= ( x + z ) 2 ( x 2 − y 2 ) + 3 z 2 x 2 + 2 xz 3
S z = 2( x 2 + y 2 + xy ) 2 − ( x 2 + y 2 + z 2 )( x + y ) 2
t ng th c c n ch ng minh t ng ng v i
S x ( y − z ) 2 + S y ( z − x ) 2 + S z ( x − y )2 ≥ 0

Không m t tính t ng quát gi s x ≥ y ≥ z > 0 . Khi ó, rõ ràng ta có S y , S z > 0.

Ta có
S x + S y = ( x 2 − y 2 )2 + 2 z 3 ( x + y ) + 2 z ( x + y )( x − y ) 2 > 0

Do ó, áp d ng tiêu chu n 2, ta suy ra pcm.


ng th c x y ra khi và ch khi x = y = z ⇔ a = b = c.

127
Bài 16. (Ph m Kim Hùng)
a ≥ b ≥ c > 0 . Ch ng minh r ng
a4 a+b+c
∑ 3a3 + 2b3 ≥ 5
cyc

Ch ng minh.
Ta có
a4 a+b+c
∑ 3a3 + 2b3 ≥ 5
cyc

25a 4
⇔∑ ≥ 5(a + b + c)
cyc 3a + 2b
3 3

 25a 4 
⇔ ∑ 3 − 11a + 6b ≥0
cyc  3a + 2b
3

2(a − b) 2 (−4a 2 + ab + 6b 2 )
⇔∑ ≥0
cyc 3a 3 + 2b3

−4b 2 + bc + 6c 2 −4c 2 + ca + 6a 2 −4a 2 + ab + 6b 2


t Sa = , S = , S =
3b3 + 2c3 3c 3 + 2a 3 3a 3 + 2b3
b c

t ng th c c n ch ng minh t ng ng v i
2S a (b − c ) 2 + 2 Sb (c − a ) 2 + 2 Sc ( a − b) 2 ≥ 0

Rõ ràng ta có Sb ≥ 0 . Ta s ch ng minh r ng

Sb + 2S c ≥ 0 (1)
a 2 Sb + 2b 2 S a ≥ 0 (2)
* Ch ng minh (1).
Ta có b t ng th c (1) t ng ng v i
2a3 ( a 2 + 2ab + 3b 2 − 6c 2 ) + 12a 2 ( ab 2 + b3 − 2c 3 ) +
+ 2( a3b 2 + a 4 c − 4b3c 2 ) + a 4c + 2ab3c + 6abc 3 + 36b 2c 3 ≥ 0
u này hi n nhiên úng do a ≥ b ≥ c > 0 . V y (1) úng.
* Ch ng minh (2).
Ta có

128
(2) ⇔ f (a ) = a 2 (3b3 + 2c 3 )(6a 2 + ac − 4c 2 ) +
+ 2b 2 (3c3 + 2a3 )(6c 2 + bc − 4b 2 ) ≥ 0

f / ( a) = 24a 2 (3ab3 + 2ac 3 − 2b 4 ) +


+ ac (3b 2 (7 ab + ac − 8bc) + 16c( ab 2 − c 3 ) + 53ab 2c) > 0 ( do a ≥ b ≥ c > 0)
⇒ f (a ) ng bi n.

⇒ f (a ) ≥ f (b) = b 2 (b 2 (2b3 + 7b 2 c + 3bc 2 − 12c3 ) + 9b3c 2 + 8bc 4 + 28c 5 ) ≥ 0


⇒ (2) úng.
Tr l i bài toán c a ta.
Ta có
2S a (b − c ) 2 + 2 Sb (c − a ) 2 + 2 Sc ( a − b) 2 ≥
(b − c) 2 (a 2 Sb + 2b 2 S a ) a
≥ ( Sb + 2 Sc )( a − b) 2 + ≥ 0 ( do a − c ≥ .(b − c) ≥ 0)
b2 b
⇒ pcm.
Bài 17. (Ph m Kim Hùng)
Cho caùc soá khoâng aâm a, b, c. Chöùng minh raèng
a3 b3 c3 a+b+c
+ 2 2+ 2 ≥
2a + b
2 2
2b + c 2c + a 2
3
Chöùng minh.
Ta coù baát ñaúng thöùc caàn chöùng minh töông ñöông vôùi
2b − a 2c − b 2a − c
.(a − b) 2 + 2 2 .(b − c) 2 + 2 .(c − a ) 2 ≥ 0
2a + b
2 2
2b + c 2c + a 2

Coù 2 tröôøng hôïp xaûy ra


+ Tröôøng hôïp 1. a ≥ b ≥ c ≥ 0 . Khi ñoù, ta coù
4b c
− 2 ≥0
2a + b
2 2
2c + a 2
−2 a 2a
+ ≥0
2a 2 + b 2 2c 2 + a 2
4b − 2a 2a − c
⇒ 2 + 2 ≥0
2a + b 2
2c + a 2
4b − 2a 2a − c
⇒ 2 .( a − b)2 + 2 .(c − a) 2 ≥ 0 (1)
2a + b 2
2c + a 2

129
(4c − 2b)b 2 (2a − c) a 2
+ ≥0
2b 2 + c 2 2c 2 + a 2
4c − 2b 2a − c
⇒ 2 2 .(b − c) 2 + 2 .(c − a )2 ≥ 0 (2)
2b + c 2c + a 2

Coäng caùc baát ñaúng thöùc (1) vaø (2) veá theo veá roâi chia caû hai veá cho 2, ta ñöôïc
2b − a 2c − b 2a − c
.(a − b) 2 + 2 2 .(b − c) 2 + 2 .(c − a ) 2 ≥ 0
2a + b
2 2
2b + c 2c + a 2

+ Tröôøng hôïp 2. c ≥ b ≥ a ≥ 0.
+ Tröôøng hôïp 2.1. 2b ≥ c + a. Khi ñoù, ta seõ chöùng minh
2b − a 4(2a − c)
+ 2 ≥0
2a + b
2 2
2c + a 2
Thaät vaäy, deã thaáy veá traùi laø haøm taêng cuûa c neân ta chæ caàn chöùng minh khi c = b,
töùc laø chöùng minh
2b − a 4(2a − b)
+ 2 ≥0
2a + b
2 2
2b + a 2
⇔ 4b3 + 2a 2b − 2ab 2 + 16a 3 − 8a 2b + 8ab 2 − 4b3 ≥ 0
⇔ 3a (5a 2 − 2ab + 2b 2 ) ≥ 0 (ñuùng)
2b − a 4(2a − c)
Do ñoù + 2 ≥0
2a + b
2 2
2c + a 2
Vaäy
2b − a 2c − b 2a − c
.( a − b ) 2
+ .(b − c ) 2
+ .(c − a) 2 ≥
2a + b
2 2
2b + c
2 2
2c + a
2 2

2b − a 2a − c
≥ .(c − a ) 2 + 2 .(c − a ) 2
4(2a + b )
2 2
2c + a 2

≥0
+ Tröôøng hôïp 2.2. 2b ≤ c + a. Khi ñoù, ta seõ chöùng minh
2b − a 6a − 3c
+ 2 ≥0 (3)
2a + b
2 2
2c + a 2
Thaät vaäy, deã thaáy veá traùi laø haøm taêng cuûa c neân chæ caàn chöùng khi c = 2b − a .
Baát ñaúng thöùc (3) trôû thaønh
2b − a 9a − 6b
+ 2 ≥0
2a + b 8b + 3a 2 − 8ab
2 2

⇔ 10b3 − 15b 2 a + 2a 2b + 15a 3 ≥ 0 (ñuùng)


Tieáp theo, ta seõ chöùng minh
2c − b 3 2a − c
+ . 2 ≥0 (4)
2b + c
2 2
2 2c + a 2

130
Thaät vaäy, vì veá traùi laø haøm giaûm theo a neân ta chæ caàn chöùng minh khi a = b , baát
ñaúng thöùc trôû thaønh
4c − 2b 6b − 3c
+ ≥0
2b 2 + c 2 2c 2 + b 2
⇔ 5c3 + 2c 2b − 2b 2c + 10b3 ≥ 0 (ñuùng)
Neáu c ≤ 2a thì ta coù baát ñaúng thöùc caàn chöùng minh ñuùng. Neáu c ≥ 2a thì töø 2 baát
3
ñaúng thöùc treân, vôùi chuù yù raèng (c − a ) 2 ≤ 3(b − a ) 2 + .(c − b)2 , ta coù
2
2b − a 2c − b 2a − c
.(a − b) 2 + 2 2 .(b − c) 2 + 2 .(c − a ) 2 ≥
2a + b
2 2
2b + c 2c + a 2

 2b − a 3(2a − c)   2c − b 3 2a − c 
≥ 2 + 2 2 
.(b − a ) 2 +  2 2 + . 2 2 
.(c − b) 2
 2a + b 2c + a   2b + c 2 2c + a 
2

≥0
Toùm laïi, ta luoân coù
2b − a 2c − b 2a − c
.(a − b) 2 + 2 2 .(b − c ) 2 + 2 .(c − a) 2 ≥ 0 (ñpcm)
2a + b
2 2
2b + c 2c + a 2

Ñaúng thöùc xaûy ra khi vaø chæ khi a = b = c.


Bài 18. (Ph m V n Thu n)
a, b, c > 0 . Ch ng minh r ng

a 2 + b2 + c 2 8abc
+ ≥2
ab + bc + ca ( a + b)(b + c)(c + a )
Ch ng minh.
Ta có b t ng th c c n ch ng minh t ng ng v i
 1 c 
∑ (a − b)2  2(ab + bc + ca) − (a + b)(b + c)(c + a)  ≥ 0
cyc  
⇔ ∑ ( a − b) 2 (( a + b − c)( ab + bc + ca) − abc) ≥ 0
cyc

t
S a = (− a + b + c )( ab + bc + ca ) − abc
Sb = ( a − b + c )( ab + bc + ca ) − abc
S c = ( a + b − c )( ab + bc + ca ) − abc
t ng th c c n ch ng minh t ng ng v i
S a (b − c ) 2 + Sb (c − a ) 2 + Sc (a − b) 2 ≥ 0

131
Không m t tính t ng quát gi s a ≥ b ≥ c . Th thì ta có Sb , Sc ≥ 0 .
Ta có
S a + Sb = 2c 2 ( a + b) ≥ 0
Do ó, áp d ng tiêu chu n 2, ta có ngay pcm.
ng th c x y ra khi và ch khi a = b = c ho c a = b, c = 0 và các hoán v .
Bài 19. (Ph m V n Thu n)
a, b, c > 0 . Ch ng minh r ng

( a + b + c) 2 1  a 3 + b 3 + c 3 a 2 + b2 + c 2 
− . − ≤2
a 2 + b2 + c2 2  abc ab + bc + ca 

Ch ng minh.
Ta có b t ng th c c n ch ng minh t ng ng v i
 
2  4 a+b+c 1 
 ∑ ( a − b)   2 + − ≥0
 cyc   a + b 2
+ c 2
abc ab + bc + ca 

a+b+c 9
t ng th c này úng do ≥ .
abc ab + bc + ca
⇒ pcm.
ng th c x y ra khi và ch khi a = b = c .
Bài 20. (Ph m Kim Hùng)
a, b, c > 0 . Ch ng minh r ng

2a 2 + 5bc 2b2 + 5ca 2c 2 + 5ab 21


+ + ≥
(b + c) 2 (c + a )2 ( a + b )2 4
Ch ng minh.
Ta có b t ng th c c n ch ng minh t ng ng v i
 2a 2 + 5bc 7 
∑  (b + c)2 − 4  ≥ 0
cyc  
8a 2 − 7b 2 − 7c 2 + 6bc
⇔∑ ≥0
cyc (b + c ) 2

132
a 2 − b2 c2 − a2 (b − c) 2
⇔ 4∑ − 4 ∑ (b + c)2 ∑ (b + c)2 ≥ 0
− 3
cyc (b + c )2 cyc cyc

 4( a + b)( a + b + 2c) 3 
⇔ ∑ ( a − b)2  − ≥0
 (a + c) (b + c ) ( a + b) 2 
2 2
cyc

⇔ ∑ ( a − b ) (4( a + b)3 (a + b + 2c ) − 3(a + c) 2 (b + c )2 ) ≥ 0


2

cyc
t
S a = 4(b + c )3 (b + c + 2a ) − 3( a + b) 2 (a + c) 2
Sb = 4( a + c)3 (a + 2b + c) − 3( a + b) 2 (b + c )2
Sc = 4( a + b)3 (a + b + 2c ) − 3(a + c) 2 (b + c )2
t ng th c c n ch ng minh t ng ng v i
S a (b − c ) 2 + Sb (c − a ) 2 + Sc (a − b) 2 ≥ 0
Không m t tính t ng quát gi s a ≥ b ≥ c > 0 .
Khi ó, ta d dàng nh n th y Sb , Sc ≥ 0 .
Ta có
Sb + Sa = 4(c + a)3 (a + 2b + c) − 3( a + b) 2 (b + c )2 +
+ 4(b + c)3 (b + c + 2a ) − 3(a + b) 2 (a + c ) 2
= 8c( a + b)((a + c) 2 + (b + c) 2 ) + ( a − b) 2 (a 2 + b 2 + 4ab + 2ac + 2bc − 2c 2 )
≥0

Do ó, áp d ng tiêu chu n 2, ta có pcm.


ng th c x y ra khi và ch khi (a, b, c) = (t , t , t ), (t , t , 0) (t > 0).
* Chú ý.
5
c ng là h ng s t t nh t c a b t ng th c
2
a 2 + kbc b 2 + kca c 2 + kab 3(k + 1)
+ + ≥
(b + c) 2 (c + a ) 2 ( a + b)2 4

133
Bài 21.
a, b, c > 0 . Ch ng minh r ng

a 2 b2 c 2 3( a 2 + b2 + c 2 )
+ + +a+b+c≥
b c a a+b+c
Ch ng minh.
Ta có b t ng th c c n ch ng minh t ng ng v i
 a2   3( a 2 + b2 + c 2 ) 
∑ b + b − 2 a  ≥ 2  − a −b −c
cyc    a+b+c 
1 2 
⇔ ∑ (a − b) 2  − ≥0
cyc b a+b+c
1 2 1 2 1 2
t Sa = − , Sb = − , Sc = −
c a+b+c a a+b+c b a+b+c
t ng th c c n ch ng minh t ng ng v i
S a (b − c ) 2 + Sb (c − a ) 2 + Sc (a − b) 2 ≥ 0
Ta có
1 1 1 6 9 6 3
S a + Sb + S c = + + − ≥ − = >0
a b c a+b+c a+b+c a+b+c a+b+c
Ta l i có

∑ a(a + b − c)(a − b + c)
S a S b + Sb S c + Sc S a =
cyc

abc( a + b + c) 2
∑ a3 + 3abc − ∑ ab(a + b)
= cyc cyc

abc( a + b + c ) 2
∑ a3 + 3abc − ∑ ab(a + b)
≥ cyc cyc

abc (a + b + c ) 2
≥ 0 ( theo bñt Schur)
ây, áp d ng tiêu chu n 5, ta có pcm.

134
Bài 22. (Ph m Kim Hùng)
Cho caùc soá döông a, b, c. Chöùng minh raèng
2 2 2
 b2   c2   c 2  12( a 3 + b3 + c3 )
a +  + b +  +  a +  ≥ .
 c   a  b  a+b+c
Ch ng minh.
Ta coù
2 2 2
 b2   c2   c 2  12(a 3 + b3 + c 3 )
a +  + b +  + a +  ≥
 c   a  b  a+b+c
ab 2 a 4 12(a 3 + b3 + c 3 )
⇔ ∑ a 2 + 2∑ +∑ 2 ≥
cyc cyc c cyc b a+b+c
 ab 2   a4 
⇔ 2∑  + ac − 2ab  + ∑  2 + b 2 − 2a 2  +
cyc  c  cyc  b 
  12(a 3 + b3 + c 3 )
+ 2  ∑ ab − ∑ a  ≥ 2
− 4(a 2 + b 2 + c 2 )
 cyc  a+b+c
 cyc 
2b 2( a + b) 
2
4a
⇔ ∑ (b − c)  2 + + − 4 ≥ 0
cyc c a+b+c c 
Ñaët
b2 4a 2(a + b)
Sa = 2 + + −4
c a+b+c c
c2 4b 2(b + c )
Sb = 2 + + −4
a a+b+c a

a2 4c 2(c + a )
Sc = + + −4
b 2
a+b+c b
Baát ñaúng thöùc caàn chöùng minh töông ñöông vôùi
S a (b − c ) 2 + Sb (c − a ) 2 + Sc (a − b) 2 ≥ 0
Coù 2 tröôøng hôïp xaûy ra
+ Tröôøng hôïp 1. c ≥ b ≥ a > 0. Khi ñoù, ta coù Sb ≥ 0.
Ta coù
b 2 c 2 4(a + b) 2( a + b) 2(b + c)
S a + Sb =
+ + + + −8 ≥ 0
c2 a 2 a + b + c c a
b2 c 2 2a 2c 2b
Vì 2 + 2 ≥ 2, + ≥ 4, ≥ 2
c a c a a

135
a 2 c 2 4(b + c ) 2( a + c) 2(b + c)
Sc + Sb = + + + + −8 ≥ 0
b2 a 2 a + b + c b a
a 2 c2 2a 2b 2c
Vì 2 + 2 ≥ 2, + ≥ 4, ≥ 2
b a b a a
Do ñoù
S a (b − c )2 + Sb (c − a ) 2 + Sc ( a − b)2 ≥ ( S a + Sb )(b − c )2 + ( Sc + Sb )(a − b) 2 ≥ 0
+ Tröôøng hôïp 2. a ≥ b ≥ c > 0. Khi ñoù, ta coù S a ≥ 1, Sc ≥ −1.
Ta coù
b 2 2c 2 8b + 4a 2( a + b) 4(b + c)
S a + 2 Sb = + + + + − 12 ≥ 0
c2 a2 a + b + c c a
4a + 8b 2a 2b 2c 2a
Vì ≥ 4, + ≥ 4, + ≥4
a+b+c c a a c
b 2 4c 2 16b + 4a 2( a + b) 8(b + c)
S a + 4 Sb = 2 + 2 + + + − 20
c a a +b +c c a
b 2 4c 2 8b + 4a 2( a + b) 8(b + c )
≥ 2+ 2 + + + − 16 = f (b)
c a a+b+c c a
Deã daøng kieåm tra f (b) laø haøm ñoàng bieán. Do ñoù
4c 2 16c 2 a
f (b) ≥ f (c) = 2 + + − 9 ≥ 2 32 − 9 > 1
a a c
+ Khaû naêng 2.1. a + c ≤ 2b ⇔ 2(b − c) ≥ a − c ≥ 0 ∧ b − c ≥ a − b ≥ 0.
Neáu Sb ≥ 0 thì ta coù ngay ñpcm. Neáu Sb ≤ 0 , thì
S a (b − c ) 2 + Sb (c − a ) 2 + Sc ( a − b) 2 ≥ ( Sa + 4Sb − 1)(b − c) 2 ≥ 0
+ Khaû naêng 2.2. a + c ≥ 2b. . Khi ñoù, ta seõ chöùng minh S c + 2 Sb ≥ 0. Thaät vaäy,
ta coù
a 2 2c 2 8b + 4c 2(a + c) 4(b + c )
Sc + 2Sb = + + + + − 12 = g (c)
b2 a 2 a + b + c b a
+ Khaû naêng 2.2.1. a ≥ 2b . Khi ñoù, do g (c ) laø haøm taêng neân
a2 8b 4b 2a
g (c) ≥ g (0) =
+ + + − 12 ≥ 0
b 2
a+b a b
a 9b a 4b a2 a −b −1
Vì + ≥ 5, + ≥ 4, 2 + ≥ 6, ≥
b a+b b a b b a+b 3
+ Khaû naêng 2.2.2. a ≤ 2b . Khi ñoù, do g (c ) laø haøm taêng neân
a 2 8b 2 4b 4a 14
g (c) ≥ g (2b − a ) = 2 + 2 + − − ≥ 0 (do 2b ≥ a ≥ b)
b a a 3b 3
Vaäy

136
S a (b − c )2 + Sb (c − a ) 2 + Sc ( a − b)2 ≥ ( S a + 2 Sb )(b − c )2 + ( Sc + 2 Sb )(a − b) 2 ≥ 0
Toùm laïi, trong moïi tröôøng hôïp, ta luoân coù
S a (b − c ) 2 + Sb (c − a ) 2 + Sc ( a − b) 2 ≥ 0 (ñpcm)
Ñaúng thöùc xaûy ra khi vaø chæ khi a = b = c.
Bài 23.
a, b, c > 0 . Ch ng minh r ng

29a 3 − b3
∑ 6a 2 + ab ≤ 4(a + b + c)
cyc

Ch ng minh.
Ta có b t ng th c c n ch ng minh t ng ng v i
 29a3 − b3 
∑  6a 2 + ab − 4a  ≤ 0
cyc  
b3 + 4a 2b − 5a 3
⇔∑ ≥0
cyc 6a 2 + ab
(b − a )(5a 2 + ab + b 2 )
⇔∑ ≥0
cyc 6a 2 + ab
(b − a )(b 2 − a 2 )
⇔∑ + ∑ (b − a ) ≥ 0
cyc 6a 2 + ab cyc

(b − a ) 2 ( a + b)
⇔∑ ≥ 0 (ñuùng)
cyc 6a 2 + ab

⇒ pcm.
ng th c x y ra khi và ch khi a = b = c .
Bài 24.
a, b, c > 0 . Ch ng minh r ng

4a 3 + 5b3 − 3a 2b + 10ab 2
∑ 3a + b
≥ 5( a 2 + b 2 + c 2 ) − ab − bc − ca
cyc

Ch ng minh.
Ta có b t ng th c c n ch ng minh t ng ng v i
 4a3 + 5b3 − 3a 2b + 10ab 2 
∑ 3a + b
− (a 2 + 4b 2 − ab)  ≥ 0
cyc  
137
4a 3 + 5b3 − 3a 2b + 10ab 2 − (3a + b)(a 2 + 4b 2 − ab)
⇔∑ ≥0
cyc 3a + b
a 3 + b3 − a 2b − ab 2
⇔∑ ≥0
cyc 3a + b
(b − a ) 2 ( a + b)
⇔∑ ≥ 0 (ñuùng)
cyc 3a + b

⇒ pcm.
ng th c x y ra khi và ch khi a = b = c .
Bài 25.
a, b, c > 0 . Ch ng minh r ng

3a3 + 7b3
∑ 2a + 3b ≥ 3(a 2 + b2 + c 2 ) − ab − bc − ca
cyc

Ch ng minh.
Ta có b t ng th c c n ch ng minh t ng ng v i
 3a 3 + 7b3 
∑  2a + 3b − (a 2 + 2b2 − ab)  ≥ 0
cyc  
3a 3 + 7b3 − (2a + 3b)(a 2 + 2b 2 − ab)
⇔∑ ≥0
cyc 2a + 3b
a 3 + b3 − a 2b − ab 2
⇔∑ ≥0
cyc 2a + 3b

(b − a ) 2 (a + b)
⇔∑ ≥ 0 (ñuùng)
cyc 2a + 3b

⇒ pcm.
ng th c x y ra khi và ch khi a = b = c .

138
Bài 26.
a, b, c > 0 . Ch ng minh r ng

2a 4
∑ a3 + b3 ≥ a + b + c
cyc

Ch ng minh.
Ta có b t ng th c c n ch ng minh t ng ng v i
4a 4
∑ a3 + b3 ≥ 2(a + b + c)
cyc

 4a 4 
⇔ ∑  3 3 − 5a + 3b  ≥ 0
cyc  a + b 
(a − b) 2 (3b 2 + ab − a 2 )
⇔∑ ≥0
cyc a 3 + b3

t
3c 2 + bc − b 2
Sa =
b3 + c3
3a 2 + ac − c 2
Sb =
c3 + a3
3b 2 + ab − a 2
Sc =
a 3 + b3
t ng th c c n ch ng minh t ng ng v i
S a (b − c ) 2 + Sb (c − a ) 2 + Sc (a − b) 2 ≥ 0
+ Tr ng h p 1. a ≤ b ≤ c . Khi ó, d th y S c , Sa ≥ 0 . Ngoài ra, ta c ng d dàng
ch ng minh c Sc + 2Sb ≥ 0, Sa + 2Sb ≥ 0 .
Do ó áp d ng tiêu chu n 3, ta suy ra pcm.
+ Tr ng h p 2. a ≥ b ≥ c . Khi ó, d th y Sb ≥ 0 .
Ngoài ra, ta c ng d dàng ch ng minh c
Sb + 2 Sc ≥ 0
a 2 Sb + 2b 2 Sa ≥ 0
Do ó

139
2S a (b − c ) 2 + 2 Sb (c − a ) 2 + 2 Sc ( a − b) 2 ≥
(b − c) 2 2
≥ ( Sb + 2 Sc ) ( a − b) +
2
2
.( a Sb + 2b 2 S a )
b
a
≥ 0 ( do a − c ≥ .(b − c ) ≥ 0)
b
⇒ pcm.
ng th c x y ra khi và ch khi a = b = c .
Bài 27.
x, y , z > 0 . Ch ng minh r ng

x2 − z 2
∑ y+z ≥0
cyc

Ch ng minh.
Ta có b t ng th c c n ch ng minh t ng ng v i
4x2 4z2
∑ y+z ∑ y+z

cyc cyc

4 x2 4z2
⇔∑ − 2( x + y + z ) ≥ ∑ − 2( x + y + z )
cyc y + z cyc y + z

 4x2   4z 2 
⇔ ∑ − ( y + z) ≥ ∑ + ( y − 3z ) 
cyc  y + z  cyc  y + z 
4 x − y − z − 2 yz
2 2 2
( y − z) 2
⇔∑ ≥∑
cyc y+z cyc y+z
4x2 − 2 y 2 − 2z 2 ( y − z )2 ( y − z )2
⇔∑ +∑ ≥∑
cyc y+z cyc y+z cyc y+z
4x2 − 2 y 2 − 2z 2
⇔∑ ≥0
cyc y+z
2x2 − y 2 − z 2
⇔∑ ≥0
cyc y+z
x2 − y 2 z 2 − x2
⇔∑ −∑ ≥0
cyc y+z cyc y + z

x2 − y 2 x2 − y2
⇔∑ −∑ ≥0
cyc y + z cyc x + z

140
( x − y)2 ( x + y )
⇔∑ ≥0
cyc ( y + z )( x + z )

ây là u hi n nhiên úng. V y b t ng th c c ch ng minh hoàn toàn.


ng th c x y ra khi và ch khi x = y = z. .
Bài 28.
a, b, c > 0 . Ch ng minh r ng

4a (b 2 + c 2 )
∑ (b + c)(2a 2 + b2 + c 2 ) ≤ 3
cyc

Ch ng minh.
Ta có b t ng th c c n ch ng minh t ng ng v i
4a (b 2 + c 2 ) 2a (b + c ) 2 2a (b + c )2
∑ (b + c)(2a 2 + b2 + c 2 ) − ∑ (b + c)(2a 2 + b 2 + c 2 ) ≤ 3 − ∑ (b + c)(2a 2 + b 2 + c 2 )
cyc cyc cyc

2a (b − c ) 2 ( a − b) 2 + (a − c) 2
⇔∑ ≤ ∑
cyc (b + c )(2a 2 + b 2 + c 2 ) cyc 2a 2 + b 2 + c 2
 1 1 2c 
⇔ ∑ ( a − b)2  2 + − ≥0
 2a + b + c a 2 + 2b 2 + c 2 (a + b)(a 2 + b 2 + 2c 2 ) 
2 2
cyc
t
1 1 2a
Sa = + −
a 2 + 2b 2 + c 2 a 2 + b 2 + 2c 2 (b + c )(2a 2 + b 2 + c 2 )
1 1 2b
Sb = 2 + 2 −
2a + b + c
2 2
a + b + 2c (c + a )( a + 2b 2 + c 2 )
2 2 2

1 1 2c
Sc = 2 + −
2a + b 2 + c 2 a 2 + 2b 2 + c 2 ( a + b)( a 2 + b 2 + 2c 2 )
t ng th c c n ch ng minh t ng ng v i
S a (b − c ) 2 + Sb (c − a ) 2 + Sc (a − b) 2 ≥ 0
Không m t tính t ng quát gi s a ≥ b ≥ c .
Ta có
1 1 2b
Sb = + 2 −
2a + b + c
2 2 2
a + b + 2c (c + a )( a + 2b 2 + c 2 )
2 2 2

141
1 1 2b 1
≥ + 2 − . 2
2a + b + c
2 2 2
a + b + 2c
2 2
a a + 2b 2 + c 2
 1 b 1   1 b 1 
= 2 − . 2 +
2   2
− . 2 2 
 2a + b + c a a + 2b + c   a + b + 2c a a + 2b + c 
2 2 2 2 2 2

(a − b)(a 2 + b 2 + c 2 − ab)  1 b 1 
= + 2 − . 2 2 
a (2a + b + c )(a + 2b + c )  a + b + 2c
2 2 2 2 2 2 2 2
a a + 2b + c 
2

1 b 1
≥ 2 − . 2
a + b + 2c
2 2
a a + 2b 2 + c 2
1 1
≥ 2 − 2
a + b + 2c
2 2
a + 2b 2 + c 2
≥0

1 1 2c
Sc = + 2 −
2a + b + c
2 2 2
a + 2b + c ( a + b)( a + b 2 + 2c 2 )
2 2 2

1 1 1
≥ 2 + 2 − 2
2a + b + c
2 2
a + 2b + c
2 2
a + b 2 + 2c 2
4 1
≥ 2 − 2
3a + 3b + 2c
2 2
a + b 2 + 2c 2
a 2 + b 2 + 6c 2
= 2
( a + b 2 + 2c 2 )(3a 2 + 3b 2 + 2c 2 )
>0
Do ó Sb , Sc ≥ 0 .
Ta l i có
Sa Sb 1 1 2
+ = + − +
a 2 b 2 a 2 ( a 2 + 2b 2 + c 2 ) a 2 ( a 2 + b 2 + 2c 2 ) a (b + c)(2a 2 + b 2 + c 2 )
1 1 2
+ 2 + 2 2 −
b (2a + b + c ) b ( a + b + 2c ) b(c + a )( a + 2b 2 + c 2 )
2 2 2 2 2 2

1 1 2
≥ 2 2 + − +
a ( a + 2b 2 + c 2 ) a 2 (a 2 + b 2 + 2c 2 ) ab(2a 2 + b 2 + c 2 )
1 1 2
+ 2 + 2 2 −
b (2a + b + c ) b (a + b + 2c ) ab( a + 2b 2 + c 2 )
2 2 2 2 2 2

 1 1 2 
= 2 2 + 2 2 − 2 
+
 a ( a + 2b + c ) b (a + b + 2c ) ab(a + 2b + c ) 
2 2 2 2 2 2

142
 1 1 2 
+ 2 2 + −  ≥0
 a ( a + b 2
+ 2c 2
) b 2
(2 a 2
+ b 2
+ c 2
) ab (2 a 2
+ b 2
+ c 2
) 

⇒ a 2 Sb + b 2 S a ≥ 0.
Do ó, áp d ng tiêu chu n 4, ta có ngay pcm.
ng th c x y ra khi và ch khi a = b = c .
Bài 29.
a, b, c > 0 và ab + bc + ca = 1 . Ch ng minh r ng

3 3 5
a 2 + b2 + c 2 + .abc ≥
4 4
Ch ng minh.
t ng th c c n ch ng minh t ng ng v i
4(a 2 + b 2 + c 2 )( ab + bc + ca ) + 3abc 3(ab + bc + ca ) ≥ 5(ab + bc + ca )2
⇔ 4∑ a 3b − 5∑ a 2b 2 − 6( a + b + c) abc + 3abc 3(ab + bc + ca ) ≥ 0
sym cyc

Áp d ng b t ng th c AM-GM, ta có
5
.∑ a 3b ≥ 5∑ a 2b 2
2 sym cyc

Do ó, ta ch c n ch ng minh
3
.∑ a 3b + 3abc 3(ab + bc + ca ) ≥ 6abc( a + b + c )
2 sym
⇔ ∑ a 3b + 2abc 3( ab + bc + ca ) ≥ 4abc( a + b + c )
sym

( )
⇔ 2abc a + b + c − 3( ab + bc + ca ) ≤ ∑ a 3b − 2abc( a + b + c)
sym

abc.∑ (a − b) 2
⇔ cyc
≤ ∑ ( ab + ac )(b − c )2
a + b + c + 3( ab + bc + ca ) cyc

 abc 
⇔ ∑ (b − c )2  ab + ac −  ≥0
 + + + + + 
cyc  a b c 3( ab bc ca ) 
u này rõ ràng úng vì

143
abc
< min{ab, bc, ca}
a + b + c + 3(ab + bc + ca )

⇒ pcm.
ng th c x y ra khi và ch khi a = b = c .
Bài 30. (Vasile Cirtoaje)
a, b, c > 0 và a + b + c = 3 . Ch ng minh r ng
a 3
∑ a + bc ≥ 2
cyc

Ch ng minh.
Ta có b t ng th c c n ch ng minh t ng ng v i
2∑ a(b + ca)(c + ab) ≥ 3( a + bc )(b + ca)(c + ab)
cyc

 
⇔ 3abc + ∑ a 2b 2 ≥ 3a 2b 2c 2 + abc  ∑ a 2 
 cyc 
cyc  
1  
⇔ 3abc + .∑ c 2 ( a − b)2 ≥ 3a 2b 2 c 2 + abc  ∑ a 2 − ∑ a 
2 cyc  cyc 
 cyc 

3  
⇔ 9abc + .∑ c 2 ( a − b)2 ≥ 9a 2b 2c 2 + abc  ∑ ( a − b)2 
2 cyc  cyc 
 
2 3c 2 abc 
⇔ ∑ ( a − b)  abc +
2
− ≥0
cyc  2 2 

3a 2 abc 3b 2 abc 3c 2 abc


t S a = a bc +2
− , Sb = ab c +
2
− , Sc = abc +
2

2 2 2 2 2 2
t ng th c c n ch ng minh t ng ng v i
S a (b − c ) 2 + Sb (c − a ) 2 + Sc (a − b) 2 ≥ 0
Không m t tính t ng quát gi s a ≥ b ≥ c . Khi ó, d th y S a > 0 .
Ta có
3b 2 abc b
Sb > − = .(3b − ac) > 0
2 2 2
3(b + c )
2 2
Sb + Sc > − abc ≥ 3bc − abc = bc(3 − a ) > 0
2
144
Do ó áp d ng tiêu chu n 2, ta có pcm.
ng th c x y ra khi và ch khi a = b = c .
Bài 31. (Nguy n Anh C ng)
a, b, c > 0 . Ch ng minh r ng

a 2 + b 2 + c 2 + ab + bc + ca
∑ 2 a 2
+ bc
≥6
cyc

Ch ng minh.
Ta có b t ng th c c n ch ng minh t ng ng v i
 2(a 2 + b 2 + c 2 + ab + bc + ca ) 
∑ 2a 2 + bc
− 4 ≥0
cyc  
−6a 2 + 2b 2 + 2c 2 + 2ab − 2bc + 2ca
⇔∑ ≥0
cyc 2a 2 + bc
(3a − b + 2c )(c − a ) − (3a + 2b − c )( a − b)
⇔∑ ≥0
cyc 2a 2 + bc
(3a − b + 2c )(c − a ) (3a + 2b − c)(a − b)
⇔∑ −∑ ≥0
cyc 2a + bc
2
cyc 2a 2 + bc
(2a + 3b − c )(a − b) (3a + 2b − c)(a − b)
⇔∑ −∑ ≥0
cyc 2b + ca
2
cyc 2a 2 + bc
⇔ ∑ ( a − b) 2 (4a 2 + 4b 2 + c 2 + 6ab − 5bc − 5ca )(2c 2 + ab) ≥ 0
cyc

t
S a = (a 2 + 4b 2 + 4c 2 + 6bc − 5ab − 5ca)(2a 2 + bc )
Sb = (4a 2 + b 2 + 4c 2 + 6ca − 5ab − 5bc )(2b 2 + ca)
S c = (4a 2 + 4b 2 + c 2 + 6ab − 5ac − 5bc )(2c 2 + ab)
t ng th c c n ch ng minh t ng ng v i
S a (b − c ) 2 + Sb (c − a ) 2 + Sc (a − b) 2 ≥ 0
Không m t tính t ng quát gi s a ≥ b ≥ c . Khi ó, d th y Sb , Sc ≥ 0 .
Ta có
a 2 Sb + b 2 Sa = a 2 (4a 2 + b 2 + 4c 2 + 6ca − 5ab − 5bc)(2b 2 + ca ) +
+ b 2 (a 2 + 4b 2 + 4c 2 + 6bc − 5ab − 5ca )(2a 2 + bc) ≥ 0
145

a 2 (2b 2 + ca ) ≥ b 2 (2a 2 + bc ) > 0

(4a 2 + b 2 + 4c 2 + 6ca − 5ab − 5bc) + ( a 2 + 4b 2 + 4c 2 + 6bc − 5ab − 5ca ) ≥ 0
Do ó, áp d ng tiêu chu n 4, ta có ngay pcm.
ng th c x y ra khi và ch khi a = b = c ho c a = b, c = 0 và các hoán v .
Bài 32.
a, b, c > 0 th a ab + bc + ca = 1 . Ch ng minh r ng

a 3 3
∑ 2 + b2 + c2 ≥ 8
cyc

Ch ng minh.
Áp d ng b t ng Bunhiacopxki, ta có
a a2 (a + b + c) 2 (a + b + c) 2
∑ 2 + b2 + c 2 ∑ 2a + ab2 + ac 2 2(a + b + c) + a 2b 3(a + b + c) − 3abc
= ≥ =
cyc cyc ∑
sym
Do ó, ta ch c n ch ng minh
(a + b + c) 2 3 3

3(a + b + c ) − 3abc 8
⇔ 8( a + b + c) 2 + 9 3abc ≥ 9 3(a + b + c)
⇔ 8( a + b + c) 2 ab + bc + ca + 9 3abc ≥ 9 3( a + b + c )( ab + bc + ca )
(
⇔ 8( a + b + c) ab + bc + ca a + b + c − ab + bc + ca ≥ )
≥ 3((a + b + c)(ab + bc + ca ) − 9abc)
8(a + b + c) ab + bc + ca    
⇔ .  ∑ ( a − b)2  ≥ 3  ∑ c( a − b)2 
a + b + c + 3( ab + bc + ca )  cyc   cyc 
 8( a + b + c) ab + bc + ca 
⇔ ∑ ( a − b) 2  − 3c  ≥ 0
 a + b + c + 3(ab + bc + ca ) 
cyc  
t
8(a + b + c) ab + bc + ca
Sa = − 3a
a + b + c + 3(ab + bc + ca )

146
8( a + b + c ) ab + bc + ca
Sb = − 3b
a + b + c + 3( ab + bc + ca )
8( a + b + c ) ab + bc + ca
Sc = − 3c
a + b + c + 3( ab + bc + ca )
t ng th c c n ch ng minh t ng ng v i
S a (b − c ) 2 + Sb (c − a ) 2 + Sc (a − b) 2 ≥ 0
Không m t tính t ng quát gi s a ≥ b ≥ c . Khi ó, d th y Sb , Sc ≥ 0 .
Ta ch ng minh
a 2 Sb + b 2 Sa ≥ 0
⇔ 4( a 2b + b 2c + ab 2 + ca 2 + a 3 + b3 ) ab + bc + ca ≥
≥ 3 ( a + b + c ) (a 2b + ab 2 ) + 3( a 2b + ab 2 ) ab + bc + ca
⇔ 4(b 2 c + ca 2 + a 3 + b3 ) ab + bc + ca +
+ ( a 2b + ab 2 ) ab + bc + ca ≥ 3(a + b + c)(a 2b + ab 2 )
Ta d dàng ch ng minh c
4(a 3 + b3 ) ab + (a 2b + ab 2 ) ab > 3(a + b)( a 2b + ab 2 ) (1)

4a 2c ab + bc + ca > 3c( a 2b + ab 2 ) (2)

(1) và (2), ta suy ra a 2 Sb + b 2 Sa ≥ 0 .


ây, áp d ng tiêu chu n 4, ta có ngay pcm.
ng th c x y ra khi và ch khi a = b = c .
Bài 33.
x, y , z > 0 th a xyz = 1 . Ch ng minh r ng
y+z
∑ x
≥ x + y + z +3
cyc

Ch ng minh.
t x = a 2 , y = b 2 , z = c 2 ( a, b, c > 0) thì abc = 1 và b t ng th c c n ch ng minh
ng ng v i

147
b2 + c2
∑ a ≥ a+b+c+3
cyc

b2 + c2
⇔∑ − 2a − 2b − 2c ≥ 3 − a − b − c
cyc a

d ng b t ng th c AM-GM, ta có 0 ≥ 3 − a − b − c . Do ó, ta ch c n ch ng
minh
b2 + c2
∑ a − 2a − 2b − 2c ≥ 0
cyc

b 2 + c 2 − 2a 2
⇔∑ ≥0
cyc a
c2 − a 2 a2 − b2
⇔∑ −∑ ≥0
cyc a cyc a
a2 − b2 a 2 − b2
⇔∑ −∑ ≥0
cyc b cyc a
(a − b) 2 (a + b)
⇔∑ ≥ 0 (ñuùng)
cyc ab

⇒ pcm.
ng th c x y ra khi và ch khi a = b = c = 1 ⇔ x = y = z = 1.
Bài 34.(Vasile Cirtoaje)
a, b, c > 0 . Ch ng minh r ng

a 2 + bc
∑ b2 + bc + c 2 ≥ 2
cyc

Ch ng minh.
*B . u a, b, c, x, y, z là sáu s th c không âm th a mãn u ki n a ≥ b ≥ c và
x ≥ y ≥ z (ho c x ≤ y ≤ z ) thì
x( a − b)(a − c) + y (b − c )(b − a ) + z (c − a )(c − b) ≥ 0
Ch ng minh.
+ Tr ng h p 1. x ≥ y ≥ z ≥ 0 .
Ta có

148
a − c ≥ b − c ≥ 0 (do a ≥ b ≥ c)
⇒ x( a − c ) ≥ y (b − c ) ≥ 0
Mà a − b ≥ 0 nên
x( a − c )(a − b) ≥ y (b − c )( a − b) ≥ 0
⇔ x( a − c )( a − b) + y (b − c)(b − a ) ≥ 0
t khác, do a ≥ b ≥ c và z ≥ 0 nên
z (c − a )(c − b) ≥ 0
Do ó
x( a − b)(a − c) + y (b − c )(b − a ) + z (c − a )(c − b) ≥ 0
+ Tr ng h p 2. 0 ≤ x ≤ y ≤ z .
Ta có
a − c ≥ a − b ≥ 0 (do a ≥ b ≥ c )
⇒ z ( a − c) ≥ y (a − b) ≥ 0
Mà b − c ≥ 0 nên
z (a − c)(b − c ) ≥ y (a − b)(b − c ) ≥ 0
⇔ z (c − a )(c − b) + y (b − c)(b − a ) ≥ 0
t khác, do a ≥ b ≥ c và x ≥ 0 nên
x( a − c )(a − b) ≥ 0
Do ó
x( a − b)(a − c) + y (b − c )(b − a ) + z (c − a )(c − b) ≥ 0
c ch ng minh hoàn toàn.
Tr l i bài toán c a ta.
Không m t tính t ng quát gi s a ≥ b ≥ c . Khi ó, ta có
0 < b 2 + bc + c 2 ≤ a 2 + ac + c 2 ≤ a 2 + ab + b 2
1 1 1
⇒ ≥ 2 ≥ 2 >0
b + bc + c
2 2
a + ac + c 2
a + ab + b 2
1 1 1
Áp d ng B trên v i x = ,y = 2 ,z = 2 ta suy ra
b + bc + c
2 2
a + ac + c 2
a + ab + b 2

149
( a − b)(a − c)
∑ b 2 + bc + c 2
≥0
cyc

a2 ab + ac − bc
⇒∑ ≥∑ 2
cyc b + bc + c
2 2
cyc b + bc + c
2

a 2 + bc ab + ac
⇒∑ 2 ≥∑ 2
cyc b + bc + c cyc b + bc + c
2 2

Do ó, ta ch c n ch ng minh
ab + ac
∑ b2 + bc + c 2 ≥ 2
cyc

 ab + ac 2a 
⇔ ∑ 2 − ≥0
cyc  b + bc + c
2
a + b + c 
ab(a − b) − ca (c − a )
⇔∑ ≥0
cyc b 2
+ bc + c 2

ab( a − b) ca (c − a )
⇔∑ −∑ 2 ≥0
cyc b + bc + c
2 2
cyc b + bc + c 2

ab( a − b) ab( a − b)
⇔∑ −∑ 2 ≥0
cyc b + bc + c
2 2
cyc a + ac + c
2

ab(a − b) 2 ( a + b + c)
⇔∑ 2 ≥ 0 (ñuùng)
cyc (b + bc + c )( a + ac + c )
2 2 2

⇒ pcm.
ng th c x y ra khi và ch khi a = b = c .
Bài 35.
a, b, c > 0 . Ch ng minh r ng

a 2 + b 2 3(a 2 + b 2 + c 2 )
3( a + b + c ) ≤ ∑
2 2 2

cyc a + b a+b+c

Ch ng minh.
Tr c tiên ta ch ng minh b t ng th c
a 2 + b2
3( a + b + c ) ≤ ∑
2 2 2
(*)
cyc a + b

Ta có

150
a2 + b2
∑ a + b − 3(a 2 + b2 + c 2 ) =
cyc

= ∑
 a 2 + b 2 (a + b) 
cyc  a + b

2 
− ( 3( a 2 + b2 + c 2 ) − a − b − c )
(a − b)2 ∑ ( a − b)2
=∑ − cyc

cyc 2(a + b) 3( a + b2 + c 2 ) + a + b + c
2

1  1 2 
= ∑ ( a − b ) 2

 a +b
− 

2 cyc
 3( a 2
+ b 2
+ c 2
) + a + b + c 
1 2 1 1 
≥ ∑ ( a − b)  −  ( theo bñt Bunhiacopxki)
2 cyc  a+b a+b+c
≥0
⇒ (*) úng.
ng th c x y ra khi và ch khi a = b = c .
Ti p theo, ta ch ng minh b t ng th c
a 2 + b 2 3(a 2 + b 2 + c 2 )
∑ a+b ≤ a+b+c
cyc

+ Cách 1.
Ta có
a 2 + b 2 3(a 2 + b 2 + c 2 )
∑ a+b ≤ a+b+c
cyc

 a 2 + b2 
⇔ ( a + b + c )  ∑  ≤ 3(a + b + c )
2 2 2

 cyc a + b 
c( a 2 + b 2 )
⇔∑ ≤ a 2 + b2 + c2
cyc a +b
 2 c (a 2 + b 2 ) 
⇔ ∑c − ≥0
cyc  a+b 
ca (c − a ) bc(b − c)
⇔∑ −∑ ≥0
cyc a+b cyc a+b

151
ab(a − b) ab(a − b)
⇔∑ −∑ ≥0
cyc b+c cyc c+a
ab( a − b) 2
⇔∑ ≥ 0 (ñuùng)
cyc (b + c )(c + a )

⇒ pcm.
+ Cách 2.
Ta có
a 2 + b 2 3(a 2 + b 2 + c 2 )
∑ a+b ≤ a+b+c
cyc

 a 2 + b 2 (a + b)  3(a 2 + b 2 + c 2 ) − (a + b + c) 2
⇔ ∑ − ≤
cyc  a + b 2  a+b+c

( a − b)2 ∑ ( a − b)2
⇔∑ ≤
cyc

cyc 2(a + b) a+b+c


 2 1 
⇔ ∑ ( a − b) 2  − ≥0
cyc  a+b+c a+b
( a − b) 2 ( a + b − c )
⇔∑ ≥0
cyc a +b

b+c−a c+ a−b a +b−c


t Sa = , Sb = , Sc =
b+c c+a a+b
t ng th c c n ch ng minh t ng ng v i
S a (b − c ) 2 + Sb (c − a ) 2 + Sc (a − b) 2 ≥ 0
Không m t tính t ng quát gi s a ≥ b ≥ c . Khi ó, d th y Sb , Sc ≥ 0 .
Ta có
a 2b ab 2 a 2b ab 2
b 2 S a + a 2 Sb = a 2 + b 2 − − > a 2 + b2 − − = (a − b) 2 ≥ 0
a+c b+c a b
ây, áp d ng tiêu chu n 4, ta suy ra pcm.
ng th c x y ra khi và ch khi a = b = c ho c a = b, c = 0 và các hoán v .

152
Bài 36. (Hojoo Lee)
a, b, c > 0 . Ch ng minh r ng

a2 bc
∑ a 2 + 2bc ≥ 1 ≥ ∑ a 2 + 2bc
cyc cyc

Ch ng minh.
* Ch ng minh
bc
1≥ ∑ (*)
cyc a + 2bc
2

*B . u a, b, c, x, y, z là sáu s th c không âm th a mãn u ki n a ≥ b ≥ c và


x ≥ y ≥ z (ho c x ≤ y ≤ z ) thì
x( a − b)(a − c) + y (b − c )(b − a ) + z (c − a )(c − b) ≥ 0
Ch ng minh.
+ Tr ng h p 1. x ≥ y ≥ z ≥ 0 .
Ta có
a − c ≥ b − c ≥ 0 (do a ≥ b ≥ c)
⇒ x( a − c ) ≥ y (b − c ) ≥ 0
Mà a − b ≥ 0 nên
x( a − c )(a − b) ≥ y (b − c )( a − b) ≥ 0
⇔ x( a − c )( a − b) + y (b − c)(b − a ) ≥ 0
t khác, do a ≥ b ≥ c và z ≥ 0 nên
z (c − a )(c − b) ≥ 0
Do ó
x( a − b)(a − c) + y (b − c )(b − a ) + z (c − a )(c − b) ≥ 0
+ Tr ng h p 2. 0 ≤ x ≤ y ≤ z .
Ta có
a − c ≥ a − b ≥ 0 (do a ≥ b ≥ c )
⇒ z ( a − c) ≥ y (a − b) ≥ 0
Mà b − c ≥ 0 nên
z (a − c)(b − c) ≥ y ( a − b)(b − c) ≥ 0
153
⇔ z (c − a )(c − b) + y (b − c)(b − a ) ≥ 0
t khác, do a ≥ b ≥ c và x ≥ 0 nên
x( a − c )(a − b) ≥ 0
Do ó
x( a − b)(a − c) + y (b − c )(b − a ) + z (c − a )(c − b) ≥ 0
c ch ng minh hoàn toàn.
Tr l i bài toán c a ta.
Ta có
 bc bc 
(*) ⇔ ∑  − 2 ≥0
cyc  ab + bc + ca a + 2bc 
bc (a − b)(a − c)
⇔∑ ≥0
cyc ( ab + bc + ca ) ( a 2 + 2bc )
abc (a − b)(a − c)
⇔ .∑ 3 ≥0
ab + bc + ca cyc a + 2abc

Không m t tính t ng quát, gi s a ≥ b ≥ c . Khi ó, ta có


a 3 + 2abc ≥ b3 + 2abc ≥ c3 + 2abc > 0
1 1 1
⇒ ≥ 3 ≥ 3 >0
c + 2abc b + 2abc a + 2abc
3

1 1 1
Áp d ng b trên v i x = ,y = 3 ,z = 3 ta suy ra c
a + 2abc
3
b + 2abc c + 2abc
( a − b)( a − c )
∑ a 3 + 2abc
≥0
cyc

y (*) úng.
* Ch ng minh
a2
∑ a 2 + 2bc ≥ 1 (**)
cyc

Ta có
 a2 a 
(**) ⇔ ∑  2 − ≥0
cyc  a + 2bc a+b+c

154
a ( ab + ac − 2bc)
⇔∑ ≥0
cyc a 2 + 2bc
ca (a − b) ab(c − a )
⇔∑ −∑ 2 ≥0
cyc a + 2bc cyc a + 2bc
2

ca (a − b) bc (a − b)
⇔∑ − ∑ ≥0
cyc a 2 + 2bc cyc b 2 + 2ca
⇔ ∑ ( a − b) 2 c(2bc + 2ca − ab)(c 2 + 2ab) ≥ 0
cyc

t
S a = a(2ab + 2ca − bc )(a 2 + 2bc)
Sb = b(2ab + 2bc − ca )(b 2 + 2ca)
Sc = c (2bc + 2ca − ab)(c 2 + 2ab)
t ng th c c n ch ng minh t ng ng v i
S a (b − c ) 2 + Sb (c − a ) 2 + Sc (a − b) 2 ≥ 0
Không m t tính t ng quát gi s a ≥ b ≥ c . Khi ó, d th y S a , Sb ≥ 0 .

th y b(b 2 + 2ca ) ≥ c(c 2 + 2ab) ≥ 0 nên

vSb + Sc ≥ c (c 2 + 2ab)(2ab + 2bc − ca + 2bc + 2ca − ab)


= c(c 2 + 2ab)(ab + 4bc + ca )
≥0
Do ó, áp d ng tiêu chu n 2, ta suy ra ngay pcm.
Bài 37. (Hojoo Lee)
a, b, c > 0 . Ch ng minh r ng

a 2 + bc
∑ b+c ≥ a+b+c
cyc

Ch ng minh.
Ta có
a 2 + bc
∑ b+c ≥ a+b+c
cyc

155
 a 2 + bc (b + c ) 
⇔ ∑ − ≥0
cyc  b + c 2 
2a 2 − b 2 − c 2
⇔∑ ≥0
cyc b+c
a2 − b2 c2 − a 2
⇔∑ −∑ ≥0
cyc b+c cyc b + c
a2 − b2 a 2 − b2
⇔∑ −∑ ≥0
cyc b + c cyc a + c

(a − b) 2 (a + b)
⇔∑ ≥ 0 (ñuùng)
cyc (b + c )( a + c)

ng th c x y ra khi và ch khi a = b = c .
Bài 38. (Gabriel Dospinescu)
x, y , z > 0 . Ch ng minh r ng
1 3 1
∑ 3x + 1 + x + y + z + 1 ≥ ∑ 2 x + y + 1
cyc sym

Ch ng minh.
1 1 1
t a = x + ,b = y + , c = z + .
3 3 3
Khi ó, b t ng th c c n ch ng minh t ng ng v i
1 3 1
∑ 3a + a + b + c ≥ ∑ 2a + b
cyc sym

 1 1 1 1 
⇔ ∑ + − − ≥0
cyc  3a a + b + c 2a + b 2a + c 
(4a + b + c)((2a + b)(2a + c ) − 3a( a + b + c))
⇔∑ ≥0
cyc a (a + b + c)(2a + b)(2a + c)
(a − b)(a − c) ( a − b)( a − c )
⇔∑ −∑ ≥0
cyc a (2a + b) cyc a(2a + c )
(a − b)(a − c) (b − a )(b − c)
⇔∑ −∑ ≥0
cyc a (2a + b) cyc b ( a + 2b )
2bc + 2ca − ab
⇔ ∑ ( a − b) 2 . ≥0
cyc ab(2a + b)( a + 2b)

156
2ab + 2ca − bc 2ab + 2bc − ca 2bc + 2ca − ab
t Sa = , Sb = , Sc =
bc (2b + c )(b + 2c) ca(2a + c )( a + 2c) ab(2a + b)( a + 2b)
t ng th c c n ch ng minh t ng ng v i
S a (b − c ) 2 + Sb (c − a ) 2 + Sc (a − b) 2 ≥ 0
Không m t tính t ng quát gi s a ≥ b ≥ c . Khi ó, d th y S a , Sb ≥ 0 .
Do b ≥ c nên
b(2a + b)(a + 2b) ≥ c(2a + c )( a + 2c ) > 0
1 1
⇒ ≥ >0
c(2a + c )( a + 2c ) b(2a + b)( a + 2b)
2ab + 2bc − ca 2bc + 2ca − ab
⇒ Sb + Sc = +
ca (2a + c)( a + 2c) ab(2a + b)( a + 2b)
2ab + 2bc − ca 2bc + 2ca − ab
≥ +
ba (2a + b)(a + 2b) ab(2a + b)( a + 2b)
ab + 4bc + ca
=
ba (2a + b)(a + 2b)
≥0
Do ó áp d ng tiêu chu n 2, ta có ngay pcm.
Bài 39. (Iran 1996)
a, b, c > 0 . Ch ng minh r ng

 1 1 1  9
(ab + bc + ca )  + + ≥
 ( a + b) (b + c) (c + a ) 2  4
2 2

Ch ng minh.
* Cách 1.
Không m t tính t ng quát gi s c ≥ b ≥ a > 0 .
b + c = 2 x a = − x + y + z
 
t c + a = 2 y ⇔ b = x − y + z ⇒ x, y , z là dài ba c nh c a m t tam giác.
a + b = 2 z c = x + y − z
 
Do c ≥ b ≥ a > 0 nên x ≥ y ≥ z > 0
t ng th c c n ch ng minh tr thành

157
 1 1 1  9
(2 xy + 2 yz + 2 zx − x 2 − y 2 − z 2 )  2 + 2 + 2  ≥
 4x 4y 4z  4
 1 1 1 
⇔ (2 xy + 2 yz + 2 zx − x 2 − y 2 − z 2 )  2 + 2 + 2  ≥ 9
x y z 
 2 1 
∑ ( x − y)2  xy − z 2  ≥ 0
cyc  
2 1 2 1 2 1
t Sx = − 2 , S y = − 2 , Sz = −
yz x zx y xy z 2
t ng th c c n ch ng minh t ng ng v i
S x ( y − z ) 2 + S y ( z − x ) 2 + S z ( x − y )2 ≥ 0

Do x ≥ y ≥ z > 0 và y + z > x nên S x ≥ 0 và S y ≥ 0

Ta ch ng minh
y2S y + z 2S z ≥ 0
⇔ y 3 + z 3 ≥ xyz
Mà y + z > x nên ta ch c n ch ng minh

y 3 + z 3 ≥ ( y + z ) yz
⇔ ( y − z ) 2 ( y + z ) ≥ 0 (ñuùng)
ây, áp d ng tiêu chu n 4, ta suy ra pcm.
* Cách 2.
. N u a, b, c, x, y, z là 6 s th c không âm th a a ≥ b ≥ c và x ≤ y ≤ z thì

x(b − c )2 (3bc + ca + ab − a 2 ) + y (c − a ) 2 (3ca + ab + bc − b 2 ) +


+ z ( a − b)2 (3ab + bc + ca − c 2 ) ≥ 0
Ch ng minh B .
Do a ≥ b ≥ c nên
3ca + ab + bc − b 2 ≥ 0
3ab + bc + ca − c 2 ≥ 0
Do ó
+ N u 3bc + ca + ab − a 2 ≥ 0 thì b hi n nhiên úng.

158
+ N u 3bc + ca + ab − a 2 ≤ 0 thì
(b − c )2 (3bc + ca + ab − a 2 ) ≤ 0
⇒ x(b − c) 2 (3bc + ca + ab − a 2 ) ≥ y (b − c) 2 (3bc + ca + ab − a 2 )
i có z ≥ y nên

z (a − b) 2 (3ab + bc + ca − c 2 ) ≥ y ( a − b) 2 (3ab + bc + ca − c 2 )
Do ó
 
∑ x(b − c)2 (3bc + ca + ab − a 2 ) ≥ y  ∑ (b − c)2 (3bc + ca + ab − a 2 ) 
cyc  cyc 
 
= 4 y  ∑ ab(a − b) 2 
 cyc 
≥0
c ch ng minh.
Tr l i bài toán c a ta.
Ta có
 4( ab + bc + ca) 
(*) ⇔ ∑  − 3 ≥ 0
cyc  ( a + b) 2

 (3a + b)(c − a ) (3a + c)(b − c ) 
⇔ ∑ − ≥0
cyc  ( a + b) 2 (a + b) 2 
(3a + b)(c − a) (3a + c )(b − c)
⇔∑ − ∑ ≥0
cyc ( a + b)2 cyc ( a + b)2
(3a + b)(c − a) (3a + c )( a − b)
⇔∑ − ∑ ≥0
cyc ( a + b)2 cyc ( a + c ) 2

3ab + bc + ca − c 2
⇔ ∑ ( a − b) . 2
≥0
cyc (b + c ) 2 ( a + c )2
⇔ ∑ ( a − b) 2 (a + b) 2 (3ab + bc + ca − c 2 ) ≥ 0
cyc

Không m t tính t ng quát gi s a ≥ b ≥ c . Khi ó, ta có


(a + b) 2 ≥ (c + a ) 2 ≥ (b + c) 2 > 0

Áp d ng B trên v i z = (a + b) 2 , y = (c + a ) 2 , x = (b + c) 2

159
ta suy ra c

∑ (a − b)2 (a + b)2 (3ab + bc + ca − c 2 ) ≥ 0


cyc

⇒ pcm.
Bài 40. (Komal)
a, b, c > 0 th a abc = 1 . Ch ng minh r ng

1 1 1 3  1 1 1 2
+ + − ≥  2 + 2 + 2 . 2
a b c a+b+c a b c  a + b2 + c 2
Ch ng minh.
Ta có
1 1 1 3  1 1 1 2
+ + − ≥  2 + 2 + 2 . 2
a b c a+b+c  a b c  a + b2 + c 2
3abc 2( a 2b 2 + b 2 c 2 + c 2 a 2 )
⇔ ab + bc + ca − ≥
a+b+c a2 + b2 + c2
9abc 2( a 2b 2 + b 2 c 2 + c 2 a 2 ) 6abc
⇔ ab + bc + ca − ≥ −
a+b+c a +b +c
2 2 2
a+b+c
c( a − b) 2 c(a − b) 2 (c 2 + bc + ca − 2ab)
⇔∑ ≥∑
cyc a + b + c cyc (a 2 + b 2 + c 2 )( a + b + c)
⇔ ∑ c (a − b) 2 ( a 2 + b 2 + 2ab − bc − ca ) ≥ 0
cyc

t
S a = a(b 2 + c 2 + 2bc − ca − ab )
Sb = b(c 2 + a 2 + 2ca − ab − bc)
Sc = c (a 2 + b 2 + 2ab − bc − ca )
t ng th c c n ch ng minh t ng ng v i
S a (b − c ) 2 + Sb (c − a ) 2 + Sc (a − b) 2 ≥ 0
Không m t tính t ng quát gi s a ≥ b ≥ c . Khi ó, d th y Sb , Sc ≥ 0 .
Ta có
a 2 Sb + b 2 Sa = ab((a − b) 2 ( a + b) + 2c (a 2 + b 2 − ab) + c( a 2 + b 2 )) > 0
ây, áp d ng tiêu chu n 4, ta suy ra pcm.

160
II. Bài t p ngh .
i các b n gi i các bài toán sau làm quen v i ph ng pháp trên và n u có th
các b n hãy th gi i các bài toán này b ng ph ng pháp khác nhé!
Bài 1.
a) (Old and New Inequalities) a, b, c > 0 và a + b + c = 1 . Ch ng minh r ng

a2 + b
∑ b+c ≥ 2
cyc

b) (Võ Qu c Bá C n) a, b, c > 0 và a + b + c = 1 . Ch ng minh r ng

a 2 + 5b
∑ b+c ≥8
cyc

c) (Võ Qu c Bá C n) i nh ng u ki n nh trên, hãy tìm h ng s k l n


nh t cho b t ng th c
a 2 + kb 3k + 1
∑ b+c ≥ 2
cyc

Bài 2. (Võ Qu c Bá C n)
a) a, b, c > 0 . Ch ng minh r ng

2a 2 + 5bc 21 59(a − b) 2 (b − c) 2 (c − a ) 2
∑ (b + c)2 ≥ 4 + 4(a + b)2 (b + c)2 (c + a)2
cyc

b) V i nh ng u ki n nh trên, hãy tìm h ng s k l n nh t cho b t ng


th c
2a 2 + 5bc 21 k (a − b) 2 (b − c)2 (c − a) 2
∑ (b + c)2 ≥ 4 + (a + b)2 (b + c)2 (c + a)2
cyc

Bài 3. (Võ Qu c Bá C n)
a) a, b, c > 0 . Ch ng minh r ng

a 2 + 2bc 3( a + b + c )
∑ b+c ≥ 2
cyc

b) V i nh ng u ki n nh trên, hãy tìm h ng s k l n nh t cho b t ng


th c

161
a 2 + kbc (k + 1)( a + b + c )
∑ b+c ≥ 2
cyc

Bài 4. (Võ Qu c Bá C n)
a, b, c > 0 và ab + bc + ca = 1 . Ch ng minh r ng

a 2 b2 c 2
+ + − 2( a 2 + b 2 + c 2 ) ≥ 3 − 2
b c a
Bài 5. (Võ Qu c Bá C n)
a, b, c > 0 . Tìm h ng s k l n nh t sao cho b t ng th c sau úng

 1 1 1  k (ab + bc + ca )
(a + b + c)  + +  + ≥9+k
a b c a2 + b2 + c2
Bài 6. ( Võ Qu c Bá C n)
a, b, c > 0 . Ch ng minh r ng

4(a 4 + b 4 + c 4 ) + 3abc (a + b + c) ≥ 7(a 3b + b 3c + c3 a )


Bài 7. (Mathlinks)
a, b, c > 0 . Ch ng minh r ng

a2 3(a 3 + b3 + c 3 )
∑ b + c 2(a 2 + b2 + c2 )

cyc

Bài 8. (Mathnfriends)
a, b, c > 0 . Ch ng minh r ng
1 21
∑ a 2 + ab + b2 ≥ 2(a 2 + b2 + c 2 ) + 5(ab + bc + ca)
cyc

Bài 9. (Olympic 30 - 4 - 2006)


a, b, c > 0 . Ch ng minh r ng
a (b + c) 6
∑ a 2 + (b + c ) 2 ≤ 5
cyc

162
Bài 10. (Mathlinks)
a, b, c > 0 . Ch ng minh r ng

a2 a2
∑ a + 2b ∑ 2a + b

cyc cyc

Bài 11. (Stronger than Vietnam TST 2006 − Võ Qu c Bá C n)


a) x, y , z ∈ [1,2] . Ch ng minh r ng

1 1 1  x y z  9( x − y ) 2 ( y − z ) 2 ( z − x )2
( x + y + z)  + +  ≥ 6  + + +
x y z  y + z z + x x + y  xyz ( x + y )( y + z )( z + x)
b) V i các u ki n nh trên, hãy tìm h ng s k t t nh t cho b t ng th c
1 1 1  x y z  k ( x − y)2 ( y − z )2 ( z − x)2
( x + y + z)  + +  ≥ 6  + +  + xyz ( x + y )( y + z )( z + x )
 x y z   y + z z + x x + y 
Bài 12. (Mathlinks)
a, b, c > 0 . Ch ng minh r ng

b2 + c 2 a 3
∑ a(b + c) − ∑ b + c ≥ 2
cyc cyc

Bài 13. (Diendantoanhoc)


a, b, c > 0 . Ch ng minh r ng
1 9
∑ a 2 + ab + b2 ≥ (a + b + c)2
cyc

Bài 14. (Gabriel Dospinescu)


a, b, c > 0 . Ch ng minh r ng

 a 2  b 2  c2  1 1 1
27 +  2 +  2 +  2 +  ≥ 6(a + b + c)  + + 
 bc  ca  ab  a b c
Bài 15. (Belarus 1997)
a, b, c > 0 . Ch ng minh r ng
a a+c
∑b ≥ ∑b+c
cyc cyc

163
Bài 16. (Belarus 1998)
a, b, c > 0 . Ch ng minh r ng
a a+b b+c
∑ b ≥ b + c + a + b +1
cyc

Bài 17. (Mathlinks)


a, b, c > 0 . Ch ng minh r ng
a b+c
∑b ≥ ∑a+c
cyc cyc

Bài 18. (Mathlinks)


a, b, c > 0 . Ch ng minh r ng

b+c  a 
∑ a ≥ 4
∑

b + c

cyc  cyc 
Bài 19. (Mildorf)
a, b, c > 0 . Ch ng minh r ng

2∑ a 6 + 16∑ a3b3 ≥ 9∑ a 2b 2 ( a 2 + b 2 )
cyc cyc cyc

Bài 20. (Vasile Cirtoaje)


a, b, c > 0 . Ch ng minh r ng

7(a 2 + b 2 + c 2 )
∑ 4b2 − bc + 4c 2 ≥ 9
cyc

Bài 21. (Mathlinks)


a, b, c > 0 và ab + bc + ca = 1 . Ch ng minh r ng

1 + a 2b 2 5
∑ ( a + b)2 ≥ 2
cyc

Bài 22. (Diendantoanhoc)


1
a, b, c > 0 và ab + bc + ca = . Ch ng minh r ng
3
1
∑ a 2 − bc + 1 ≤ 3
cyc

164
Bài 23. (Japan 2004)
a, b, c > 0 và a + b + c = 1 . Ch ng minh r ng

 a 1+ a
2 ∑  ≥ ∑
 cyc b  cyc 1 − a
 
Bài 24. (Vasile Cirtoaje)
a , b, c > 0 , t E (a, b, c) = ∑ a (a − b)( a − c ) . Ch ng minh r ng
cyc

a) (a + b + c) E (a, b, c) ≥ ∑ ab( a − b)2


cyc

1 1 1
b)  + +  E ( a, b, c) ≥ ∑ a 2 − ∑ ab
a b c cyc cyc

Bài 25. (Vasile Cirtoaje)


x, y , z > 0, xyz = 1 . Ch ng minh r ng
( x + y )( y + z )( z + x) + 7 ≥ 5( x + y + z )
Bài 26. (Vasile Cirtoaje)
a) x, y , z > 0 . Ch ng minh r ng

 
3  ∑ x 4 − ∑ x3 y  ≥ ∑ z 2 ( x − y)2
 cyc  cyc
 cyc 
b) Ch ng minh r ng b t ng th c trên c ng úng cho x, y , z ∈ R .
Bài 27. (Mathlinks)
1
a, b, c > 0 và ab + bc + ca = . Ch ng minh r ng
3
a 1
∑ a 2 − bc + 1 ≥ a + b + c
cyc

Bài 28. (Võ Qu c Bá C n)


a) a, b, c > 0 . Ch ng minh r ng

2a 2 + bc 9
∑ b2 + c 2 ≥ 2
cyc

b) V i các u ki n nh trên, tìm h ng s k t t nh t cho b t ng th c


165
a 2 + kbc 3( k + 1)
∑ b2 + c 2 ≥ 2
cyc

c) i các u ki n nh trên, tìm h ng s k t t nh t cho b t ng th c


2a 2 + bc 9 k (a − b) 2 (b − c)2 (c − a) 2
∑ b2 + c 2 ≥ 2 + (a 2 + b2 )(b2 + c2 )(c 2 + a 2 )
cyc

Bài 29. (Mathlinks)


a, b, c > 0 . Ch ng minh r ng
1 2a
∑ b + c ≥ ∑ 3a 2 + bc
cyc cyc

Bài 30.
a, b, c > 0 . Ch ng minh r ng

a2 a
∑ b2 + c2 ≥ ∑ b + c
cyc cyc

Bài 31. (VMO 2006B)


a, b, c > 0, abc = 1 . Tìm k max sao cho
1 1 1
+ + + 3k ≥ (k + 1)( a + b + c )
a 2 b2 c 2
Bài 32. (Võ Qu c Bá C n)
a) a, b, c > 0 . Ch ng minh r ng

(b + c ) 2
∑ a 2 + 2bc ≥ 4
cyc

b) i các u ki n nh trên, tìm h ng s k t t nh t cho b t ng th c


(b + c ) 2 k (a − b) 2 (b − c)2 (c − a )2
∑ a 2 + 2bc ≥ 4 + (a 2 + 2bc)(b2 + 2ca)(c 2 + 2ab)
cyc

Bài 33.
a) (Mathlinks) a, b, c > 0 . Ch ng minh r ng

a2 1
∑ (2a + b)(2a + c) ≤ 3
cyc

166
b) (Võ Qu c Bá C n) V i các u ki n nh trên, tìm h ng s k t t nh t cho
t ng th c
a2 1 k ( a − b) 2 (b − c ) 2 (c − a ) 2
∑ (2a + b)(2a + c) ≤ 3 − (2a + b)(2b + c)(2c + a)(2a + c)(2c + b)(2b + a)
cyc

Bài 34. (Mathinks)


a, b, c > 0 và p ≥ 3 + 7 . Ch ng minh r ng
1 9
∑ pa 2 + bc ≥ ( p + 1)(ab + bc + ca)
cyc

Bài 35. (Mathlinks)


a, b, c > 0 và p > −2 . Ch ng minh r ng
ab + ( p − 1)bc + ca 3( p + 1)
∑ b 2 + pbc + c 2

p+2
cyc

Bài 36. (Stronger than Schur - Nguy n Anh C ng)


a, b, c > 0 . Ch ng minh r ng

a3 + b3 + c 3 + 3abc ≥ ab 2( a 2 + b 2 ) + bc 2(b 2 + c 2 ) + ca 2(c 2 + a 2 )


Bài 37. (JBMO 2002)
a, b, c > 0 . Ch ng minh r ng

a3 a2
∑ b2 ≥ ∑ b
cyc cyc

Bài 38. (Mathlinks)


a, b, c > 0 . Ch ng minh r ng
ab 3
∑ a 2 + b 2 + 3c2 ≤ 5
cyc

Bài 39. (Ph m Kim Hùng)


a) a, b, c > 0 . Ch ng minh r ng

 
∑ ∑ a 4
+ a 3
b ≥ 2
∑
 ab 3

cyc cyc  cyc 
b) Ch ng minh r ng b t ng th c trên c ng úng cho a, b, c ∈ Rr.
167
Bài 40. (Diendantoanhoc)
a, b, c > 0 . Ch ng minh r ng

   
∑ a4 + 2  ∑ a3b  ≥
 cyc  ( )
2 + 1  ∑ ab3 
 cyc 
cyc    
Bài 41. (Võ Qu c Bá C n)
a) a, b, c > 0 . Ch ng minh r ng

2a 2 + 3bc
∑ b2 + bc + c 2 ≥ 5
cyc

b) V i các u ki n nh trên, tìm h ng s k t t nh t cho b t ng th c


a 2 + kbc
∑ b 2 + bc + c 2 ≥ k + 1
cyc

c) i các u ki n nh trên, tìm h ng s k t t nh t cho b t ng th c


2a 2 + 3bc k ( a − b ) 2 (b − c ) 2 ( c − a ) 2
∑ b2 + bc + c 2 ≥ 5 + (a 2 + ab + b2 )(b2 + bc + c2 )(c2 + ca + a2 )
cyc

Bài 42. (Vasile Cirtoaje)


a, b, c > 0 , p ∈ R . Ch ng minh r ng

∑ (a − pb)(a − pc)(a − b)(a − c) ≥ 0


cyc

Bài 43. (Mathlinks)


a, b, c > 0 . Ch ng minh r ng

b+c ( a 2 + b 2 + c 2 )(ab + bc + ca )
∑ a ≥ 3+ abc( a + b + c)
cyc

Bài 44. (Diendantoanhoc)


a, b, c > 0 . Ch ng minh r ng

b + c ab + bc + ca
∑ a
+ 2
a + b2 + c 2
≥ 6 +1
cyc

168
Bài 45. (Mathlinks)
a, b, c > 0 . Ch ng minh r ng
ab + bc + ca 3
∑ ab + bc + ca + 3a 2 ≥ 2
cyc

Bài 46. (Mathlinks)


a, b, c > 0 và a + b + c = 3 . Ch ng minh r ng
1 1 1
2
+ 2 + 2 ≥ a2 + b2 + c2
a b c
Bài 47. (Mathlinks)
a, b, c > 0 . Ch ng minh r ng

a 2 + bc 18 a 2 + b 2 + c 2
∑ a 2 + (b + c)2 5 . (a + b + c)2

cyc

Bài 48. (Mathnfriend)


a, b, c > 0 . Ch ng minh r ng

 1 1 1  9 15 ( a − b)2 (b − c) 2 (c − a ) 2
(ab + bc + ca )  + + ≥ + .
 ( a + b) (b + c) (c + a) 2  4 4 (a + b) 2 (b + c)2 (c + a) 2
2 2

Bài 49. (Mathnfriend)


a, b, c là dài ba c nh c a m t tam giác. Ch ng minh r ng

 1 1 1  9 47 (a − b) 2 (b − c )2 (c − a ) 2
(ab + bc + ca )  + + ≥ + .
 ( a + b) (b + c) (c + a ) 2  4 4 ( a + b)2 (b + c) 2 (c + a ) 2
2 2

Bài 50. (Vasile Cirtoaje).


a) a, b, c > 0 . Ch ng minh r ng

b 2 + c 2 − 4a 2
∑ a(b + c) + 3 ≥ 0
cyc

b) V i các u ki n nh trên, tìm h ng s k t t nh t cho b t ng th c


b 2 + c 2 − ka 2 3( k − 2)
∑ a(b + c) + 2 ≥ 0
cyc

169
Bài 51. (Toán H c Tu i Tr 1998)
a, b, c > 0 . Ch ng minh r ng

1 a 2 + b2 + c 2 a b c 1 ab + bc + ca
+ ≥ + + ≥ 2− . 2
2 ab + bc + ca b + c c + a a + b 2 a + b2 + c 2
Bài 52. (Mathlinks)
a, b, c > 0 và a + b + c = 1 . Ch ng minh r ng
2
1  8(a 2 + b 2 + c 2 )2
∑  a  (1 − a)(1 − b)(1 − c)
− 2 ≥
cyc

Bài 53. (Mathlinks)


a, b, c > 0 . Ch ng minh r ng
3
    
2∑ a 3 + 9∑ a 2b +  ∑ a  ≥ 12  ∑ a 2  ∑ a 
 cyc   cyc  cyc 
cyc cyc     
Bài 54. (Mathlinks)
a, b, c > 0 và a + b + c = 1 . Ch ng minh r ng
1 1 10
+ 2 2 2≥
abc a + b + c ab + bc + ca
Bài 55. (Mathnfriend)
a, b, c > 0 . Ch ng minh r ng

a3 a+b+c
∑ 2a 2 + b2 + c 2 ≥ 4
cyc

Bài 56. (Mathlinks)


a, b, c > 0 . Ch ng minh r ng
1 2 1
∑ a 2 + 2bc ≥ ab + bc + ca + a2 + b2 + c2
cyc

Bài 57. (Mathlinks)


a) a, b, c > 0 . Ch ng minh r ng

∑a a 2 + 2bc ≥ 3( ab + bc + ca)
cyc

170
b) a, b, c > 0 . Ch ng minh r ng

∑a a 2 + 3bc ≥ 2( ab + bc + ca)
cyc

c) a, b, c > 0 . Tìm h ng s k t t nh t cho b t ng th c

∑a a 2 + kbc ≥ k + 1( ab + bc + ca )
cyc

Bài 58.
a, b, c > 0 và a + b + c = 1 . Ch ng minh r ng
5
(
ab + bc + ca + . (a + b) ab + (b + c) bc + (c + a ) ca ≤ 2
2
)
Bài 59.
a) (Mathlinks) a, b, c > 0 . Ch ng minh r ng
b+c 6
∑ 2a 2 + bc ≥ a + b + c
cyc

b) (Võ Qu c Bá C n) i các u ki n nh trên, tìm h ng s k t t nh t cho


t ng th c
 b+c  k ( a − b) 2 (b − c) 2 (c − a ) 2
(a + b + c)  ∑ 2 ≥ +
 cyc 2a + bc 
6
  (2a 2 + bc )(2b 2 + ca )(2c 2 + ab)

Bài 60. (Mathlinks)


a, b, c > 0 . Ch ng minh r ng
1 bc
∑ a(b + c) ≤ ∑ a3 (b + c)
cyc cyc

Bài 61. (Mathlinks)


a, b, c > 0 . Ch ng minh r ng
2
 a2 
4 ∑ ≥ 3(a 2 + b 2 + c 2 )
 cyc b + c 
 

171
Bài 62. (Japan 1997)
a, b, c > 0 . Ch ng minh r ng

(b + c − a ) 2 3
∑ a 2 + (b + c)2 ≥ 5
cyc

Bài 63. (USA 2003)


a, b, c > 0 . Ch ng minh r ng

(2a + b + c) 2
∑ 2a 2 + (b + c)2 ≤ 8
cyc

Bài 64. (Poland 1992)


a, b, c ∈ R . Ch ng minh r ng

(a + b − c) 2 (b + c − a ) 2 (c + a − b) 2 ≥ ( a 2 + b 2 − c 2 )(b 2 + c 2 − a 2 )(c 2 + a 2 − b 2 )
Bài 65. (Mathlinks)
a, b, c > 0 . Ch ng minh r ng
1 1 1 3
+ + ≥
11a + bc
2
11b + ca
2
11c + ab
2 2 ab + bc + ca
Bài 66.
a) (Mathinks) a, b, c > 0 và ab + bc + ca = 1 . Ch ng minh r ng

1 + a 2b 2 1 + b 2c 2 1 + c 2 a 2 5
+ + ≥
( a + b) 2 (b + c) 2 (c + a )2 2
b) (Võ Qu c Bá C n) V i các u ki n nh trên, hãy tìm h ng s k t t nh t
cho b t ng th c
1 + a 2b 2 1 + b 2c 2 1 + c 2 a 2 5 k (a − b) 2 (b − c) 2 (c − a ) 2
+ + ≥ +
( a + b) 2 (b + c) 2 (c + a) 2 2 ( a + b) 2 (b + c ) 2 (c + a )2
Bài 67. (Ph m Kim Hùng)
a, b, c > 0 . Ch ng minh r ng
a abc 5
∑ b + c + 2(a3 + b3 + c3 ) ≥ 3
cyc

172
Bài 68. (Ph m Kim Hùng)
a, b, c > 0 . Ch ng minh r ng

a 3 + b3 + c 3 54abc
+ ≥5
abc ( a + b + c )3
Bài 69. (Ph m Kim Hùng)
a, b, c > 0 . Ch ng minh r ng

a 4 + b4 + c 4 3abc 2
+ ≥ .( a 2 + b2 + c 2 )
ab + bc + ca a + b + c 3
Bài 70. (Ph m Kim Hùng)
a, b, c > 0 và ab + bc + ca = 1 , k ≥ 2 + 3 . Ch ng minh r ng
1 + bc 1 + ca 1 + ab 12
+ + ≥
ka 2 + bc kb 2 + ca kc 2 + ab k + 1
Bài 71. (Võ Qu c Bá C n)
a) a, b, c > 0 . Ch ng minh r ng
1 9
∑ 4b2 − bc + 4c 2 ≥ 7(a 2 + b 2 + c2 )
cyc

b) i các u ki n nh trên, tìm h ng s k t t nh t cho b t ng th c


1 9
∑ b2 + kbc + c 2 ≥ (k + 2)(a 2 + b2 + c2 )
cyc

Bài 72. (VMO 1991)


x ≥ y ≥ z > 0 . Ch ng minh r ng

x2 y y2 z z 2 x
+ + ≥ x2 + y 2 + z 2
z x y
Bài 73. (Mathinks)
a, b, c > 0 . Tìm h ng s k t t nh t cho b t ng th c
a k ( ab + bc + ca ) 3
∑b+c + a2 + b2 + c2
≥ +k
2
cyc

173
Bài 74.
a, b, c > 0 . Ch ng minh r ng

abc ( )
a + b + c + ( a + b + c) 2 ≥ 4 3abc (a + b + c)

Bài 75. (Ph m Kim Hùng)


a, b, c > 0 . Tìm h ng s k t t nh t cho b t ng th c

a 3 + b3 + c 3 k (ab + bc + ca ) 3 k
+ ≥ +
( a + b)(b + c )(c + a ) (a + b + c) 2 8 3
Bài 76. (Vasile Cirtoaje)
a, b, c, k > 0 . Ch ng minh r ng
ab + (k − 3)bc + ca 3( k − 1)
∑ (b − c ) 2 + kbc

k
cyc

Bài 77. (Võ Qu c Bá C n)


Ch ng minh r ng v i m i a, b, c > 0, k ≥ 1 ta luôn có
a (b + c ) 6
∑ b2 + kbc + c 2 ≥ k + 2
cyc

Bài 78. (Võ Qu c Bá C n)


a, b, c > 0 . Tìm h ng s k t t nh t cho b t ng th c
a (2 a + b + c) 12
∑ ka 2 + bc

k +1
cyc

Bài 79. (Toán H c Tu i Tr 2002)


a, b, c > 0 . Ch ng minh r ng

(a + b + c)3 ( a + b − c )( a − b + c )(b + c − a ) ≤ 27 a 2b2 c 2


Bài 80. (Manlio Marangelli)
a, b, c > 0 . Ch ng minh r ng

3( a 2b + b 2 c + c 2 a)( ab 2 + bc 2 + ca 2 ) ≥ abc(a + b + c)3

174
Bài 81. (Võ Qu c Bá C n)
a, b, c > 0 . Ch ng minh r ng

b2 + c 2 − a2 c 2 + a 2 − b2 a 2 + b2 − c2
+ 2 + 2 ≥1
a 2 + 2bc b + 2ca c + 2ab
Bài 82. (Toán H c Tu i Tr 2005)
a, b, c > 0 . Ch ng minh r ng
1 1 1 b+c c+a a+b
+ + ≥ 2 + 2 + 2
a b c a + bc b + ca c + ab
Bài 83. (Võ Qu c Bá C n)
a, b, c > 0 . Tìm h ng s k t t nh t cho b t ng th c
( a + b)(b + c)(c + a) kabc 8 k
+ 3 3 3≥ +
( a + b + c)( ab + bc + ca) a + b + c 9 3
Bài 84. (Võ Qu c Bá C n)
a, b, c > 0 . Tìm h ng s k t t nh t cho b t ng th c
a b c kabc 3 k
+ + + 3 3 3≥ +
b+c c+a a+b a +b +c 2 3
Bài 85. (Mathlinks)
a, b, c > 0 . Ch ng minh r ng
1 1
∑ 5a 2 − ab + 5b2 ≥ a2 + b2 + c 2
cyc

175
T TÌM TÒI NH V B T NG TH C
Voõ Quoác Baù Caån

Baát ñaúng thöùc laø moät trong nhöõng lónh vöïc hay, khoù vaø loâi cuoán nhaát cuûa toaùn
hoïc. Baïn coù theå deã daøng kieåm chöùng ñöôïc ñieàu naøy qua caùc trang web toaùn hoïc,
trong forum baát ñaúng thöùc cuûa caùc trang web naøy, luoân chieám soá löôïng baøi vieát
nhieàu nhaát. Baøi vieát sau ñaây, toâi xin giôùi thieäu moät phöông phaùp hay, khaù hieäu
quaû ñeå chöùng minh baát ñaúng thöùc ñoái xöùng ba bieán maø toâi tình côø tìm ñöôïc qua
vieäc giaûi toaùn. Do trình ñoä coøn haïn heïp vaø ñaây chæ laø moät tìm toøi nhoû cuûa toâi neân
khoù loøng traùnh khoûi nhöõng sai soùt, mong baïn ñoïc thoâng caûm.
Phöông phaùp naøy raát ñôn giaûn nhöng khaù hieäu quaû vaø noù ñaõ giuùp toâi giaûi ñöôïc
khaù nhieàu baøi toaùn khoù maø nhöõng phöông phaùp maïnh khaùc nhö S.O.S, doàn
bieán… ñaønh baát löïc.
Xin ñöôïc noùi sô qua veà cô sôû cuûa phöông phaùp naøy, noù ñöôïc xaây döïng hoaøn toaøn
döïa treân 2 Boå ñeà raát cô baûn sau
* Boå ñeà 1. (baát ñaúng thöùc Schur) ∀a, b, c ≥ 0 thì

4 pq − p 3
r≥
9
trong ñoù p = a + b + c, q = ab + bc + ca, r = abc.

* Boå ñeà 2. ∀a, b, c ∈ R thì toàn taïi caùc soá thöïc x0 , y0 , x1, y1 sao cho

p = a + b + c = 2 x0 + y0 = 2 x1 + y1
q = ab + bc + ca = x02 + 2 x0 y0 = x12 + 2 x1 y1
x02 y0 ≤ r = abc ≤ x12 y1

Ngoaøi ra, neáu a, b, c ≥ 0 thì x0 , x1, y1 ≥ 0 . Trong ñoù

+ Neáu p 2 ≥ 4q thì y0 ≤ 0
+ Neáu p 2 ≤ 4q thì y0 ≥ 0
176
Caùc keát quaû treân chöùng minh töông ñoái ñôn giaûn, caùc baïn neân töï chöùng minh laáy,
xem nhö laø baøi taäp.
Ñeå hieåu roõ hôn tính hieäu quaû cuûa phöông phaùp naøy, caùc baïn haõy cuøng theo doõi
caùc ví duï sau
Ví duï 1. (Vasile Cirtoaje)

Cho a, b, c > 0 thoûa a 2 + b 2 + c 2 = a + b + c. Chöùng minh raèng

ab + bc + ca ≥ a 2b 2 + b 2c 2 + c 2 a 2
Lôøi giaûi.
Ta coù baát ñaúng thöùc caàn chöùng minh töông ñöông vôùi
2
 a 2 + b2 + c2 
(ab + bc + ca ).  ≥ a b +b c +c a
2 2 2 2 2 2

 a+b+c 
Do caû 2 veá cuûa baát ñaúng thöùc naøy ñoàng baäc neân khoâng maát tính toång quaùt, ta coù
1
theå giaû söû a + b + c = 1. Ñaët q = ab + bc + ca, r = abc ⇒ 0 ≤ q ≤ . Khi ñoù, baát ñaúng
3
thöùc caàn chöùng minh trôû thaønh

q(1 − 2 q) 2 ≥ q 2 − 2r
f (r ) = 18r + 9q (4q − 1)( q − 1) ≥ 0 (*)

* Tröôøng hôïp 1. 4q ≤ 1 thì (*) hieån nhieân ñuùng.

* Tröôøng hôïp 2. 4q ≥ 1 , theá thì theo Boå ñeà 1, ta coù

4q − 1
r≥ ≥0
9
Do ñoù
f (r ) = 18r + 9q (4q − 1)( q − 1) ≥ 2(4q − 1) + 9q(4q − 1)(q − 1)
= (4q − 1)(2 − 3q )(1 − 3q) ≥ 0
⇒ (*) ñuùng.
⇒ ñpcm.

177
Ví duï 2. (Vasile Cirtoaje)
Cho a, b, c > 0 thoûa a + b + c = 3. Chöùng minh raèng

a b c 3
P (a, b, c ) = + + ≥
a + bc b + ca c + ab 2
Lôøi giaûi.
Ñaët q = ab + bc + ca, r = abc ⇒ 0 ≤ q ≤ 3. Ta coù baát ñaúng thöùc töông ñöông vôùi:

3r 2 + 2r (6 − q) − q 2 ≤ 0

Töø Boå ñeà 1, ta coù

3r 2 + 2r (6 − q ) − q 2 ≤ 3( x12 y1 )2 + ( x12 y1 )(6 − q ) − q 2

Do ñoù, ñeå chöùng minh baát ñaúng thöùc ñaõ cho, ta chæ caàn chöùng minh

3( x12 y1 ) 2 + ( x12 y1 )(6 − q ) − q 2 ≤ 0


3
⇔ P ( x1 , x1 , y1 ) ≥
2
2 y1 3
⇔ + ≥
y1 + 1 y1 + x12 2
2 y1 3
⇔ + ≥
y1 + 1  3 − y1 
2
2
y1 +  
 2 
2 4 y1 3
⇔ + 2 ≥
y1 + 1 y1 − 2 y1 + 9 2
⇔ ( y1 − 1) 2 (3 − y1 ) ≥ 0 (ñuùng)
⇒ ñpcm.

Ñaúng thöùc xaûy ra khi vaø chæ khi a = b = c hoaëc a = 3, b = c → 0 vaø caùc hoaùn vò.

178
Ví duï 3. (Phaïm Kim Huøng)
Cho a, b, c ≥ 0 thoûa a + b + c = 1. Chöùng minh raèng

ab + bc + ca ≥ 8( a 2b 2 + b 2c 2 + c 2 a 2 )( a 2 + b 2 + c 2 + 16abc )

Lôøi giaûi.

 q, r ≥ 0

Ñaët q = ab + bc + ca, r = abc ⇒  1 theá thì theo baát ñaúng thöùc Schur, ta coù
 q ≤
3
4q − 1
r≥ . Töø caùch ñaët, ta coùù
9
a 2b 2 + b 2c 2 + c 2 a 2 = q 2 − 2r
a 2 + b 2 + c 2 = 1 − 2q

Do ñoù, baát ñaúng thöùc caàn chöùng minh trôû thaønh

q ≥ 8( q 2 − 2r )(16r + 1 − 2q)
⇔ f ( r ) = 8(2r − q 2 )(16 r + 1 − 2q ) + q ≥ 0

Ta coù f / ( r ) = 6(32r − (4q − 1)(2q + 1))

Coù 2 tröôøng hôïp xaûy ra

* Tröôøng hôïp 1. 1 ≥ 4q ⇒ f / (r ) ≥ 0 ⇒ f (r ) laø haøm ñoàng bieán ∀r ≥ 0.

4q − 1
* Tröôøng hôïp 2. 4q ≥ 1 ⇒ r ≥ ≥ 0 . Do ñoù
9

 32(4q − 1) 
f / ( r ) = 6(32r − (4q − 1)(2q + 1)) ≥ 6  − (4q − 1)(2q + 1) 
 9 
2(4q − 1)(23 − 18q )
=
3
≥0

⇒ f (r ) laø haøm ñoàng bieán ∀r ≥ 0.

Toùm laïi, trong moïi tröôøng hôïp, ta luoân coù f (r ) laø haøm ñoàng bieán ∀r ≥ 0. Do ñoù

f (r ) ≥ f (0) = q (4q − 1)2 ≥ 0


⇒ ñpcm.
179
* Chuù yù.
Caùc baïn neân chuù yù raèng phöông phaùp naøy chæ ñaëc bieät coù hieäu quaû ñoái vôùi nhöõng
baát ñaúng thöùc maø daáu baèng xaûy ra khi a = b = c hoaëc trong ba soá a, b, c coù moät soá

baèng 0, hai soá coøn laïi baèng nhau.

180
BAØI TAÄP

Baøi 1. (Iran 1996)


Cho a, b, c > 0. Chöùng minh raèng

 1 1 1  9
(ab + bc + ca )  + + ≥
 ( a + b) (b + c) (c + a ) 2  4
2 2

Baøi 2. (Phaïm Kim Huøng)


Cho a, b, c laø caùc soá thöïc döông thoûa maõn abc = 1 . Chöùng minh raèng

64(1 + a 3 )(1 + b3 )(1 + c 3 ) ≤ ( a + b + c )6

Baøi 3.

Cho a, b, c ∈ R thoûa a 2 + b 2 + c 2 = 9. Tìm giaù trò lôùn nhaát cuûa bieåu thöùc
P = 2(a + b + c) − abc

Baøi 4.
Cho a, b, c > 0. Chöùng minh raèng

abc 2 ab + bc + ca
a) + ≥ 2
a + b + c 3 a + b2 + c2
3 3 3

2
a 3 + b3 + c3 1  a 2 + b 2 + c 2 
b) + ≥ 
4abc 4  ab + bc + ca 

Baøi 5.
Cho x, y , z > 0. Chöùng minh raèng

x4 + y 4 + z 4 2( xy + yz + zx)
+ ≥ 1+ 2
x y +y z +z x
2 2 2 2 2 2
x2 + y2 + z 2

Baøi 6. (Vietnam TST 1996)


Cho a, b, c ∈ R. Chöùng minh raèng
4
(a + b) 4 + (b + c )4 + (c + a )4 ≥ .(a 4 + b 4 + c 4 )
7

181
Baøi 7.
Cho a, b, c > 0. Chöùng minh raèng

1 1 1 9  1 1 1 
+ + + ≥ 4 + + 
a b c a+b+c  a+b b+c c+a
Baøi 8.
Cho a, b, c > 0 thoûa ab + bc + ca = 3. Chöùng minh raèng

1 1 1
+ + ≤1
2+a 2
2+b 2
2 + c2
Baøi 9. (Vasile Cirtoaje)
Cho a, b, c > 0 thoûa ab + bc + ca = 3. Chöùng minh raèng

1 1 1 3
+ + ≥
1+ a 1+ b 1+ c
2 2 2
2
Baøi 10. (Kvant 1993)
Cho a, b, c, d > 0 thoûa a + b + c = 1.Chöùng minh raèng

1 1 d 
a3 + b3 + c 3 + abcd ≥ min  , + 
 4 9 27 
Baøi 11. (Mihai Piticari, Dan Popescu)
Cho a, b, c > 0 thoûa a + b + c = 1.Chöùng minh raèng

5( a 2 + b 2 + c 2 ) ≤ 6(a 3 + b3 + c3 ) + 1

Baøi 12.
Cho a, b, c > 0. Chöùng minh raèng

a 2 + b2 + c 2 8abc
+ ≥2
ab + bc + ca ( a + b)(b + c)(c + a )

Baøi 13.
Cho x, y , z > 0 thoûa x + y + z = 1. Chöùng minh raèng

xy yz zx 1
+ + ≤
xy + yz yz + zx zx + xy 2
182
Baøi 14. (Phaïm Vaên Thuaän)
Cho a, b, c > 0. Chöùng minh raèng

( a + b + c) 2 1  a 3 + b3 + c 3 a 2 + b 2 + c 2 
+ . − ≥4
a 2 + b2 + c2 2  abc ab + bc + ca 

Baøi 15. (Toaùn Hoïc Tuoåi Treû 2002)

Cho a, b, c ∈ R thoûa a 2 + b 2 + c 2 = 1. Tìm giaù trò lôùn nhaát cuûa bieåu thöùc:
P = 3(a + b + c ) − 22abc

Baøi 16. (Voõ Quoác Baù Caån)


Cho a, b, c > 0 thoûa a + b + c = 3. Chöùng minh raèng

(1 + a k +1 )(1 + b k +1 )(1 + c k +1 ) ≥ (1 + a k )(1 + b k )(1 + c k ) ∀k ≥ 1

Baøi 17. (Vasile Cirtoaje)

Cho a, b, c ≥ 0 thoûa a 2 + b2 + c 2 = 3. Chöùng minh raèng


12 + 9abc ≥ 7( ab + bc + ca )

Baøi 18. (Vasile Cirtoaje)


Cho a, b, c ≥ 0 thoûa ab + bc + ca = 3. Chöùng minh raèng

a 3 + b3 + c3 + 7 abc ≥ 10
Baøi 19.
Cho a, b, c ≥ 0 thoûa a + b + c = 3. Chöùng minh raèng

1 1 1 3
+ + ≤
6 − ab 6 − bc 6 − ca 5
Baøi 20. (Vasile Cirtoaje)

Cho a, b, c ≥ 0 thoûa a 2 + b2 + c 2 = 3. Chöùng minh raèng

3
5( a + b + c ) + ≥ 18
abc

183
HAØM LOÀI (LOÕM),
HAØM NÖÛA LOÀI NÖÛA LOÕM VAØ BAÁT ÑAÚNG THÖÙC
Voõ Quoác Baù Caån

I. Caùc ñònh nghóa.


1. Ñònh nghóa haøm loài (loõm).
Haøm soá f ( x) ñöôïc goïi laø loài treân taäp [ a, b] ⊂ R neáu vôùi moïi x, y ∈ [ a, b] vaø vôùi

moïi caëp soá khoâng aâm α , β coù toång baèng 1, ta ñeàu coù
f (α x + β y) ≥ α f ( x) + β f ( y)

Haøm soá f ( x) ñöôïc goïi laø loõm treân taäp [ a, b] ⊂ R neáu vôùi moïi x, y ∈ [ a, b] vaø vôùi

moïi caëp soá khoâng aâm α , β coù toång baèng 1, ta ñeàu coù
f (α x + β y) ≤ α f ( x) + β f ( y)

Keát quaû sau ñaây chuùng ta thöôøng duøng ñeå nhaän bieát moät haøm laø loài hay loõm
Neáu f ( x) khaû vi baäc hai treân [ a, b] thì f ( x) loài (loõm) treân [ a, b] khi vaø chæ khi

f // ( x ) ≤ 0 ( f // ( x ) ≥ 0) ∀x ∈ [a, b] .

2. Ñònh nghóa haøm nöûa loài nöûa loõm.


Haøm soá f ( x) ñöôïc goïi laø nöûa loài nöûa loõm treân [ a, b] ⊂ R neáu toàn taïi duy nhaát

haèng soá c ( a < c < b) sao cho f ( x) loài treân [ a, c] vaø loõm treân [c, b] (hoaëc ngöôïc

laïi).
II. Moät soá tính chaát.

1. Tính chaát 1.

b ≥ x ≥ z ≥ y ≥ a
Neáu f ( x) laø moät haøm loõm treân [ a, b] thì vôùi moïi  ta coù
x + y − z ≥ a
f ( x) + f ( y ) ≥ f ( z ) + f ( x + y − z )

Chöùng minh.

184
Ta coù ∀h thoûa 0 ≤ h ≤ x − y thì toàn taïi α ∈ [0,1] sao cho h = α ( x − y)

Do ñoù x − h = (1 − α ) x + α y ∈ [a, b] neân theo ñònh nghóa haøm loõm, ta coù


f ( x − h) = f ((1 − α ) x + α y) ≤ (1 − α ) f ( x) + α f ( y )

Töông töï, ta coù y + h = α x + (1 − α ) y ∈ [a, b] neân theo ñònh nghóa haøm loõm, ta

cuõng coù
f ( y + h) = f (α x + (1 − α ) y ) ≤ α f ( x) + (1 − α ) f ( y )

Do ñoù
f ( x − h) + f ( y + h) ≤ f ( x) + f ( y ) (*)

Roõ raøng ta coù 0 ≤ x − z ≤ x − y neân aùp duïng (*) vôùi h = x − z, ta ñöôïc


f ( x) + f ( y ) ≥ f ( z ) + f ( x + y − z )

Tính chaát 1 ñöôïc chöùng minh hoaøn toaøn.


Töø tính chaát 1 ta suy ra ñöôïc tính chaát 2 nhö sau

2. Tính chaát 2.
Neáu f ( x) laø moät haøm loõm treân [ a, b] thì vôùi moïi x1 , x2 ,..., xn ∈ [ a, b] thoûa maõn

x1 + x2 + ... + xn − (n − 1)a ≤ b thì ta coù

f ( x1 ) + f ( x2 ) + ... + f ( xn ) ≤ ( n − 1) f ( a) + f ( x1 + x2 + ... + xn − ( n − 1) a )

Chöùng minh.
Ta chöùng minh baèng quy naïp theo n .
Deã thaáy khaúng ñònh ñuùng cho 1 bieán soá.
Giaû söû khaúng ñònh ñuùng cho n bieán soá, töùc laø ta coù
f ( x1 ) + f ( x2 ) + ... + f ( xn ) ≤ ( n − 1) f ( a) + f ( x1 + x2 + ... + xn − ( n − 1) a )

Ta seõ chöùng minh khaúng ñònh ñuùng cho n + 1 bieán soá, töùc laø chöùng minh
f ( x1 ) + f ( x2 ) + ... + f ( xn +1 ) ≤ nf (a ) + f ( x1 + x2 + ... + xn+1 − na )

Khoâng maát tính toång quaùt, ta coù theå giaû söû xn+1 = max{x1 , x2 ,..., xn +1} . AÙp duïng giaû

thieát quy naïp, ta coù

185
f ( x1 ) + f ( x2 ) + ... + f ( xn ) ≤ ( n − 1) f ( a) + f ( x1 + x2 + ... + xn − ( n − 1) a )

Do ñoù, ñeå chöùng minh khaúng ñònh ñuùng cho n + 1 bieán soá, ta chæ caàn chöùng minh
f ( x1 + x2 + ... + xn − (n − 1)a ) + f ( xn+1 ) ≤ f (a ) + f ( x1 + x2 + ... + xn +1 − na)

Do xn+1 = max{x1 , x2 ,..., xn +1} vaø b ≥ xi ≥ a ∀i = 1, n neân ta coù

b ≥ x1 + x2 + ... + xn+1 − na ≥ xn+1 ≥ a

Do ñoù theo tính chaát 1, ta coù


f (a ) + f ( x1 + x2 + ... + xn +1 − na) ≥
≥ f (( x1 + x2 + ... + xn+1 − na ) + a − xn +1 ) + f ( xn+1 )
= f ( x1 + x2 + ... + xn − ( n − 1) a ) + f ( xn+1 )

Vaäy khaúng ñònh ñuùng cho n + 1 bieán soá. Theo nguyeân lyù quy naïp, khaúng ñònh
ñuùng vôùi moïi n ≥ 1.
Tính chaát 2 ñöôïc chöùng minh.

3. Tính chaát 2’.


 x , x ,..., xn ∈ [a, b]
Neáu f ( x) laø moät haøm loài treân [ a, b] thì vôùi moïi  1 2 thì
 x1 + x 2 + ... + x n − ( n − 1) a ≤ b

ta coù
f ( x1 ) + f ( x2 ) + ... + f ( xn ) ≥ ( n − 1) f ( a) + f ( x1 + x2 + ... + xn − ( n − 1) a )

4. Tính chaát 3. (Heä quaû cuûa ñònh lyù Larange)


+ Neáu f ( x) khaû vi baäc 2 treân [ a, b] vaø loõm treân [ a, b] thì vôùi moïi x, x0 ∈ [ a, b] ta

coù

f ( x) ≥ f / ( x0 )( x − x0 ) + f ( x0 )

+ Neáu f ( x) khaû vi baäc 2 treân [ a, b] vaø loài treân [ a, b] thì vôùi moïi x, x0 ∈ [ a, b] ta

coù

f ( x) ≤ f / ( x0 )( x − x0 ) + f ( x0 )

Töø tính chaát treân, ta suy ra ñöôïc baát ñaúng thöùc Jensen noåi tieáng. Caùc baïn haõy thöû
chöùng minh laïi baèng caùch söû duïng tính chaát 3 xem nhö laø baøi taäp.
186
III. ÖÙng duïng vaøo baát ñaúng thöùc.
Caùc ñònh lyù sau ñaây coù theå xem nhö laø moät phöông phaùp chöùng minh baát ñaúng
thöùc khaù hieäu quaû. (baïn cuõng neân töï chöùng minh laáy xem nhö laø baøi taäp, löu yù laø
ñeå chöùng minh chuùng, ta chæ caàn duøng caùc tính chaát treân laø ñuû).
1. Ñònh lyù 1.
x1 , x2 ,..., xn laø n soá thöïc thoûa maõn

i) x1 ≤ x2 ≤ ... ≤ xn
ii) xi ∈ [a, b] ∀i = 1, n
iii) x1 + x2 + ... + xn = C (C laø haèng soá)

vaø f laø moät haøm treân [ a, b] thoûa maõn f loài treân [ a, c] vaø loõm treân [c, b] .

Ñaët F = f ( x1 ) + f ( x2 ) + ... + f ( xn )

Khi ñoù
F ñaït min khi x1 = x2 = x3 = ... = xk −1 = a, xk +1 = ... = xn ∈ [a, b] (k = 1, 2,..., n)

F ñaït max khi x1 = x2 = ... = xk −1 ∈ [ a, b], xk +1 = xk + 2 = ... = xn = b ( k = 1, 2,..., n)

2. Ñònh lyù 1’.


x1 , x2 ,..., xn laø n soá thöïc thoûa maõn

i) x1 ≤ x2 ≤ ... ≤ xn
ii) xi ∈ [a, b] ∀i = 1, n
iii) x1 + x2 + ... + xn = C (C laø haèng soá)

vaø f laø moät haøm treân [ a, b] thoûa maõn f loõm treân [ a, c] vaø loài treân [c, b] .

Ñaët F = f ( x1 ) + f ( x2 ) + ... + f ( xn )

Khi ñoù
F ñaït max khi x1 = x2 = x3 = ... = xk −1 = a, xk +1 = ... = xn ∈ [a, b] ( k = 1, 2,..., n)

F ñaït min khi x1 = x2 = ... = xk −1 ∈ [a, b], xk +1 = xk + 2 = ... = xn = b (k = 1, 2,..., n)

187
3. Ñònh lyù 2.
x1 , x2 ,..., xn laø n soá thöïc thoûa maõn

i) x1 ≤ x2 ≤ ... ≤ xn
ii) xi ∈ ( −∞, +∞) ∀i = 1, n
iii) x1 + x2 + ... + xn = C (C laø haèng soá)

vaø f laø moät haøm treân [ a, b] thoûa maõn f loài treân (−∞, c] vaø loõm treân [c, +∞) .

Ñaët F = f ( x1 ) + f ( x2 ) + ... + f ( xn )

Khi ñoù,
F ñaït min khi x1 ≤ x2 = x3 = ... = xn

F ñaït max khi x1 = x2 = ... = xn−1 ≤ xn .

4. Ñònh lyù 2’.


x1 , x2 ,..., xn laø n soá thöïc thoûa maõn

i) x1 ≤ x2 ≤ ... ≤ xn
ii) xi ∈ ( −∞, +∞) ∀i = 1, n
iii) x1 + x2 + ... + xn = C (C laø haèng soá)

vaø f laø moät haøm treân [ a, b] thoûa maõn f loõm treân (−∞, c] vaø loài treân [c, +∞) .

Ñaët F = f ( x1 ) + f ( x2 ) + ... + f ( xn )

Khi ñoù,
F ñaït max khi x1 ≤ x2 = x3 = ... = xn

F ñaït min khi x1 = x2 = ... = xn−1 ≤ xn .

IV. Moät soá aùp duïng.

Ví duï 1. (VMEO 2004)


Cho tam giaùc nhoïn ABC. Tìm giaù trò nhoû nhaát cuûa bieåu thöùc
P = tgA + 2tgB + 5tgC

Lôøi giaûi.

188
 π
Xeùt haøm soá f ( x) = tgx vôùi x ∈  0,  .
 2
Ta coù
f / ( x) = tg 2 x + 1
f // ( x) = 2tgx( tg 2 x + 1) > 0

 π
⇒ f ( x) laø haøm loõm treân  0,  .
 2
Do ñoù, theo tính chaát haøm loõm, ta coù

f ( A) ≥ f (arctg3) + f / (arctg3)( A − arctg3) = 3 + 10( A − arctg3)

Töông töï, ta coù

f ( B) ≥ f (arctg 2) + f / (arctg 2)( B − arctg 2) = 2 + 5( B − arctg 2)


⇒ 2 f ( B ) ≥ 4 + 10( B − arctg 2)

f (C ) ≥ f (arctg1) + f / (arctg1)(C − arctg1) = 1 + 2(C − arctg1)


⇒ 5 f (C ) ≥ 5 + 10(C − arctg1)

Do ñoù
P = f ( A) + 2 f ( B ) + 5 f (C )
≥ 12 + 10( A + B + C − arctg3 − arctg 2 − arctg1)
= 12 ( vì A + B + C = arctg3 + arctg 2 + arctg1 = π )


 A = arctg3

Ñaúng thöùc xaûy ra khi vaø chæ khi  B = arctg 2 .
 π
C =
 4
Vaäy
min P = 12.

189
Ví duï 2.
Cho caùc soá döông a, b, c thoûa 21ab + 2bc + 8ca ≤ 12. Tìm giaù trò nhoû nhaát cuûa bieåu

thöùc
1 2 3
P= + +
a b c
Lôøi giaûi.
1 2 3
Ñaët x = , y = , z = , baøi toaùn chuyeån veà
a b c
x, y , z > 0 thoûa 2 x + 4 y + 7 z ≤ 2 xyz. Tìm giaù trò nhoû nhaát cuûa bieåu thöùc
P= x+ y+z

Khoâng maát tính toång quaùt, ta chæ caàn xeùt tröôøng hôïp 2 x + 4 y + 7 z = 2 xyz laø ñuû (taïi

7 2 7
sao?). Ñaët x = 7m, y = .n, z = . p thì ta coù m + n + p = mnp . Do ñoù, toàn
2 7
taïi tam giaùc nhoïn ABC sao cho m = tgA, n = tgB, p = tgC. Khi ñoù, ta coù

7
P= .(14tgA + 7 tgB + 4tgC )
14
 π
Xeùt haøm soá f ( x) = tgx vôùi x ∈  0,  .
 2
Ta coù
f / ( x) = tg 2 x + 1
f // ( x) = 2tgx( tg 2 x + 1) > 0

 π
⇒ f ( x) laø haøm loõm treân  0,  .
 2
Do ñoù, theo tính chaát haøm loõm, ta coù
 3 7  3 7  3 7
f ( A) ≥ f  arctg  + f  arctg
/
 A − arctg 
 7   7  7 

190
3 7 16  3 7
= + . A − arctg 
7 7  7 
 3 7
⇒ 14 f ( A) ≥ 6 7 + 32  A − arctg 
 7 

Töông töï, ta coù


 5 7 / 5 7  5 7
f ( B) ≥ f  arctg  + f  arctg  B − arctg 
 7   7  7 
5 7 32  5 7
= + . B − arctg 
7 7  7 
 5 7
⇒ 7 f ( B ) ≥ 5 7 + 32  B − arctg 
 7 

( ) ( )(
f (C ) ≥ f arctg 7 + f / arctg 7 C − arctg 7 )
(
= 7 + 8 C − arctg 7 )
(
⇒ 4 f (C ) ≥ 4 7 + 32 C − arctg 7 )
Do ñoù

7
P= .(14 f ( A) + 7 f ( B) + 4 f (C ))
14
7   3 7 5 7 
≥ .15 7 + 32  A + B + C − arctg − arctg − arctg 7  
14   7 7 

15 3 7 5 7
= (vì A + B + C = arctg + arctg + arctg 7 = π )
2 7 7
 3 7  1
 A = arctg a =
 7 m = 3  3
 
 5 7  5  4
Ñaúng thöùc xaûy ra khi vaø chæ khi  B = arctg ⇔  n = ⇔ b = .
 7  2  5
C = arctg 7  p = 2  3
 c = 2
 

Vaäy
191
15
min P = .
2

Ví duï 3. (Phaïm Kim Huøng)


Cho a1 , a2 ,..., an > 0 thoûa a1a2 ...an = 1 . chöùng minh raèng vôùi moïi k > 0 thì

1 1 1  n
+ + ... + ≥ min 1, k 
(1 + a1 ) k
(1 + a2 ) k
(1 + an ) k
 2 

Chöùng minh.
Neáu n = 1 thì baát ñaúng thöùc ñaõ cho hieån nhieân ñuùng.
Neáu n = 2
+ Neáu 0 < k < 1 thì ta coù
1 1 1 1 1 1
+ ≥ + = + =1
(1 + a1 ) k
(1 + a2 ) 1 + a1 1 + a2 1 + a1 1 + 1
k

a1

+ Neáu k ≥ 1 thì ta coù

1 1 a1k + 1 1
+ = ≥ k −1 (theo baát ñaúng thöùc Holder)
(1 + a1 ) k
(1 + a2 ) k
( a1 + 1) k
2

Xeùt n ≥ 3
n
Ta chöùng minh baát ñaúng thöùc ñuùng cho giaù trò tôùi haïn 1 = ⇔ k = log2 n .
2k
Do n ≥ 3 neân n − 1 > k > 1 .
Khi ñoù
+ ∀m ≥ k , ta coù
m m
n
1 1  n 1 k 1  n k n
∑ (1 + a )m m −1  ∑ (1 + a )k  ≥ m −1 . 2k  = 2m
≥ .
nk  
i =1 i i =1 i
nk
+ ∀m ≤ k , ta coù
k
 n 1 m n
1 n
1
∑ m 
≥ ∑ ≥ 1 ⇒ ∑ ≥1
 i =1 (1 + ai )  i =1 (1 + ai ) k
i =1 (1 + ai ) m

192
Khoâng maát tính toång quaùt giaû söû 0 < a1 ≤ a2 ≤ ... ≤ an .

 x1 ≤ x2 ≤ ... ≤ xn
Ñaët x1 = ln a1 , x2 = ln a2 ,..., xn = ln an thì 
 x1 + x2 + ... + xn = 0 (do a1a2 ...an = 1)
1
Xeùt haøm soá f ( x) =
(e + 1) k
x

Ta coù
ke x .(ke x − 1)
f ( x) =
//

(e x + 1) k + 2
f // ( x ) = 0 ⇔ x = − ln k

Töø ñoù, ta coù f loài treân (−∞, − ln k ] vaø loõm treân [ − ln k , +∞)

⇒ Theo Ñònh lyù 2, ta coù


n n
1 1
P=∑ = ∑ ñaït min khi x1 ≤ x2 = x3 = ... = xn
i =1 (1 + ai ) i =1 (1 + e )
k xi k

 n −1 1 
⇒ min P ≥ min  t + − ( n−1) t k 
(t ≥ 0)
 (e + 1) + 1) 
k
(e
 n −1 x ( n−1) k 
= min  + n −1 k 
( x = et ≥ 1) (1)
 ( x + 1) ( x + 1) 
k

n −1 x ( n−1) k
Tieáp theo, ta seõ tìm min cuûa haøm soá g ( x) = + vôùi x ≥ 1
( x + 1) k ( x n−1 + 1) k

 x ( n−1) k −1 1 
Ta coù g / ( x) = ( n − 1) k  n−1 k +1
− k +1 
 ( x + 1) ( x + 1) 

g / ( x) = 0 ⇔ x ( n −1) k −1.( x + 1) k +1 = ( x n −1 + 1) k +1
( n −1) k −1
⇔x k +1
.( x + 1) = x n −1 + 1 (2)
1
Ñaët t = x k +1 ⇒ t ≥ 1 . Khi ñoù, phöông trình (2) trô ûthaønh
t ( n−1) k −1.(t k +1 + 1) = t ( n−1)( k +1) + 1
⇔ t ( n−1)( k +1) − t nk − t ( n−1) k −1 + 1 = 0

193
Xeùt haøm soá h(t ) = t ( n −1)( k +1) − t nk − t ( n −1) k −1 + 1 vôùi t ≥ 1

Ta coù h / (t ) = t ( n −1) k − 2 .(( n − 1)( k + 1)t n − nkt k +1 − (n − 1)k + 1)

h / (t ) = 0 ⇔ (n − 1)(k + 1)t n − nkt k +1 − (n − 1)k + 1 = 0

Xeùt tieáp haøm soá m(t ) = (n − 1)(k + 1)t n − nkt k +1 − (n − 1) k + 1 vôùi t ≥ 1

Ta coù m / (t ) = n( k + 1)t k ((n − 1)t n − k −1 − k )

Chuù yù raèng n − 1 > k neân m / (t ) ≥ n(k + 1)t k ((n − 1) − k ) > 0

⇒ m(t ) laø haøm ñoàng bieán treân [1, +∞)

Ta laïi coù m(1) = (n − 1)(k + 1) − nk − ( n − 1)k + 1 = n(1 − k ) < 0, lim m(t ) = +∞


t →+∞

Neân phöông trình m(t ) = 0 coù nghieäm duy nhaát t0 > 1

⇒ Phöông trình h / (t ) = 0 coù nghieäm duy nhaát t0 > 1

Baûng bieán thieân cuûa h(t )

t 1 t0 +∞

h / (t ) _ 0 +
0 +∞
h(t )

Caên cöù vaøo baûng bieán thieân, ta coù


h(t ) = 0 coù 2 nghieäm phaân bieät laø 1 vaø t1 > t0 > 1

Do ñoù g / ( x) = 0 coù 2 nghieäm phaân bieät laø 1 vaø t1k +1 > 1 .

Baûng bieán thieân cuûa g ( x)


x 1 t1k +1 +∞
g / ( x) + 0 _

g ( x)
1 1

Caên cöù vaøo baûng bieán thieân, ta suy ra

194
{ }
g ( x ) ≥ min g (1), lim g ( x) = 1 ∀x ≥ 1
x →∞
(3)

Töø (1) vaø (3), ta suy ra ñpcm.


Ví duï 4. (Vasile Cirtoaje)
2n − 1
Cho n ≥ 3, n ∈ N , 0 < k ≤ vaø a1 , a2 ,..., an > 0 thoûa a1a2 ...an = 1 . Chöùng minh
(n − 1) 2

raèng
1 1 1 n
+ + ... + ≤
1 + ka1 1 + ka2 1 + kan k +1

Lôøi giaûi.
2n − 1
Ñaët yi = kai (i = 1, n) ⇒ y1 y2 ... yn = k n vôùi k = n y1 y2 ... yn ≤ . Khi ñoù, baát
(n − 1) 2

ñaúng thöùc caàn chöùng minh trôû thaønh


1 1 1 n
+ + ... + ≤
1 + y1 1 + y2 1 + yn k +1

Khoâng maát tính toång quaùt giaû söû 0 < y1 ≤ y2 ≤ ... ≤ yn .

 x ≤ x2 ≤ ... ≤ xn
Ñaët x1 = ln y1, x2 = ln y2 ,..., xn = ln yn thì  1
 x1 + x2 + ... + xn = n ln k (do a1a2 ...an = 1)
1
Xeùt haøm soá f ( x) =
(e + 1)1 2
x

e x .(e x − 2) //
Ta coù f ( x) =
//
; f ( x ) = 0 ⇔ x = ln 2
4(e x + 1)5 2

Töø ñoù, ta coù f loài treân (−∞,ln 2] vaø loõm treân [ln 2, +∞)

⇒ Theo Ñònh lyù 2, ta coù


n n
1 1
P=∑ = ∑ ñaït max khi x1 = x2 = ... = xn −1 ≤ xn .
i =1 (1 + y i )12
i =1 (1 + e xi 1 2
)

 n −1 1 
⇒ max P ≤ max  +  (t ≤ ln k )
 e +1
t
e n ln k −( n−1) t + 1 
195
 n −1

 n −1 x2 
= max  +  (x = e ≤ k)
t
(1)
 x +1 x n−1 + k n 
 
n −1
n −1 x 2
Tieáp theo, ta seõ tìm max cuûa haøm soá g ( x) = + vôùi x ≤ k
x +1 n −1
x + kn

 n −3


.  
n 2
n 1 k x 1
Ta coù g / ( x) = 3
− 3 
2  n−1 
 (x + k ) ( x + 1) 2
n 2

n −3 3 3
n −1
g ( x) = 0 ⇔ k x
/ n 2
.( x + 1) = ( x 2
+k ) n 2

2n n −3
⇔k 3x 3
.( x + 1) = x n−1 + k n (2)
2 2
Ñaët t = x ⇒ t ≤ k . Khi ñoù, phöông trình (2) trô ûthaønh
3 3

2 n n −3
 23  3( n2−1)
k t3 2 .  t + 1 = t + kn
 
3( n −1) 2n n 2 n n −3
⇔t 2 −k t −k t
3 2 3 2 + kn = 0
3( n −1) 2n n 2 n n −3 2
Xeùt haøm soá h(t ) = t 2 −k t −k t 3 2 3 2 + k vôùi t ≤ k
n 3

1
n −5
 2n 3 2n

Ta coù h / (t ) = .t 3
. 3( n − 1)t n − nk 3 t 2 − (n − 3)k 3 
2  
2n 3 2n
h (t ) = 0 ⇔ 3( n − 1)t − nk t − (n − 3)k
/ n 3 2 3
=0
2n 3 2n 2
Xeùt tieáp haøm soá m(t ) = 3( n − 1)t − nk t − ( n − 3) k n 3 2 3
vôùi t ≤ k 3

3n 2  
1 2 n −3 2n
Ta coù m (t ) = t . 2( n − 1)t
/ 2
− k 3

2 
 
2
  2n 2 n −3

m (t ) = 0 ⇔ t0 = 
/ k  3


 2(n − 1) 
 
196
2n − 1
2
Do 0 < k ≤ neân t0 < k 3 . Qua t0 thì m / (t ) ñoåi daáu töø aâm sang döông neân
(n − 1) 2

 2

m(t ) nghòch bieán treân (0, t0 ] vaø ñoàng bieán treân t0 , k 3  .
 

 2 2n
 2n -1 
Ta laïi coù m(0) = 3 − n ≤ 0, m  k 3  = nk 3 (2 − k ) > 0  do 2 > ≥ k 
  (n -1) 2
   
2
Neân phöông trình m(t ) = 0 coù nghieäm duy nhaát 0 < t1 < k . 3

2
⇒ Phöông trình h (t ) = 0 coù nghieäm duy nhaát 0 < t1 < k
/ 3

Baûng bieán thieân cuûa h(t )

t 0 t1 k2 3

h / (t ) _ 0 +
kn 0
h(t )

Caên cöù vaøo baûng bieán thieân, ta coù


h(t ) = 0 coù 2 nghieäm döông phaân bieät laø k 2 3 vaø t2 < t1 .

Do ñoù g / ( x) = 0 coù 2 nghieäm döông phaân bieät laø k vaø t23 2 < t13 2 < k .

Baûng bieán thieân cuûa g ( x)


x 0 t23 2 k
g / ( x) _ 0 + 0

g ( x)

Caên cöù vaøo baûng bieán thieân, ta suy ra


n
g ( x) ≤ max { g (0), g ( k )} = ∀x ≤ k (3)
k +1

Töø (1) vaø (3), ta suy ra ñpcm.


197
Ví duï 5. (Voõ Quoác Baù Caån)
 x, y , z ∈ − 3, 3
Cho caùc soá thöïc x, y , z thoûa 
( )
. Tìm giaù trò lôùn nhaát vaø giaù trò
+
 x y z 1+ =

nhoû nhaát cuûa bieåu thöùc


x y z
P ( x, y , z ) = + 2 + 2
x +1 y +1 z +1
2

Lôøi giaûi.
Khoâng maát tính toång quaùt, ta coù theå giaû söû x ≤ y ≤ z

Xeùt haøm soá f ( x) =


x
x +1
2
vôùi x ∈ − 3, 3( )
Ta coù

x( x 2 − 3)
f ( x) = 2
//
( x + 1)3
f // ( x ) = 0 ⇔ x = 0 ∨ x = ± 3

(
Qua 0 thì f // ( x ) ñoåi daáu töø döông sang aâm neân f ( x) loõm treân − 3,0  vaø loài

treân  0, 3 . )
Do ñoù, theo Ñònh lyù 1’, ta coù P( x, y, z ) = f ( x) + f ( y) + f ( z ) ñaït max khi

x = y = z  x = − 3, y = z  x = y = − 3
 ∨ ∨
 x + y + z = 1 
 x + y + z = 1  x + y + z = 1

Hay
x = − 3
1   x = y = − 3
x= y = z = ∨ ∨  (loaïi)
3  y = z = 3 + 1  z = 1 + 2 3
 2
Ta laïi coù

1 1 1 9
P , ,  =
 3 3 3  10

198
 3 +1 3 +1 5+4 3 9
P  − 3, , = <
 2 2  4 4 + 3 10 ( )
Do ñoù
9
max P ( x, y , z ) = .
10
Cuõng theo Ñònh lyù 1’, ta coù P( x, y, z ) = f ( x) + f ( y) + f ( z ) ñaït min khi

x = y = z  x = y, z = 3  y = z = 3
 ∨ ∨
 x + y + z = 1  x + y + z = 1  x + y + z = 1

Hay
 1− 3 
1 x = y = x = 1− 2 3
x= y = z = ∨ 2 ∨ (loaïi)
3   y = z = 3
z = 3
Ta laïi coù

1 1 1 9
P , ,  =
 3 3 3  10
1− 3 1− 3  5−4 3
P , 3 =
( )
,
 2 2  4 4− 3

Vaäy

5−4 3
min P( x, y, z ) = .
(
4 4− 3 )
Keát luaän
9
max P ( x, y, z ) =
10

5−4 3
min P( x, y, z ) =
(
4 4− 3 )

199
Ví duï 6. (Crux mathematicorum)
Cho caùc soá khoâng aâm x1 , x2 ,..., xn (n ≥ 2) thoûa x1 + x2 + ... + xn = 1. Chöùng minh

raèng

1 − x1 1 − x2 1 − xn 2
P= + + ... + ≤ n−2+
1 + x1 1 + x2 1 + xn 3

Chöùng minh.

1− x
Xeùt haøm soá f ( x ) = vôùi x ∈ [0,1] .
1+ x

Ta coù

(1 + x) 2 (1 − 2 x )
f // ( x) =
(1 + x)3 (1 − x) (1 + x)3 (1 − x)
1
f // ( x) = 0 ⇔ x =
2
1  1
Qua thì f // ( x ) ñoåi daáu töø döông sang aâm neân f ( x) loõm treân  0, 2  vaø loài treân
2

1 
 2 ,1 . Do ñoù, theo Ñònh lyù 1’, ta coù P = f ( x1 ) + f ( x2 ) + ... + f ( xn ) ñaït max khi

 x1 = x2 = ... = xm = 0, xm+1 = xm+ 2 = ... = xn ( m = 0, n − 1)



 x1 + x2 + ... + xn = 1

Hay
 x1 = x2 = ... = xm = 0

 1 ( m = 0, n − 1)
 m+1
x = x m+2 = ... = x =
n−m
n

+ Neáu m = n − 1 thì ta coù


P = f ( x1 ) + f ( x2 ) + ... + f ( xn−1 ) + f ( xn )
≤ ( n − 1) f (0) + f (1)
= n −1
2
< n−2+ (1)
3
200
+ Neáu m < n − 1 thì ta coù
P = f ( x1 ) + f ( x2 ) + ... + f ( xn −1 ) + f ( xn )
 1 
≤ mf (0) + (n − m) f  
n−m
n − m −1
= m + (n − m)
n − m +1
t −1
= n−t +t
t +1
= g (t )

Trong ñoù t = n − m ∈ [2, n] .

Ta coù

t 2 − (t + 1)3 (t − 1)
g (t ) =
/

(t + 1)3 (t − 1)
t 4 − (t + 1)3 (t − 1)
=
(
(t + 1)3 (t − 1) t 2 + (t + 1)3 (t − 1) )
−2t 3 + 2t 2 + 1
=
(
(t + 1)3 (t − 1) t 2 + (t + 1)3 (t − 1) )
< 0 (do t ≥ 2)

⇒ g (t ) laø haøm nghòch bieán treân [2, n].


2
⇒ g (t ) ≤ g (2) = n − 2 + ∀t ∈ [2, n]
3
2
⇒ P ≤ n−2+ (2)
3

Töø (1) vaø (2) suy ra trong moïi tröôøng hôïp, ta luoân coù
2
P ≤ n−2+
3

⇒ ñpcm.

201
Ví duï 7.
Cho caùc soá thöïc döông a, b, c thoûa abc = 1. Chöùng minh raèng

1 1 1
+ 2 + 2 ≤3
a − a +1 b − b +1 c − c +1
2

Nhaän xeùt.

Ta coù theå ñaët a = e x , b = e y , c = e z ( x, y, z ∈ R ) thì ta coù x + y + z = 0 . Ñeán ñaây,

1
neáu laøm theo caùch laøm treân, ta seõ xeùt haøm soá f (t ) = ñeå xem f (t ) coù
e − et + 1
2t

laø haøm nöûa loài nöûa loõm hay khoâng. Nhöng ruûi thay, f (t ) laïi khoâng phaûi laø haøm

et (4e3t − 3e 2t − 3et + 1)
nöûa loài nöûa loõm. Thaät vaäy, ta coù f // (t ) = . Deã thaáy f // (t )
(e − e + 1)
2t t 3

coù 2 nghieäm phaân bieät neân f (t ) khoâng phaûi laø haøm nöûa loài nöûa loõm. Vaäy phaûi

laøm sao baây giôø? Laøm theá naøo ñeå vöôït qua noù ñaây? Sau ñaây giaûi phaùp cuûa toâi cho
vaán ñeà treân
Chöùng minh.
Ta coù Boå ñeà sau

Boå ñeà. Vôùi moïi soá thöïc döông a, b, c thoûa abc = 1 , ta coù

1 1 1
P (a, b, c ) = + 2 + 2 ≥1
a + a +1 b + b +1 c + c +1
2

Chöùng minh.

Do a, b, c > 0 vaø abc = 1 neân toàn taïi x, y , z ∈ R sao cho a = e x , b = e y , c = e z . Khi

1
ñoù, ta coù x + y + z = 0 vaø P (a, b, c) = f ( x) + f ( y ) + f ( z ) vôùi f (t ) = .
e + et + 1
2t

Ta coù

et (4e3t + 3e 2t − 3et − 1)
f // (t ) =
(e 2t + et + 1)3

202
Deã thaáy f // (t ) = 0 coù duy nhaát moät nghieäm t0 vaø qua t0 thì f // (t ) ñoåi daáu töø aâm

sang döông neân f (t ) loài treân (−∞, t0 ] vaø loõm treân [t0 , +∞) neân theo Ñònh lyù 2, ta

coù P (a, b, c) ñaït min khi x ≤ y = z . Do ñoù, ta chæ caàn chöùng minh
f ( x) + 2 f ( y ) ≥ 1
1 2
⇔ 2x + 2y ≥1 (*)
e + e +1 e + ey +1
x

Vôùi x, y ∈ R thoûa x + 2 y = 0 .

Ta coù
2 1
(*) ⇔ + ≥ 1 (m = e y )
m + m +1
2 1
+
1
+1
m4 m2
2 m4
⇔ 2 + ≥1
m + m + 1 m4 + m2 + 1
2 m4
⇔ 2 + ≥1
m + m + 1 ( m 2 + m + 1)( m 2 − m + 1)
⇔ 2( m 2 − m + 1) + m 4 ≥ m 4 + m 2 + 1
⇔ ( m − 1) 2 ≥ 0 (ñuùng)

Töø ñaây, ta suy ra ñöôïc


P (a, b, c) ≥ 1

Boå ñeà ñöôïc chöùng minh hoaøn toaøn.


Ñaúng thöùc xaûy ra khi vaø chæ khi a = b = c = 1.
Trôû laïi baøi toaùn cuûa ta
1 1 1
Töø Boå ñeà treân, thay a, b, c laàn löôït bôûi , , , ta ñöôïc
a 2 b2 c2
1 1 1
+ + ≥1
1 1 1 1 1 1
+ +1 + +1 + +1
a4 a2 b4 b 2 c4 c 2
a4 b4 c4
⇔ 4 + + ≥1
a + a 2 + 1 b4 + b2 + 1 c4 + c 2 + 1

203
a2 + 1 b2 + 1 c2 + 1
⇔ + + ≤2
a 4 + a 2 + 1 b 4 + b2 + 1 c4 + c2 + 1
2( a 2 + 1) 2(b 2 + 1) 2(c 2 + 1)
⇔ 4 + + ≤4
a + a 2 + 1 b 4 + b2 + 1 c4 + c2 + 1
(a 2 + a + 1) + (a 2 − a + 1) (b 2 + b + 1) + (b 2 − b + 1)
⇔ + +
(a 2 + a + 1)( a 2 − a + 1) (b 2 + b + 1)(b 2 − b + 1)
(c 2 + c + 1) + (c 2 − c + 1)
+ ≤4
(c 2 + c + 1)(c 2 − c + 1)
 1   1 
⇔ ∑ 2 +∑ 2 ≤4
 cyc a − a + 1   cyc a + a + 1 
 (**)
   
Laïi aùp duïng Boå ñeà treân, ta coù
1
∑ a2 + a + 1 ≥ 1
cyc

Neân töø (**), ta suy ra ñöôïc


1
∑ a2 − a + 1 ≤ 3
cyc

⇒ ñpcm.
Ñaúng thöùc xaûy ra khi vaø chæ khi a = b = c = 1.

204
Baøi taäp.

U Baøi 1 . (VMEO 2005)


U

Cho a, b, c laø caùc soá thöïc döông cho tröôùc vaø x, y , z laø caùc soá thöïc döông thoûa

maõn ax + by + cz = xyz. Tìm giaù trò nhoû nhaát cuûa bieåu thöùc
P= x+ y+z

U Baøi 2 . (Phaïm Kim Huøng)


U

Cho a, b, c laø caùc soá thöïc khoâng aâm. Chöùng minh raèng
k k k
 a   b   c   3
  +  +  ≥ min  2, k  ∀k ≥ 0
b+c c+a  a+b  2 
U Baøi 3 . (Crux mathematicorum)
U

Chöùng minh raèng vôùi moïi soá khoâng aâm a, b, c ta coù

48a 48b 48c


1+ + 1+ + 1+ ≥ 15
b+c c+a a+b

U Baøi 4 .
U

Cho tam giaùc khoâng tuø ABC. Chöùng minh raèng

cos 2 A cos 2 B cos 2 C 1


+ + ≥
cos A + 1 cos B + 1 cos C + 1 2
U Baøi 5 .
U

Xeùt caùc soá thöïc khoâng aâm x1 , x2 ,..., xn thoûa maõn

x1 + x2 + ... + xn n −1
=r≥
n n

Chöùng minh raèng


1 1 1 n
+ + ... + ≥
1 + x12 1 + x22 1 + xn2 1 + r 2
205
U Baøi 6 .
U

Xeùt caùc soá thöïc khoâng aâm x1 , x2 ,..., xn thoûa maõn

x1 + x2 + ... + xn n −1
=r≤
n n − n +1
2

Chöùng minh raèng


1 1 1 n
+ + ... + ≤
1 + x1 1 + x2
2 2
1 + xn 1 + r 2
2

U Baøi 7 .
U

Xeùt caùc soá thöïc khoâng aâm x1 , x2 ,..., xn thoûa maõn

1
n x1 x2 ...xn = p ≤
n −1
Chöùng minh raèng
1 1 1 n
+ + ... + ≤
1 + x1 1 + x2 1 + xn 1 + p

U Baøi 8 .
U

Xeùt caùc soá thöïc khoâng aâm x1 , x2 ,..., xn thoûa maõn

n
n x1 x2 ...xn = p ≤ −1
n −1

Chöùng minh raèng


1 1 1 n
+ + ... + ≤
(1 + x1 ) 2
(1 + x2 ) 2
(1 + xn ) 2
(1 + p ) 2

U Baøi 9 .
U

Cho tam giaùc ABC. Tìm giaù trò nhoû nhaát cuûa caùc bieåu thöùc

P = sin A.sin 2 B.sin 3 C

Q = sin m A.sin n B.sin p C (m, n, p laø caùc soá thöïc döông cho tröôùc)

206
CAÙC BAØI TOAÙN CHOÏN LOÏC
----oOo----

Baøi toaùn 1. (Phaïm Kim Huøng)


Cho a, b, c laø caùc soá thöïc khoâng aâm. Chöùng minh raèng

ab bc ca a+b+c
+ + ≤
a + 4b + 4c b + 4c + 4a c + 4a + 4b 9
Lôøi giaûi.

* Caùch 1.
Ta coù baát ñaúng thöùc caàn chöùng minh töông ñöông vôùi
9∑ ab(4a + 4b + c)(4a + b + 4c) ≤
cyc

≤ (a + b + c)(4a + 4b + c)(4a + b + 4c)(a + 4b + 4c)


⇔ 4∑ a 4 + 16∑ ab 3 ≥ 11∑ a 3b + 3∑ a 2b 2 + 6∑ a 2bc
cyc cyc cyc cyc cyc

⇔ 4∑ a 4 + 11( a − b)(b − c)(c − a)( a + b + c) + 5∑ ab3 ≥ 3∑ a 2b 2 + 6∑ a 2bc


cyc cyc cyc cyc

Khoâng maát tính toång quaùt, giaû söû a = min {a, b, c}

Ñaëêt b = a + x, c = a + y

Khi ñoù, ta coù

∑ a 4 = 3a 4 + 4( x + y)a3 + 6( x 2 + y 2 )a 2 + 4( x3 + y3 )a + x4 + y 4
cyc

∑ ab3 = 3a 4 + 4( x + y )a3 + 3( x2 + y 2 + xy)a 2 + ( x3 + y3 + 3xy 2 )a + xy 3


cyc

∑ a 2b2 = 3a 4 + 4( x + y)a3 + 2( x 2 + y 2 + 2 xy)a 2 + 2( x2 y + xy 2 )a + x 2 y 2


cyc

∑ a 2bc = 3a 4 + 4( x + y)a3 + ( x 2 + y 2 + 5 xy)a 2 + ( x 2 y + xy 2 )a


cyc

(a − b)(b − c )(c − a )(a + b + c ) = − xy ( x − y )(3a + x + y )


= −3 xy ( x − y ) a − x3 y + xy 3

207
Do ñoù, baát ñaúng thöùc caàn chöùng minh töông ñöông vôùi
27( x 2 + y 2 − xy ) a 2 + (21x 3 + 21y 3 − 45 x 2 y + 36 xy 2 )a +
+ 4 x 4 + 4 y 4 − 11x 3 y − 3x 2 y 2 + 16 xy 3 ≥ 0

⇔ 27( x 2 + y 2 − xy )a 2 + (21x 3 + 21 y 3 − 45 x 2 y + 36 xy 2 )a +
+ ( x − 2 y ) 2 (4 x 2 + 5 xy + y 2 ) ≥ 0 (ñuùng)
⇒ ñpcm.

Ñaúng thöùc xaûy ra khi vaø chæ khi a = b = c hoaëc a = 0, b = 2c vaø caùc hoaùn vò töông

öùng.

* Caùch 2.
Ta coù

 3ab  b
∑ 
cyc  a + 4b + 4c
+ b  = 4( a + b + c).∑
 cyc a + 4b + 4c

Do ñoù, baát ñaúng thöùc caàn chöùng minh töông ñöông vôùi
a b c 1
+ + ≤ (*)
4a + 4b + c 4b + 4c + a 4c + 4a + b 3
Khoâng maát tính toång quaùt, ta coù theå giaû söû a + b + c = 3 . Khi ñoù, ta coù
a b c
(*) ⇔ + + ≤1
4−c 4−a 4−b
⇔ ∑ a (4 − a )(4 − b) ≤ (4 − a )(4 − b)(4 − c )
cyc

⇔ a 2b + b 2c + c 2 a + abc ≤ 4

Nhö vaäy, ñeå chöùng minh baát ñaúng thöùc ñaõ cho, ta caàn phaûi chöùng minh
a 2b + b 2c + c 2 a + abc ≤ 4 (**)

Khoâng maát tính toång quaùt, ta coù theå giaû söû b naèm giöõa a vaø c.
Do ñoù
c( a − b)(c − b) ≤ 0
⇔ b 2c + c 2 a ≤ bc 2 + abc
⇔ a 2b + b 2c + c 2 a + abc ≤ a 2b + bc 2 + 2abc = b( a + c) 2

208
Maët khaùc, aùp duïng baát ñaúng thöùc AM-GM, ta coù

 2b + ( a + c ) + (a + c) 
2

2b(a + c) = 2b.(a + c).( a + c) ≤ 


2
 =8
 3 
⇒ b(a + c) ≤ 4
2

Vaäy

a 2b + b 2c + c 2a + abc ≤ 4
⇒ ñpcm.

Ñaúng thöùc xaûy ra khi vaø chæ khi a = b = c hoaëc a = 0, b = 2c vaø caùc hoaùn vò töông

öùng.

Baøi toaùn 2. (Phaïm Kim Huøng)


Cho a, b, c laø caùc soá thöïc khoâng aâm thoûa maõn a + b + c = 3 . Chöùng minh raèng

36( ab + bc + ca) ≥ ( a 3 + b3 + c3 )(a 3b3 + b3c3 + c3a 3 )

Lôøi giaûi.

Ñaët f (a, b, c) = 36( ab + bc + ca ) − (a 3 + b3 + c 3 )(a 3b3 + b 3c3 + c3a 3 )

Khoâng maát tính toång quaùt, giaû söû a ≥ b ≥ c ≥ 0


Khi ñoù, ta coù
ab + bc + ca ≥ a (b + c)
a 3 + (b + c)3 ≥ a 3 + b3 + c3 ≥ 0
a 3 (b + c )3 = a 3b3 + a 3c 3 + 3a 3b 2c + 3a 3bc 2 ≥ a 3b 3 + a 3c 3 + b3c3 ≥ 0
( )
⇒ a 3 + (b + c )3 a 3 (b + c)3 ≥ (a 3 + b3 + c 3 )(a 3b3 + a 3c 3 + b3c3 )
⇒ 36(ab + bc + ca ) − ( a 3 + b3 + c 3 )( a 3b3 + a 3c 3 + b3c3 ) ≥
( )
≥ 36a (b + c) − a 3 + (b + c )3 a 3 (b + c)3

Do ñoù
f (a, b, c) ≥ f (a, b + c, 0)
= f ( a,3 − a,0)
= 36a (3 − a ) − a 3 (3 − a)3 (a 3 + (3 − a )3 )

= 9a(3 − a )(a 2 − 3a + 2) 2 ( a (3 − a) + 1) ≥ 0
209
⇒ f ( a, b, c) ≥ 0 (ñpcm)

Ñaúng thöùc xaûy ra khi vaø chæ khi (a, b, c) = (2,1,0).

Baøi toaùn 3. (Phaïm Kim Huøng)


Cho a, b, c, d ≥ 0 thoûa maõn a + b + c + d = 4 . Chöùng minh raèng

(1 + a 4 )(1 + b 4 )(1 + c 4 )(1 + d 4 ) ≥ (1 + a 3 )(1 + b3 )(1 + c3 )(1 + d 3 )

Lôøi giaûi.
+ Caùch 1.
Khoâng maát tính toång quaùt giaû söû a ≥ b ≥ c ≥ d ≥ 0
1 1
* Tröôøng hôïp 1. a ≥ 3 ⇒ b + c + d ≤ 1 ⇒ 0 ≤ b ≤ 1,0 ≤ c ≤ ,0 ≤ d ≤ .
2 3
⇒ (1 + a 4 )(1 + b 4 )(1 + c 4 )(1 + d 4 ) − (1 + a 3 )(1 + b3 )(1 + c3 )(1 + d 3 ) ≥
  1 3   1 3 
≥ (1 + a ) − (1 + a )(1 + 1 ) 1 +   1 +   
4 3 3
  2    3  
  
7 4
= a 4 − .a 3 −
3 3
7 4
≥ 3a 3 − .a 3 −
3 3
2
= .( a 3 − 2) > 0
3
⇒ (1 + a 4 )(1 + b 4 )(1 + c 4 )(1 + d 4 ) ≥ (1 + a 3 )(1 + b3 )(1 + c3 )(1 + d 3 )

* Tröôøng hôïp 2. 3 ≥ a ≥ b ≥ c ≥ d ≥ 0
Ta coù baát ñaúng thöùc caàn chöùng minh töông ñöông vôùi

∑ ( 2ln(1 + a 4 ) − 2ln(1 + a3 ) ) ≥ 0
cyc

Xeùt haøm soá f ( x) = 2ln (1 + x 4 ) − 2ln(1 + x 3 ) − x + 1 vôùi 0 ≤ x ≤ 3

Ta coù

8 x3 6x2 ( x − 1)( − x 6 + x5 + x4 + 7 x 2 + x + 1)
f ( x) = 4
/
− −1 =
x + 1 x3 + 1 ( x 4 + 1)( x 3 + 1)

210
Deã thaáy f / ( x) = 0 chæ coù 2 nghieäm döông phaân bieät laø 1 vaø x0 ∈ (2,3).
Qua 1 thì f / ( x ) ñoåi daáu töø aâm sang döông, qua x0 thì f / ( x ) ñoåi daáu töø döông
sang aâm neân
f ( x) ≥ min { f (1), f (3)} = min {0, 2(ln41 − ln14 − 1)} = 0 ∀x ∈ [ 0,3]

⇒ 2ln(1 + x 4 ) − 2ln (1 + x3 ) ≥ x − 1 ∀x ∈ [ 0,3]

( )
⇒ ∑ 2ln (1 + a 4 ) − 2ln (1 + a 3 ) ≥ ∑ (a − 1) = 0
cyc cyc

⇒ ñpcm.

Ñaúng thöùc xaûy ra khi vaø chæ khi a = b = c = d = 1.

* Nhaän xeùt.
Baèng caùch laøm hoaøn toaøn töông töï, ta coù keát quaû sau
Cho a, b, c, d ≥ 0 thoûa maõn a + b + c + d = 4 . Khi ñoù, ta coù

(1 + a k +1 )(1 + b k +1 )(1 + c k +1 )(1 + d k +1 ) ≥ (1 + a k )(1 + b k )(1 + c k )(1 + d k ) ∀k ≥ 2.

+ Caùch 2.
Ta seõ chöùng minh baát ñaúng thöùc ñaõ baèng phöông phaùp phaûn chöùng.
Giaû söû ngöôïc laïi toàn taïi boán soá khoâng aâm (a, b, c, d ) thoûa a + b + c + d = 4 sao

cho

(1 + a 4 )(1 + b 4 )(1 + c 4 )(1 + d 4 ) < (1 + a 3 )(1 + b3 )(1 + c3 )(1 + d 3 )

Khoâng maát tính toång quaùt, ta coù theå giaû söû a ≤ b ≤ c ≤ d .

Ñaët Fk = (1 + a k )(1 + b k )(1 + c k )(1 + d k ) . Theá thì theo baát ñaúng thöùc Bunhiacopxki,

ta coù

F4 .F2 ≥ F32 , F3 .F1 ≥ F22 , F2 .F0 ≥ F12 (1)

Theo giaû thieát phaûn chöùng thì


F4 < F3 (2)

Töø (1) vaø (2), ta suy ra ñöôïc


F4 < F3 < F2 < F1 < F0 = 16 (3)
211
Töø (3), ta coù d < 2.
Ñeå daãn tôùi maâu thuaãn vôùi (3), ta seõ chöùng minh
F3 ≥ F1 (4)

Thaät vaäy

(4) ⇔ (1 − a + a 2 )(1 − b + b 2 )(1 − c + c 2 )(1 − d + d 2 ) ≥ 1


 3 (2a − 1)2  3 (2b − 1)2  3 (2c − 1) 2  3 (2d − 1) 2 
⇔ +  +  +  +  ≥1
4 4  4 4  4 4  4 4 
 (2a − 1) 2  (2b − 1) 2  (2c − 1) 2  (2d − 1) 2   4  4
⇔ 1 +  1 + 1 + 1 + ≥ 
 3  3  3  3  3
4
  x + y + z + t 2 
⇔ (1 + x )(1 + y )(1 + z )(1 + t ) ≥ 1 + 
2 2 2 2
  (5)
  4  

2a − 1 2b − 1 2c − 1 2d − 1
Trong ñoù x = ,y = ,z = ,t =
3 3 3 3

Töø ñoù, xeùt baát ñaúng thöùc


2
  A + B 2 
(1 + A )(1 + B ) ≥ 1 + 
2 2

  2  
(6)
 
1
⇔ .( A − B )2 (8 − A2 − 6 AB − B 2 ) ≥ 0
8
Ta thaáy neáu A + B ≤ 2 thì baát ñaúng thöùc treân ñuùng.
x + t < 2
Töø a ≤ b ≤ c ≤ d < 2, ta deã daøng chöùng minh ñöôïc  . Do ñoù, theo (6), ta
y + z < 2

coù
2
  x + t 2 
(1 + x )(1 + t ) ≥ 1 + 
  2  
2 2

 
2
  y + z 2 
(1 + y )(1 + z ) ≥ 1 + 
  2  
2 2

 

212
2
  x + t 2   y + z  2  
⇒ (1 + x )(1 + y )(1 + z )(1 + t ) ≥ 1 + 
2 2 2 2
  1 +    
  2     2   

x + t < 2 x+t y+ z
Töø  ta coù + < 2 . Do ñoù, theo (6), ta laïi coù
y + z < 2 2 2
2
  x + t 2   y + z 2    x + y + z + t 2 
1 +   1 +  ≥ 1 +  
  2    2     4  
   
Do ñoù
4
  x + y + z + t 2 
(1 + x )(1 + y )(1 + z )(1 + t ) ≥  1 + 
2 2 2 2
 
  4  

⇒ (5) ñuùng
⇒ (4) ñuùng.
Töø ñaây daãn ñeán maâu thuaãn.
Vaäy ta phaûi coù

(1 + a 4 )(1 + b 4 )(1 + c 4 )(1 + d 4 ) ≥ (1 + a 3 )(1 + b3 )(1 + c3 )(1 + d 3 ) (ñpcm)

Ñaúng thöùc xaûy ra khi vaø chæ khi a = b = c = d = 1.

* Ghi chuù.
Ngoaøi 2 caùch chöùng minh treân, ta coøn coù moät caùch chöùng minh nöõa laø chöùng minh
baát ñaúng thöùc maïnh hôn nhö sau
Cho a, b, c, d ≥ 0 thoûa maõn a + b + c + d = 4 . Khi ñoù, ta coù

(1 + a 2 )(1 + b 2 )(1 + c 2 )(1 + d 2 ) ≥ (1 + a )(1 + b)(1 + c)(1 + d )

Chöùng minh.
Ta coù baát ñaúng thöùc caàn chöùng minh töông ñöông vôùi

f (a, b, c, d ) = (1 + a 2 )(1 + b 2 )(1 + c 2 )(1 + d 2 ) − (1 + a)(1 + b)(1 + c )(1 + d ) ≥ 0

Ta coù Nhaän xeùt sau

Nhaän xeùt. Neáu a + b ≤ 2 vaø a ≥ x ≥ b thì


213
f (a, b, c, d ) ≥ f ( x, a + b − x, c, d )

Thaät vaäy
f (a, b, c, d ) − f ( x, a + b − x, c, d ) =
= ( a − x)( x − b)((c + 1)( d + 1) − (c 2 + 1)(d 2 + 1)( ab − x 2 + ax + bx − 2))

Töø ñaây, söû duïng giaû thieát, ta deã daøng chöùng minh ñöôïc
f (a, b, c, d ) ≥ f ( x, a + b − x, c, d )

Nhaän xeùt ñöôïc chöùng minh.


Trôû laïi baøi toaùn cuûa ta
a+b+c
Khoâng maát tính toång quaùt, ta coù theå giaû söû a ≤ b ≤ c ≤ d vaø ñaët x = theá
3
thì ta coù a + c ≤ 2 vaø c ≥ x ≥ a. Do ñoù, theo Nhaän xeùt treân, ta coù
f (a, b, c, d ) ≥ f (a + c − x, b, x, d ) (1)

(a + c − x ) + b + x  x = min{x, b, a + c − x}
Chuù yù raèng x = neân neáu  x = max{x, b, a + c − x} thì ta coù x = b =
3 

= a + c − x neân f (a + c − x, b, x, d ) = f ( x, x, x, d )

Giaû söû ngöôïc laïi, khi ñoù coù 2 tröôøng hôïp xaûy ra
b< x < a+c− x (2)
b > x > a+c−x (3)

Laïi söû duïng Nhaän xeùt, ta ñöôïc


(2) ⇒ f (a + c − x, b, x, d ) ≥ f ( x, a + b + c − 2 x, x, d ) = f ( x, x, x, d )
(3) ⇒ f (a + c − x, b, x, d ) ≥ f (a + b + c − 2 x, x, x, d ) = f ( x, x, x, d )

Toùm laïi, trong moïi tröôøng hôïp, ta luoân coù


f (a + c − x, b, x, d ) ≥ f ( x, x, x, d )
⇒ f ( a, b, c, d ) ≥ f ( x, x, x, d ) (4)

Maët khaùc, ta coù


f ( x, x, x, d ) =
4−d 4−d 4−d 
= f , , ,d 
 3 3 3 
214
1
= .(d − 1)2 (d 6 − 22d 5 + 223d 4 − 1268d 3 + 4210d 2 − 7564d + 6364)
729
≥0
Neân töø (4), ta suy ra ñöôïc
f (a, b, c, d ) ≥ 0
⇒ ñpcm.

Baøi toaùn 4. (Phaïm Kim Huøng)


Cho x1,x2 ,..., xn laø caùc soá thöïc döông thoûa maõn
n n
1

i =1
xi = ∑
i=1 x i

Chöùng minh raèng


n
1

i =1 x 2
+ n − 1
≤1
i

Lôøi giaûi.
Neáu n = 1, n = 2 thì baát ñaúng thöùc ñaõ cho trôû thaønh ñaúng thöùc.

Xeùt n ≥ 3
n n
1 1
Ñaët yi =
xi
(i = 1, n) thì ∑
i =1
yi = ∑ . Khi ñoù, baát ñaúng thöùc caàn chöùng minh töông
i =1 y i

ñöông vôùi
n
yi2
∑ (n − 1) y 2 + 1 ≤ 1
i =1 i
n
1
⇔∑ ≥1
i =1 (n − 1) yi2 + 1
n
1
Giaû söû ngöôïc laïi ∑
i =1 ( n − 1) y + 1
2
< 1.
i

1 n
1 − ai
Ñaët ai = (i = 1, n) thì ai > 0 ∀i = 1, n, ∑ ai < 1 vaø yi = ∀i = 1, n
(n − 1) yi + 1
2
i =1 ( n − 1) ai

215
∑aj n
Ñaët bi = j ≠i
∀i = 1, n, a = ∑ ai
n −1 i =1

Ta coù
n
1 − ai bi

i =1
ai < 1 ⇒ 1 − a i > ∑ a j = ( n − 1)bi
j ≠i
∀i = 1, n ⇒ yi =
( n − 1) ai
>
ai
∀i = 1, n

n n
bi
⇒ ∑ yi > ∑
i =1 i =1 ai

Ta chöùng minh
n n
bi a

i =1
≥∑ i
ai i =1 bi
(*)

Thaät vaäy, ta coù


n 
b a  n ( n − 1)bi − ( n − 1)ai
n − 1.∑  i − i  = ∑
i =1  ai bi  i =1 ai ( n − 1)bi

n ∑ a j − (n − 1)ai
=∑ j ≠i

i =1 ai ( a − ai )

 1 1 
= ∑ i j  a (a − a ) a (a − a ) 
( a − a )  −
1≤i < j ≤ n
 j j i i 

(ai − a j ) 2 . ∑ ak
= ∑ k ≠i ,k ≠ j

1≤i < j ≤ n ai a j ( a − ai )( a − a j ). ( ai ( a − ai ) + a j ( a − a j ) )
≥0
( ai − a j ) 2 ( a − ai − a j )
= ∑
1≤i < j ≤ n ai a j ( a − ai )( a − a j ). ( ai ( a − ai ) + a j ( a − a j ) )
 b
n
a 
⇒ ∑  i − i  ≥ 0
i =1  ai bi 
n n
bi a
⇒∑ ≥∑ i
i =1 ai i =1 bi
⇒ (*) ñuùng.
216
n n n n
bi ai 1
Vaäy ∑ yi > ∑ ≥∑ > ∑ . Ñieàu naøy traùi vôùi giaû thieát.
ai i =1 bi i =1 yi
i =1 i =1
n
1
Vaäy ta phaûi coù ∑ (n − 1) y 2 + 1 ≥ 1 (ñpcm).
i =1 i

Baøi toaùn 5. (Phaïm Kim Huøng)


Cho a, b, c laø caùc soá thöïc döông thoûa maõn abc = 1 . Chöùng minh raèng

i. 81(1 + a 2 )(1 + b 2 )(1 + c 2 ) ≤ 8(a + b + c) 4


ii. 64(1 + a 3 )(1 + b3 )(1 + c 3 ) ≤ (a + b + c) 6

Lôøi giaûi.

i. Ñaët f (a, b, c) = 8(a + b + c) 4 − 81(1 + a 2 )(1 + b 2 )(1 + c 2 )

Tröôùc heát ta chöùng minh raèng

 1 
f  x, x, 2  ≥ 0
 x 
4
 1   1 
⇔ 8  2 x + 2  − 81(1 + x 2 ) 2 1 + 4  ≥ 0
 x   x 
⇔ 8(2 x + 1) − 81x (1 + x ) (1 + x 4 ) ≥ 0
3 4 4 2 2

⇔ ( x − 1) 2 (47 x10 + 94 x 9 − 21x8 + 120 x 7 +


+ 99 x 6 + 78 x5 + 87 x 4 + 96 x 3 + 24 x 2 + 16 x + 8) ≥ 0 (ñuùng)
Vaäy

 1 
f  x, x, 2  ≥ 0 (*)
 x 

Tieáp theo, khoâng maát tính toång quaùt giaû söû a ≥ b ≥ c . Ta chöùng minh

f (a, b, c) ≥ f ( ab , ab , c )
⇔ 8( a + b + c) 4 − 81(1 + a 2 )(1 + b 2 )(1 + c 2 ) ≥

( )
4
≥ 8 2 ab + c − 81(1 + ab) 2 (1 + c 2 )

( )
a − b  (a + b + c) 2 + c + 2 ab  ( ) ( )
+ 2c  ≥
2 2 2
⇔8  a+ b
  
≥ 81(a − b) (1 + c 2 )
2

217
⇔ 8  (a + b + c ) 2 + c + 2 ab  ( ) ( ) + 2c  −
2 2
 a+ b
  

( )
2
− 81 a + b (1 + c 2 ) ≥ 0
 2
( ) ( ) ( )
2
⇔ 8   + c − 2 ab  + c + 2 ab  
 + 2c  −
2 2
a+ b a+ b
   

( )
2
− 81 a + b (1 + c 2 ) ≥ 0 (**)

( )
2
Ñaët t = a+ b ⇒ t ≥ 4 ab ≥ 4 ≥ 4c

Xeùt haøm soá g (t ) = 8  t + c − 2 ab ( ) + (c + 2 )  (t + 2c) − 81t (1 + c 2 )


2 2
ab 
 
Ta caàn chöùng minh g (t ) ≥ 0
Ta coù

(
g / (t ) = 8  t + c − 2 ab + c + 2 ab ) ( ) (
 + 16(t + 2c) t + c − 2 ab − 81(1 + c 2 )
)
2 2

 

(
≥ 8  4 ab + c − 2 ab + c + 2 ) ( )
ab  +
2 2

 
( )(
+ 16 4 ab + 2c 4 ab + c − 2 ab − 81(1 + c 2 ) )
= 3 16 c + 2 ab ( ) − 27(1 + c 2 ) 
2

 
(
= 3 64ab − 27 + 64c ab − 11c 2 ≥ 0 )
⇒ g (t ) ñoàng bieán.

( )
⇒ g (t ) ≥ f 4 ab = 4  8 c + 2 ab − 81(1 + c 2 ) ab  ( )
3

 
 1 
3
 1 
= 4  8  2 + 2t  − 81t  1 + 4   (t = ab ≥ 1)
 t   t  
 
4(64t − 81t + 96t − 33t + 1)
9 7 6 3
= ≥ 0 ∀t ≥ 1
t6
⇒ g (t ) ≥ 0
⇒ (**) ñuùng.

⇒ f ( a, b, c) ≥ f ( 
) 1 
ab , ab , c = f  ab , ab ,  ≥ 0 (do (*))
 ab 

218
⇒ ñpcm.

ii. Tröôùc heát xin ñöôïc nhaéc laïi khoâng chöùng minh keát quaû sau
Cho caùc soá thöïc döông a, b, c . Khi ñoù, toàn taïi caùc soá thöïc x0 , y0 , x1 , y1 ( x0 , x1 , y1 ≥ 0)

sao cho
2 x0 + y0 = 2 x1 + y1 = a + b + c
x02 + 2 x0 y0 = x12 + 2 x1 y1 = ab + bc + ca
x02 y0 ≤ abc ≤ x12 y1

Ngoaøi ra
+ Neáu (a + b + c) 2 ≥ 4( ab + bc + ca) thì y0 ≤ 0
+ Neáu (a + b + c) 2 ≤ 4( ab + bc + ca) thì y0 ≥ 0

Trôû laïi baøi toaùn cuûa ta


Ta coù baát ñaúng thöùc caàn chöùng minh töông ñöông vôùi
 
64 1 + ∑ a 3 + ∑ a 3b3 + a 3b3c 3  ≤ ( a + b + c )6
 cyc cyc 

 
⇔ 64  2 + ∑ a 3 + ∑ a 3b3  ≤ ( a + b + c) 6
 cyc cyc 

 
⇔ 64  2a 2b 2c 2 + abc ∑ a 3 + ∑ a 3b3  ≤ (a + b + c)6
 cyc cyc 
Khoâng maát tính toång quaùt giaû söû a + b + c = 1 . Ñaët q = ab + bc + ca, r = abc . Khi

ñoù, baát ñaúng thöùc caàn chöùng minh töông ñöông vôùi
1
2r 2 + r (1 − 3q + 3r ) + q3 − 3qr + 3r 2 ≤
64
1
⇔ f ( r ) = 8r 2 + (1 − 6q ) r + q 3 − ≤0
64
Ta coù
f / ( r ) = 16r + 1 − 6q
f // (r ) = 16 > 0
219
⇒ f ( r ) laø haøm loõm.

1
* Tröôøng hôïp 1. 4q ≥ 1 ⇒ y0 ≥ 0 ⇒ 0 ≤ x0 ≤
2

{
⇒ f ( r ) ≤ max f ( x02 y0 ), f ( x12 y1 ) }
Ta coù
f ( x02 y0 ) =
1
= 8 x04 y02 + (1 − 6( x02 + 2 x0 y0 )) x02 y0 + ( x02 + 2 x0 y0 )3 −
64
(2 x0 − 1)(1024 x05 − 368 x04 + 264 x03 − 60 x02 + 2 x0 + 1) 1
= ≤ 0 (do 0 ≤ x0 ≤ )
64 2
Töông töï, ta coù f ( x12 y1 ) ≤ 0

⇒ f (r ) ≤ 0 (1)

* Tröôøng hôïp 2. 4q ≤ 1

{
⇒ f ( r ) ≤ max f (0), f ( x12 y1 ) }
Theo treân, ta coù f ( x12 y1 ) ≤ 0
3
1 1 1
Ta laïi coù f (0) = q − ≤  −
3
=0
64  4  64
⇒ f (r ) ≤ 0 (2)

Töø (1) vaø (2) ta suy ra ñpcm.


Baøi toaùn 6. (Greece 2002)
Cho a, b, c > 0 thoûa a 2 + b2 + c 2 = 1 . Chöùng minh raèng

a b c 3
( )
2
+ 2 + 2 ≥ . a a +b b +c c
b +1 c +1 a +1 4
2

Lôøi giaûi.
AÙp duïng baát ñaúng Bunhiacopxki, ta coù

220
(a )
2
a a 3 a +b b +c c
∑ b2 + 1 = ∑ a 2b 2 + a 2 ≥
cyc cyc ∑ (a 2 + a 2b2 )
cyc

(a )
2
a +b b +c c
=
1 + ∑ a 2b 2
cyc

(a )
2
a +b b +c c

1
1 + .( a 2 + b 2 + c 2 ) 2
3

(a )
2
a +b b +c c
=
1
1+
3
3
( )
2
= . a a +b b +c c
4
⇒ ñpcm.

1
Ñaúng thöùc xaûy ra khi vaø chæ khi a = b = c = .
3

Baøi toaùn 7. (Vasile Cirtoaje)

a) Cho a, b, c > 0 . Chöùng minh raèng

 1 1 1  9
(ab + bc + ca )  + + ≥
 a (a + b) b(b + c ) c (c + a )  2
b) Cho a, b, c, d > 0 . Chöùng minh raèng

 1 1 1 1 
(ab + bc + cd + da )  + + + ≥8
 a (a + b) b(b + c ) c (c + d ) d ( d + a ) 
Lôøi giaûi.
a) AÙp duïng baát ñaúng thöùc AM−GM, ta coù
1 1 1 3
+ + ≥
a (a + b) b(b + c ) c (c + a ) 3 abc( a + b)(b + c)(c + a)

Do ñoù, ta chæ caàn chöùng minh


221
2(ab + bc + ca ) ≥ 3 3 abc( a + b)(b + c)(c + a)
⇔ a (b + c) + b(c + a ) + c (a + b) ≥ 3 3 a (b + c ).b(c + a ).c( a + b)

Ñieàu naøy hieån nhieân ñuùng theo bñt AM-GM.


⇒ ñpcm.

b) AÙp duïng baát ñaúng thöùc AM−GM, ta coù


1 1 2 8
+ ≥ ≥
a ( a + b) c ( c + d ) ac (a + b)(c + d ) (a + c)(a + b + c + d )

Töông töï, ta coù


1 1 8
+ ≥
b(b + c) d (d + a ) (b + d )( a + b + c + d )

Do ñoù
1 1 1 1
+ + + ≥
a (a + b) b(b + c) c(c + d ) d (d + a )
8 8
≥ +
(a + c)(a + b + c + d ) (b + d )( a + b + c + d )
8
=
(a + c )(b + d )
8
=
ab + bc + cd + da
Suy ra

 1 1 1 1 
(ab + bc + cd + da )  + + +  ≥ 8 (ñpcm)
 a (a + b) b(b + c ) c (c + d ) d ( d + a ) 
Baøi toaùn 8. (Vasile Cirtoaje)
Cho a, b, c > 0 thoûa abc ≥ 1 . Chöùng minh raèng
a b c
a b .b c .c a ≥ 1
Lôøi giaûi.

Do a, b, c > 0 vaø abc ≥ 1 neân ñaët a = ka / , b = kb / , c = kc / vôùi k ≥ 1, a / , b / , c / > 0 vaø

a / b/ c / = 1 . Khi ñoù, baát ñaúngthöùc caàn chöùng minh töông ñöông vôùi
222
a / b/ c/ a/ b/ c/
+ +
k b/ c/ a/
.a / b/ / c/
.b .c / a/
≥1
Do k ≥ 1 neân ta chæ caàn chöùng minh
a/ b/ c/

a / b .b / c .c / a ≥ 1
/ / /

Do ñoù, khoâng maát tính toång quaùt coù theå giaû söû abc = 1 . Khi ñoù, baát ñaúng thöùc caàn
chöùng minh töông ñöông vôùi
a ln a b ln b c ln c
+ + ≥0
b c a
Coù 2 tröôøng hôïp xaûy ra

* Tröôøng hôïp 1. a ≥ b ≥ c ⇒ ln a ≥ ln b ≥ ln c
a b c
+ Tröôøng hôïp 1.1. 0 < b ≤ 1 ⇒ ≥ ≥
b c a
⇒ Theo baát ñaúng thöùc Chebyshev, ta coù
a ln a b ln b c ln c 1  a b c 
+ + ≥ . + +  (ln a + ln b + ln c ) = 0
b c a 3 b c a

+ Tröôøng hôïp 1.2. b ≥ 1 ⇒ ln b ≥ 0


Ta coù baát ñaúng thöùc caàn chöùng minh töông ñöông vôùi
a ln a b ln b c ln c
+ + ≥ ln a + ln b + ln c
b c a
(a − b) ln a (b − c )ln b (c − a ) ln c
⇔ + + ≥0
b c a
 ln a ln c   ln b ln c 
⇔ ( a − b)  −  + (b − c)  − ≥0
 b a   c a 
(a − b)(a ln a − b ln c) (b − c)(a ln b − c ln c)
⇔ + ≥0
ab ac
ln c ≤ 0
Chuù yù raèng a ≥ b ≥ c vaø abc = 1 neân c ≤ 1 vaø a ≥ b ≥ 1 ⇒ 
ln a ≥ ln b ≥ 0
 a ln a − b ln c ≥ 0
⇒
 a ln b − c ln c ≥ 0

223
(a − b)(a ln a − b ln c) (b − c )( a ln b − c ln c )
⇒ + ≥0
ab ac
* Tröôøng hôïp 2. a ≤ b ≤ c ⇒ c ≥ 1 ≥ a > 0
Theo treân, ta coù baát ñaúng thöùc caàn chöùng minh töông ñöông vôùi
(a − b)(a ln a − b ln c ) (b − c)( a ln b − c ln c )
+ ≥0
ab ac
(b − a )(b ln c − a ln a) (c − b)(c ln c − a ln b)
⇔ + ≥0
ab ac
Do a ≤ b ≤ c neân ln a ≤ ln b ≤ ln c vaø ln c ≥ 0.
b ln c − a ln a ≥ a ln c − a ln a = a (ln c − ln a ) ≥ 0
⇒
c ln c − a ln b ≥ a ln c − a ln b = a (ln c − ln b) ≥ 0
(b − a )(b ln c − a ln a ) (c − b)(c ln c − a ln b)
⇒ + ≥0
ab ac
Toùm laïi, ta luoân coù
a ln a b ln b c ln c
+ + ≥ 0 (ñpcm)
b c a
Ñaúng thöùc xaûy ra khi vaø chæ khi a = b = c = 1 .
Baøi toaùn 9. (Phaïm Kim Huøng)
Cho a1 , a2 ,..., an > 0 thoûa a1a2 ...an = 1 . chöùng minh raèng vôùi moïi k > 0 thì

1 1 1  n
+ + ... + ≥ min 1, k 
(1 + a1 ) k (1 + a2 ) k (1 + an ) k  2 

Lôøi giaûi.
+ Caùch 1.
Ta coù Boå ñeàø sau
Boå ñeà. x1 , x2 ,..., xn laø n soá thöïc döông thoûa maõn

i) x1 ≤ x2 ≤ ... ≤ xn
ii) xi ∈ (−∞, +∞) ∀i = 1, n
iii) x1 + x2 + ... + xn = C

224
vaø f laø moät haøm treân (−∞, +∞) thoûa maõn f loài treân (−∞, c ] vaø loõm treân [c, +∞)

Ñaët F = f ( x1 ) + f ( x2 ) + ... + f ( xn )

Khi ñoù, F ñaït min khi x1 ≤ x2 = x3 = ... = xn .

Chöùng minh.
Giaû söû x1 , x2 ,..., xi ∈ ( −∞, c] , do f loài treân (−∞, c ] neân

f ( x1 ) + f ( x2 ) + ... + f ( xi ) ≥ (i − 1) f (c) + f ( x1 + x2 + ... + xi − (i − 1)c )

Maët khaùc do f loõm treân [c, +∞) neân

 (i − 1)c + xi +1 + xi + 2 + ... + xn 
(i − 1) f (c) + f ( xi +1 ) + f ( xi + 2 ) + ... + f ( xn ) ≥ (n − 1) f  
 n −1 
Do ñoù
n
 (i − 1)c + xi +1 + xi + 2 + ... + xn 
F = ∑ f ( xk ) ≥ (n − 1) f   + f ( x1 + x2 + ... + xi − (i − 1)c)
k =1  n −1 
Boå ñeà ñöôïc chöùng minh.
Trôû laïi baøi toaùn cuûa ta
Neáu n = 1 thì baát ñaúng thöùc ñaõ cho hieån nhieân ñuùng.
Neáu n = 2
+ Neáu 0 < k < 1 thì ta coù
1 1 1 1 1 1
+ ≥ + = + =1
(1 + a1 ) k
(1 + a2 ) 1 + a1 1 + a2 1 + a1 1 + 1
k

a1

+ Neáu k ≥ 1 thì ta coù

1 1 a1k + 1 1
+ = ≥ k −1 (theo baát ñaúng thöùc Holder)
(1 + a1 ) k
(1 + a2 ) k
( a1 + 1) k
2

Xeùt n ≥ 3
n
Ta chöùng minh baát ñaúng thöùc ñuùng cho giaù trò tôùi haïn 1 = ⇔ k = log2 n .
2k
Do n ≥ 3 neân n − 1 > k > 1 .
Khi ñoù
225
+ ∀m ≥ k , ta coù
m m
n
1 1  n
1  1  n k
k n

i =1 (1 + ai )
m
≥ m . ∑ k 
i =1 (1 + ai ) 
≥ m . k  = m
−1  2 
nk 
−1 2
nk
+ ∀m ≤ k , ta coù
k
 n
1  m n
1 n
1
∑ m 
≥ ∑ ≥ 1 ⇒ ∑ ≥1
 i =1 (1 + a i )  i =1 (1 + ai ) k
i =1 (1 + a i ) m

Khoâng maát tính toång quaùt giaû söû 0 < a1 ≤ a2 ≤ ... ≤ an .

 x ≤ x2 ≤ ... ≤ xn
Ñaët x1 = ln a1 , x2 = ln a2 ,..., xn = ln an thì  1
 x1 + x2 + ... + xn = 0 (do a1a2 ...an = 1)
1
Xeùt haøm soá f ( x) =
(e + 1) k
x

Ta coù
ke x .( ke x − 1)
f ( x) =
//

(e x + 1) k + 2
f // ( x) = 0 ⇔ x = − ln k

Töø ñoù, ta coù f loài treân (−∞, − ln k ] vaø loõm treân [ − ln k , +∞)

⇒ Theo Boå ñeà treân, ta coù


n n
1 1
P=∑ = ∑ ñaït min khi x1 ≤ x2 = x3 = ... = xn
i =1 (1 + ai ) i =1 (1 + e )
k xi k

 n −1 1 
⇒ min P ≥ min  t +  (t ≥ 0)
 (e + 1)
k
(e − ( n−1) t + 1) k 
 n −1 x ( n−1) k 
= min  + n −1 k 
( x = et ≥ 1) (1)
+
 ( x 1)
k
(x + 1) 

n −1 x ( n−1) k
Tieáp theo, ta seõ tìm min cuûa haøm soá g ( x) = + vôùi x ≥ 1
( x + 1) k ( x n−1 + 1) k

226
 x ( n−1) k −1 1 
Ta coù g ( x) = ( n − 1) k  n−1
/
k +1
− 
 ( x + 1) ( x + 1) k +1 

g / ( x) = 0 ⇔ x ( n−1) k −1.( x + 1) k +1 = ( x n−1 + 1) k +1


( n −1) k −1
⇔x k +1
.( x + 1) = x n −1 + 1 (2)
1
Ñaët t = x k +1 ⇒ t ≥ 1 . Khi ñoù, phöông trình (2) trô ûthaønh
t ( n−1) k −1.(t k +1 + 1) = t ( n−1)( k +1) + 1
⇔ t ( n−1)( k +1) − t nk − t ( n−1) k −1 + 1 = 0
Xeùt haøm soá h(t ) = t ( n −1)( k +1) − t nk − t ( n −1) k −1 + 1 vôùi t ≥ 1

Ta coù h / (t ) = t ( n −1) k − 2 .(( n − 1)( k + 1)t n − nkt k +1 − (n − 1)k + 1)

h / (t ) = 0 ⇔ (n − 1)(k + 1)t n − nkt k +1 − (n − 1)k + 1 = 0

Xeùt tieáp haøm soá m(t ) = (n − 1)(k + 1)t n − nkt k +1 − (n − 1) k + 1 vôùi t ≥ 1

Ta coù m / (t ) = n( k + 1)t k ((n − 1)t n − k −1 − k )

Chuù yù raèng n − 1 > k neân m / (t ) ≥ n(k + 1)t k ((n − 1) − k ) > 0

⇒ m(t ) laø haøm ñoàng bieán treân [1, +∞)

Ta laïi coù m(1) = (n − 1)(k + 1) − nk − ( n − 1)k + 1 = n(1 − k ) < 0, lim m(t ) = +∞


t →+∞

Neân phöông trình m(t ) = 0 coù nghieäm duy nhaát t0 > 1

⇒ Phöông trình h / (t ) = 0 coù nghieäm duy nhaát t0 > 1

Baûng bieán thieân cuûa h(t )

t 1 t0 +∞

h / (t ) _ 0 +
0 +∞
h(t )

Caên cöù vaøo baûng bieán thieân, ta coù

227
h(t ) = 0 coù 2 nghieäm phaân bieät laø 1 vaø t1 > t0 > 1

Do ñoù g / ( x) = 0 coù 2 nghieäm phaân bieät laø 1 vaø t1k +1 > 1 .

Baûng bieán thieân cuûa g ( x)

x 1 t1k +1 +∞
g / ( x) + 0 _

g ( x)
1 1

Caên cöù vaøo baûng bieán thieân, ta suy ra

{ }
g ( x ) ≥ min g (1), lim g ( x ) = 1 ∀x ≥ 1
x →∞
(3)

Töø (1) vaø (3), ta suy ra ñpcm.

+ Caùch 2.
1
Ñaët f (t ) =
(t + 1) k

Ta coù boå ñeà sau

Boå ñeà. Neáu 0 < a ≤ b ≤ c ≤ d vaø ad = bc thì


f (a ) + f ( d ) ≥ min { f (b) + f (c),1}

Chöùng minh.
−k
m m 
Ñaët m = ad = bc vaø g (t ) = f ( mt ) + f   = (mt + 1) − k +  + 1 vôùi moïi soá
t  t 
c d
döông t . Ñaët t1 = , t2 = . Ta coù 1 ≤ t1 ≤ t2 .
m m
Ñeå chöùng minh Boå ñeà, ta caàn chöùng minh g (t2 ) ≥ min { g (t1 ),1}

Deã thaáy lim g (t ) = 1.


t →+∞

Xeùt tính ñôn ñieäu cuûa haøm g treân khoaûng [1, +∞ ) , ta coù

228
/
 − k −1 1 m 
− k −1

g (t ) = mk  −(mt + 1) + 2 .  + 1 
 t  t 
 
k +1
k +1 m 
g (t ) > 0 ⇔ ( mt + 1)
/
> t .  + 1
2

t 
2 1− k
⇔ t k +1 − mt + mt 1+k − 1 < 0
2 1− k
Xeùt haøm soá h(t ) = t +1
k − mt + mt +k
1 − 1 vôùi t ≥ 1.

Ta coù
h(1) = 0
k −1 −2 k
2 k +1 1− k
h (t ) =
/
.t − m + .mt k +1
k +1 1+ k
2k  1 
h / (1) = . − m 
k +1  k 
1+3 k
2(1 − k ) − 1+ k
h // (t ) = .t .(t − mk )
(1 + k ) 2

Tuøy thuoäc vaøo caùc giaù trò cuûa m vaø k , ta coù caùc tröôøng hôïp sau

* Tröôøng hôïp 1. k = 1, m ≤ 1 . Khi ñoù, ta coù h(t ) = (1 − m)(t − 1) ≥ 0 ∀t > 1 , do ñoù

h ≥ 0 treân khoaûng (1, +∞).

* Tröôøng hôïp 2. k = 1, m > 1 . Khi ñoù, ta coù h(t ) = (1 − m)(t − 1) < 0 ∀t > 1 , do ñoù

h < 0 treân khoaûng (1, +∞).

1
* Tröôøng hôïp 3. k < 1, m ≤ . Khi ñoù, ta coù h // > 0 ∀t > 1, vì h / (1) ≥ 0 vaø t > 1 ,
k

neân h / > 0 treân khoaûng (1, +∞) . Vì h(1) = 0 neân h > 0 treân khoaûng (1, +∞) .

1
* Tröôøng hôïp 4. k < 1, m > . Khi ñoù, ta coù h / (1) < 0 vaø h // < 0 treân (1, mk ) . Do
k

ñoù suy ra h / < 0 treân (1, mk ). Vì h(1) = 0 neân h < 0 treân (1, mk ] . Treân khoaûng

(mk , +∞) , ta coù h // > 0, coù nghóa h laø haøm loõm treân ( mk , +∞). Vì h(mk ) < 0 vaø

229
lim h(t ) = +∞ neân toàn taïi duy nhaát moät soá thöïc p > 1 sao cho h < 0 treân (1, p) vaø
t →+∞

h > 0 treân ( p, +∞).

Trong caùc tröôøng hôïp noùi treân

+ Neáu h(t2 ) ≥ 0 thì h ≥ 0 treân (t2 , +∞) , töùc laø g / ≤ 0. Suy ra haøm g ñôn

ñieäu giaûm treân khoaûng [t2 , +∞ ) vaø g (t2 ) ≥ lim g (t ) = 1.


t →+∞

+ Neáu h(t2 ) ≤ 0 thì h ≥ 0 treân khoaûng (1, t2 ), hay g / ≥ 0. Vaäy g laø haøm

ñôn ñieäu taêng, suy ra g (t1 ) ≤ g (t2 ).

Boå ñeà ñöôïc chöùng minh hoaøn toaøn.


Trôû laïi baøi toaùn cuûa ta
Ta seõ chöùng minh baát ñaúng thöùc ñaõ cho baèng quy naïp theo n .
Neáu n = 1 thì baát ñaúng thöùc ñaõ cho trôû thaønh ñaúng thöùc.
Xeùt n ≥ 2. Goïi m laø trung bình nhaân cuûa a1 , a2 ,..., an thì ta coù m = 1 . Ta coù, baát

ñaúng thöùc caàn chöùng minh töông ñöông vôùi


f (a1 ) + f (a2 ) + ... + f ( an ) ≥ min {nf (m),1}

 1 
Neáu n = 2 thì ta coù min {2 f (m),1} = min  k −1 ,1
2 
+ Neáu 0 < k < 1 thì ta coù
1 1 1 1 1 1
+ ≥ + = + =1
(1 + a1 ) k
(1 + a2 ) k
1 + a1 1 + a2 1 + a1 1 + 1
a1

+ Neáu k ≥ 1 thì ta coù

1 1 a1k + 1 1
+ = ≥ k −1 ( theo baát ñaúng thöùc Holder)
(1 + a1 ) k
(1 + a2 ) k
(a1 + 1) k
2

Vaäy khaúng ñònh ñuùng khi n = 2 .

230
Giaû söû khaúng ñònh ñuùng cho soá caùc bieán beù hôn n ( n ≥ 2). Ta seõ chöùng minh

khaúng ñònh ñuùng cho soá bieán baèng n . Deã thaáy raèng trong daõy a1 , a2 ,..., an luoân

chöùa ít nhaát moät soá khoâng lôùn hôn m vaø ít nhaát moät soá khoâng nhoû hôn m . Khoâng
maát tính toång quaùt, ta coù theå giaû söû a1 ≤ m ≤ a2 .

 aa   aa 
Kyù hieäu x1 = min  m, 1 2  , x2 = max m, 1 2  .
 m   m 

Khi ñoù, ta coù a1 ≤ x1 ≤ x2 ≤ a2 vaø x1 x2 = a1a2 . Töø ñaây, theo keát quaû cuûa boå ñeà treân,

ta coù

 aa  
f (a1 ) + f (a2 ) ≥ min { f ( x1 ) + f ( x2 ),1} = min  f (m) + f  1 2  ,1
  m  
a1a2
Trung bình nhaân cuûa , a3 ,..., an cuõng baèng m vaø soá bieán laø n − 1 < n neân theo
m
giaû thieát quy naïp, ta coù

aa 
f  1 2  + f ( a3 ) + ... + f (an ) ≥ min {( n − 1) f ( m),1}
 m 
Suy ra
f (a1 ) + f (a2 ) + ... + f (an ) ≥
 aa  
≥ min  f ( m) + f  1 2  ,1 + f ( a3 ) + ... + f ( an )
  m  

 aa  
= min  f (m) + f  1 2  + f ( a3 ) + ... + f (an ),1 + f ( a3 ) + ... + f ( an ) 
  m  
 aa  
≥ min  f (m) + f  1 2  + f (a3 ) + ... + f (an ),1
  m  
≥ min { f ( m) + min {(n − 1) f ( m),1} ,1}
≥ min {nf ( m),1}
⇒ f (a1 ) + f (a2 ) + ... + f (an ) ≥ min {nf (m),1}
⇒ khaúng ñònh ñuùng vôùi soá bieán soá baèng n.

231
Theo nguyeân lyù quy naïp, ta suy ra khaúng ñònh ñuùng vôùi moïi n. Ñaây chính laø ñieàu
ta caàn phaûi chöùng minh.
Baøi toaùn 10. (Moldova 1999)
Cho a, b, c > 0 . Chöùng minh raèng

ab bc ca a b c
+ + ≥ + +
c(c + a ) a (a + b) b(b + c ) c + a a + b b + c

Lôøi giaûi.
a b c
Ñaët x = , y = , z = thì ta coù x, y , z > 0 vaø xyz = 1 .
b c a
Khi ñoù, ta coù
b
ab y

cyc c( c + a )
=∑ c
c
=∑
cyc 1 + z
cyc 1+
a
a 1 1

cyc c + a
=∑
c
=∑
cyc 1 + z
cyc 1+
a
Do ñoù, baát ñaúng thöùc caàn chöùng minh töông ñöông vôùi
x y z 1 1 1
+ + ≥ + +
y +1 z +1 x +1 x +1 y +1 z +1

x
∑ (1 + x)(1 + y)
⇔∑ ≥ cyc
cyc y + 1 (1 + x)(1 + y )(1 + z )
3 + 2∑ x + ∑ xy
x
⇔∑ ≥ cyc cyc

cyc y + 1 2 + ∑ x + ∑ xy
cyc cyc

Aùp duïng baát ñaúng thöùc Bunhiacopxki, ta coù


2 2
   
x2
 ∑ x  ∑ x
x  cyc   cyc 
∑ y + 1 = ∑ x + xy ≥ ( x + xy ) =
cyc cyc ∑ ∑ x + ∑ xy
cyc cyc cyc

232
Do ñoù, ñeå chöùng minh baát ñaúng thöùc ñaõ cho, ta chæ caàn chöùng minh
2
 
∑ x 3 + 2∑ x + ∑ xy
 cyc  ≥ cyc cyc

∑ x + ∑ xy 2 + ∑ x + ∑ xy
cyc cyc cyc cyc
2
      
⇔  ∑ x   2 + ∑ x + ∑ xy  ≥  ∑ x + ∑ xy  3 + 2∑ x + ∑ xy 
 cyc   cyc cyc   cyc cyc  cyc cyc 
3 2 2
          
⇔  ∑ x  +  ∑ x   ∑ xy  ≥ 3∑ x + 3∑ xy + 3  ∑ x  ∑ xy  +  ∑ xy 
 cyc   cyc   cyc  cyc cyc  cyc  cyc   cyc 
Söû duïng baát ñaúng thöùc AM-GM vaø giaû thieát, ta deã daøng chöùng minh ñöôïc caùc baát
ñaúng thöùc sau


cyc
x≥3

∑ xy ≥ 3
cyc
2
   
 ∑ x  ≥ 3  ∑ xy 
 cyc   cyc 
Do ñoù, ta coù
3 2
       
 ∑ x  =  ∑ x  ∑ x  ≥ 3  ∑ x  ∑ xy 
 cyc   cyc  cyc   cyc  cyc 
3
    
⇒ 3  ∑ x  ≥ 9  ∑ x  ∑ xy  (1)
 cyc   cyc  cyc 
2
       
 ∑ x   ∑ xy  =  ∑ x  ∑ x  ∑ xy  ≥ 9∑ x (2)
 cyc   cyc   cyc  cyc  cyc  cyc
2
   
 ∑ x   ∑ xy  ≥ 9∑ xy (3)
 cyc   cyc  cyc
2 2
     
 ∑ x   ∑ xy  ≥ 3  ∑ xy  (4)
 cyc   cyc   cyc 

233
Coäng caùc baát ñaúng thöùc (1), (2), (3) vaø (4) veá theo veá, roài chia caû hai veá cho 3, ta
thu ñöôïc
3 2 2
          
 ∑ x  +  ∑ x   ∑ xy  ≥ 3∑ x + 3∑ xy + 3  ∑ x  ∑ xy  +  ∑ xy  (ñpcm)
 cyc   cyc   cyc  cyc cyc  cyc  cyc   cyc 
Ñaúng thöùc xaûy ra khi vaø chæ khi x = y = z = 1 ⇔ a = b = c.

Baøi toaùn 11.


Cho a, b, c ∈ R thoûa (a − b)(b − c)(c − a ) ≠ 0 . Chöùng minh raèng

(1 − a 2 )(1 − b 2 ) (1 − b 2 )(1 − c 2 ) (1 − c 2 )(1 − a 2 )


+ + ≥ −1
( a − b) 2 (b − c ) 2 (c − a ) 2

Lôøi giaûi.
Ta coù baát ñaúng thöùc caàn chöùng minh töông döông vôùi

 (1 − a 2 )(1 − b 2 ) 
∑ 
cyc  ( a − b) 2
+ 1 ≥ 2

 1 − ab 
2

⇔ ∑  ≥2 (*)
cyc  a − b 

1 − ab 1 − bc 1 − ca
Ñaët x = ,y = ,z = thì ta coù
a −b b−c c−a
( x + 1)( y + 1)( z + 1) = ( x − 1)( y − 1)( z − 1)
⇒ xy + yz + zx = −1

Khi ñoù
(*) ⇔ x 2 + y 2 + z 2 ≥ −2( xy + yz + zx)
⇔ ( x + y + z )2 ≥ 0 (ñuùng)
⇒ ñpcm.

234
Baøi toaùn 12.
Cho a, b, c > 0 . Chöùng minh raèng
3
c−3a 3a−3b 3b−3c
+ + ≥0
2a + b 2b + c 2c + a
Lôøi giaûi.
Khoâng maát tính toång quaùt coù theå giaû söû a = max {a, b, c}

Coù 2 tröôøng hôïp xaûy ra


* Tröôøng hôïp 1. a ≥ b ≥ c

2a + b ≥ 2b + c > 0
Khi ñoù, ta coù  . Do ñoù
 2 a + b ≥ 2c + a > 0
3
c−3a 3a−3b 3b−3c 3c−3a 3a−3b 3b−3c
+ + ≥ + + =0
2a + b 2b + c 2c + a 2a + b 2a + b 2a + b
* Tröôøng hôïp 2. a ≥ c ≥ b

2c + a ≥ 2b + c > 0
Khi ñoù, ta coù  . Do ñoù
2a + b ≥ 2b + c > 0
3
c−3a 3a−3b 3b−3c 3c−3a 3a−3b 3b−3c
+ + ≥ + + =0
2a + b 2b + c 2c + a 2b + c 2b + c 2b + c
Toùm laïi, ta luoân coù
3
c−3a 3a−3b 3b−3c
+ + ≥ 0 (ñpcm)
2a + b 2b + c 2c + a
Ñaúng thöùc xaûy ra khi vaø chæ khi a = b = c.

* Nhaän xeùt.
Baèng caùch laøm hoaøn toaøn töông töï, ta coù

cn − an an − bn bn − cn
+ + ≥ 0 ∀a, b, c, n > 0
2a + b 2b + c 2c + a

235
Baøi toaùn 13.
Cho a, b, c > 0 . Chöùng minh raèng

2a n − b n − c n

cyc b − bc + c
2 2
≥0

trong ñoù n > 0 laø haèng soá cho tröôùc.

Lôøi giaûi.
Khoâng maát tính toång quaùt giaû söû a ≥ b ≥ c > 0 .
Ta coù baát ñaúng thöùc caàn chöùng minh töông ñöông vôùi

 2 1 1 
(a n − b n )  2 − 2 − 2 2 
+
 b − bc + c a − ac + c a − ab + b 
2 2

 1 1 2 
+ (b n − c n )  2 + 2 − 2 2 
≥0
 b − bc + c a − ac + c a − ab + b 
2 2

a − ab + b ≥ b − bc + c > 0
2 2 2 2
Do a ≥ b ≥ c > 0 neân  2
a − ac + c ≥ b − bc + c > 0
2 2 2

2 1 1
⇒ − 2 − 2 ≥0
b − bc + c
2 2
a − ac + c a − ab + b 2
2

Do ñoù, ta chæ caàn chöùng minh


1 1 2
+ 2 − 2 ≥0
b − bc + c
2 2
a − ac + c a − ab + b 2
2

1 1 1 1
⇔ 2 − ≥ −
b − bc + c 2 a 2 − ab + b 2 a 2 − ab + b 2 a 2 − ac + c 2
⇔ ( a − c )(a + c − b)(a 2 − ac + c 2 ) ≥ (b − c)(a − b − c)(b 2 − bc + c 2 )

+ Neáu a ≤ b + c thì ta coù ngay ñpcm.


a − c ≥ b − c ≥ 0

+ Neáu a > b + c thì ta coù a + c − b ≥ a − b − c > 0
 2
a − ac + c ≥ b − bc + c > 0
2 2 2

⇒ ( a − c)(a + c − b)(a 2 − ac + c 2 ) ≥ (b − c)(a − b − c)(b 2 − bc + c 2 )


⇒ ñpcm.

236
Ñaúng thöùc xaûy ra khi vaø chæ khi a = b = c.

Baøi toaùn 14. (Toaùn Hoïc Tuoåi Treû 2006)


k k
Cho a1 , a2 ,..., an laø caùc soá thöïc döông thoûa maõn ∑ ai ≤ ∑ i(i + 1) ∀k = 1, n . Chöùng
i =1 i =1

minh raèng
1 1 1 n
+ + ... + ≥
a1 a2 an n + 1

Lôøi giaûi.
Tröôùc heát, ta chöùng minh boå ñeà sau

Boå ñeà. Cho n ≥ 3, n ∈ N . Khi ñoù, vôùi hai daõy soá thöïc ( xn ) vaø ( yn ), ta coù
n n −1 i n

∑ xi yi = ∑ ( xi − xi+1 ).∑ y j + xn .∑ yi
i =1 i =1 j =1 i =1

Chöùng minh.
Ta coù
n −1 i n

∑ ( xi − xi +1 ).∑ y j + xn .∑ yi =
i =1 j =1 i =1
n −1 i n −1 i n
= ∑ xi .∑ y j − ∑ xi +1.∑ y j + xn .∑ yi
i =1 j =1 i =1 j =1 i =1
n −1 i n−2 i n −1 n
= x1 y1 + ∑ xi .∑ y j − ∑ xi +1.∑ y j − xn .∑ yi + xn .∑ yi
i =2 j =1 i =1 j =1 i =1 i =1
n −1 i

n −1

i −1 n n −1
= x1 y1 + ∑ xi .∑ y j − ∑ xi .∑ y j + xn . ∑ yi − ∑ yi 
i =2 j =1 i =2 j =1  i =1 i =1 
 i
n −1 i −1 
= x1 y1 + ∑ xi .  ∑ y j − ∑ y j  + xn yn
i=2  j =1 j =1 
n −1
= x1 y1 + ∑ xi yi + xn yn
i =2
n
= ∑ xi yi
i =1

237
n n −1 i n
Vaäy ∑ xi yi = ∑ ( xi − xi +1 ).∑ y j + xn .∑ yi . Boå ñeà ñöôïc chöùng minh.
i =1 i =1 j =1 i =1

Trôû laïi baøi toaùn cuûa ta


1 1
+ Neáu n = 1 thì hieån nhieân ta coù ≥ (do giaû thieát a1 ≤ 2) (1)
a1 2

a ≤ 2 a ≤ 2
+ Neáu n = 2 thì theo giaûthieát, ta coù  1 ⇒ 1
a1 + a2 ≤ 8 a2 ≤ 8 − a1
Do ñoù
1 1 1 1
+ ≥ +
a1 a2 a1 8 − a1

8
=
a1 (8 − a1 )
8
=
− a12 + 8a1
8
=
−(a1 − 2) + 4a1 + 4
2

8

4.2 + 4
2
= (2)
3
+ Neáu n ≥ 3
AÙp duïng baát ñaúng thöùc Bunhiacopxki, ta coù
2 2
 1   n 1 
 
i (i + 1)   ∑ 
  i =1 i (i + 1)  n2
n n
1 1
∑ = ∑
i =1  
≥ n
ai
=
( n + 1) 2
. n
ai
 i 2 (i + 1) 2  ∑ i 2 (i + 1) 2 ∑
i =1 ai ai
i =1 i (i + 1)
2 2
  i =1

1
AÙp duïng boå ñeà vôùi xi = ; yi = ai ∀i = 1, 2,..., n, ta coù
i (i + 1)
n
a
∑ i 2 (i +i 1)2 =
i =1

238
n −1
 1 1  i 1 n
= ∑ 2 − 2  ∑ j
. a + 2 ∑ i
. a
i =1  i (i + 1) 2
(i + 1) 2
(i + 2)  j =1 n 2
( n + 1) i =1


n −1
1 1  i 1 n
≤ ∑ 2 − .∑ j ( j + 1) + 2 .∑ i(i + 1) ( gt)
i =1  i (i + 1)
2
(i + 1) 2 (i + 2) 2  j =1 n ( n + 1) 2 i =1

1
Laïi aùp duïng boå ñeà vôùi xi = , yi = i (i + 1) ∀i = 1, 2,..., n, ta coù
i (i + 1) 2
2

n n
1 1
∑ i(i + 1) = ∑ i 2 (i + 1)2 .i(i + 1)
i =1 i =1


n −1
1 1  i 1 n
= ∑ 2 −  .∑ j ( j + 1) + 2 .∑ i (i + 1)
i =1  i (i + 1) (i + 1) 2 (i + 2) 2  j =1 n (n + 1) 2 i =1
2

n n
ai 1 n
Do ñoù ∑ i 2 (i + 1)2 ∑ i(i + 1) = n + 1

i =1 i =1

Suy ra
n
1 n2 1 n2 1 n
∑ a (n + 1)2 .
≥ n
a
≥ 2
.
(n + 1)  n  n + 1
=
i =1 i
∑ i 2 (i +i 1)2  n +1
 
i =1

n
1 n
Vaäy ∑
i =1 a

n +1
(3)
i

Töø (1), (2) vaø (3), ta suy ra ñpcm.


Ñaúng thöùc xaûy ra khi vaø chæ khi ai = i (i + 1) ∀i = 1, 2,..., n.

* Nhaän xeùt.
Baèng caùch laøm hoaøn toaøn töông töï, ta coù keát quaû sau
b1 ≤ b2 ≤ ... ≤ bn

(an ),(bn ) laø hai daõy soá thöïc döông thoûa maõn  k k .
 ∑ ai ≤ ∑ bi ∀k = 1, n
 i =1 i =1

Khi ñoù, ta coù


n n
1 1

i =1 ai
≥ ∑
i =1 bi

239
Baøi toaùn 15. (Phaïm Kim Huøng)
Cho n ≥ 4, n ∈ N vaø a1 , a2 ,..., an > 0 thoûa a1a2 ...an = 1. Chöùng minh raèng

1 1 1 3n
+ + ... + + ≥ n +3
a1 a2 an a1 + a2 + ... + an

Lôøi giaûi.
+ Caùch 1.
1 1 1 3n
Ñaët f (a1, a2 ,..., an ) = + + ... + + . Ta caàn tìm min f .
a1 a2 an a1 + a2 + ... + an

Giaû söû vôùi boä soá ( x1 , x2 ,..., xn ) thoaû x1 x2 ...xn = 1 thì f ( x1, x2 ,..., xn ) = min f .

Khoâng maát tính toång quaùt giaû söû 0 < x1 ≤ x2 ≤ ... ≤ xn .

Ta chöùng minh raèng ñeå f ( x1, x2 ,..., xn ) = min f thì x1 = x2 = ... = xn−1.

Thaät vaäy, giaû söû x1 = x2 = ... = xi (1 ≤ i ≤ n − 3) . Ta chöùng minh xi +1 = xi = ... = x1 .

Giaû söû ngöôïc laïi xi +1 > xi . Ta seõ chöùng minh

(
f ( x1,..., xi , xi +1,..., xn ) > f x1 ,..., xi −1 , xi xi +1 , xi xi +1 , xi + 2 ,..., xn )
1 1 1 1 3n 1 1
⇔ + ... + + + ... + + > + ... + +
x1 xi xi +1 xn x1 + ... + xi + xi +1 + ... + xn x1 xi −1
2 1 1 1 3n
+ + + + ... + +
xi xi +1 xi + 2 xi +3 xn x1 + ... + xi −1 + 2 xi xi +1 + xi + 2 + ... + xn

( ) ( x− x )
2 2
xi − xi +1 3n i i +1
⇔ >
xi xi +1 ( x1 + ... + xi + xi +1 + ... + x ) ( x + ... + x + 2
n 1 i −1 xi xi +1 + xi + 2 + ... + xn )
(
⇔ ( x1 + ... + xi + xi +1 + ... + xn ) x1 + ... + xi −1 + 2 xi xi +1 + xi + 2 + ... + xn > 3nxi xi +1 )
(
⇔ (ixi + xi +1 + xi + 2 + ... + xn ) (i − 1) xi + 2 xi xi +1 + xi + 2 + ... + xn > 3nxi xi +1 )
Ta coù
 xn ≥ xn−1 ≥ ... ≥ xi +1 > xi = xi −1 = ... = x1 > 0

1 ≤ i ≤ n − 3

240
ixi + xi +1 + xi + 2 + ... + xn ≥ ixi + (n − i ) xi +1 > ( n − i ) xi +1 ≥ 3 xi +1 > 0
⇒
(i − 1) xi + 2 xi xi +1 + xi + 2 + ... + xn > nxi > 0
( )
⇒ (ixi + xi +1 + xi + 2 + ... + xn ) (i − 1) xi + 2 xi xi +1 + xi + 2 + ... + xn > 3nxi xi +1

( )
Vaäy f ( x1,..., xi , xi +1,..., xn ) > f x1 ,..., xi −1 , xi xi +1 , xi xi +1 , xi + 2 ,..., xn . Ñieàu naøy voâ lyù

vì f ( x1, x2 ,..., xn ) = min f . Vaäy ta phaûi coù xi +1 = xi = ... = x1 .

Baèng laäp luaän töông töï, ta ñi ñeán keát quaû sau ñeå f ( x1, x2 ,..., xn ) = min f thì ta

phaûi coù xn− 2 = xn−3 = ... = x1.

Tieáp theo, ta seõ chöùng minh xn−1 = xn −2 = ... = x1.

Giaû söû ngöôïc laïi xn−1 > xn− 2

Khi ñoù, ta seõ chöùng minh

(
f ( x1, x2 ,..., xn− 2 , xn−1 , xn ) > f x1 , x2 ,..., xn−3 , xn−2 xn−1 , xn− 2 xn−1 , xn )
( )
⇔ (( n − 2) xn− 2 + xn−1 + xn ) ( n − 3) xn− 2 + 2 xn− 2 xn−1 + xn > 3nxn− 2 xn−1

Do xn ≥ xn−1 > xn −2 = xn−3 = ... = x1 > 0 neân

(n − 2) xn− 2 + xn−1 + xn ≥ 2 xn−1 + (n − 2) xn−2 > 0



(n − 3) xn−2 + 2 xn− 2 xn−1 + xn > xn−1 + (n − 1) xn −2 > 0
(
⇒ (( n − 2) xn− 2 + xn−1 + xn ) ( n − 3) xn− 2 + 2 xn −2 xn−1 + xn > )
> (2 xn−1 + (n − 2) xn−2 )( xn−1 + ( n − 1) xn− 2 )

Do ñoù, ta chæ caàn chöùng minh


(2 xn−1 + (n − 2) xn−2 )( xn−1 + ( n − 1) xn− 2 ) > 3nxn−1 xn− 2
⇔ 2 xn2−1 − 4 xn−1 xn− 2 + (n 2 − 3n + 2) xn2−2 > 0 (ñuùng)

(
Vaäy ta coù f ( x1, x2 ,..., xn− 2 , xn−1 , xn ) > f x1 , x2 ,..., xn−3 , xn− 2 xn−1 , xn−2 xn−1 , xn . )
Ñieàu naøy voâ lyù vì f ( x1, x2 ,..., xn ) = min f . Vaäy ta phaûi coù xn−1 = xn −2 = ... = x1.

Nhö vaäy, ta ñi ñeán keát quaû


Ñeå f ( x1, x2 ,..., xn ) = min f thì xn ≥ xn−1 = xn− 2 = ... = x1.

241
⇒ min f (a1, a2 ,..., an ) ≥
1 1 1 
≥ min f  , ,..., , x n−1  ( x ≥ 1)
x x x 
 1 3nx 
= min (n − 1) x + n−1 + n  ( x ≥ 1) (1)
 x x + n − 1
1 3nx
Tieáp theo, ta seõ tìm min cuûa haøm soá g ( x ) = (n − 1) x + n −1
+ vôùi x ≥ 1
x x + n −1
n

( n − 1)( x n − 1)( x 2 n − (n + 2) x n + ( n − 1) 2 )
Ta coù g ( x) =
/

x n ( x n + n − 1) 2

Theo baát ñaúng thöùc AM-GM, ta coù


x 2n + (n − 1) 2 ≥ 2( n − 1) x n ≥ (n + 2) x n (do n ≥ 4)

⇒ g / ( x ) ≥ 0 ∀x ≥ 1
⇒ g ( x ) ñoàng bieán treân [1,+∞ ) .

⇒ g ( x ) ≥ g (1) = n + 3 ∀x ≥ 1 (2)

Töø (1) vaø (2), ta suy ra


min f ( a1 , a2 ,..., an ) ≥ n + 3
1 1 1 3n
⇒ + + ... + + ≥ n + 3 (ñpcm)
a1 a2 an a1 + a2 + ... + an

Ñaúng thöùc xaûy ra khi vaø chæ khi a1 = a2 = ... = an = 1.

+ Caùch 2.
Ta seõ chöùng minh keát quaû maïnh hôn nhö sau
Cho n soá thöïc döông a1 , a2 ,..., an thoûa a1a2 ...an = 1. Khi ñoù, ta coù

1 1 1 k k
+ + ... + + ≥ n+
a1 a2 an a1 + a2 + ... + an n

Vôùi k = 4( n − 1) (noùi rieâng neáu n ≥ 4 thì k ≥ 3n )

Ta seõ chöùng minh baèng doàn bieán. n = 1, n = 2 laø caùc tröôøng hôïp taàm thöôøng neân ôû

ñaây ta seõ khoâng xeùt tôùi. Ta seõ xeùt tröôøng hôïp n ≥ 3 .

242
1 1 1 k k
Ñaët f (a1 , a2 ,..., an ) = + + ... + + −n−
a1 a2 an a1 + a2 + ... + an n

Ta coù Nhaän xeùt sau


a ≤ x ≤ a2
(i) Neáu  1 thì
a1a2 ≤ 1

 aa 
f (a1 , a2 ,..., an ) ≥ f  x, 1 2 ,..., an 
 x 
  n  n 
(ii) Neáu (1 − a1 )(1 − a2 )  ka1a2 −  ∑ ai  ∑ ai + a1a2 + 1  ≥ 0 thì
  i=1  i =3 
f (a1 , a2 ,..., an ) ≥ f (1, a1a2 ,..., an )

Chöùng minh Nhaän xeùt.


n
Ñeå tieän vieäc trình baøy xin ñöôïc kyù hieäu A = ∑ ai .
i=3

(i) Ta coù
 aa 
f (a1 , a2 ,..., an ) − f  x, 1 2 ,..., an  =
 x 
1 1 1 x k k
= + − − + −
a1 a2 x a1a2 a1 + a2 + A x + a1a2 + A
x

  aa  
( x − a1 )(a2 − x ) ( a1 + a2 + A)  x + 1 2 + A  − ka1a2 
  x  
=
a1a2 (a1 + a2 + A)( x + Ax + a1a2 )
2

AÙp duïng baát ñaúng thöùc AM-GM, ta coù

 aa 
(a1 + a2 + A)  x + 1 2 + A  ≥ n 2 ≥ 4( n − 1) = k ≥ ka1a2
 x 
Do ñoù

 aa 
f (a1 , a2 ,..., an ) ≥ f  x, 1 2 ,..., an 
 x 
(i) ñöôïc chöùng minh.
243
(ii) Khai trieån töông töï nhö treân, ta coù
f (a1 , a2 ,..., an ) − f (1, a1a2 , a3 ,..., an ) =
(1 − a1 )(1 − a2 )[( a1 + a2 + A)(1 + a1a2 + A) − ka1a2 ]
=
a1a2 ( a1 + a2 + A)(1 + A + a1a2 )
≥0
⇒ f ( a1 , a2 ,..., an ) ≥ f (1, a1a2 , a3 ,..., an )

(ii) ñöôïc chöùng minh.


Nhaän xeùt ñöôïc chöùng minh hoaøn toaøn. Löu yù raèng caùc bieán a1 , a2 ,..., an bình ñaúng

neân a1 , a2 trong Nhaän xeùt coù theå thay baèng ai , a j (i ≠ j ) tuøy yù.

Trôû laïi baøi toaùn cuûa ta


Ta seõ chöùng minh raèng luoân coù theå ñöa veà tröôøng hôïp trong n bieán coù n − 1 bieán
khoâng lôùn hôn 1.
Thaät vaäy, giaû söû trong n bieán coù nhieàu hôn 1 bieán lôùn hôn 1, khoâng maát tính toång
quaùt, ta coù theå giaû söû laø a1 , a2 . Xeùt 2 tröôøng hôïp

 n  n 
* Tröôøng hôïp 1. ka1a2 ≥  ∑ ai  ∑ ai + a1a2 + 1 . Khi ñoù, theo Nhaän xeùt (ii), ta
 i=1  i=3 

coù
f (a1 , a2 ,..., an ) ≥ f (1, a1a2 , a3 ,..., an )

Nhö vaäy, ta coù theå thay boä soá (a1 , a2 ,..., an ) bôûi boä soá (1, a1a2 , a3 ,..., an ) ñeå f

khoâng taêng. Khi ñoù, soá bieán baèng 1 taêng leân ít nhaát laø 1.
 n  n 
* Tröôøng hôïp 2. ka1a2 ≤  ∑ ai  ∑ ai + a1a2 + 1 .
 i =1  i =3 

Khi ñoù, vôùi moïi a j < 1 ≤ a2 ( a j luoân toàn taïi vì a1a2 ...an = 1 ) ta ñeàu coù

 n  
ka1a j ≤  ∑ ai   ∑ ai + a1a j + 1
 i=1   i ≠1,i≠ j 

Thaät vaäy, ta coù

244
n
∑ ai + a1a j + 1 ∑ ai + a1a j + 1 − a1 − a j
i ≠1,i ≠ j
= i =1
ka1a j ka1a j
n
∑ ai + 1 − a1 − a j
= i =1
+1
ka1a j
n
∑ ai + 1 − a1 − a2
≥ i =1
+1
ka1a2
n
∑ ai + a1a2 + 1
= i =3
ka1a2
 n  
⇒ ka1a j ≤  ∑ ai   ∑ ai + a1a j + 1
 i=1   i ≠1,i≠ j 

  n  
Do ñoù (1 − a1 )(1 − a j )  ka1a j −  ∑ ai   ∑ ai + a1a j + 1  ≥ 0 . Söû duïng Nhaän xeùt
  i =1   i≠1,i ≠ j  

(ii), ta coù theå thay boä soá (a1 , a2 ,..., an ) bôûi boä soá (1, a2 ,..., a1a j ,..., an ) ñeå f khoâng

taêng. Khi ñoù soá bieán baèng 1 cuõng taêng leân ít nhaát laø 1.
Toùm laïi, neáu vaãn coøn 2 bieán lôùn hôn 1 thì ta luoân coù theå thay boä soá ñang xeùt bôûi
moät boä soá khaùc maø soá bieán baèng 1 taêng leân ít nhaát laø 1 ñeå f khoâng taêng. Vieäc

thay theá naøy chæ coù theå ñöôïc thöïc hieän khoâng quaù n laàn (vì coù khoâng quaù n baèng
1). Do ñoù, sau moät soá böôùc höõu haïn (khoâng quaù n ), ta seõ ñöa baøi toaùn veà tröôøng
hôïp trong n bieán coù n − 1 bieán khoâng lôùn hôn 1.
Tieáp theo, ta seõ chöùng minh coù theå thay n − 1 bieán khoâng lôùn hôn 1 bôûi trung bình
nhaân cuûa chuùng. Thaät vaäy, khoâng maát tính toång quaùt, giaû söû a1 ≤ a2 ≤ ... ≤ an −1 ≤ 1 .

Ñaët x = n−1 a1a2 ...an−1 ≤ 1. Neáu a1 = x ∨ an−1 = x thì a1 = a2 = ... = an−1 = x. Neáu toàn

taïi a j (1 < j < n − 1) sao cho a j ≠ x thì ta coù a1 < x < an−1. Söû duïng Nhaän xeùt (i), ta

245
 aa 
coù theå thay boä soá (a1 , a2 ,..., an ) bôûi boä soá  x, a2 ,..., 1 n −1 , an  ñeå f khoâng taêng.
 x 
a1an−1 a1
Khi ñoù, soá bieán baèng x taêng leân ít nhaát laø 1. Ta cuõng chuù yù raèng ≤ ≤1
x x
neân vieäc thay nhö treân vaãn ñaûm baûo trong n bieán coù n − 1 bieán khoâng lôùn hôn 1,
ñieàu ñoù cho pheùp vieäc thay theá nhö treân coù theå thöïc hieän lieân tieáp. Tuy nhieân,
vieäc thay theá naøy chæ coù theå ñöôïc thöïc hieän khoâng quaù n laàn (vì coù khoâng quaù n
baèng x ). Do ñoù, sau moät soá laàn thay (khoâng quaù n ), ta ñöa ñöôïc veà tröôøng hôïp
trong n bieán coù n − 1 bieán khoâng lôùn hôn 1 baèng nhau.
Cuoái cuøng, ñeå chöùng minh baát ñaúng thöùc ñaõ cho, ta chæ caàn chöùng minh
 1 
f  x, x,..., n −1  ≥ 0 (0 < x ≤ 1)
 x 
n − 1 n−1 k k
⇔ g ( x) = +x + − n − ≥ 0 (0 < x ≤ 1)
x 1 n
nx + n −1
x
Ta coù
2
( n − 1)( x n − 1)  ( n − 1) x n − 1 
g ( x) =
/
.  ≤ 0 ∀x ∈ (0,1]
x2  (n − 1) x + 1 
n

⇒ g ( x ) nghòch bieán treân (0,1].


⇒ g ( x ) ≥ g (1) = 0 ∀x ∈ (0,1]
⇒ ñpcm.

Ñaúng thöùc xaûy ra khi vaø chæ khi a1 = a2 = ... = an = 1.

246
Baøi toaùn 16.
Cho a, b, c ∈ R thoûa (a − b)(b − c)(c − a ) ≠ 0 . Chöùng minh raèng

(a − 2b) 2 + ( a − 2c )2 (b − 2c ) 2 + (b − 2a ) 2 (c − 2a ) 2 + (c − 2b) 2
+ + ≥ 22
(b − c) 2 (c − a ) 2 ( a − b) 2

Lôøi giaûi.
Ta coù

(a − 2b) 2 + (a − 2c) 2
∑ (b − c) 2
≥ 22
cyc

2a 2 − 4(b + c) a + 4b 2 + 4c 2
⇔∑ ≥ 22
cyc (b − c) 2
a 2 − 2(b + c) a + 2b 2 + 2c 2
⇔∑ ≥ 11
cyc (b − c) 2
a 2 − 2(b + c) a + (b + c) 2 + (b − c)2
⇔∑ ≥ 11
cyc (b − c) 2
b+c−a
2

⇔ ∑  ≥8
cyc  b − c 

b+c−a c+a−b a+b−c


Ñaët x = ,y = ,z = thì ta coù
b−c c−a a −b
( x − 2)( y − 2)( z − 2) = ( x + 2)( y + 2)( z + 2)
⇒ xy + yz + zx = −4

Ta coù baát ñaúng thöùc caàn chöùng minh töông ñöông vôùi
x2 + y 2 + z 2 ≥ 8
⇔ x 2 + y 2 + z 2 ≥ −2( xy + yz + zx )
⇔ ( x + y + z )2 ≥ 0 (ñuùng)
⇒ ñpcm.

247
Baøi toaùn 17. (APMO 2005)
Cho a, b, c > 0 thoûa abc = 8. Chöùng minh raèng

a2 b2 c2 4
+ + ≥
(1 + a )(1 + b )
3 3
(1 + b )(1 + c )
3 3
(1 + c )(1 + a )
3 3 3

Lôøi giaûi.
Ta coù
1 2
≥ ∀x > 0 (*)
1 + x3 2 + x2

Thaät vaäy, ta coù


(*) ⇔ (2 + x 2 ) 2 ≥ 4(1 + x 3 )
⇔ x 2 ( x − 2) 2 ≥ 0 (ñuùng)

Vaäy (*) ñuùng.


Do ñoù

a2 4a 2 2S (a, b, c) 2
∑ ≥∑ = =
(1 + a )(1 + b ) cyc (2 + a )(2 + b ) 36 + S (a, b, c) 1 +
3 3 2 2 36
cyc
S (a, b, c)

trong ñoù S (a, b, c) = 2(a 2 + b 2 + c 2 ) + a 2b 2 + b 2c 2 + c 2 a 2

Theo baát ñaúng thöùc AM-GM, ta coù

a 2 + b 2 + c 2 ≥ 3 3 ( abc) 2 = 12
a 2b 2 + b 2c 2 + c 2 a 2 ≥ 3 3 (abc) 4 = 48
⇒ S ( a, b, c ) ≥ 72

Do ñoù
a2 2 2 4
∑ ≥
36
≥ =
36 3
cyc (1 + a 3 )(1 + b3 ) 1+ 1+
S ( a , b, c ) 72
⇒ ñpcm.

Ñaúng thöùc xaûy ra khi vaø chæ khi a = b = c = 2.

248
Baøi toaùn 18. (Vasile Cirtoaje)
Cho a, b, c ≥ 0. Chöùng minh raèng

a2 b2 c2 2
+ 2 + 2 ≥
3b + 3c − 2bc 3c + 3a − 2ca 3a + 3b − 2ab 3
2 2 2 2

Lôøi giaûi.
AÙp duïng baát ñaúng thöùc Bunhiacopxki, ta coù

a2 a4

cyc 3b + 3c − 2bc
2 2
= ∑
cyc a (3b + 3c − 2bc )
2 2 2

(a 2 + b2 + c 2 )2

∑ a 2 (3b 2 + 3c 2 − 2bc)
cyc

(a 2 + b2 + c 2 )2
=
6(a 2b 2 + b 2c 2 + c 2 a 2 ) − 2abc( a + b + c)

Do ñoù, ñeå chöùng minh baát ñaúng thöùc ñaõ cho, ta chæ caàn chöùng minh
(a 2 + b2 + c 2 )2 2

6(a b + b c + c a ) − 2abc( a + b + c ) 3
2 2 2 2 2 2

⇔ 3( a 2 + b2 + c 2 ) 2 ≥ 12( a 2b 2 + b2 c 2 + c 2a 2 ) − 4abc( a + b + c )
⇔ 3  a 4 + b4 + c 4 + abc( a + b + c ) − 2( a 2b 2 + b2 c 2 + c 2a 2 )  + abc( a + b + c ) ≥ 0
Theo baát ñaúng thöùc Schur thì

a 4 + b 4 + c 4 + abc(a + b + c) ≥ ∑ ab( a 2 + b 2 )
cyc

≥ 2(a 2b 2 + b 2c 2 + c 2 a 2 ) (theo AM-GM)

Vaäy ta coù

3  a 4 + b 4 + c 4 + abc (a + b + c) − 2( a 2b 2 + b 2c 2 + c 2 a 2 )  + abc( a + b + c ) ≥ 0

⇒ ñpcm.
Ñaúng thöùc xaûy ra khi vaø chæ khi (a, b, c) = (t , t , 0) (t > 0).

249
Baøi toaùn 18. (Vasile Cirtoaje)
Cho a, b, c ≥ 0. Chöùng minh raèng

a2 b2 c2
+ + ≥2
b 2 − bc + c 2 c 2 − ca + a 2 a 2 − ab + b 2
Lôøi giaûi.
+ Caùch 1.
AÙp duïng baát ñaúng thöùc Bunhiacopxki, ta coù

a2 a4
∑ b2 − bc + c 2 ∑ a 2 (b2 − bc + c2 )
=
cyc cyc

(a 2 + b 2 + c 2 ) 2

∑ a 2 (b2 − bc + c2 )
cyc

(a 2 + b 2 + c 2 ) 2
=
2(a 2b 2 + b 2c 2 + c 2 a 2 ) − abc( a + b + c )

Do ñoù, ñeå chöùng minh baát ñaúng thöùc ñaõ cho, ta chæ caàn chöùng minh

(a 2 + b 2 + c 2 )2
≥2
2(a 2b 2 + b 2c 2 + c 2 a 2 ) − abc( a + b + c)

⇔ ( a 2 + b 2 + c 2 ) 2 ≥ 4(a 2b 2 + b 2c 2 + c 2 a 2 ) − 2abc (a + b + c)
⇔  a 4 + b 4 + c 4 + abc (a + b + c) − 2(a 2b 2 + b 2c 2 + c 2 a 2 )  + abc( a + b + c ) ≥ 0

Theo baát ñaúng thöùc Schur thì

a 4 + b 4 + c 4 + abc (a + b + c) ≥ ∑ ab( a 2 + b 2 )
cyc

≥ 2(a 2b 2 + b 2 c 2 + c 2 a 2 ) (theo AM-GM)

Vaäy ta coù

 a + b + c + abc(a + b + c) − 2(a b + b c + c a )  + abc (a + b + c) ≥ 0


4 4 4 2 2 2 2 2 2

⇒ ñpcm.
Ñaúng thöùc xaûy ra khi vaø chæ khi (a, b, c) = (t , t , 0) (t > 0).

250
+ Caùch 2.
Khoâng maát tính toång quaùt coù theå giaû söû 0 ≤ a ≤ b ≤ c.
a 2 − ac ≤ 0 0 ≤ c 2 − ca + a 2 ≤ c 2
Khi ñoù, ta coù  2 ⇒
a − ab ≤ 0 0 ≤ a − ab + b ≤ b
2 2 2

Do ñoù

a2 b2 c2 a2 b2 c2
+ + ≥ + +
b 2 − bc + c 2 c2 − ca + a 2 a 2 − ab + b 2 b 2 − bc + c 2 c 2 b 2

b2 c2
≥ + ≥ 2 (theo AM-GM)
c 2 b2
⇒ ñpcm.

Ñaúng thöùc xaûy ra khi vaø chæ khi (a, b, c) = (t , t , 0) (t > 0).

Baøi toaùn 19. (VMEO 2004)


Cho x, y , z > 0 thoûa x + y + z = 1 . Chöùng minh raèng

( y − z )2 ( z − x) 2 ( x − y)2
x+ + y+ + z+ ≤ 3
12 12 12

Lôøi giaûi.
Khoâng maát tính toång quaùt coù theå giaû söû x ≥ y ≥ z > 0
⇒ 0 ≤ x − y ≤ x + y − 2 z = 1 − 3z

Ta seõ chöùng minh

x + u 2 + y + v 2 ≤ 2( x + y ) + (u + v )2 (*)

y−z x−z
trong ñoù u = ,v =
12 12
Thaät vaäy

(*) ⇔ x + y + 2uv ≥ 2 ( x + u 2 )( y + v 2 )

⇔ ( x − y ) 2 + 4(u − v )( xv − yu ) ≥ 0

251
1
⇔ ( x − y ) 2 − .( x − y )2 ( x + y − z ) ≥ 0
3
⇔ ( x − y ) 2 (3 + z − x − y) ≥ 0 (ñuùng)

Vaäy (*) ñuùng. Do ñoù

( y − z )2 ( x + y − 2 z )2 ( x − y)2

cyc
x+
12
≤ 2( x + y ) +
12
+ z+
12
( x + y − 2 z )2 (1 − 3 z ) 2
≤ 2( x + y ) + + z+ = 3 (ñpcm)
12 12
1
Ñaúng thöùc xaûy ra khi vaø chæ khi x = y = z = .
3
Baøi toaùn 20.
Cho x, y , z ≥ 0 thoûa x + y + z = 1. Tìm giaù trò lôùn nhaát cuûa bieåu thöùc

f ( x, y, z ) = x n y + y n z + z n x

trong ñoù n ≥ 2, n ∈ R laø haèng soá cho tröôùc.

Lôøi giaûi.
Giaû söû vôùi boä soá ( x0 , y0 , z0 ) thì f ( x0 , y0 , z0 ) = max f .

Khoâng maát tính toång quaùt coù theå giaû söû x0 = max { x0 , y0 , z0 } .

Khi ñoù, neáu y0 ≤ z0 thì ta coù

f ( x0 , z0 , y0 ) − f ( x0 , y0 , z0 ) = ( z0 − y0 ) x0n + ( y0n − z0n ) x0 + y0 z0n − y0n z0 = g ( x0 )

Ta coù
g / ( x0 ) = n( z0 − y0 ) x0n−1 + y0n − z0n ≥ n( z0 − y0 ) z0n−1 + y0n − z0n = ( n − 1)( z0n − y0n ) ≥ 0
⇒ g ( x0 ) ñoàng bieán.

⇒ g ( x0 ) ≥ g ( z0 ) = 0

⇒ f ( x0 , z0 , y0 ) ≥ f ( x0 , y0 , z0 )

Do ñoù, khoâng maát tính toång quaùt, ta chæ xeùt x ≥ y ≥ z ≥ 0 laø ñuû.

Theo baát ñaúng thöùc Becnulli, ta coù

252
n
 z n  nz  n −1
( x + z ) = x .1 +
n n
 ≥ x . 1 +  = x + nx z ≥ 0
n

 x  x 

Do ñoù, ta coù
f ( x + z , y, 0) = ( x + z ) n y
≥ ( x n + nx n−1z ) y
≥ ( x n + 2 x n−1 z ) y (do n ≥ 2)
= x n y + x n−1 yz + x n−1 yz

≥ x n y + y n z + z n x ( do x ≥ y ≥ z ≥ 0)

= f ( x, y , z )

Ta laïi coù
f ( x + z , y , 0) = ( x + z ) n y
= (1 − y ) n y
1
= .(ny ).(1 − y ) n
n
n +1
1  ny + n(1 − y ) 
≤ . 
n  n +1 
nn
=
(n + 1) n+1

Do ñoù

nn
f ( x, y, z ) ≤
( n + 1) n+1
n 1
Ñaúng thöùc xaûy ra chaúng haïn khi x = ,y = ,z = 0.
n +1 n +1
Keát luaän

nn
max f = .
(n + 1) n+1

253
Baøi toaùn 21. (Voõ Quoác Baù Caån)
Cho a, b, c > 0 thoûa a + b + c = 3. Tìm giaù trò nhoû nhaát cuûa bieåu thöùc

a b c
P= + +
b ab + 2 c bc + 2 a ca + 2

Lôøi giaûi.
Ta coù
2a  2a 
2P = ∑ = ∑ − a + ∑a
cyc b ab + 2 cyc  b ab + 2  cyc
a 3 2b 3 2
= 3− ∑
cyc a1 2b3 2 + 2
a 3 2b 3 2
≥ 3− ∑ (theo bñt AM-GM)
cyc 3 3 a1 2b 3 2
1
= 3 − .∑ a 4 3b
3 cyc
1 a 2 + 2ab + 3a 4 3b 4 3
≥ 3 − .∑ (theo bñt AM-GM)
3 cyc 6

1 1
= 3− .∑ ( a 2 + 2ab) − .∑ a 4 3b 4 3
18 cyc 6 cyc
1 1
= 3− .(a + b + c) 2 − .∑ a 4 3b 4 3
18 6 cyc
5 1
= − .∑ a 4 3b 4 3
2 6 cyc
5 1
= − .∑ ab.a1 3b1 3
2 6 cyc
5 1 ab(a + b + 1)
≥ − .∑
2 6 cyc 3
5 1
= − .∑ ab(4 − c )
2 18 cyc
5 1
= − .(4(ab + bc + ca ) − 3abc )
2 18
Maët khaùc, aùp duïng baát ñaúng thöùc Schur, ta coù

254
a 3 + b3 + c3 + 3abc ≥ ∑ ab(a + b)
cyc

⇒ a 3 + b 3 + c3 + 3∑ ab(a + b) + 6abc ≥ 4∑ ab( a + b) + 3abc


cyc cyc

⇔ ( a + b + c )3 ≥ 4∑ ab( a + b) + 3abc
cyc

⇔ 27 ≥ 4∑ ab(3 − c ) + 3abc
cyc

⇔ 27 ≥ 12(ab + bc + ca ) − 9abc
⇒ 4( ab + bc + ca ) − 3abc ≤ 9
5 1 5 1
⇒ 2P ≥ − .(4(ab + bc + ca ) − 3abc) ≥ − .9 = 2
2 18 2 18
⇒ P ≥1
Ñaúng thöùc xaûy ra khi vaø chæ khi a = b = c = 1.
Vaäy min P = 1.

Baøi toaùn 22. (Voõ Quoác Baù Caån)


Cho a, b, c > 0. Chöùng minh raèng

 b 2c 2  c 2 a 2  a 2b 2   a b c 
27 +  4 + 2  4 + 2  4 + 2  ≥ 36  + + 
 a  b  c  b+c c+a a+b

Lôøi giaûi.
Ta coù Boå ñeà sau

Boå ñeà. x, y , z > 0. Khi ñoù, ta coù

( x 2 + 2)( y 2 + 2)( z 2 + 2) ≥ 9( xy + yz + zx )

Chöùng minh.

Theo nguyeân lyù Dirichlet, ta coù trong 3 soá x 2 − 1, y 2 − 1 vaø z 2 − 1 luoân toàn taïi ít

nhaát 2 soá cuøng daáu. Khoâng maát tính toång quaùt coù theå giaû söû x 2 − 1 vaø y 2 − 1 cuøng

daáu.

⇒ ( x 2 − 1)(y 2 − 1) ≥ 0
⇒ x 2 y 2 ≥ x2 + y 2 − 1
255
⇒ x 2 y 2 + 2 x 2 + 2 y 2 + 4 ≥ 3( x 2 + y 2 + 1)
⇔ ( x 2 + 2)(y 2 + 2) ≥ 3( x 2 + y 2 + 1)

Do ñoù
( x 2 + 2)( y 2 + 2)( z 2 + 2) ≥ 3( x 2 + y 2 + 1)( z 2 + 2)
= 3( x 2 + y 2 + 1)(1 + 1 + z 2 )
≥ 3( x + y + z ) 2 (theo bñt Bunhiacopxki)
≥ 9( xy + yz + zx )

Boå ñeà ñöôïc chöùng minh hoaøn toaøn.


Ñaúng thöùc xaûy ra khi vaø chæ khi x = y = z = 1.

Trôû laïi baøi toaùn cuûa ta


bc ca ab
AÙp duïng Boå ñeà treân vôùi x = 2
, y = 2 , z = 2 , ta coù
a b c
 b2c2  c 2 a 2  a 2b 2   bc ca ca ab ab bc  9( a + b + c )
3 3 3

 4 + 2  4 + 2  4 + 2  ≥ 9  2 . 2 + 2 . 2 + 2 . 2  =
 a  b  c  a b b c c a  abc

Do ñoù, ta chæ caàn chöùng minh

9(a 3 + b 3 + c 3 )  a b c 
27 + ≥ 36  + + 
abc b+c c+a a+b
a 3 + b3 + c3  a b c 
⇔ 3+ ≥ 4 + + 
abc b+c c+a a+b
Chuù yù raèng
(a + b + c).∑ (a − b) 2
a +b +c
3 3 3
= cyc
+3
abc 2abc

 a b c  (a − b) 2
2 + + =∑ +3
 b + c c + a a + b  cyc ( a + c )(b + c )

Do ñoù

a 3 + b3 + c 3 a
3+ − 4.∑ =
abc cyc b + c

256
( a + b + c).∑ ( a − b) 2
 ( a − b) 2 
= 3+ 3+ cyc
− 2 ∑ + 3
2abc  cyc ( a + c)(b + c ) 
1 (a − b) ((a + b + c)(a + c)(b + c ) − 4abc )
2
= .∑
2 cyc abc (a + c)(b + c )
1 (a − b) 2 ((a + c) 2 (b + c) − 4abc)
≥ .∑
2 cyc abc( a + c)(b + c)
1 (a − b) 2 (4ac(b + c) − 4abc )
≥ .∑ (theo bñt AM-GM)
2 cyc abc( a + c)(b + c)
c (a − b) 2
= 2.∑
cyc b(a + c)(b + c)
≥0
a 3 + b3 + c3  a b c 
⇒ 3+ ≥ 4 + + 
abc b+c c +a a +b 
⇒ ñpcm.

Ñaúng thöùc xaûy ra khi vaø chæ khi a = b = c.

Baøi toaùn 23. (Voõ Quoác Baù Caån)


1
Cho 0 ≤ x, y, z ≤ thoûa x + y + z = 1. Chöùng minh raèng
2
1 9
≤ x3 + y 3 + z 3 + 4 xyz ≤
4 32
Lôøi giaûi.
Tröôùc heát, ta chöùng minh
1
x 3 + y 3 + z 3 + 4 xyz ≥
4
⇔ 4 x3 + 4 y 3 + 4 z 3 + 16 xyz ≥ ( x + y + z )3
10
⇔ x3 + y 3 + z 3 + xyz ≥ ∑ xy ( x + y ) (*)
3 cyc

Theo bñt Schur thì


x 3 + y 3 + z 3 + 3 xyz ≥ ∑ xy ( x + y )
cyc

257
⇒ (*) ñuùng.

Tieáp theo, ta seõ chöùng minh


9
x 3 + y 3 + z 3 + 4 xyz ≤
32
23
⇔ 3( xy + yz + zx ) − 7 xyz ≥
32
23
⇔ 3x ( y + z ) + yz (3 − 7 x) ≥
32
1 1
Khoâng maát tính toång quaùt giaû söû x ≥ y ≥ z ⇒ ≤x≤
3 2
Coù 2 tröôøng hôïp xaûy ra
1 3 3
* Tröôøng hôïp 1. ≤ x≤ ⇒0≤ z≤ y≤
3 7 7
3  3 
⇒  − y  − z  ≥ 0
7  7 
9 3 9 3 12 3
⇒ yz ≥ − + ( y + z ) = − + (1 − x ) = − x
49 7 49 7 49 7
Do ñoù

 12 3  36 23
3 x( y + z ) + (3 − 7 x ) yz ≥ 3 x(1 − x) + (3 − 7 x)  − x  = >
 49 7  49 32
3 1
* Tröôøng hôïp 2. ≤ x ≤ . Khi ñoù ta coù
7 2
1
( y + z ) (3 − 7 x) (theo bñt AM-GM)
2
3 x( y + z ) + (3 − 7 x ) yz ≥ 3 x(1 − x) +
4
1
(1 − x ) (3 − 7 x)
2
= 3 x(1 − x) +
4
1 23 23 1
= (1 − 2 x)(28 x 2 − 6 x + 1) + ≥ (do x ≤ )
32 32 32 2
Toùm laïi, ta luoân coù
1 9
≤ x3 + y 3 + z 3 + 4 xyz ≤ (ñpcm)
4 32
258
Baøi toaùn 24. (Jack Grafunkel)
Tìm haèng soá k nhoû nhaát sao cho baát ñaúng thöùc sau ñuùng vôùi moïi x, y , z ≥ 0

x y z
+ + ≤k x+ y+z
x+ y y+z z+x

Lôøi giaûi.
5
Ta seõ chöùng minh baát ñaúng thöùc ñaõ cho ñuùng khi k = . Ñaây chính laø haèng soá toát
4
nhaát cuûa baát ñaúng thöùc ñaõ cho vì ta coù ñaúng thöùc xaûy ra khi x = 0, y = 3, z = 1 .

Ñaët x + y = c 2 , y + z = a 2 , z + x = b 2 ( a, b, c ≥ 0) ⇒ a 2 , b2 , c 2 laø ñoä daøi ba caïnh cuûa

moät tam giaùc (coù theå suy bieán).


Baát ñaúng thöùc caàn chöùng minh trôû thaønh

−a 2 + b 2 + c 2 a 2 − b 2 + c 2 a 2 + b 2 − c 2 5
+ + ≤ . a 2 + b 2 + c 2 (1)
c a b 2 2

Khoâng maát tính toång quaùt, ta coù theå giaû söû a ≥ b, c.

AÙp duïng baát ñaúng thöùc Bunhiacopxki, ta coù

a + b2 + c2
≤ a 2 + b2 + c 2
2

Do ñoù, ñeå chöùng minh (1), ta chæ caàn chöùng minh

−a 2 + b 2 + c 2 a 2 − b 2 + c 2 a 2 + b 2 − c 2 5
c
+
a
+
b (
≤ . a + b2 + c 2
4
)
⇔ a+b+c+
( a + b + c)( a − b)( a − c )(c − b) 5
abc (
≤ . a + b2 + c 2
4
)
(
⇔ 4abc (a + b + c) + 4( a + b + c )(a − b)(a − c)(c − b) ≤ 5abc a + b 2 + c 2 )
Töø ñaây suy ra khoâng maát tính toång quaùt, ta chæ caàn xeùt tröôøng hôïp a ≥ c ≥ b laø ñuû.
Ta coù

(
4abc( a + b + c) + 4(a + b + c)(a − b)(a − c)(c − b) ≤ 5abc a + b 2 + c 2 )
259
⇔ f ( a ) = 4a 3 (c − b) − a 2bc + 4bc(c 2 − b 2 ) +

( )
+ a 4b3 + 4b 2c + 4bc 2 − 4c 3 − 5bc b 2 + c 2 ≤ 0

Do a 2 , b2 , c 2 laø ñoä daøi ba caïnh cuûa moät tam giaùc (coù theå suy bieán) neân ta coù

a ≤ b 2 + c 2 . Do ñoù, ta caàn chöùng minh f (a ) ≤ 0 vôùi b ≤ c ≤ a ≤ b 2 + c 2 (2) .

+ Neáu b = c thì

(
f (a ) = − ab 2  (a − b) + 5 2 − 7 b  ≤ 0
  )
+ Neáu b < c thì f (a ) laø moät haøm ña thöùc baäc ba coù heä soá cao nhaát vaø thaáp nhaát

döông. Ta coù
lim f (a ) = −∞
a→−∞

f (0) > 0
lim f (a ) = +∞
a→+∞

Ta laïi coù

(
f (c ) = −bc 2 5 b 2 + c 2 − 4b − 3c < 0 )
Vì 25(b 2 + c 2 ) − (3c + 4b)2 = (3b − 4c ) 2 > 0

Ngoaøi ra,

f ( ) (
b 2 + c 2 = 2bc 4b b 2 + c 2 − 5b 2 − c 2 )
( )
2
= −2bc b + c − 2b
2 2

≤0

Do ñoù f coù ba nghieäm phaân bieät (1 nghieäm aâm, 1 nghieäm thuoäc (0, c ) vaø 1

nghieäm khoâng nhoû hôn b 2 + c 2 ). Do c ≤ a ≤ b 2 + c 2 neân f (a ) ≤ 0 .

⇒ ñpcm.

260
a = 2 x = 0
a = b 2 + c 2  
Ñaúng thöùc xaûy ra khi vaø chæ khi  ⇔ b = 1 ⇔  y = 3
 b + c = 2b
2 2
 z = 1
c = 3 

Vaäy
5
kmin = .
4
Baøi toaùn 25.
Cho a, b, c > 0 thoûa abc = 1. Chöùng minh raèng

b+c c+a a+b 1 1 1


+ 3 + 3 ≤ 2+ 2+ 2
a + bc b + ca c + ab a
3
b c
Lôøi giaûi.
Tröôùc heát, ta chöùng minh caùc Boå ñeà sau

Boå ñeà 1. x, y , z > 0. Khi ñoù, ta coù

( xy ( x + y ) + yz ( y + z ) + zx( z + x)) 2 ≥ 4( xy + yz + zx)( x 2 y 2 + y 2 z 2 + z 2 x 2 )

Chöùng minh.
Ta coù

( xy ( x + y ) + yz ( y + z ) + zx( z + x )) 2 − 4( xy + yz + zx)( x 2 y 2 + y 2 z 2 + z 2 x 2 ) =
= ∑ x 2 y 2 ( x − y ) 2 + 2 xyz ( x3 + y 3 + z 3 + 3 xyz − xy ( x + y ) − yz ( y + z ) − zx( z + x))
cyc

∑ x 2 y 2 ( x − y ) 2 ≥ 0

Do  cyc
 x 3 + y 3 + z 3 + 3 xyz − xy ( x + y ) − yz ( y + z ) − zx( z + x) ≥ 0 (theo bñt Schur)

Neân
( xy ( x + y ) + yz ( y + z ) + zx( z + x)) 2 − 4( xy + yz + zx )( x 2 y 2 + y 2 z 2 + z 2 x 2 ) ≥ 0
⇒ ( xy ( x + y ) + yz ( y + z ) + zx( z + x)) 2 ≥ 4( xy + yz + zx)( x 2 y 2 + y 2 z 2 + z 2 x 2 )

Boå ñeà 1 ñöôïc chöùng minh hoaøn toaøn.

261
Boå ñeà 2. x, y , z > 0. Khi ñoù, ta coù

x 2 ( y + z ) y 2 ( z + x) z 2 ( x + y ) 2( x 2 + y 2 + z 2 )
+ + ≤
x 2 + 2 yz y 2 + 2 zx z 2 + 2 xy x+ y+ z

Chöùng minh.
Ta coù

x2 ( y + z) 2 yz ( y + z )
= y+z− 2
x + 2 yz
2
x + 2 yz
y 2 ( z + x) 2 zx( z + x )
= z+x− 2
y + 2 zx
2
y + 2 zx
z 2 ( x + y) 2 xy ( x + y )
= x+y− 2
z + 2 xy
2
z + 2 xy

Do ñoù

x2 ( y + z)  yz ( y + z ) 
∑ x 2 + 2 yz = 2  ( x + y + z ) − ∑ x2 + 2 yz 
cyc  cyc 

x 2 ( y + z ) 2( x 2 + y 2 + z 2 )
⇒∑ 2 ≤
cyc x + 2 yz x+ y+z

yz ( y + z ) x 2 + y 2 + z 2
⇔ ( x + y + z) − ∑ 2 ≤
cyc x + 2 yz x+ y+z
yz ( y + z ) x2 + y2 + z 2
⇔∑ ≥ x + y + z −
cyc x 2 + 2 yz x+ y+z
yz ( y + z ) 2( xy + yz + zx)
⇔∑ ≥
cyc x 2 + 2 yz x+ y+ z

AÙp duïng baát ñaúng thöùc Bunhiacopxki, ta coù

yz ( y + z ) ( xy ( x + y )) 2
∑ x 2 + 2 yz ∑ ( z 2 + 2 xy ) xy ( x + y)
=
cyc cyc

( xy ( x + y ) + yz ( y + z ) + zx ( z + x))2

∑ ( z 2 + 2 xy) xy( x + y )
cyc

262
( xy ( x + y ) + yz ( y + z ) + zx ( z + x )) 2
=
2( x + y + z )( x 2 y 2 + y 2 z 2 + z 2 x 2 )
2( xy + yz + zx)
≥ (theo Boå ñeà 1)
x+ y+z

Boå ñeà 2 ñöôïc chöùng minh hoaøn toaøn.


Boå ñeà 3. a, b, c > 0. Khi ñoù, ta coù

b+c c+a a+b 1 1 1


+ + ≤ + +
a 3 + bc.
3
b3 + ca.
3
c3 + ab.
3 a 2 b2 c2
1 a +1 b +1 c 1 a +1 b +1 c 1 a +1 b +1 c
Chöùng minh.
1 1 1
Ñaët x = , y = , z = ⇒ x, y , z > 0 . Khi ñoù, ta coù
a b c
b+c x3 ( y + z )( x + y + z )
= 3
a 3 + bc.
3 2 x + ( x 3 + yz 2 + y 2 z + xyz )
1 a +1 b +1 c
x3 ( y + z )( x + y + z )
≤ (theo bñt AM-GM)
2 x3 + 4 xyz
x + y + z x 2 ( y + z)
= . 2
2 x + 2 yz

Töông töï, ta coù

c+a x + y + z y 2 ( z + x)
≤ . 2
b3 + ca.
3 2 y + 2 zx
1 a +1 b +1 c

a+b x + y + z z 2 ( x + y)
≤ . 2
c3 + ab.
3 2 z + 2 xy
1 a +1 b +1 c

Do ñoù

b+c x+ y+ z x2 ( y + z)
∑ ≤ .∑ 2
cyc x + 2 yz
3 2
cyc a 3 + bc.
1 a +1 b +1 c

263
x + y + z 2( x 2 + y 2 + z 2 )
≤ . (theo Boå ñeà 2)
2 x+ y+ z
= x2 + y 2 + z 2
1 1 1
= 2+ 2+ 2
a b c
Boå ñeà 3 ñöôïc chöùng minh hoaøn toaøn.
Trôû laïi baøi toaùn cuûa ta
AÙp duïng baát ñaúng thöùc AM-HM, ta coù
3 3
3
abc ≥ ⇒1≥ (do abc = 1)
1 a +1 b +1 c 1 a +1 b +1 c

Do ñoù
b+c b+c 1 1 1
∑ a3 + bc ≤ ∑ 3

a 2
+ 2 + 2 (theo Boå ñeà 3)
b c
cyc cyc a 3 + bc.
1 a +1 b +1 c
⇒ ñpcm.

Ñaúng thöùc xaûy ra khi vaø chæ khi a = b = c = 1.

Baøi toaùn 26. (Voõ Quoác Baù Caån)


Cho a, b, c ≥ 0 thoûa ( a + b)(b + c )(c + a ) > 0. Chöùng minh raèng

a (b + c) b(c + a ) c ( a + b) abc( a + b)(b + c)(c + a)


+ + ≥ 2 + 2 1+ 4
a + bc
2
b + ca
2
c + ab
2
( a 2 + bc )(b 2 + ca)(c 2 + ab)

Lôøi giaûi.

a (b + c ) b(c + a) c (a + b)
Ñaët x = ,y= ,z = . Khi ñoù, baát ñaúng thöùc caàn chöùng
a + bc
2
b + ca
2
c 2 + ab
minh töông ñöông vôùi

x + y + z ≥ 2 + 2 1 + 4 xyz

Ta seõ chöùng minh


x2 + y2 + z 2 ≥ 2 (1)
x 2 y 2 + y 2 z 2 + z 2 x2 ≥ 1 (2)
264
Khi ñoù, ta coù

( x + y + z ) 2 = x 2 + y 2 + z 2 + 2( xy + yz + zx )
≥ 2 + 2( xy + yz + zx)
= 2 + 2 x 2 y 2 + y 2 z 2 + z 2 x 2 + 2 xyz ( x + y + z )
≥ 2+2
=4
⇒ x+ y+ z ≥ 2

Vaø do ñoù

( x + y + z ) 2 ≥ 2 + 2 x 2 y 2 + y 2 z 2 + z 2 x 2 + 2 xyz ( x + y + z )
≥ 2 + 2 x 2 y 2 + y 2 z 2 + z 2 x 2 + 4 xyz
≥ 2 + 2 1 + 4 xyz

⇒ x + y + z ≥ 2 + 2 1 + 4 xyz

Ñaây chính laø ñieàu chuùng ta caàn phaûi chöùng minh.


Vaäy nhieäm vuï cuûa chuùng ta baây giôø chæ laø chöùng minh tính ñuùng ñaén cuûa caùc baát
ñaúng thöùc (1) vaø (2) nöõa thoâi.

* Chöùng minh (1).


a (b + c) b(c + a ) c (a + b)
(1) ⇔ + + ≥2
a 2 + bc b 2 + ca c 2 + ab
Ta coù

a (b + c ) ( a − b) 2 (b − c) 2 (c − a ) 2 + abc(a + b)(b + c)(c + a )


∑ a 2 + bc − 2 = (a 2 + bc)(b2 + ca )(c 2 + ab)
≥0
cyc

a (b + c )
⇒∑ ≥2
cyc a 2 + bc

Vaäy (1) ñuùng.


* Chöùng minh (2).
ab(a + c)(b + c ) bc(b + a)(c + a) ca(c + b)( a + b)
(2) ⇔ + 2 + 2 ≥1
( a + bc )(b + ca ) (b + ca )(c + ab) (c + ab)(a 2 + bc)
2 2 2

265
Ta coù
ab( a + c )(b + c) 2abc (a + b)(b + c)(c + a )
∑ (a 2 + bc)(b2 + ca) − 1 = (a 2 + bc)(b2 + ca)(c 2 + ab) ≥ 0
cyc

ab( a + c )(b + c)
⇒∑ ≥1
cyc (a 2 + bc)(b 2 + ca )

Vaäy (2) ñuùng.


⇒ ñpcm.
Ñaúng thöùc xaûy ra khi vaø chæ khi (a, b, c) = (t , t , 0) (t > 0).

Baøi toaùn 27. (Voõ Quoác Baù Caån)


Cho a, b, c ≥ 0 thoûa a + b + c = 1 vaø (a + b)(b + c)(c + a) > 0. Chöùng minh raèng

a 2 + 5b b 2 + 5c c 2 + 5a
+ + ≥8
b+c c+a a+b
Lôøi giaûi.
Ta coù Boå ñeà sau
x + y + z ≥ 0
Boå ñeà. x, y , z laø caùc soá thöïc thoûa  . Khi ñoù, ta coù
 xy + yz + zx ≥ 0

x(b − c) 2 + y (c − a ) 2 + z ( a − b)2 ≥ 0 ∀a, b, c ∈ R

Boå ñeà treân chöùng minh raát ñôn giaûn (chæ caàn duøng tam thöùc baäc hai laø ñöôïc) neân
ôû ñaây ta khoâng nhaéc laïi chöùng minh cuûa noù.
Trôû laïi baøi toaùn cuûa ta
Ta coù baát ñaúng thöùc caàn chöùng minh töông ñöông vôùi

 2a 2   2a 
 ∑ − 1 + 5  ∑
 
− 3 ≥ 0

 cyc b + c   cyc a + b 
Chuù yù raèng

2a 2 ( a − b) 2 (a + b + c ) ( a − b) 2
∑b+c = a + b + c + ∑ (a + c)(b + c) = 1 + ∑ (a + c)(b + c) (theo gt)
cyc cyc cyc

266
2a 2 ( a − b)2
⇒∑ −1 = ∑
cyc b + c cyc ( a + c)(b + c)

2a
∑ ( a − b)3
∑ a + b = 3 − 3(a + cycb)(b + c)(c + a )
cyc

2a
∑ (a − b)3
⇒∑ −3 = − cyc

cyc a+b 3( a + b)(b + c)(c + a )

Do ñoù, baát ñaúng thöùc caàn chöùng minh töông ñöông vôùi

5∑ ( a − b)3
( a − b) 2

∑ (a + c)(b + c) − 3(a + bcyc)(b + c)(c + a) ≥ 0


cyc

⇔ ∑ ( a − b) 2 (4b − a ) ≥ 0
cyc

Ñaët S a = 4c − b, Sb = 4a − c, Sc = 4b − a . Khi ñoù, baát ñaúng thöùc caàn chöùng minh

töông ñöông vôùi

S a (b − c ) 2 + Sb (c − a ) 2 + Sc (a − b) 2 ≥ 0

Coù 2 tröôøng hôïp xaûy ra

* Tröôøng hôïp 1. a ≥ b ≥ c ≥ 0 ⇒ Sb ≥ 0 . Khi ñoù, ta coù

S a + Sb = 4a − b + 3c ≥ 0
Sb + S c = 3a + 4b − c ≥ 0

Chuù yù raèng (a − c) 2 ≥ (a − b) 2 + (b − c) 2 . Do ñoù

S a (b − c ) 2 + Sb (c − a ) 2 + Sc ( a − b) 2 ≥ ( Sa + Sb )(b − c ) 2 + ( Sb + S c )(a − b) 2 ≥ 0

* Tröôøng hôïp 2. 0 ≤ a ≤ b ≤ c ⇒ S a , S c ≥ 0 . Neáu Sb ≥ 0 thì ta coù ngay ñpcm, do

ñoù ta chæ caàn xeùt Sb ≤ 0 laø ñuû.

+ Tröôøng hôïp 2.1. 2b ≥ c. Khi ñoù, ta coù:


S a + 2 Sb = 8a − b + 2c ≥ 0
S c + 2 Sb = 6a + 4b − 2c ≥ 0

267
Maët khaùc, theo baát ñaúng thöùc Bunhiacopxki, ta coù

(c − a ) 2 ≤ 2(a − b) 2 + 2(b − c ) 2

Do ñoù

S a (b − c )2 + Sb (c − a ) 2 + Sc ( a − b)2 ≥ ( S a + 2 Sb )(b − c )2 + ( Sc + 2 Sb )(a − b) 2 ≥ 0

+ Tröôøng hôïp 2.2. c ≥ 2b

- Tröôøng hôïp 2.2.1. a + ( )


3 − 1 c ≥ 3b ⇔ 3(b − c) 2 ≥ (c − a) 2 .

Khi ñoù, ta coù


S a + 3Sb = 12a − b + c ≥ 0

Do ñoù

S a (b − c ) 2 + Sb (c − a ) 2 + Sc ( a − b) 2 ≥ ( Sa + 3Sb )(b − c )2 ≥ 0

3 −1
- Tröôøng hôïp 2.2.2. a + ( )
3 − 1 c ≤ 3b ⇒ b ≥
3
.c ≥
2c
5

Khi ñoù, ta coù


S a + Sb + Sc = 3(a + b + c) ≥ 0
S a Sb + Sb S c + Sc Sa = (4c − b)(4a − c) + (4a − c)(4b − a) + (4b − a )(4c − b)
= 13( ab + bc + ca ) − 4(a 2 + b 2 + c 2 )
≥ 13bc − 4(b 2 + c 2 )
2c  2  c 2 
≥ 13. .c − 4  c +   
5   2  

c2
=
5
≥0

AÙp duïng boå ñeà treân vôùi x = S a , y = Sb , z = Sc , ta suy ra

S a (b − c ) 2 + Sb (c − a ) 2 + Sc (a − b) 2 ≥ 0

Toùm laïi, trong moïi tröôøng hôïp, ta luoân coù

S a (b − c ) 2 + Sb (c − a ) 2 + Sc (a − b) 2 ≥ 0 (ñpcm)

268
1
Ñaúng thöùc xaûy ra khi vaø chæ khi a = b = c = .
3
Baøi toaùn 28. (Phaïm Kim Huøng)
Cho a, b, c ≥ 0 thoûa ( a + b)(b + c )(c + a ) > 0. Chöùng minh raèng

a (b + c) b(c + a ) c (a + b)
+ + ≥2
a 2 + bc b 2 + ca c 2 + ab
Lôøi giaûi.
Ta coù 2 caùch giaûi
* Caùch 1. (tham khaûo lôøi giaûi baøi toaùn 26)

* Caùch 2.
Khoâng maát tính toång quaùt coù theå giaû söû a ≥ b ≥ c.
Baát ñaúng thöùc caàn chöùng minh töông ñöông vôùi
(a − b)(a − c) (b − c)(a − c) b(c + a )
+ ≤ 2
a 2 + bc c 2 + ab b + ca
Ta coù
(a − b)(a − c ) (b − c)(a − c )  a−b b−c 
+ = (a − c)  2 + 2 
a + bc
2
c + ab
2
 a + bc c + ab 
 a −b b−c
≤ (a − c)  2 + 
 a ab 
(a − c )(2ab − b 2 − ac)
=
a 2b
2ab − b 2 − ac

ab
Do ñoù, ñeå chöùng minh baát ñaúng thöùc ñaõ cho, ta chæ caàn chöùng minh

2ab − b 2 − ac b(c + a )
≤ 2
ab b + ca
⇔ (2ab − b 2 − ac)(b 2 + ca ) ≤ ab 2 (c + a )
⇔ ( a − b) 2 b 2 + a 2c 2 ≥ 2(a − b)bac (ñuùng theo bñt AM-GM)

⇒ ñpcm.

269
Ñaúng thöùc xaûy ra khi vaø chæ khi (a, b, c) = (t , t , 0) (t > 0).

Baøi toaùn 29. (Phaïm Kim Huøng)


Cho a, b, c ≥ 0 thoûa a + b + c = 2. Tìm giaù trò lôùn nhaát cuaû bieåu thöùc

P = ( a 2 − ab + b 2 )(b 2 − bc + c 2 )(c 2 − ca + a 2 )

Lôøi giaûi.
Khoâng maát tính toång quaùt giaû söû a ≥ b ≥ c ≥ 0. Khi ñoù, ta coù

0 ≤ a 2 − ac + c 2 ≤ a 2
0 ≤ b 2 − bc + c 2 ≤ b 2
Do ñoù

P ≤ a 2b 2 ( a 2 − ab + b 2 ) = v 2 (u 2 − 3v ) (trong ñoù u = a + b, v = ab)

AÙp duïng baátñaúng thöùc AM-GM, ta coù


3
 3v 3v 
4  + + u 2 − 3v  4u 6 4( a + b) 6 4(a + b + c)6 28
P ≤ v 2 (u 2 − 3v) ≤ . 2 2  = = ≤ =
9  3  243 243 243 243
 
4 2
Ñaúng thöùc xaûy ra khi vaø chæ khi a = , b = , c = 0 vaø caùc hoaùn vò.
3 3
Vaäy

28
max P =
243
Baøi toaùn 30. (Voõ Quoác Baù Caån)
Cho x, y , z > 0 thoûa xy + yz + zx + xyz = 4. Chöùng minh raèng

1 1 1 1
+ + ≥
5x + 1 5 y + 1 5z + 1 2

Lôøi giaûi.
Ta coù
1 1 1
+ + =
2+ x 2+ y 2+ z
270
(2 + x )(2 + y ) + (2 + y )(2 + z ) + (2 + z )(2 + x)
=
(2 + x )(2 + y )(2 + z )
12 + 4( x + y + z ) + ( xy + yz + zx )
=
8 + 4( x + y + z ) + 2( xy + yz + zx ) + xyz
8 + 4( x + y + z ) + ( xy + yz + zx) + 4
=
8 + 4( x + y + z ) + 2( xy + yz + zx ) + xyz
8 + 4( x + y + z ) + ( xy + yz + zx ) + ( xy + yz + zx + xyz )
= (theo gt)
8 + 4( x + y + z ) + 2( xy + yz + zx) + xyz
8 + 4( x + y + z ) + 2( xy + yz + zx ) + xyz
=
8 + 4( x + y + z ) + 2( xy + yz + zx ) + xyz
=1
1 1 1
Vaäy + + =1
2+ x 2+ y 2+ z

 1
0 < a, b, c < 2
1 1 1 
Ñaët a = ,b = ,c = thì ta coù a + b + c = 1
2+ x 2+ y 2+ z  1- 2a 1 − 2b 1 − 2c
x = ,y= ,z =
 a b c

Do ñoù
1 1 a
= =
5 x + 1 5(1 − 2a) + 1 5 − 9a
a
Töông töï, ta coù
1 a
=
5 y + 1 5 − 9b
1 c
=
5 z + 1 5 − 9c
1 1 1 a b c
⇒ + + = + +
5 x + 1 5 y + 1 5 z + 1 5 − 9a 5 − 9b 5 − 9c

1  9a 9b 9c 
= . + + 
9  5 − 9a 5 − 9b 5 − 9c 
1  (9a − 5) + 5 (9b − 5) + 5 (9c − 5) + 5 
= . + + 
9  5 − 9a 5 − 9b 5 − 9c 
271
1 5  1 1 1 
= − + . + + 
3 9  5 − 9a 5 − 9b 5 − 9c 
1 5 9
≥− + .
3 9 (5 − 9a ) + (5 − 9b) + (5 − 9c )
1 5
=− +
3 15 − 9( a + b + c)
1 5
=− +
3 15 − 9
1
=
2
Vaäy
1 1 1 1
+ + ≥ (ñpcm)
5x + 1 5 y + 1 5z + 1 2

Ñaúng thöùc xaûy ra khi vaø chæ khi x = y = z = 1.

* Nhaän xeùt.
Coù theå ñaây laø moät baøi toaùn khoâng khoù nhöng ñieàu ñaëc saéc cuûa noù chính laø ôû choã
töø giaû thieát xy + yz + zx + xyz = 4 ta coù theå suy ra ñöôïc ñaúng thöùc

1 1 1
+ + =1
2+ x 2+ y 2+ z

Vaø chính nhôø ñaúng thöùc naøy maø baøi toaùn cuûa ta ñaõ trôû neân cöïc kyø ñôn giaûn. Baèng
caùch söû duïng ñaúng thöùc naøy, ta coù theå deã daøng chöùng minh ñöôïc caùc keát quaû sau
(1) x + y + z ≥ xy + yz + zx
1 1 1 1 1 1
(2) + + ≤ + + ∀m > 1 > n > 0
2 + xm 2 + ym 2 + zm 2 + xn 2 + yn 2 + zn

272
Baøi toaùn 31. (Voõ Quoác Baù Caån)
Cho a, b, c > 0. Chöùng minh raèng

(b + c − a ) 2 (c + a − b) 2 (a + b − c) 2 3( a 2 + b2 + c 2 )
+ + ≥
2a 2 + (b + c) 2 2b 2 + (c + a ) 2 2c 2 + ( a + b)2 2(a + b + c) 2

Lôøi giaûi.
Do caû hai veá cuûa baát ñaúng thöùc ñaõ cho ñoàng baäc neân khoâng maát tính toång quaùt, ta
coù theå giaû söû a + b + c = 3.
Khi ñoù, baát ñaúng thöùc caàn chöùng minh töông ñöông vôùi
(3 − 2a ) 2 (3 − 2b) 2 (3 − 2c) 2 1
+ 2 + 2 ≥ .( a 2 + b 2 + c 2 )
2a + (3 − a )
2 2
2b + (3 − b) 2
2c + (3 − c) 2
6
2(3 − 2a ) 2 2(3 − 2b) 2 2(3 − 2c) 2
⇔ 2 + 2 + 2 ≥ a2 + b2 + c2
a − 2a + 3 b − 2b + 3 c − 2c + 3
Ta seõ chöùng minh

2(3 − 2 x ) 2
≥ x 2 − 6 x + 6 ∀x ∈ (0,3) (*)
x − 2x + 3
2

Thaät vaäy
(*) ⇔ 2(3 − 2 x) 2 ≥ ( x 2 − 6 x + 6)( x 2 − 2 x + 3)
⇔ x( x − 1) 2 (6 − x) ≥ 0 (ñuùng do 0 < x < 3)

Vaäy (*) ñuùng.


Do ñoù, ta coù

2(3 − 2a ) 2
≥ a 2 − 6a + 6
a − 2a + 3
2

2(3 − 2b) 2
≥ b 2 − 6b + 6
b − 2b + 3
2

2(3 − 2c )2
≥ c 2 − 6c + 6
c − 2c + 3
2

2(3 − 2a ) 2 2(3 − 2b) 2 2(3 − 2c) 2


⇒ + 2 + 2 ≥ a 2 + b 2 + c 2 − 6( a + b + c) + 18
a − 2a + 3 b − 2b + 3 c − 2c + 3
2

= a2 + b2 + c2
273
2(3 − 2a ) 2 2(3 − 2b)2 2(3 − 2c) 2
⇒ + 2 + 2 ≥ a 2 + b 2 + c 2 (ñpcm)
a − 2a + 3 b − 2b + 3 c − 2c + 3
2

Ñaúng thöùc xaûy ra khi vaø chæ khi a = b = c.


Baøi toaùn 32. (Voõ Quoác Baù Caån)
Cho a, b, c > 0. Chöùng minh raèng

a 2 + b 2 + c 2 1  a 3 + b3 + c3 2(a + b + c) 2 
+ . + 2 ≥4
ab + bc + ca 3  abc a + b2 + c2 

Lôøi giaûi.
Do hai veá cuûa baát ñaúng thöùc ñaõ cho ñoàng baäc neân khoâng maát tính toång quaùt, ta coù

theå giaû söû a 2 + b 2 + c 2 = 3. Ñaët p = ab + bc + ca thì ta coù 0 < p ≤ 3.

Khi ñoù, ta coù

a 2 + b 2 + c 2 1  a 3 + b3 + c 3 2( a + b + c ) 2 
+ . + 2 =
ab + bc + ca 3  abc a + b2 + c2 

3 1 a+b+c 2(3 + 2 p) 
= + . .(3 − p) + 3 + 
p 3  abc 3 
3 1  1 1 1  4p 
= + .   + +  .(3 − p ) + + 5
p 3   ab bc ca  3 
3 1 9 4p 
≥ + .  .(3 − p ) + + 5
p 3p 3 
12 4 p 4
= + −
p 9 3
8 1 p 4
= + 4 +  −
p p 9 3
8 1 p 4
≥ + 4.2 . − (theo bñt AM-GM)
3 p 9 3
=4
a 2 + b 2 + c 2 1  a 3 + b3 + c 3 2( a + b + c )2 
⇒ + . + 2  ≥ 4 (ñpcm)
ab + bc + ca 3  abc a + b2 + c2 

Ñaúng thöùc xaûy ra khi vaø chæ khi a = b = c.

274
Baøi toaùn 33. (Voõ Quoác Baù Caån)
Cho a, b, c > 0. Chöùng minh raèng

1 1 1 3
+ + ≥
a 3a + 2b b 3b + 2c c 3c + 2a 5abc

Lôøi giaûi.
1 1 1
Ñaët x = , y = , z = thì ta coù x, y , z > 0. Khi ñoù, baát ñaúng thöùc caàn chöùng minh
a b c
töông ñöông vôùi
x y z 3
+ + ≥
3 zx + 2 yz 3 xy + 2 zx 3 yz + 2 xy 5
x y z 3
⇔ + + ≥
5 z . 3x + 2 y 5x . 3 y + 2 z 5 y . 3z + 2 x 5

AÙp duïng baát ñaúng thöùc AM-GM vaø baát ñaúng thöùc Bunhiacopxki, ta coù
x y z
+ + ≥
5z . 3x + 2 y 5x. 3 y + 2 z 5 y . 3z + 2 x
 x y z 
≥ 2 + + 
 3x + 2 y + 5 z 5 x + 3 y + 2 z 2 x + 5 y + 3 z 

2( x + y + z ) 2

x(3 x + 2 y + 5 z ) + y (5 x + 3 y + 2 z ) + z (2 x + 5 y + 3 z )
2( x + y + z )2
=
3( x 2 + y 2 + z 2 ) + 7( xy + yz + zx)
2( x + y + z ) 2
=
1 20
3( x 2 + y 2 + z 2 ) + .( xy + yz + zx ) + .( xy + yz + zx )
3 3
2( x + y + z ) 2

1 20
3( x 2 + y 2 + z 2 ) + .( x 2 + y 2 + z 2 ) + .( xy + yz + zx)
3 3
3( x + y + z ) 2
3
= =
5( x + y + z + 2 xy + 2 yz + 2 zx) 5
2 2 2

x y z 3
⇒ + + ≥
5 z . 3x + 2 y 5x . 3 y + 2 z 5 y . 3z + 2 x 5
275
⇒ ñpcm.

Ñaúng thöùc xaûy ra khi vaø chæ khi x = y = z ⇔ a = b = c.

Baøi toaùn 34. (R. Stanojevic ù)


Cho a, b, c > 0 thoûa abc = 1. Chöùng minh raèng

1 1 1
+ + ≥ 2
1 1 1 1 1 1
a+ + c+ + c+ +
c 2 b 2 b 2

Lôøi giaûi.
x z y
Do abc = 1 neân toàn taïi caùc soá x, y , z > 0 sao cho a = , b = , c = , chaúng haïn
y x z

x = 3 ca 2 , y = 3 bc 2 , z = 3 ab 2 . Khi ñoù, ta coù

1 1 2y
= =
1 1 x z 1 2x + y + 2 z
a+ + + +
c 2 y y 2

Töông töï, ta coù

1 2z
=
1 1 2x + 2 y + z
c+ +
b 2

1 2x
=
1 1 x + 2 y + 2z
b+ +
a 2

Khi ñoù, baát ñaúng thöùc caàn chöùng minh trôû thaønh

x y z
+ + ≥1
x + 2 y + 2z y + 2z + 2 x z + 2x + 2 y

AÙp duïng baát ñaúng thöùc AM-GM, ta coù


x 2x 2x x
= ≥ =
x + 2 y + 2z 2 x. x + 2 y + 2z x + ( x + 2 y + 2z) x + y + z

Töông töï, ta coù

276
y y

y + 2z + 2x x + y + z
z z

z + 2x + 2 y x + y + z

x y z
⇒ + + ≥
x + 2 y + 2z y + 2z + 2 x z + 2x + 2 y
x y z
≥ + + =1
x+ y+ z x+ y+ z x+ y+ z
⇒ ñpcm.

Baøi toaùn 35. (Taiwanese Mathematical Olympiad 1992)


Cho n ≥ 3, n ∈ N vaø x1 , x2 ,..., xn ≥ 0 thoûa x1 + x2 + ... + xn = 1. Chöùng minh raèng

4
x12 x2 + x22 x3 + ... + xn2 x1 ≤
27
Lôøi giaûi.

Ñaët f ( x1 , x2 ,..., xn ) = x12 x2 + x22 x3 + ... + xn2 x1

Khoâng maát tính toång quaùt, coù theå giaû söû x1 = max xi (i = 1, n) .

Goïi xk = max xi (i = 2, n) .

Khi ñoù, ta coù


f (1 − xk , xk , 0,
14 24 30) = f ( x1 + x2 + ... + xk −1 + xk +1 + ... + xn , xk , 0,
0,..., 14 0,...,0)
24 3
n− 2 soá 0 n− 2 soá 0

= ( x1 + x2 + ... + xk −1 + xk +1 + ... + xn ) xk
2

≥ ( x12 + 2 x1 ( x2 + ... + xk −1 + xk +1 + ... + xn )) xk


≥ x12 x2 + x22 x3 + ... + xn2 x1 (do x1 ≥ xk = max xi (i = 2, n))

= f ( x1 , x2 ,..., xn )

Ta laïi coù
3
1 1  2 xk + 2(1 − xk )  4
f (1 − xk , xk , 0,
1 0,...,0)
424 3 = (1 − xk ) xk
2
= .(2 xk ).(1 − xk ) 2
≤ .   =
n− 2 soá 0
2 2  3  27

277
4
Vaäy f ( x1 , x2 ,..., xn ) ≤ (ñpcm) .
27
Baøi toaùn 36.
Cho a, b, c ≥ 0 thoûa a + b + c + abc = 1. Chöùng minh raèng

(2 + abc )(1 + 2 abc)


ab + bc + ca ≤
7 − abc
Lôøi giaûi.
a+b+c
Ñaët m = ⇒ 3m + abc = 1. Khi ñoù, ta coù
3
(2 + abc)(1 + 2abc) (3 − 3m)(3 − 6m) 3(1 − m)(1 − 2m)
A= = =
7 − abc 6 + 3m 2+m
3(1 − m)(1 − 2m) 3(1 − 3m − m3 ) 3( abc − m3 )
⇒ A − 3m =
2
− 3m =
2
=
2+m 2+m 2+m
3( abc − m )3
⇒ A= + 3m 2
2+m
Do ñoù, baát ñaúng thöùc caàn chöùng minh töông ñöông vôùi

3( abc − m3 )
+ 3m 2 ≥ ab + bc + ca
2+m
3(m3 − abc)
⇔ 3m − ab − bc − ca ≥
2
2+m
(a + b + c)3 − 27 abc
⇔ 3(a + b + c) − 9(ab + bc + ca ) ≥
2
2+m
⇔ 3(2 + m)(( a + b + c )2 − 3( ab + bc + ca )) ≥ ( a + b + c)3 − 27 abc

1 7(a + b + c) 4( a + b + c)
Do 3m + abc = 1 neân m ≤ ⇒ 2 + m ≥ 7m = ≥ .
3 3 3
Do ñoù, ta chæ caàn chöùng minh
4(a + b + c)(( a + b + c )2 − 3( ab + bc + ca )) ≥ ( a + b + c)3 − 27 abc
 
⇔ 4  ∑ a 3 − 3abc  ≥ ∑ a 3 + 3∑ ab(a + b) − 21abc
 cyc  cyc cyc

⇔ 3∑ a 3 + 9abc ≥ 3∑ ab(a + b)
cyc cyc

278
⇔ ∑ a 3 + 3abc ≥ ∑ ab(a + b) (ñuùng theo Schur)
cyc cyc

⇒ ñpcm.

Baøi toaùn 37. (Voõ Quoác Baù Caån)

Cho a, b, c > 0 thoûa abc = 2 ab + bc + ca + 4. Chöùng minh raèng

a7 b7 c7 128
+ + ≥
1 + ab 1 + bc 1 + ca
2 2 2
3
Lôøi giaûi.
AÙp duïng baát ñaúng thöùc Bunhiacopxki, ta coù

a7 b7 c7 a 7c b7 a c 7b
+ + = + +
1 + ab 2 1 + bc 2 1 + ca 2 c + ab 2c a + abc 2 b + ca 2b
( a 7 2 .c1 2 + b7 2 .a1 2 + c 7 2 .b1 2 )2

a + b + c + a 2bc + ab 2 c + abc 2
(a 7 2 .c1 2 + b7 2 .a1 2 + c 7 2 .b1 2 ) 2
=
(a + b + c)(1 + abc)

AÙp duïng baát ñaúng thöùc AM−GM, ta coù

23a 7 2 .c1 2 + 11b 7 2 .a1 2 + 9c 7 2 .b1 2 ≥ 43a 2bc


23b 7 2 .a1 2 + 11c 7 2 .b1 2 + 9a 7 2 .c1 2 ≥ 43ab 2 c
23c 7 2 .b1 2 + 11a 7 2 .c1 2 + 9b7 2 .a1 2 ≥ 43abc 2
⇒ 43(a 7 2 .c1 2 + b7 2 .a1 2 + c 7 2 .b1 2 ) ≥ 43abc( a + b + c )
⇔ a 7 2 .c1 2 + b7 2 .a1 2 + c 7 2 .b1 2 ≥ abc( a + b + c )

Do ñoù
a7 b7 c7 (a 7 2 .c1 2 + b 7 2 .a1 2 + c 7 2 .b1 2 ) 2
+ + ≥
1 + ab 2 1 + bc 2 1 + ca 2 ( a + b + c )(1 + abc)
( abc( a + b + c )) 2

( a + b + c)(1 + abc )
a 2b 2c 2 ( a + b + c)
=
1 + abc
Ta coù

279
abc = 2 ab + bc + ca + 4 ≥ 4 4 ab.bc.ca.4 = 4 4 2abc
⇒ abc ≥ 8
Do ñoù

a 2b2 c 2 (a + b + c) a 2b 2c 2 .3 3 abc

1 + abc 1 + abc
6 a 2b 2 c 2

1 + abc
6.82 t2
≥ (do f (t ) = ñoàng bieán treân (0, +∞))
1+ 8 1+ t
128
=
3

a7 b7 c7 a 2b 2c 2 (a + b + c ) 128
⇒ + + = ≥ (ñpcm)
1 + ab 2 1 + bc 2 1 + ca 2 1 + abc 3
Ñaúng thöùc xaûy ra khi vaø chæ khi a = b = c = 2.

Baøi toaùn 38. (Phaïm Kim Huøng)


Cho a, b, c > 0 thoûa abc = 1. Chöùng minh raèng

1 1 1 2
a) + + + ≥1
(1 + a ) 2
(1 + b) (1 + c)
2 2
(1 + a)(1 + b)(1 + c )
a+3 b+3 c+3
b) + + ≥3
(1 + a ) 2 (1 + b) 2 (1 + c) 2

Lôøi giaûi.
2 2 2
a) Ñaët x = − 1, y = − 1, z = − 1 ⇒ x, y, z ∈ [ −1,1]
1+ a 1+ b 1+ c

 2  2  2 
⇒ (1 − x)(1 − y )(1 − z ) =  2 −  2 −  2 − 
 1 + a  1 + b  1 + c 
8abc
=
(1 + a )(1 + b)(1 + c)
8
=
(1 + a)(1 + b)(1 + c )
= (1 + x)(1 + y )(1 + z )

280
⇒ x + y + z + xyz = 0

Do ñoù, baát ñaúng thöùc caàn chöùng minh trôû thaønh


( x + 1)2 + ( y + 1) 2 + ( z + 1)2 + ( x + 1)( y + 1)( z + 1) ≥ 4
⇔ x 2 + y 2 + z 2 + xy + yz + zx + 3( x + y + z ) + xyz ≥ 0
⇔ x 2 + y 2 + z 2 + xy + yz + zx − 2 xyz ≥ 0
⇔ 2( x 2 + y 2 + z 2 ) + 2( xy + yz + zx ) − 4 xyz ≥ 0
⇔ x 2 + y 2 + z 2 + ( x + y + z ) 2 − 4 xyz ≥ 0
⇔ x 2 + y 2 + z 2 + x 2 y 2 z 2 − 4 xyz ≥ 0

AÙp duïng baát ñaúng thöùc AM−GM, ta coù

x 2 + y 2 + z 2 + x 2 y 2 z 2 ≥ 4 4 x 2 . y 2 .z 2 .x 2 y 2 z 2 = 4 xyz ≥ 4 xyz
⇒ x 2 + y 2 + z 2 + x 2 y 2 z 2 − 4 xyz ≥ 0 (ñpcm)

2 2 2
b) Ñaët x = − 1, y = − 1, z = − 1 ⇒ x, y, z ∈ [ −1,1]
1+ a 1+ b 1+ c

 2  2  2 
⇒ (1 − x)(1 − y )(1 − z ) =  2 −  2 −  2 − 
 1 + a  1 + b  1+ c 
8abc
=
(1 + a )(1 + b)(1 + c)
8
=
(1 + a)(1 + b)(1 + c )
= (1 + x)(1 + y )(1 + z )
⇒ x + y + z + xyz = 0

Baát ñaúng thöùc caàn chöùng minh töông ñöông vôùi


( x + 1)( x + 2) + ( y + 1)( y + 2) + ( z + 1)( z + 2) ≥ 0
⇔ x 2 + y 2 + z 2 ≥ −3( x + y + z )
⇔ x 2 + y 2 + z 2 ≥ 3 xyz

AÙp duïng baát ñaúng thöùc AM−GM, ta coù

x 2 + y 2 + z 2 ≥ 3 3 x 2 y 2 z 2 ≥ 3xyz (do x, y , z ∈ [ −1,1])


⇒ x 2 + y 2 + z 2 ≥ 3xyz (ñpcm)

281
Baøi toaùn 39. (Voõ Quoác Baù Caån)
Cho a, b, c ≥ 0. Chöùng minh raèng

a (b + c ) b(c + a) c( a + b)
+ + ≥2
b +c
2 2
c +a
2 2
a 2 + b2

Lôøi giaûi.
Ta seõ chöùng minh
a (b + c)
∑ b2 + c 2 ≥2 (1)
cyc

ab(a + c)(b + c)
∑ (a 2 + c2 )(b 2 + c2 ) ≥ 1 (2)
cyc

Khi ñoù, ta coù


2
 a (b + c ) b(c + a ) c (a + b)  a (b + c ) ab( a + c )(b + c)
 + +  = ∑ + 2∑
 b +c c2 + a 2 a 2 + b2  cyc b + c (a 2 + c 2 )(b 2 + c 2 )
2 2 2 2
cyc

ab( a + c )(b + c)
≥ 2 + 2∑
cyc ( a 2 + c 2 )(b 2 + c 2 )
ab( a + c )(b + c)
≥ 2 + 2∑ ≥4
cyc (a 2 + c 2 )(b 2 + c 2 )
a (b + c ) b(c + a ) c (a + b)
⇒ + + ≥2
b +c
2 2
c +a
2 2
a 2 + b2
Ñaây chính laø ñieàu ta caàn phaûi chöùng minh, vaäy nhieäm vuï cuûa ta baây giôø chæ laø
chöùng minh tính ñuùng ñaén cuûa caùc baát ñaúng thöùc (1) vaø (2) nöõa thoâi.
* Chöùng minh (1).
Ta coù

a (b + c )
∑ ab(a − b)2 (a 2 + b2 + 2c2 ) + 8a 2b2c2
∑ b2 + c2 −2= cyc

( a 2 + b2 )(b 2 + c 2 )(c 2 + a 2 )
≥0
cyc

a(b + c)
⇒∑ ≥2
cyc b2 + c2
⇒ (1) ñuùng.

282
* Chöùng minh (2).
Ta coù
ab(a + c)(b + c ) 2abc(( a + b + c)( a 2 + b2 + c 2 ) − abc )
∑ (a 2 + c2 )(b2 + c2 ) − 1 =
( a 2 + b2 )(b 2 + c 2 )(c 2 + a 2 )
≥0
cyc

⇒ (2) ñuùng
⇒ ñpcm.
Ñaúng thöùc xaûy ra khi vaø chæ khi (a, b, c) = (t , t , 0) (t > 0).

* Nhaän xeùt.
Ngoaøi ra, ta coøn coù moät baát ñaúng thöùc maïnh hôn nhö sau
Cho a, b, c ≥ 0. Khi ñoù

a (b + c) b(c + a ) c ( a + b) abc( a + b)(b + c)(c + a )


+ + ≥ 2 + 2 1+ 4
b +c
2 2
c +a
2 2
a +b
2 2
( a 2 + b 2 )(b 2 + c 2 )(c 2 + a 2 )

Baøi toaùn 40. (Voõ Quoác Baù Caån)


Cho a, b, c > 0 thoûa a + b + c = 1. Chöùng minh raèng

b c c a a b
+ + ≥1
(
a 2 c + 3ab ) ( b 2 a + 3bc ) ( c 2 b + 3ca )
Lôøi giaûi.
Ta coù baát ñaúng thöùc caàn chöùng minh töông ñöông vôùi

bc ca ab
a + b + c ≥1
ca ab bc
a 3+2 b 3+2 c 3+2
b c a

bc ca ab  x, y , z > 0
Ñaët x = ,y = ,z = thì ta coù 
a b c  xy + yz + zx = 1 (do a + b + c = 1)
Baát ñaúng thöùc caàn chöùng minh trôû thaønh
x y z
+ + ≥1
2 y + yz 3 2 z + zx 3 2 x + xy 3
283
AÙp duïng baát ñaúng thöùc Bunhiacopxki, ta coù

x y z x2 y2 z2
+ + = + +
2 y + yz 3 2 z + zx 3 2 x + xy 3 2 xy + xyz 3 2 yz + xyz 3 2 zx + xyz 3
( x + y + z )2

2( xy + yz + zx ) + 3 xyz 3
3( xy + yz + zx )

2( xy + yz + zx ) + 3 xyz 3
3
=
2 + 3 xyz 3
3

3
 xy + yz + zx 
2 + 3 3.  
 3 
=1
x y z
⇒ + + ≥1
2 y + yz 3 2 z + zx 3 2 x + xy 3
⇒ ñpcm.

Baøi toaùn 41. (Voõ Quoác Baù Caån)


Cho x, y , z laø ñoä daøi ba caïnh cuûa moät tam giaùc (coù theå suy bieán). Tìm haèng soá k

lôùn nhaát sao cho

1 1 1  x y z  k ( x − y ) 2 ( y − z ) 2 ( z − x) 2
( x + y + z)  + +  ≥ 6  + + +
x y z  y + z z + x x + y  xyz ( x + y )( y + z)( z + x)
Lôøi giaûi.
Ta coù baát ñaúng thöùc ñaõ cho töông ñöông vôùi

∑ ( x − y)2 ( z 2 + xz + yz − 2 xy ) z( x + y ) ≥
cyc

≥ k ( x − y ) 2 ( y − z ) 2 ( z − x )2 (1)

Do x, y , z laø ñoä daøi ba caïnh cuûa moät tam giaùc (coù theå suy bieán) neân toàn taïi caùc soá

khoâng aâm a, b, c sao cho x = b + c, y = c + a, z = a + b .

Thay vaøo (1), ta coù baát ñaúng thöùc (1) trôû thaønh

284
∑ (a − b)2 (a 2 + b 2 − c 2 + ab)(a + b)(a + b + 2c) ≥
cyc

k
≥ .( a − b) 2 (b − c ) 2 (c − a )2 (2)
2
Cho c → 0 + , ta ñöôïc (2) trôû thaønh

2(b3 (b 2 − a 2 )(2a + b) + a 3 ( a 2 − b 2 )(a + 2b) +


k
+ (a − b) 2 (a 2 + b 2 + ab)(a + b) 2 ) ≥ .a 2b 2 ( a − b)2
2
⇔ 2(( a − b) (a + b) + ( a − b) ( a + b + ab)( a + b) ) ≥ k .a 2b 2 ( a − b) 2
2 4 2 2 2 2

⇔ 2(( a + b) 4 + ( a 2 + b 2 + ab)(a + b) 2 ) ≥ k .a 2b 2

Cho a = b = 1, ta suy ra ñöôïc: k ≤ 56.

Ta seõ chöùng minh kmax = 56, töùc laø chöùng minh

∑ (a − b)2 (a 2 + b2 − c 2 + ab)(a + b)(a + b + 2c) ≥ 28(a − b)2 (b − c)2 (c − a )2


cyc

Khoâng maát tính toång quaùt giaû söû a ≥ b ≥ c ≥ 0 .


Ta coù baát ñaúng thöùc treân töông ñöông vôùi

((b − c) 2 (b 2 − a 2 )(b + c)(2a + b + c) + (a − c) 2 ( a 2 − b 2 )( a + c)( a + 2b + c)) +


+ ( a − b) 2 (a 2 + b 2 + ab)( a + b)(a + b + 2c) +
+ ((ac + c 2 )( a − c ) 2 ( a + c )( a + 2b + c ) − c 2 ( a − b) 2 (a + b)(a + b + 2c)) +
+ (bc + c 2 )(b − c ) 2 (b + c)(2a + b + c) ≥

≥ 28(a − b) 2 (b − c)2 (c − a )2

Do a ≥ b ≥ c ≥ 0 neân

(bc + c 2 )(b − c )2 (b + c)(2a + b + c) ≥ 0 (3)

(ac + c 2 )( a − c) 2 ( a + c )( a + 2b + c) − c 2 ( a − b) 2 ( a + b)( a + b + 2c ) ≥
≥ 2c 2 (a − b) 2 (a + c)(a + 2b + c) − c 2 (a − b) 2 ( a + b)( a + b + 2c )
= c 2 (a − b) 2 (2( a + c)( a + 2b + c ) − ( a + b)(a + b + 2c))
= c 2 (a − b) 2 ( a 2 + 2ab − b 2 + 2c( a + 2b + c ) − 2bc ) ≥ 0

285
⇒ ( ac + c 2 )(a − c) 2 (a + c)(a + 2b + c) ≥
≥ c 2 (a − b) 2 ( a + b)( a + b + 2c ) (4)

a
Laïi do a ≥ b ≥ c ≥ 0 neân a − c ≥ .(b − c) ≥ 0
b
⇒ (b − c) 2 (b 2 − a 2 )(b + c )(2a + b + c ) + (a − c) 2 (a 2 − b 2 )(a + c)(a + 2b + c) ≥
a2
≥ (b − c) (b − a )(b + c)(2 a + b + c) + 2 .(b − c ) 2 ( a 2 − b 2 )(a + c)( a + 2b + c)
2 2 2
b
(b − c) 2 (a 2 − b 2 )( a 2 (a + c)(a + 2b + c) − b 2 (b + c)(2a + b + c ))
=
b2
(b − c ) 2 (a 2 − b 2 )(( a 4 − b 4 ) + 2ab( a 2 − b 2 ) + 2c( a 3 − b3 ) + 2abc( a − b) + c 2 ( a 2 − b 2 ))
=
b2

(b − c) 2 (a 2 − b 2 )(( a 4 − b 4 ) + 2ab( a 2 − b 2 ))

b2
(b − c) 2 (a − b) 2 ( a + b)4
=
b2
(b − c) 2 (a − b) 2 .16a 2b 2
≥ (theo bñt AM-GM)
b2
= 16( a − b) 2 (b − c ) 2 a 2

≥ 16(a − b) 2 (b − c )2 ( a − c )2 (5)

Ta laïi coù
(a − b) 2 ( a 2 + b 2 + ab)(a + b)( a + b + 2c ) ≥
≥ ( a − b) 2 ( a 2 + b 2 + ab)( a + b)2
≥ ( a − b) 2 (2ab + ab)4ab (theo bñt AM-GM)
= 12( a − b)2 a 2b 2
≥ 12(a − b) 2 (b − c) 2 (c − a ) 2

Töø (3),(4),(5) vaø (6), ta suy ra

∑ (a − b)2 (a 2 + b2 − c 2 + ab)(a + b)(a + b + 2c) ≥ 28(a − b)2 (b − c)2 (c − a )2


cyc

Vaäy kmax = 56.

286
Baøi toaùn 42. (Poland 2005)
Cho a, b, c ∈ [0,1] . Chöùng minh raèng

a b c
+ + ≤2
bc + 1 ca + 1 ab + 1
Lôøi giaûi.
a a
Do a, b, c ∈ [0,1] neân bc + 1 ≥ abc + 1 > 0 ⇒ ≤
bc + 1 abc + 1
Töông töï, ta coù
b b

ca + 1 abc + 1
c c

ab + 1 abc + 1
a b c a+b+c
⇒ + + ≤
bc + 1 ca + 1 ab + 1 abc + 1
Do ñoù, ñeå chöùng minh baát ñaúng thöùc ñaõ cho, ta chæ caàn chöùng minh
a+b+c
≤2
abc + 1
⇔ a + b + c ≤ 2(1 + abc )

Do a, b ∈ [0,1] neân (1 − a )(1 − b) ≥ 0 ⇒ a + b ≤ 1 + ab

⇒ a + b + c ≤ 1 + ab + c
Laïi do a, b, c ∈ [0,1] neân (1 − ab)(1 − c) ≥ 0 ⇒ ab + c ≤ 1 + abc
⇒ a + b + c ≤ 1 + ab + c ≤ 2 + abc ≤ 2(1 + abc )
⇒ ñpcm.

Baøi toaùn 43. (China 2006)


Cho x, y , z > 0 thoûa x + y + z = 1. Chöùng minh raèng

xy yz zx 2
+ + ≤
xy + yz yz + zx zx + xy 2

Lôøi giaûi.
Ta coù baát ñaúng thöùc caàn chöùng minh töông ñöông vôùi
287
x y y z z x 2
+ + ≤
z+x x+ y y+z 2

AÙp duïng baát ñaúng thöùc Bunhiacopxki, ta coù

z+x ≥
1
2
. ( z+ x >0 )
x y x y
⇒ ≤ 2.
z+x z+ x
Töôïng töï, ta coù

y z y z
≤ 2.
x+ y x+ y
z x z x
≤ 2.
y+z y+ z

x y y z z x  x y y z z x 
⇒ + + ≤ 2. + + 
z+x x+ y y+z  z+ x x+ y y + z 

Do ñoù, ñeå chöùng minh baát ñaúng thöùc ñaõ cho, ta chæ caàn chöùng minh

x y y z z x 1
+ + ≤
z+ x x+ y y+ z 2
 x y  1
⇔ ∑ − xy  + ∑ xy ≤

cyc  z + x
 cyc 2

1
⇔ 2∑ xy ≤ 1 + 2 xyz .∑
cyc cyc z+ x

1
⇔ 2∑ xy ≤ ∑ x + 2 xyz .∑ (*)
cyc cyc cyc z+ x

Ñaët a = x , b = y , c = z thì ta coù a, b, c > 0 . Khi ñoù, baát ñaúng thöùc (*) trôû thaønh

 1 1 1 
a 2 + b2 + c 2 + 2abc  + +  ≥ 2(ab + bc + ca ) (**)
a+b b+c c+a

288
Do caû hai veá cuûa baát ñaúng thöùc treân ñoàng baäc neân khoâng maát tính toång quaùt, coù
1
theå giaû söû a + b + c = 1. Ñaët q = ab + bc + ca, r = abc ⇒ 0 ≤ q ≤ . Khi ñoù, ta coù
3
a 2 + b 2 + c 2 = 1 − 2q
1 1 1 1+ q
+ + =
a+b b+c c+a q−r

Do ñoù
2r (1 + q )
(**) ⇔ 1 − 4q + ≥0
q−r
⇔ (1 − 4q)( q − r ) + 2r (1 + q) ≥ 0
⇔ q (1 − 4q ) + r (1 + 6q ) ≥ 0

Coù 2 tröôøng hôïp xaûy ra


1
* Tröôøng hôïp 1. 0 ≤ q ≤ . Trong tröôøng hôïp naøy, baát ñaúng thöùc treân hieån nhieân
4
ñuùng.
1 1
* Tröôøng hôïp 2. ≤q≤ .
4 3
4q − 1
AÙp duïng baát ñaúng thöùc Schur, ta coù r ≥ ≥ 0.
9
Do ñoù
(4q − 1)(1 + 6q ) (4q − 1)(1 − 3q )
q(1 − 4q) + r (1 + 6q ) ≥ q (1 − 4q) + = ≥0
9 9
Toùm laïi, ta luoân coù
q(1 − 4q) + r (1 + 6q ) ≥ 0
⇒ ñpcm.

1
Ñaúng thöùc xaûy ra khi vaø chæ khi x = y = z = .
3

289
Baøi toaùn 44. (Phaïm Kim Huøng)
Cho a, b, c, d ≥ 0. Tìm giaù trò nhoû nhaát cuûa bieåu thöùc
2 2 2 2
 a   b   c   d 
P=  +  +  + 
 a+b+c b+c+d  c+d +a d +a+b
Lôøi giaûi.
4
Ta seõ chöùng minh raèng min P = .
9
Trong caùc soá a, b, c, d , goïi p laø soá lôùn nhaát, soá lôùn nhaát trong 3 soá coøn laïi laø q,

soá lôùn nhaát trong 2 soá coøn laïi laø r vaø s laø soá nhoû nhaát.

p ≥ q ≥ r ≥ s

Khi ñoù, ta coù  1 1 1 1
≤ ≤ ≤
p+q+r p+q+s p+r +s q+r+s

Do ñoù, theo baát ñaúng thöùc saép xeáp laïi, ta coù


2 2 2 2
 a   b   c   d 
P=  +  +  + 
 a+b+c b+c+d  c+d +a d +a+b
2 2 2 2
 p   q   r   s 
≥  +  +  + 
 p+q+r  p+q+s  p+r+s q+r+s

Khoâng maát tính toång quaùt, coù theå giaû söû p + q + r + s = 1. Khi ñoù, ñeå chöùng minh

4
P≥ , ta chæ caàn chöùng minh
9

p2 q2 r2 s2 4
+ + + ≥
(1 − s )2 (1 − r ) 2 (1 − q) 2 (1 − p) 2 9

p2 s2
Ñaët m = p + s, n = ps, t = + thì ta coù 0 ≤ m ≤ 1 vaø
(1 − s ) 2 (1 − p) 2

m 2 − 2n − 2m3 + 6mn + m 4 − 4m 2 n + 2n 2
t=
(1 − m + n) 2

⇒ n 2 (2 − t ) − 2n(m − 1)(2m − 1 − t ) + (m − 1) 2 ( m 2 − t ) = 0 (*)

290
+ Neáu t ≥ 2 thì hieån nhieân ta coù

p2 q2 r2 s2 4
+ + + ≥
(1 − s ) (1 − r ) (1 − q)
2 2 2
(1 − p) 2
9

 m = 1, n = 0 p2
+ Neáu  thì ta coù t ≥ 1 vì = 1 . Do ñoù
t = 2 (1 − s ) 2

p2 q2 r2 s2 4
+ + + ≥
(1 − s ) (1 − r ) (1 − q)
2 2 2
(1 − p) 2
9

+ Neáu t < 2, m < 1 . Xem (*) laø phöông trình baäc hai ñoái vôùi n . Do n luoân toàn taïi

neân ta phaûi coù


∆ / = (m − 1) 2 (2m − 1 − t ) 2 − (2 − t )( m − 1) 2 (m 2 − t ) ≥ 0
⇔ (2m − 1 − t ) 2 ≥ (2 − t )(m 2 − t )
−2 m 2 + 4 m − 1
⇒t ≥
(2 − m)2

Töông töï, ñaët l = q + r ⇒ l = 1 − m . Baèng laäp luaän töông töï nhö treân, roõ raøng ta chæ

q2 r2
caàn xeùt tröôøng hôïp l < 1 vaø + < 2 laø ñuû. Khi ñoù, ta coù
(1 − r )2 (1 − q ) 2

q2 r2 −2l 2 + 4l − 1 1 − 2m 2
+ ≥ =
(1 − r )2 (1 − q ) 2 (2 − l ) 2 (1 + m) 2

Do ñoù
p2 q2 r2 s2 −2 m 2 + 4 m − 1 1 − 2 m 2
+ + + ≥ +
(1 − s ) 2 (1 − r ) 2 (1 − q) 2 (1 − p )2 (2 − m) 2 (1 + m) 2
(2m − 1) 2 (11 + 10m − 10m 2 ) 4
= +
9(2 − m) 2 ( m + 1) 2 9
4

9
4
⇒P≥
9
Ñaúng thöùc xaûy ra khi vaø chæ khi a = b = c = d .

291
4
Vaäy min P = .
9
Baøi toaùn 45. (Phaïm Kim Huøng)
Cho a, b, c > 0 thoûa a + b + c = 3 vaø k ∈ R laø moät haèng soá cho tröôùc. Tìm haèng soá

Ck nhoû nhaát sao cho

Ck (a k + b k + c k ) ≥ ab + bc + ca

Lôøi giaûi.
Ta coù Boå ñeà sau

Boå ñeà. a, b, c > 0 thoûa a + b + c = 3 . Khi ñoù, ta coù

 32 k 
(ab) + (bc ) + (ca ) ≤ max 3, 2 k  ∀k ∈ R
k k k

 2 
Chöùng minh.
Ta chöùng minh baát ñaúng thöùc ñuùng cho giaù trò tôùi haïn

32 k ln 3
3= ⇔k=
2 2k
2ln 3 − 2 ln 2
Khi ñoù
+ ∀m ≥ k , ta coù
m
m
 32 k  k 32 m
(ab) m + (bc ) m + (ca) m ≤ ((ab) k + (bc ) k + k k
(ca) ) ≤  2 k  = 2m
2  2

 32 m 
⇒ ( ab) + (bc) + (ca ) ≤ max 3, 2 m 
m m m

 2 
+ ∀m < k , ta coù
k k k k
−1 −1
(( ab) + (bc )
m m
+ (ca ) )
m m
≤ 3 ((ab) k
m + (bc ) + (ca ) ) ≤
k k
3 .3 =
m 3 m

⇒ ( ab) m + (bc )m + (ca ) m ≤ 3


 32 m 
⇒ ( ab) m + (bc )m + (ca ) m ≤ max 3, 2 m 
 2 
292
Khoâng maát tính toång quaùt, coù theå giaû söû a ≤ b ≤ c . Ta chöùng minh veá traùi ñaït max
khi b = c.
Thaät vaäy, ñaët b + c = 2 z , c − b = 2t ⇒ z ≥ t ≥ 0 ∧ a ≤ z − t . Khi ñoù, ta coù

( )
VT = a k ( z + t ) k + ( z − t ) k + ( z 2 − t 2 )k = f (t )

Ta coù f / (t ) = ka k (( z + t )k −1 − ( z − t ) k −1 ) − 2tk ( z 2 − t 2 ) k −1

Xeùt haøm soá g ( x ) = x k −1 vôùi x ≥ 0 .

Ta coù

g / ( x) = (k − 1) x k −2

g // ( x) = (k − 1)( k − 2) x k −3 ≤ 0

⇒ theo ñònh lyù Larange, ta coù g ( x) − g ( y ) ≤ ( x − y ) g / ( y ) ∀0 ≤ y ≤ x

AÙp duïng cho y = z − t , x = z + t , ta ñöôïc ( z + t ) k −1 − ( z − t ) k − ≤ 2t ( k − 1)( z − t ) k −2

Do ñoù
f / (t ) ≤ 2tk ( z − t ) k −2 ( a k (k − 1) − ( z + t ) k −1 ( z − t ))
≤ 2tk ( z − t ) k −1 ( a k −1 ( k − 1) − ( z + t ) k −1 ) (do a ≤ z − t )

≤ 2tk ( z − t ) k −1 ( a k −1 − ( z + t ) k −1 ) ≤ 0 (do a ≤ z − t ≤ z + t )

⇒ f (t ) laø haøm nghòch bieán treân [ 0, + ∞ )


⇒ f (t ) ≤ f (0) = 2b k (3 − 2b) k + b 2 k

Baây giôø ta coøn phaûi chöùng minh


3
g (b) = 2b k (3 − 2b) k − 2b k ≤ 3 ∀1 ≤ b <
2
Ta coù

2 k −1
 3 − 2b k  3 − 2b 
k −1

g (b) = 2kb
/
  − 2   + 1
 b   b  

293
3 − 2b
Ñaët x = (*) thì 0 < x ≤ 1 vaø roõ raøng öùng vôùi moãi x ∈ (0,1] thì ta coù duy nhaát
b

 3
b ∈ 1,  thoûa maõn (*).
 2

Xeùt haøm soá h( x) = x k − 2 x k −1 + 1 vôùi x ∈ (0,1]

Ta coù
h / ( x) = x k −2 ( kx − 2( k − 1))
⇒ h / ( x) coù toái ña 1 nghieäm
⇒ h( x ) coù toái ña 2 nghieäm (theo ñònh lyù Rolle)

 1  8 − 15 1  2 −3
k k
Ta laïi coù h(1) = 0, h   = > 0, h   = <0
8 8k 2 2k

1 1
⇒ h( x ) coù ñuùng 2 nghieäm laø x0 ∈  ,  , x = 1
8 2
3
⇒ g / (b) coù ñuùng 2 nghieäm laø b0 = ,b = 1
x0 + 2

Baûng bieán thieân cuûa g (b)

3 3
b 1
x0 + 2 2

g / (b ) 0 _ 0 +

g (b)

Caên cöù vaøo baûng bieán thieân, ta thaáy

  3   3
g (b) ≤ max  g (1), g    = 3 ∀b ∈ 1, 
  2   2

Boå ñeà ñöôïc chöùng minh hoaøn toaøn.


Trôû laïi baøi toaùn cuûa ta

294
* Neáu k ≥ 1 ∨ k ≤ 0 thì ta coù a k + bk + c k ≥ 3 ≥ ab + bc + ca vaø daáu baèng ñaït taïi
a = b = c = 1 neân hieån nhieân Ck = 1.

* Xeùt k ∈ (0,1)

Cho a = b = c = 1 ta suy ra Ck ≥ 1 .

3 32−k
Cho a = b → , c → 0, ta ñöôïc Ck ≥ 3−k .
2 2
 32−k 
Ngöôïc laïi, ta seõ chöùng minh Ck = max 1, 3−k  thoûa maõn ñieàu kieän cuûa ñeà baøi,
 2 
nghóa laø ta phaûi chöùng minh

Ck (a k + b k + c k )( a + b + c ) 2−k ≥ 32−k ( ab + bc + ca ) (1)

AÙp duïng baát ñaúng thöùc Holder, ta coù


3− k
2− k
 2 2 2

(a + b + c )( a + b + c )
k k k
≥ a +b + c k 
3− k 3− k 3−
 
 
Do ñoù, (1) laø heä quaû cuûa
3− k
 2 2 2

Ck  a + b + c k
3− k 3− k 3−
 ≥ 32−k (ab + bc + ca ) (2)

 

3−k
2 2 2
Ñaët A = 3−
a k ,B = 3−
b k ,C = 3−
c k vaø λ = thì (2) töông ñöông vôùi
2

 A+ B +C  ( AB) λ + ( BC )λ + (CA)λ
Ck   ≥ (3)
 3  3

Do caû hai veá cuûa (3) ñoàng baäc neân khoâng maát tính toång quaùt, coù theå giaû söû
A + B + C = 3 . Khi ñoù, (3) trôû thaønh

( AB )λ + ( BC ) λ + (CA)λ ≤ 3Ck

λ λ
  3 2 λ  λ
⇔ ( AB) + ( BC ) + (CA) ≤ max 3,    (4)
  2  

295
AÙp duïng keát quaû cuûa Boå ñeà treân, ta suy ra (4) ñuùng.
⇒ ñpcm.
Keát luaän
+ k ≥ 1 ∨ k ≤ 0 ⇒ Ck = 1

 32−k 
+ 0 < k < 1 ⇒ Ck = max 1, 3−k  .
 2 

Baøi toaùn 46.


Cho a, b, c ∈ R. Tìm taát caû caùc soá nguyeân döông n sao cho

a ( a + b ) n + b(b + c ) n + c ( c + a ) n ≥ 0

Lôøi giaûi.
Nhaän xeùt raèng n phaûi leû.
1 9
Neáu n ≥ 6 thì cho a = , b = − , c = 1. Khi ñoù, ta coù
4 4
n− 2
5
a( a + b) + b(b + c) + c(c + a ) = 13. 
n n n
− 2n− 2 < 0 ∀n ≥ 6
 4
Do ñoù n ≤ 5 maø n leû neân n = 1 ∨ n = 3 ∨ n = 5. Ta seõ chöùng minh ñoù laø taát caû
nhöõng giaù trò caàn tìm, töùc laø chöùng minh
a( a + b) + b(b + c) + c (c + a ) ≥ 0 (1)
a( a + b) + b(b + c ) + c(c + a) ≥ 0
3 3 3
(2)
a( a + b)5 + b(b + c)5 + c(c + a )5 ≥ 0 (3)

* Chöùng minh (1).


Ta coù
1
(1) ⇔ .(( a + b)2 + (b + c ) 2 + (c + a) 2 ) ≥ 0 (ñuùng)
2
* Chöùng minh (2).

296
2 z = a + b a = − x + y + z
 
Ñaët 2 y = c + a ⇔ b = x − y + z
2 x = b + c c = x + y − z
 
Ta coù

(2) ⇔ 8( x 4 + y 4 + z 4 + xy 3 + yz 3 + zx3 − x 3 y − y 3 z − z 3 x ) ≥ 0
 
⇔ 4  ∑ ( x 2 − y 2 − xy ) 2 + ∑ x 2 y 2  ≥ 0 (ñuùng)
 cyc 
 cyc 

* Chöùng minh (3).

2 z = a + b a = − x + y + z
 
Ñaët 2 y = c + a ⇔ b = x − y + z
2 x = b + c c = x + y − z
 
Ta coù

(3) ⇔ 32( x 6 + y 6 + z 6 + xy 5 + yz 6 + zx5 − x 5 y − y 5 z − z 5 x) ≥ 0


⇔ 16∑ ( x 2 + y 2 )( x 2 − y 2 − xy ) 2 ≥ 0 (ñuùng)
cyc

Vaäy taát caû caùc giaù trò n caàn tìm laø n = 1, n = 3, n = 5.

Baøi toaùn 47.


Cho a, b, c, d > 0 thoûa a + b + c + d = 4. Chöùng minh raèng

a2 b2 c2 d2
+ + + ≥1
b2 + 3 c2 + 3 d 2 + 3 a 2 + 3
Lôøi giaûi.
AÙp duïng baát ñaúng thöùc Bunhiacopxki, ta coù
a2 b2 c2 d2
+ + + ≥
b2 + 3 c2 + 3 d 2 + 3 a 2 + 3
a4 b4 c4 d4
= + + +
a 2b 2 + 3a 2 b 2 c 2 + 3b 2 c 2 d 2 + 3c 2 d 2 a 2 + 3d 2
(a 2 + b 2 + c 2 + d 2 ) 2

3( a 2 + b 2 + c 2 + d 2 ) + a 2b 2 + b 2 c 2 + c 2 d 2 + d 2 a 2

297
AÙp duïng baát ñaúng thöùc AM-GM, ta laïi coù
1
a 2b 2 + b 2c 2 + c 2 d 2 + d 2 a 2 = (a 2 + c 2 )(b 2 + d 2 ) ≤ .( a 2 + b 2 + c2 + d 2 ) 2
4
Do ñoù, ñeå chöùng minh baát ñaúng thöùc ñaõ cho, ta chæ caàn chöùng minh
1
(a 2 + b 2 + c 2 + d 2 ) 2 ≥ 3( a 2 + b 2 + c 2 + d 2 ) + .( a 2 + b 2 + c 2 + d 2 ) 2
4
⇔ a +b +c +d ≥ 4
2 2 2 2

1
⇔ a 2 + b 2 + c 2 + d 2 ≥ .(a + b + c + d ) 2 (ñuùng theo bñt Bunhiacopxki)
4
⇒ ñpcm.

Baøi toaùn 48.


n
1
Cho a1 , a2 ,..., an laø n soá thöïc döông thoûa ∑1+ a = 1. Chöùng minh raèng
i =1 i

n n
1
∑ ai ≥ (n − 1)∑
ai
i =1 i =1

Lôøi giaûi.
1 n
1 − xi
Ñaët xi = (i = 1, n) thì ta coù xi > 0 (i = 1, n), ∑ xi = 1 vaø ai = (i = 1, n).
1 + ai i =1 xi

Khi ñoù, ta coù baát ñaúng thöùc caàn chöùng minh töông ñöông vôùi
n
1 − xi n
xi
∑ ≥ ( n − 1) ∑
i =1 xi i =1 1 − xi
n
1 − nxi
⇔∑ ≥0
i =1 xi (1 − xi )
n
x1 + x2 + ... + xn − nxi
⇔∑ ≥0
i =1 xi (1 − xi )
 1 1 
⇔ ∑ ( xi − x j )  − ≥0
 x j (1 − x j ) x (1 − x ) 
i≠ j  i i 

⇔∑
( xi − x j ) ( xi (1 − xi ) − x j (1 − x j ) )≥0
i≠ j xi x j (1 − xi )(1 − x j )

298
( xi − x j ) 2 (1 − xi − x j )
⇔∑ ≥ 0 (ñuùng)
i≠ j ( xi (1 − xi ) + x j (1 − x j ) ) xi x j (1 − xi )(1 − x j )
⇒ ñpcm.

Baøi toaùn 49. (Poland 1990)


Cho n ≥ 3 vaø x1 , x2 ,..., xn > 0. Chöùng minh raèng
n
xi2
∑ x2 + x x ≤ n − 1
i =1 i i +1 i + 2

trong ñoù xn+1 = x1 vaø xn+ 2 = x2 .

Lôøi giaûi.
Ta chöùng minh baèng quy naïp.
+ n = 3 Khi ñoù, baát ñaúng thöùc caàn chöùng minh töông ñöông vôùi

x2
∑ x 2 + yz ≤ 2
cyc

yz
⇔∑ ≥1
cyc x + yz
2

AÙp duïng baát ñaúng thöùc Bunhiacopxki, ta coù


yz y2z2
∑ x 2 + yz = ∑ x2 yz + y 2 z 2
cyc cyc

( xy + yz + zx ) 2
≥ 2 2
x y + y 2 z 2 + z 2 x 2 + x 2 yz + xy 2 z + xyz 2
( xy + yz + zx) 2
=
( xy + yz + zx )2 − x 2 yz − xy 2 z − xyz 2

( xy + yz + zx) 2 yz
Nhöng ta laïi coù
( xy + yz + zx) 2 − x 2 yz − xy 2 z − xyz 2
≥ 1 , do ñoù ∑ x2 + yz ≥ 1
cyc

Vaäy khaúng ñònh ñuùng khi n = 3.


+ Giaû söû khaúng ñònh ñuùng cho n bieán soá, ta seõ chöùng minh noù cuõng ñuùng cho
n + 1 bieán soá.

299
Khoâng maát tính toång quaùt, ta coù theå giaû söû xn+1 = max { x1 , x2 ,..., xn +1} .

Ta caàn phaûi chöùng minh


n +1
xi2
∑ x2 + x x ≤ n (*)
i =1 i i +1 i + 2

xi2
n
Theo giaû thieát quy naïp, ta coù ∑ 2 ≤ n − 1 . Do ñoù, ñeåchöùng minh (*), ta
i =1 xi + xi +1 xi + 2

chæ caàn chöùng minh

xn2+1 xn2 xn2−1 xn2 xn2−1


+ + − − ≤1
xn2+1 + x1 x2 xn2 + xn +1 x1 xn2−1 + xn xn+1 xn2 + x1 x2 xn2−1 + xn x1

 x2   1 1 
⇔ 1 − 2 n+1  + xn2 .  2 − 2 +
 xn+1 + x1 2 
x x +
 n 1 2 x x x n + x n+1 1 
x
 1 1 
+ xn2−1. 2 − 2 ≥0
 xn −1 + xn x1 xn −1 + xn xn+1 
x1 x2 xn2 x1 ( xn+1 − x2 ) xn2−1 xn ( xn+1 − x1 )
⇔ + + ≥ 0 (ñuùng)
xn2+1 + x1 x2 ( xn2 + x1 x2 )( xn2 + xn +1 x1 ) ( xn2−1 + xn x1 )( xn2−1 + xn xn+1 )
Vaäy khaúng ñònh ñuùng cho n + 1 bieán soá. Theo nguyeân lyù quy naïp, khaúng ñònh
ñuùng cho moïi n ≥ 3.
⇒ ñpcm.

Baøi toaùn 50.

Cho a, b, c, d laø caùc soá thöïc thoûa maõn a 2 + b 2 + c2 + d 2 ≤ 1 . Tìm giaù trò lôùn nhaát

cuûa bieåu thöùc

P = ( a + b ) 4 + ( a + c ) 4 + ( a + d ) 4 + (b + c ) 4 + (b + c ) 4 + ( c + d ) 4

Lôøi giaûi.
Ta coù

(a + b) 4 ≤ (a − b) 4 + ( a + b) 4 = 2(a 4 + b 4 + 6a 2b 2 )

Töôïng töï, ta coù

300
(a + c) 4 ≤ 2(a 4 + c 4 + 6a 2 c 2 )
(a + d )4 ≤ 2( a 4 + d 4 + 6a 2 d 2 )
(b + c )4 ≤ 2(b 4 + c 4 + 6b 2c 2 )
(b + d ) 4 ≤ 2(b 4 + d 4 + 6b 2 d 2 )
(c + d ) 4 ≤ 2(c 4 + d 4 + 6c 2 d 2 )

Do ñoù
P ≤ 6( a 4 + b 4 + c 4 + d 4 + 2a 2b2 + 2a 2c 2 + 2a 2d 2 + 2b 2 c 2 + 2b 2 d 2 + 2c 2d 2 )
= 6( a 2 + b 2 + c 2 + d 2 )2
≤6

1
Ñaúng thöùc xaûy ra khi vaø chæ khi a = b = c = d = ± .
2
Vaäy max P = 6.

Baøi toaùn 51. (Phaïm Kim Huøng)


Cho a, b, c > 0. Chöùng minh raèng

1 1 1 4
f (a, b, c) = + + ≥
4a 2 + bc 4b 2 + ca 4c 2 + ab a+b+c

Lôøi giaûi.
Khoâng maát tính toång quaùt, coù theå giaû söû a ≥ b ≥ c > 0.
a+b
Ta seõ chöùng minh f (a, b, c) ≥ f (t , t , c), trong ñoù t = ≥ c.
2
Thaät vaäy, aùp duïng baát ñaúng thöùc AM-GM, ta coù
1 1 2
+ ≥
4a 2 + bc 4b 2 + ca 4 (4a 2 + bc )(4b 2 + ca)

Maët khaùc, ta coù


2
  a + b  2 ( a + b)c 
 4.   +  − (4a + bc)(4b + ca ) =
2 2

  2  2 
 c2 
= ( a − b)2  a 2 + b 2 + 6ab + − 3ac − 3bc  ≥ 0 (do a ≥ b ≥ c > 0)
 4 
301
1 1 2 2
⇒ + ≥ ≥
4a 2 + bc 4b 2 + ca 4 (4a 2 + bc)(4b 2 + ca ) 4t 2 + tc

Cuõng theo baát ñaúng thöùc AM-GM, ta coù


1 1 1
≥ =
4c + ab
2
a+b
2
4c 2 + t 2
4c + 
2

 2 

Do ñoù
1 1 1
f (a, b, c) = + +
4a 2 + bc 4b 2 + ca 4c 2 + ab
2 1
≥ +
4t + tc
2
4c 2 + t 2
= f (t , t , c )

4
Vaäy ñeå chöùng minh f (a, b, c) ≥ , ta chæ caàn chöùng minh
a+b+c
4
f (t , t , c) ≥
2t + c
2 1 4
⇔ + ≥
4t 2 + tc 4c 2 + t 2 2t + c

 2 1  1 1  4 2
⇔ − + − ≥ − 
 4t + tc t   4c + t
2 2 2 t   2t + c t 

 
 2 1 4c 
⇔ c.  − − ≥0


t (2t + c ) 4t + tc . 2t + 4t + tc
2 2
(
t 4c + t . t + 4c + t 
2 2 2 2
 ) ( )
2 1 4c
⇔ − − ≥0
t (2t + c)
(
4t 2 + tc . 2t + 4t 2 + tc t 4c 2 + t 2 . t + 4c 2 + t 2) ( )
 
 1 1 
⇔ − +
 3t (2t + c)

4t 2 + tc . 2t + 4t 2 + tc ( ) 

302
 
 5 4c 
+ − ≥0
 3t (2t + c ) t 4c + t . t + 4c + t

2 2 2 2
( ) 

4
Nhö vaäy, ñeå chöùng minh f (t , t , c) ≥ , ta chæ caàn chöùng minh
2t + c
1 1
− ≥0
( )
(1)
3t (2t + c) 4t + tc . 2t + 4t + tc
2 2

5 4c
− ≥0
( )
(2)
3t (2t + c) t 4c 2 + t 2 . t + 4c 2 + t 2

* Chöùng minh (1).


Ta coù
1 1
− ≥0
3t (2t + c)
(
4t + tc . 2t + 4t + tc
2 2
)
1 1
⇔ − ≥0
3(2t + c ) (
4t + c . 2 t + 4t + c )
(
⇔ 4t + c . 2 t + 4t + c ≥ 3(2t + c ))
⇔ 2 4t 2 + tc + 4t + c ≥ 6t + 3c
⇔ 4t 2 + tc ≥ t + c (ñuùng do t ≥ c )

⇒ (1) ñuùng.

* Chöùng minh (2).


Ta coù
5 4c
− ≥0
(
3t (2t + c) t 4c 2 + t 2 . t + 4c 2 + t 2
)
5 4c
⇔ − ≥0
3(2t + c ) 2 2
(
4c + t . t + 4c + t
2 2
)
( )
⇔ 5 4c 2 + t 2 . t + 4c 2 + t 2 ≥ 12c (2t + c)

303
⇔ 5t 4c 2 + t 2 + 5(4c 2 + t 2 ) ≥ 12c(2t + c )
⇔ 5t 4c 2 + t 2 + 8c 2 + 5t 2 ≥ 24tc
⇔ 25t 2 (4c 2 + t 2 ) ≥ (8c 2 + 5t 2 − 24tc) 2
⇔ 4c (60t 3 − 139t 2c + 96tc 2 − 16c 3 ) ≥ 0 (ñuùng do t ≥ c)
⇒ (2) ñuùng.
⇒ ñpcm.

Ñaúng thöùc xaûy ra khi vaø chæ khi a = b, c = 0 vaø caùc hoaùn vò töông öùng.

Baøi toaùn 52. (Vasile Cirtoaje)


Cho x, y , z > 0. Chöùng minh raèng

1 1 1  1 1 1 
( x + y + z)  + +  ≥ 1 + 1 + ( x 2 + y 2 + z 2 )  2 + 2 + 2 
x y z x y z 

Lôøi giaûi.

Ñaët a 2 = x + y + z , b 2 = xy + yz + zx, c 2 = xyz ( a, b, c > 0) thì ta coù ab ≥ 3c > 0.

Khi ñoù, ta coù


1 1 1 b2
+ + =
x y z c2
x 2 + y 2 + z 2 = a 4 − 2b 2
1 1 1 b 4 − 2a 2c 2
+ + =
x2 y2 z 2 c4

Do ñoù, baát ñaúng thöùc caàn chöùng minh töông ñöông vôùi

ab ( a 4 − 2b 2 )(b 4 − 2a 2 c 2 )
≥1+ 1+
c c4

⇔ ab − c ≥ c 2 + ( a 4 − 2b 2 )(b 4 − 2a 2c 2 )

⇔ ( ab − c) 2 ≥ c 2 + ( a 4 − 2b 2 )(b 4 − 2a 2 c 2 )

⇔ a 2b 2 − 2abc ≥ (a 4 − 2b 2 )(b 4 − 2a 2c 2 )

⇔ ( a 2b 2 − 2abc) 2 ≥ (a 4 − 2b 2 )(b 4 − 2a 2c 2 )

304
⇔ 2(b3 − a 3c) 2 ≥ 0 (ñuùng)
⇒ ñpcm.

Ñaúng thöùc xaûy ra khi vaø chæ khi x = y = z.

Baøi toaùn 53. (Mildorf)


cho a, b, c > 0, k ∈ R. Chöùng minh raèng

∑ max(a k , bk ).(a − b)2 ≥ 2∑ a k (a − b)(a − c) ≥ ∑ min(a k , bk ).(a − b)2


cyc cyc cyc

Lôøi giaûi.
Khoâng maát tính toång quaùt, coù theå giaû söû a ≥ b ≥ c > 0.
Coù 2 tröôøng hôïp xaûy ra

* Tröôøng hôïp 1. k ≥ 0 ⇒ a k ≥ b k ≥ c k > 0.


Tröôùc heát, ta chöùng minh

∑ max(a k , bk ).(a − b)2 ≥ 2∑ a k (a − b)(a − c)


cyc cyc

⇔ a (a − b) + a ( a − c ) + b k (b − c) 2 ≥ 2∑ a k ( a − b)( a − c )
k 2 k 2

cyc

Chuù yù raèng (a − b) 2 + ( a − c ) 2 = (b − c) 2 + 2( a − b)(a − c) , neân baát ñaúng thöùc treân

töông ñöông vôùi

a k (b − c) 2 + b k (b − c )2 + 2a k ( a − b)( a − c ) ≥ 2∑ a k ( a − b)(a − c )
cyc

⇔ a k (b − c) 2 + b k (b − c) 2 ≥ 2b k (b − a)(b − c) + 2c k (c − a)(c − b)
⇔ a k (b − c) + b k (b − c ) + 2b k ( a − b) − 2c k ( a − c ) ≥ 0
⇔ ( a k + b k − 2c k )(b − c) + 2(b k − c k )( a − b) ≥ 0 (ñuùng)

Tieáp theo, ta seõ chöùng minh


2∑ a k ( a − b)( a − c ) ≥ ∑ min( a k , b k ).(a − b) 2
cyc cyc

⇔ 2∑ a k ( a − b)( a − c) ≥ b k ( a − b)2 + ck (a − c) 2 + c k (b − c) 2
cyc

305
Chuù yù raèng (a − c) 2 + (b − c) 2 = ( a − b) 2 + 2(c − a )(c − b) , neân baát ñaúng thöùc treân

töông vôùi

2∑ a k ( a − b)( a − c ) ≥ b k (a − b) 2 + c k (a − b) 2 + 2c k (c − a )(c − b)
cyc

⇔ 2a k (a − b)( a − c ) + 2b k (b − a )(b − c) ≥ b k (a − b) 2 + c k (a − b) 2
⇔ 2a k (a − c) − 2b k ( a − b)(b − c) ≥ b k (a − b) + c k (a − b)
⇔ 2( a k − b k )( a − b) + (2a k − b k − c k )(b − c ) ≥ 0 (ñuùng)

Vaäy trong tröôøng hôïp naøy, ta coù

∑ max(a k , bk ).(a − b)2 ≥ 2∑ a k (a − b)(a − c) ≥ ∑ min(a k , bk ).(a − b)2


cyc cyc cyc

* Tröôøng hôïp 2. k < 0 ⇒ a k ≤ b k ≤ c k .


Laäp luaän töông töï tröôøng hôïp 1, ta cuõng coù

∑ max(a k , bk ).(a − b)2 ≥ 2∑ a k (a − b)(a − c) ≥ ∑ min(a k , bk ).(a − b)2


cyc cyc cyc

Toùm laïi, trong moïi tröôøng hôïp, ta luoân coù

∑ max(a k , bk ).(a − b)2 ≥ 2∑ a k (a − b)(a − c) ≥ ∑ min(a k , bk ).(a − b)2


cyc cyc cyc

⇒ ñpcm

Baøi toaùn 54. (Vasile Cirtoaje)


Cho ∆ABC . Chöùng minh raèng

1 1 1
+ + ≥2
2 − cos A 2 − cos B 2 − cos C
Lôøi giaûi.
Ta coù baát ñaúng thöùc caàn chöùng minh töông ñöông vôùi

∑ (2 − cos A)(2 − cos B) ≥ 2(2 − cos A)(2 − cos B)(2 − cos C )


cyc

⇔ 4∑ cos A − 2∑ cos A.cos B + 3cos A.cos B.cos C − 4 ≥ 0


cyc cyc

C C C
⇔ 4cos C + 8sin .t − 6sin .cos C.t − 3t 2 + 3cos 2 +
2 2 2

306
C
+ 2cos C.t 2 − 2cos 2 .cos C − 4 ≥ 0 (*)
2
A− B C  A- B A+ B π 
trong ñoù t = cos ⇒ 1 ≥ t ≥ sin  do 0 ≤ ≤ < 
2 2  2 2 2

Ñaët VT (*) = f (t )

Ta coù
f // (t ) = 2(2 cos C − 3) < 0
 C 
⇒ f (t ) laø haøm loài treân sin ,1 .
 2 
  C 
⇒ f (t ) ≥ min  f  sin  , f (1) 
  2 
Ta coù

 C C C C
f  sin  = 4cos C + 8sin 2 − 6sin 2 .cos C − 3sin 2 +
 2 2 2 2
C C C
+ 3cos 2
+ 2 cos C.sin 2 − 2cos2 .cos C − 4
2 2 2
C C C C
= 4cos C + 5sin 2 + 3cos 2 − 4sin 2 .cos C − 2cos 2 .cos C − 4
2 2 2 2
C C
= 4cos C + 2sin 2 + 3 − 2sin 2 .cos C − 2cos C − 4
2 2
C
= 2cos C + 2sin 2 .(1 − cos C ) − 1
2
= 2cos C + (1 − cos C ) 2 − 1
= cos 2 C
≥0

Ta laïi coù
C C C
f (1) = 4cos C + 8sin − 6sin .cos C − 3 + 3cos 2 +
2 2 2
C
+ 2cos C − 2cos 2 .cos C − 4
2
C C C C
= 6cos C + 8sin − 3sin 2 − 6sin .cos C − 2cos 2 .cos C − 4
2 2 2 2
307
2
C  C   C
= sin . 2sin − 1  2 − sin  ≥ 0
2  2   2

 C   C 
Vaäy ta coù f  sin  ≥ 0 vaø f (1) ≥ 0 ⇒ min  f  sin  , f (1)  ≥ 0
 2   2 
  C 
⇒ f (t ) ≥ min  f  sin  , f (1)  ≥ 0
  2 
⇒ ñpcm.

π π
Ñaúng thöùc xaûy ra khi vaø chæ khi A = B = C = hoaëc A = B → , C → 0 vaø caùc
3 2
hoaùn vò töông öùng.

Baøi toaùn 55.


Cho a, b, c > 0. Chöùng minh raèng

a b c
(a + b + c)  + +  ≥ 3 3( a 2 + b 2 + c 2 )
b c a
Lôøi giaûi.
Ta coù
a b c
(a + b + c)  + +  ≥ 3 3( a 2 + b 2 + c 2 )
b c a
a2 ab
⇔ ∑ + ∑ + a + b + c ≥ 3 3( a 2 + b2 + c 2 )
cyc b cyc c

 a2
cyc  b
  ab

  cyc c

⇔ ∑  + b − 2a  +  ∑ − a − b − c  ≥ 3


( 3(a 2 + b 2 + c 2 ) − a − b − c )
3∑ (a − b) 2
(a − b) 2
1 c( a − b) 2
⇔∑ + .∑ ≥ cyc

cyc b 2 cyc ab 3( a 2 + b2 + c 2 ) + a + b + c

AÙp duïng baát ñaúng thöùc Bunhiacopxki, ta coù


3∑ (a − b) 2 ∑ ( a − b)2
3 cyc
≤ .
cyc

3( a 2 + b 2 + c 2 ) + a + b + c 2 a + b + c

308
Do ñoù, ñeå chöùng minh baát ñaúng thöùc ñaõ cho, ta chæ caàn chöùng minh

( a − b) 2
1 c(a − b) 2
3 cyc
∑ (a − b) 2
∑ b
+ .∑
2 cyc ab
≥ .
2 a+b+c
cyc

(a − b)2
c( a − b) 2 ∑ (a − b) 2
⇔ 2∑ +∑ ≥ 3 cyc
cyc b cyc ab a+b+c
1 1  2 c 1 
⇔ 2∑ ( a − b) 2  −  + ∑ ( a − b)  − ≥0
cyc  b a + b + c  cyc  ab a + b + c 
(a − b) 2 (a + c) 1
⇔ 2∑ + .∑ ( a − b) 2 (c 2 + ac + bc − ab ) ≥ 0
cyc b( a + b + c ) abc( a + b + c ) cyc

( a − b) 2 (a + c)
Deã thaáy 2∑ ≥ 0 . Do ñoù, ta chæ caàn chöùng minh
cyc b( a + b + c )

∑ (a − b)2 (c 2 + ac + bc − ab) ≥ 0
cyc

Khoâng maát tính toång quaùt, giaû söû a ≥ b ≥ c > 0 ⇒ a − c ≥ a − b ≥ 0.


Khi ñoù, ta coù

∑ (a − b)2 (c 2 + ac + bc − ab) =
cyc

= (b − c) 2 (a 2 + ab + ac − bc) + (a − c) 2 (b 2 + ab + bc − ac ) +
+ ( a − b) 2 (c 2 + ac + bc − ab)

≥ ( a − c) 2 (b 2 + ab + bc − ac) + (a − b) 2 (c 2 + ac + bc − ab)

≥ ( a − b) 2 (b 2 + ab + bc − ac) + ( a − b) 2 (c 2 + ac + bc − ab)
= ( a − b)2 (b + c) 2
≥0
Vaäy ∑ (a − b)2 (c 2 + ac + bc − ab) ≥ 0
cyc

⇒ ñpcm.

Ñaúng thöùc xaûy ra khi vaø chæ khi a = b = c.

309
Baøi toaùn 56. (Leâ Trung Kieân)
Cho a, b, c > 0. Chöùng minh raèng

a (b + c) b(c + a ) c(a + b) 2( a − b)2 (b − c) 2 (c − a ) 2


+ + ≤ 2+ 2
a 2 + 2bc b 2 + 2ca c 2 + 2 ab ( a + 2bc)(b2 + 2ca )(c 2 + 2ab)

Lôøi giaûi.
Ta coù baát ñaúng thöùc caàn chöùng minh töông ñöông vôùi
3a(b + c) 6( a − b) 2 (b − c) 2 (c − a ) 2
∑ a 2 + 2bc ≤ 6 +
( a 2 + 2bc)(b2 + 2ca )(c 2 + 2ab)
cyc

 3a(b + c)  6(a − b) 2 (b − c)2 (c − a )2


⇔ ∑ 2 − 2 + ≥0
cyc  a + 2bc  (a 2 + 2bc)(b2 + 2ca )(c 2 + 2ab)
2a 2 − 3a(b + c) + 4bc 6(a − b) 2 (b − c)2 (c − a )2
⇔∑ + 2 ≥0
cyc a 2 + 2bc (a + 2bc)(b2 + 2ca )(c 2 + 2ab)
(a − 2c)( a − b) − (a − 2b)(c − a) 6( a − b)2 (b − c) 2 (c − a ) 2
⇔∑ + ≥0
cyc a 2 + 2bc ( a 2 + 2bc)(b 2 + 2ca)(c 2 + 2ab)
(a − 2c)( a − b) ( a − 2b)(c − a ) 6(a − b) 2 (b − c ) 2 (c − a ) 2
⇔∑ −∑ + 2 ≥0
cyc a 2 + 2bc cyc a 2 + 2bc (a + 2bc )(b2 + 2ca )(c 2 + 2ab)
(a − 2c)( a − b) (b − 2c )( a − b) 6( a − b) 2 (b − c) 2 (c − a ) 2
⇔∑ − ∑ b2 + 2ca + ≥0
cyc a 2 + 2bc cyc (a 2 + 2bc)(b2 + 2ca )(c 2 + 2ab)
(a − b) 2 (−4c 2 + 4c( a + b) − ab) 6(a − b) 2 (b − c )2 (c − a )2
⇔∑ + ≥0
cyc ( a 2 + 2bc )(b 2 + 2ca) (a 2 + 2bc)(b 2 + 2ca )(c 2 + 2ab)
⇔ ∑ ( a − b) 2 ( −4c 2 + 4c (a + b) − ab)(c 2 + 2ab) + 6(a − b) 2 (b − c )2 (c − a )2 ≥ 0
cyc

⇔ 3∑ ab( a − b) 2 (c 2 + 2ab) + ∑ (a − b) 2 ( −4c 2 + 4c(a + b) − 4ab)(c 2 + 2ab) +


cyc cyc

1 + 6(a − b) 2 (b − c) 2 (c − a ) 2 ≥ 0
⇔ 3∑ ab(a − b) 2 (c 2 + 2ab) + 4( a − b)(b − c)(c − a ).∑ ( a − b)(c 2 + 2ab) +
cyc cyc

1 + 6( a − b)2 (b − c) 2 (c − a ) 2 ≥ 0

⇔ 3∑ ab( a − b) 2 (c 2 + 2ab) − 12(a − b) 2 (b − c )2 (c − a )2 +


cyc

+ 6( a − b) 2 (b − c ) 2 (c − a ) 2 ≥ 0

310
⇔ ∑ ab(a − b) 2 (c 2 + 2ab) − 2( a − b) 2 (b − c) 2 (c − a ) 2 ≥ 0
cyc

Khoâng maát tính toång quaùt, giaû söû a ≥ b ≥ c > 0. Khi ñoù, ta coù

∑ ab(a − b)2 (c 2 + 2ab) ≥ ab(a − b)2 (c 2 + 2ab)


cyc

≥ 2a 2b 2 (a − b) 2
≥ 2( a − b) 2 (b − c) 2 (c − a ) 2

Vaäy

∑ ab(a − b)2 (c 2 + 2ab) − 2(a − b)2 (b − c)2 (c − a)2 ≥ 0


cyc

a (b + c) b(c + a) c (a + b) 2(a − b) 2 (b − c)2 (c − a )2


⇒ + + ≤ 2 + (ñpcm)
a 2 + 2bc b 2 + 2ca c 2 + 2ab (a 2 + 2bc)(b 2 + 2ca )(c 2 + 2ab)
Ñaúng thöùc xaûy ra khi vaø chæ khi a = b = c hoaëc a = b, c = 0 vaø caùc hoaùn vò.

Baøi toaùn 57.


Cho a, b, c > 0. Chöùng minh raèng

2(1 + a 2 )(1 + b 2 )(1 + c 2 ) ≥ (1 + a )(1 + b)(1 + c) − 2(1 + abc)

Lôøi giaûi.

* Caùch 1.
Ta coù baát ñaúng thöùc caàn chöùng minh töông ñöông vôùi

2(1 + a 2 )(1 + b 2 )(1 + c 2 ) ≥ −1 + a + b + c + ab + bc + ca − abc


⇔ 2(1 + a 2 )(1 + b 2 )(1 + c 2 ) ≥ (−1 + a + b + c + ab + bc + ca − abc )2

Chuù yù raèng (1 + a 2 )(1 + b 2 )(1 + c 2 ) = ( −1 + ab + bc + ca) 2 + (a + b + c − abc) 2

Do ñoù, baát ñaúng thöùc caàn chöùng minh töông ñöông vôùi
2(−1 + ab + bc + ca ) 2 + 2( a + b + c − abc) 2 ≥ ( −1 + a + b + c + ab + bc + ca − abc ) 2
⇔ ( −1 + ab + bc + ca − a − b − c + abc) 2 ≥ 0 (ñuùng)
⇒ ñpcm.
* Caùch 2.
Ta coù baát ñaúng thöùc caàn chöùng minh töông ñöông vôùi

311
2(1 + a 2 )(1 + b2 )(1 + c 2 ) ≥ −1 + a + b + c + ab + bc + ca − abc

⇔ 2(1 + a 2 )(1 + b 2 )(1 + c 2 ) ≥ (−1 + a + b + c + ab + bc + ca − abc )2

 π
Ñaët a = tgA, b = tgB, c = tgC  0 < A, B, C <  . Khi ñoù, baát ñaúng thöùc caàn chöùng
 2

minh töông ñöông vôùi


2
2  sin A sin A.sin B sin A.sin B.sin C 
≥ 
cos 2 A.cos 2 B.cos 2 C 
−1 + ∑ cos A ∑ cos A.cos B − cos A.cos B.cos C 
+
cyc cyc 

⇔ 2 ≥  ∑ sin A.cos B.cos C − sin A.sin B.sin C +
 cyc

2

+ ∑ sin A.sin B.cos C − cos A.cos B.cos C 

cyc 
Chuù yù raèng
sin( A + B + C ) = ∑ sin A.cos B.cos C − sin A.sin B.sin C
cyc

cos( A + B + C ) = cos A.cos B.cos C − ∑ sin A.sin B.cos C


cyc

Do ñoù, baát ñaúng thöùc caàn chöùng minh töông ñöông vôùi
2 ≥ (sin( A + B + C ) − cos( A + B + C )) 2 (hieån nhieân ñuùng)
⇒ ñpcm.

Baøi toaùn 58. (France 2004)


Cho a, b, c, d , e, f ∈ R thoûa a + b + c + d + e + f = 0. Chöùng minh raèng

1
ab + bc + cd + de + ef + fa ≤ .(a 2 + b 2 + c 2 + d 2 + e 2 + f 2 )
2
Lôøi giaûi.
* Caùch 1.

Ta coù (a + c + e)(b + d + f ) = −( a + c + e) 2 ≤ 0

Maët khaùc, ta coù

312
(a + c + e)(b + d + f ) = ( ab + bc + cd + de + ef + fa ) + ( ad + be + fc)

Do ñoù
ab + bc + cd + de + ef + fa ≤ −ad − be − fc
a 2 + d 2 b2 + e2 c 2 + f 2
≤ + +
2 2 2
1
= .(a 2 + b 2 + c 2 + d 2 + e2 + f 2 )
2
⇒ ñpcm.

* Caùch 2.
Ñaët
A = a 2 + b 2 + c2 + d 2 + e2 + f 2
B = ab + bc + cd + de + ef + fa
C = ac + bd + ce + df + ea + fb
D = ad + be + cf

Khi ñoù, ñeå chöùng minh baát ñaúng thöùc ñaõ cho, ta caàn chöùng minh A ≥ 2 B
Theo giaû thieát, ta coù

(a + b + c + d + e + f )2 = A + 2 B + 2C + 2 D = 0

Ta laïi coù

(a + d )2 + (b + e) 2 + (c + f ) 2 = A + 2 D
(a + c + e) 2 + (b + d + f ) 2 = A + 2C

Vì toång caùc bình phöông luoân khoâng aâm neân ta coù

( a + d ) 2 + ( b + e ) 2 + ( c + f ) 2 + ( a + c + e ) 2 + (b + d + f ) 2 = 2 A + 2 C + 2 D ≥ 0
Theo treân, ta coù A + 2 B + 2C + 2 D = 0
Do ñoù
2 A + 2C + 2 D ≥ A + 2 B + 2C + 2 D
⇒ A ≥ 2B
⇒ ñpcm.

313
Baøi toaùn 59. (Voõ Quoác Baù Caån)
Cho x, y > 0. Chöùng minh raèng

4 y 2 ( x 2 + 3 y 2 )( y 2 + 3 x 2 ) + 8 x 2 y x 2 + 3 y 2 +
+ 4 x( x 2 + y 2 ) y 2 + 3 x 2 ≤ 3( x 2 + 3 y 2 )( y 2 + 3 x 2 )

Lôøi giaûi.
Ta coù Boå ñeà sau

 π
Boå ñeà. ∀x ∈  0,  thì
 2

1 3 3
sin x + .sin 2 x ≤
2 4
Chöùng minh.
1  π
Xeùt haøm soá f ( x) = sin x + .sin 2 x vôùi x ∈  0,  .
2  2

Ta coù f / ( x) = cos x + cos 2 x = 2cos 2 x + cos x − 1 = (1 + cos x)(2cos x − 1)

π
f / ( x) = 0 ⇔ x = .
3
π
Qua thì f / ( x ) ñoåi daáu töø döông sang aâm, neân
3

π  3 3  π
f ( x) ≤ f   = ∀x ∈  0, 
3 4  2
Boå ñeà ñöôïc chöùng minh hoaøn toaøn.
Trôû laïi baøi toaùn cuûa ta
Ta coù baát ñaúng thöùc caàn chöùng minh töông ñöông vôùi

y2 3 2x2 y 3 x( x 2 + y 2 ) 3 3 3
+ + ≤
( x + 3 y )( y + 3 x )
2 2 2 2
( y + 3x ) x + 3 y
2 2 2 2
( x + 3 y ) y + 3x
2 2 2 2 4
Chuù yù raèng

314
y2 3 x2 y
= 1− 2 .
( x 2 + 3 y 2 )( y 2 + 3x 2 ) x + 3y 2
y 2 + 3x 2
2x2 y 3 y2 2 xy
= 1− 2 .
( y 2 + 3x 2 ) x 2 + 3 y 2 y + 3x 2
( x 2 + 3 y 2 )( y 2 + 3 x 2 )
x( x 2 + y 2 ) 3 4x2 y2 x
= 1− .
( x 2 + 3 y 2 ) y 2 + 3x2 ( x + 3 y )( y + 3 x ) x 2 + 3 y 2
2 2 2 2

x y 2 xy
Maët khaùc 0 < , , < 1.
x2 + 3 y2 y 2 + 3x 2 ( x 2 + 3 y 2 )( y 2 + 3 x 2 )

Do ñoù, ta coù theå ñaët


x y 2 xy
cos A = ,cos B = , cos C =
x2 + 3 y2 y 2 + 3x 2 ( x 2 + 3 y 2 )( y 2 + 3x 2 )

π
trong ñoù 0 < A, B, C < .
2
Khi ñoù, baát ñaúng thöùc caàn chöùng minh töông ñöông vôùi

3 3
sin A.cos B + sin B.cos C + sin C.cos A ≤
4
Maët khaùc, töø caùch ñaët, ta coù

cos 2 A + cos 2 B + cos 2 C + 2cos A.cos B.cos C = 1


⇒ A + B + C = π.
π
Do > A, B, C > 0 neân (sin A,sin B,sin C ) vaø (cos A,cos B,cos C ) laø 2 daõy ñôn
2
ñieäu ngöôïc chieàu.
⇒ Theo baát ñaúng thöùc saép xeáp laïi, ta coù
sin A.cos B + sin B.cos C + sin C .cos A ≤ sin A.cos C + sin B.cos B + sin C.cos A
1
= sin B + .sin 2 B
2
3 3
≤ (theo Boå ñeà treân)
4
⇒ ñpcm.
315
π
Ñaúng thöùc xaûy ra khi vaø chæ khi A = B = C = ⇔ x = y.
3
Baøi toaùn 60. (Phaïm Kim Huøng)
Cho a, b, c > 0. Tìm haèng soá k nhoû nhaát sao cho baát ñaúng thöùc sau ñuùng
k k k
 a   b   c  3
  +  +  ≥ k
b+c c+a  a+b 2

Lôøi giaûi.
ln3
Cho b = c = 1, a → 0+ , ta suy ra k ≥ − 1 = n.
ln 2
Ta seõ chöùng minh kmin = n, töùc laø chöùng minh
n n n
 a   b   c  3
  +  +  ≥ n
b+c c+a  a+b 2

0 < a ≤ b ≤ c 1
Khoâng maát tính toång quaùt, coù theå giaû söû  ⇒0<a≤ .
a + b + c = 1 3

b = t − m
c + b = 2t 
Ñaët  ⇒ c = t + m
c − b = 2 m t > m ≥ 0

(t + m)n (t − m) n
Xeùt haøm soá f (m) = + vôùi m ≥ 0.
(t − m + a) n (t + m + a ) n

 (t + m) n −1 (t − m) n −1 
Ta coù f ( m) = n(2t + a ) 
/
n+1
− 
 (t − m + a ) (t + m + a ) n+1 

Ta seõ chöùng minh f / ( m) ≥ 0 ∀m ≥ 0.

Thaät vaäy
f / ( m) ≥ 0
⇔ (1 − n)(ln(t + m) − ln(t − m)) ≤ (n + 1)(ln(t + m + a ) − ln(t − m + a ))
1+ n
Do > 2 neân ta chæ caàn chöùng minh
1− n

316
ln(t + m) − ln(t − m) ≤ 2(ln(t + m + a ) − ln(t − m + a ))

Xeùt haøm soá g ( m) = ln(t + m) − ln(t − m) − 2ln(t + m + a ) + 2ln(t − m + a )

1 1 2 2
Ta coù g / (m) = − − − ≤ 0 (do a ≤ b ≤ c)
m+t t −m a +t + m a +t −m
⇒ g ( m) laø haøm nghòch bieán treân [0,+∞).
⇒ g ( m) ≤ g (0) = 0 ∀m ≥ 0
⇒ ln(t + m) − ln(t − m) ≤ 2(ln(t + m + a ) − ln(t − m + a ))
⇒ f / (m) ≥ 0

⇒ f ( m) laø haøm ñoàng bieán treân [0,+∞).


n n
 t   1- a 
⇒ f ( m) ≥ f (0) = 2   = 2  ∀m ≥ 0
t+a 1+ a 
Do ñoù
n n n n
 a   b   c  (t + m)n (t − m) n  a 
 +
  +
   = + + 
b+c c+a  a+b (t − m + a) (t + m + a )  1 − a 
n n

n n
 1− a   a 
≥ 2  + 
1+ a  1− a 
= h( a)

 a n−1 4(1 − a ) n−1 


Ta coù h ( a) = n 
/
n+1
− 
 (1 − a ) (1 + a ) n+1 

h / ( a) = 0
⇔ ln 4 = (n + 1)ln(1 + a ) − 2n ln(1 − a ) + (n − 1)ln a

Ñaët ϕ (a ) = ( n + 1)ln(1 + a ) − 2ln(1 − a) + (n − 1)ln a

n + 1 2n 1 − n (3n + 1)a + n − 1
Ta coù ϕ / ( a) = + − =
1+ a 1− a a a (1 − a 2 )

1− n 1  ln 3 1
⇒ ϕ / (a ) coù 1 nghieäm döông duy nhaát laø a0 = <  do n = −1 > 
3n + 1 3  ln 2 3

317
1
Qua a0 thì ϕ / (a ) ñoåi daáu töø aâm sang döông vaø lim ϕ ( a ) = +∞,ϕ   = ln 4 neân
a →0 3
1 1
phöông trình ϕ (a ) = ln 4 coù 2 nghieäm döông phaân bieät laø vaø 0 < a1 < .
3 3
1
⇒ phöông trình h / ( a) = 0 coù 2 nghieäm döông phaân bieät laø vaø a1.
3
1
Qua a1 thì h / (a ) ñoåi daáu töø döông sang aâm, qua thì h / (a ) ñoåi daáu töø aâm sang
3
döông neân ta coù

  1  3
h(a ) ≥ min h(0), h    = n
  3  2
n n n
 a   b   c  3
⇒  +  +  ≥ n
b+c c+a a+b 2

Vaäy kmin = n.

Baøi toaùn 61. (Traàn Nam Duõng)


Cho x > 0. Tìm haèng soá s döông nhoû nhaát sao cho

 1   1
2  x s + s + 1 ≥ 3  x + 
 x   x

Lôøi giaûi.
Roõ raøng vôùi x = 1 thì baát ñaúng thöùc ñaõ cho trôû thaønh ñaúng thöùc. Do ñoù, khoâng maát
tính toång quaùt, ta chæ caàn xeùt x > 1 laø ñuû. Khi ñoù, ta coù

 1   1
2  x s + s + 1 ≥ 3  x + 
 x   x
2( x s − 1) 2 3( x − 1) 2
⇔ ≥
xs x
s −1
xs − 1 3
⇔ ≥ .x 2
x −1 2
Theo ñònh lyù Lagrange, toàn taïi y ∈ (1, x) sao cho

318
xs −1
= ( y s ) / = sy s −1
x −1
Do ñoù
s −1
xs −1 3
≥ .x 2
x −1 2
s −1
s −1 3
⇔ sy ≥ .x 2
2

3
Cho x → 1+ thì y → 1+ , ta suy ra ñöôïc s ≥ . Ta seõ chöùng minh ñaây laø giaù trò
2
s −1
xs −1
caàn tìm. Ñeå coù ñieàu naøy, ta chæ caàn chöùng minh ≥ s.x 2 ∀x, s > 1 laø ñuû.
x −1
Ta coù
s −1
xs −1
≥ s.x 2
x −1
s +1 s −1
⇔ f ( x) = x − 1 − s
sx 2 + sx 2 ≥0
s −3 
 s +1  ( s + 1)( x − 1) 
Ta coù f ( x ) = sx . x 2 − 1 − 
 
/ 2
 2 
  

Theo ñònh lyù Lagrange, toàn taïi z ∈ (1, x) sao cho


/
s +1
 s +1  s −1
( s + 1)( x − 1) ( s + 1)( x − 1)
x2 − 1 =  z  .( x − 1) = z 2 .
2 > (do s , z > 1)
  2 2
 

⇒ f / ( x) > 0 ∀x > 1

⇒ f ( x) laø haøm ñoàng bieán treân (1, +∞) .


⇒ f ( x ) ≥ lim f ( x) = 0 ∀x > 1
x→1

3
Vaäy smin = .
2

319
Baøi toaùn 62. (Bulgaria 2003)
Cho a, b, c > 0 thoûa a + b + c = 3. Chöùng minh raèng

a b c 3
+ 2 + 2 ≥
b +1 c +1 a +1 2
2

Lôøi giaûi.
* Caùch 1.
Baèng caùch quy ñoàng maãu soá vaø thu goïn, ta coù baát ñaúng thöùc caàn chöùng minh
töông ñöông vôùi
2(a 2b 3 + b 2c3 + c 2 a 3 + a 3 + b 3 + c 3 + a 2b + b 2c + c 2 a + a + b + c ) ≥
≥ 3( a 2b 2c 2 + a 2b 2 + b 2c 2 + c 2 a 2 + a 2 + b 2 + c 2 )
⇔ 2( a 2b3 + b 2c 3 + c 2a 3 + a 3 + b3 + c 3 + a 2b + b 2 c + c 2 a + 3) ≥
≥ 3( a 2b 2c 2 + a 2b 2 + b 2c 2 + c 2 a 2 + a 2 + b 2 + c 2 )

AÙp duïng baát ñaúng thöùc AM-GM, ta coù


3 2 3
.( a b + a 2b) ≥ 3a 2b 2
2
3 2 3
.(b c + b 2c ) ≥ 3b 2c 2
2
3 2 3 2
.(c a + c a) ≥ 3c 2a 2
2
3    
⇒ . ∑ a 2b 3 + ∑ a 2b  ≥ 3  ∑ a 2b 2  (1)
2  cyc cyc   cyc 
Laïi aùp duïng baát daúng thöùc AM-GM, ta coù
2a 3 + 1 ≥ 3a
2b3 + 1 ≥ 3b
2c3 + 1 ≥ 3c
⇒ 2( a 3 + b3 + c 3 ) + 3 ≥ 3( a + b + c ) = 9
⇒ 2(a 3 + b3 + c 3 ) ≥ 6 (2)

Tieáp tuïc aùp duïng baát ñaúng thöùc AM-GM, ta coù

320
1 2 3
.( a b + b 2c 3 + c 2a 3 + a 2b + b 2 c + c 2 a) ≥ 3a 4 3b 4 3c 4 3
2
3a 2b 2c 2
=
3 2 2 2
abc

3a 2b 2c 2

a+b+c
2

 
 3 

= 3a 2b2 c 2

1
⇒ .( a 2b3 + b 2 c3 + c 2 a 3 + a 2b + b 2 c + c 2a ) ≥ 3a 2b 2c 2 (3)
2
Coäng caùc baát ñaúng thöùc (1),(2) vaø (3) veá theo veá, ta ñöôïc

2(a 2b 3 + b 2c3 + c 2a 3 + a 3 + b 3 + c 3 + a 2b + b 2c + c 2a + 3) ≥
≥ 3( a 2b 2c 2 + a 2b 2 + b 2c 2 + c 2 a 2 + a 2 + b 2 + c 2 )
⇒ ñpcm.

* Caùch 2.
Ta coù baát ñaúng thöùc caàn chöùng minh töông ñöông vôùi

 a   b   c  3
 2 − a +  2 −b +  2 − c ≥ + a + b + c
 b +1   c +1   a +1  2
ab 2 bc 2 ca 2 3
⇔ 2 + 2 + 2 ≤
b +1 c +1 a +1 2
AÙp duïng baát ñaúng thöùc AM-GM, ta coù

ab 2 ab 2 1
≤ = .ab
b 2 + 1 2b 2
Töông töï, ta coù
bc 2 1
≤ .ab
c +1 2
2

ca 2 1
≤ .ab
a +1 2
2

Do ñoù

321
ab 2 bc 2 ca 2 1 3
+ + ≤ .(ab + bc + ca ) ≤
b +1 c +1 a +1 2
2 2 2
2
⇒ ñpcm.

Ñaúng thöùc xaûy ra khi vaø chæ khi a = b = c = 1.


Baøi toaùn 63.
Cho n ≥ 4, n ∈ N vaø a1 , a2 ,..., an ≥ 0 thoûa maõn a1 + a2 + ... + an = 2. Tìm giaù trò nhoû

nhaát cuûa bieåu thöùc


a1 a a
P= + 2 2 + ... + 2 n
a + 1 a3 + 1
2
2 a1 + 1

Lôøi giaûi.
Ta coù Boå ñeà sau
Boå ñeà. n ≥ 4, n ∈ N vaø a1, a2 ,..., an ≥ 0. Khi ñoù, ta coù

4(a1a2 + a2a3 + ... + an a1 ) ≤ (a1 + a2 + ... + an ) 2

Chöùng minh.
Ñaët f n (a1 , a2 ,..., an ) = 4( a1a2 + a2 a3 + ... + an a1 ) − ( a1 + a2 + ... + an ) 2 . Ñeå chöùng minh

Boå ñeà treân, ta seõ chöùng minh baèng quy naïp theo n raèng
f n ( a1 , a2 ,..., an ) ≤ 0

+ n = 4, ta coù

f 4 ( a1 , a2 , a3 , a4 ) = 4( a1a2 + a2 a3 + a3a4 + a4 a1 ) − (a1 + a2 + a3 + a4 ) 2


= 4( a1 + a3 )( a2 + a4 ) − ( a1 + a2 + a3 + a4 ) 2
= −(a1 − a2 + a3 − a4 ) 2
≤0
⇒ f 4 ( a1 , a2 , a3 , a4 ) ≤ 4

Vaäy khaúng ñònh ñuùng khi n = 4.


Giaû söû khaúng ñònh ñuùng cho n − 1 bieán soá (n ≥ 5) , ta seõ chöùng minh khaúng ñònh

ñuùng cho n bieán soá.

322
Khoâng maát tính toång, quaùt coù theå giaû söû a1 = max{a1 , a2 ,..., an }. Khi ñoù, ta coù

f n ( a1 , a2 ,..., an ) − f n−1 (a1, a2 ,..., an−3 , an− 2 , an−1 + an ) =


= 4( an− 2 an−1 + an−1an + an a1 − an− 2 (an−1 + an ) − (an −1 + an ) a1 )
= 4( an−1an − an− 2an − an−1a1 )
≤0
⇒ f n (a1, a2 ,..., an ) ≤ f n−1 ( a1 , a2 ,..., an−2 , an−1 + an )

Theo giaû thieát quy naïp, ta coù


f n−1 ( a1 , a2 ,..., an− 2 , an−1 + an ) ≤ 0

Do ñoù
f n ( a1 , a2 ,..., an ) ≤ 0

Vaäy khaúng ñònh ñuùng cho n bieán soá. Theo nguyeân lyù quy naïp, khaúng ñònh ñuùng
vôùi moïi n ≥ 4.
Boå ñeà ñöôïc chöùng minh hoaøn toaøn.
Trôû laïi baøi toaùn cuûa ta
Ta coù
a1 a2 an
P= + + ... +
a22 + 1 a32 + 1 a12 + 1
n
 a  n
= ∑  2 i − ai  + ∑ ai
i =1  ai +1 + 1  i =1
n
a a2
= −∑ 2i i +1 + 2
i =1 ai +1 + 1
n
ai ai2+1
≥ −∑ + 2 (theo bñt AM-GM)
i =1 2ai +1
1 n
= − .∑ ai ai +1 + 2
2 i =1
1
≥ − .( a1 + a2 + ... + an ) 2 + 2 (theo Boå ñeà treân)
8
3
=
2

323
Ñaúng thöùc xaûy ra chaúng haïn khi a1 = a2 = 1, a3 = a4 = ... = an = 0.

Vaäy
3
min P =
2
Baøi toaùn 64.
Cho a, b laø caùc soá thöïc thoûa a + b ≠ 0 vaø x, y > 1 laø caùc haèng soá döông cho tröôùc.

Tìm giaù trò nhoû nhaát cuûa bieåu thöùc

( a 2 + 1) x (b 2 + 1) y
f (a, b) =
( a + b) 2

Lôøi giaûi.
AÙp duïng baát ñaúng thöùc AM-GM môû roäng, ta coù
x
1  1  x −1 x+ y−2 
(a + 1) = x  . a 2 +
2 x x
+ . 
x  x + y −1  x ( x + y − 1)( x − 1) 
( x + y − 2) x −1  1 
≥x . x
x −1
. a2 +
x −1  
( x + y − 1) ( x − 1)  x + y −1 

Töông töï, ta coù

( x + y − 2) y −1  1 
(b + 1) ≥ y .
2 y y
. b2 +
y −1 
y −1
( x + y − 1) ( y − 1)  x + y − 1 

Do ñoù

( x + y − 2) x+ y − 2
(a + 1) (b + 1) ≥ x y .
2 x 2 y x y
x
( x + y − 1) x+ y − 2 ( x − 1) x −1 ( y − 1) y −1
 1  2 1 
x  a2 +  b + 
 x + y − 1  x + y −1

AÙp duïng baát ñaúng thöùc Bunhiacopxki, ta coù


2
 2 1  2 1   1 1  1
a +  b +  ≥  a. + b.  = .( a + b) 2
 x + y − 1  x + y −1  x + y −1 x + y −1  x + y −1

Do ñoù

324
x x y y ( x + y − 2) x + y −2
(a 2 + 1) x (b 2 + 1) y ≥ x + y −1 x −1 y −1
.(a + b) 2
( x + y − 1) ( x − 1) ( y − 1)
x x y y ( x + y − 2) x+ y − 2
⇒ f ( a, b) ≥
( x + y − 1) x+ y −1 ( x − 1) x −1 ( y − 1) y −1

 y −1
a =
 ( x + y − 1)( x − 1)
 x −1
Ñaúng thöùc xaûy ra khi vaø chæ khi  b =
 ( x + y − 1)( y − 1)
ab > 0



x x y y ( x + y − 2) x + y − 2
Vaäy min f (a, b) =
( x + y − 1) x + y −1 ( x − 1) x−1 ( y − 1) y −1

* Ghi chuù.
Ñeå coù ñöôïc moät lôøi giaûi ngaén goïn nhö treân, ta phaûi traûi qua moät böôùc choïn ñieåm
rôi nhö sau
Giaû söû M (a0 , b0 ) laø ñieåm cöïc trò cuûa haøm soá f (a, b) thì (a0 , b0 ) laø nghieäm cuûa heä

phöông trình

 f a/ = 0
 /
 f b = 0
( x − 1)a 2 + xab − 1 = 0
⇔
( y − 1)b + yab − 1 = 0
2

( x − 1)a 2 + xab − 1 = 0


⇔
( x − 1)a + ( x − y )ab − ( y − 1)b = 0
2 2

( x − 1)a 2 + xab − 1 = 0
⇔
(a + b)(( x − 1) a − ( y − 1)b) = 0
( x − 1)a 2 + xab − 1 = 0
⇔
( x − 1)a = ( y − 1)b

325
 y −1
a =
 ( x + y − 1)( x − 1)
 x −1
⇔ b =
 ( x + y − 1)( y − 1)
ab > 0



Töø ñaây, ta ñi ñeán moät lôøi giaûi hôi “choaùng” nhö treân.
Baøi toaùn 65. (Vasile Cirtoaje)
2n − 1
Cho n ≥ 3, n ∈ N , 0 < k ≤ vaø a1 , a2 ,..., an > 0 thoûa a1a2 ...an = 1 . Chöùng minh
( n − 1) 2

raèng
1 1 1 n
+ + ... + ≤
1 + ka1 1 + ka2 1 + kan k +1

Lôøi giaûi.
Ta coù boå ñeàø sau

Boå ñeà. a1 , a2 ,..., an laø n soá thöïc döông thoûa maõn

i) a1 ≤ a2 ≤ ... ≤ an
ii) ai ∈ ( −∞, +∞) ∀i = 1, n
iii) a1 + a2 + ... + an = C

vaø f laø moät haøm treân (−∞, +∞) thoûa maõn f loài treân (−∞, c ] vaø loõm treân [c, +∞)

Ñaët F = f ( a1 ) + f (a2 ) + ... + f ( an )

Khi ñoù, F ñaït max khi a1 = a2 = ... = an−1 ≤ an .

Chöùng minh.
Giaû söû ai , ai +1 ,..., an ∈ [c, +∞) , do f loõm treân [c, +∞) neân

f (ai ) + f ( ai +1 ) + ... + f (an ) ≤ (i − 1) f (c) + f (ai + ai +1 + ... + an − (i − 1)c )

Maët khaùc do f loài treân (−∞, c] neân

326
 (i − 1)c + a1 + a2 + ... + ai −1 
(i − 1) f (c) + f (a1 ) + f (a2 ) + ... + f (ai −1 ) ≤ ( n − 1) f  
 n −1 
Do ñoù
n
 (i − 1)c + a1 + a2 + ... + ai −1 
F = ∑ f ( ak ) ≤ (n − 1) f  +
k =1  n −1 
+ f ( ai + ai +1 + ... + an − (i − 1)c )

Boå ñeà ñöôïc chöùng minh.


Trôû laïi baøi toaùn cuûa ta
2n − 1
Ñaët yi = kai (i = 1, n) ⇒ y1 y2 ... yn = k n vôùi k = n y1 y2 ... yn ≤ . Khi ñoù, baát
(n − 1) 2

ñaúng thöùc caàn chöùng minh trôû thaønh


1 1 1 n
+ + ... + ≤
1 + y1 1 + y2 1 + yn k +1

Khoâng maát tính toång quaùt giaû söû 0 < y1 ≤ y2 ≤ ... ≤ yn .

 x ≤ x2 ≤ ... ≤ xn
Ñaët x1 = ln y1, x2 = ln y2 ,..., xn = ln yn thì  1
 x1 + x2 + ... + xn = n ln k (do a1a2 ...an = 1)
1
Xeùt haøm soá f ( x) =
(e + 1)1 2
x

e x .(e x − 2) //
Ta coù f // ( x ) = , f ( x) = 0 ⇔ x = ln 2
4(e x + 1)5 2

Töø ñoù, ta coù f loài treân (−∞,ln 2] vaø loõm treân [ln 2, +∞)

⇒ Theo boå ñeà treân, ta coù


n n
1 1
P=∑ = ∑ ñaït max khi x1 = x2 = ... = xn−1 ≤ xn .
i =1 (1 + yi ) i =1 (1 + e )
12 xi 1 2

 n −1 1 
⇒ max P ≤ max  +  (t ≤ ln k )
n ln k − ( n −1) t
 e t
+ 1 e + 1 

327
 n −1

 n −1 x 2

= max  +  ( x = et ≤ k ) (1)
 x + 1 x n−1 + k n 
 
n −1
n −1 x2
Tieáp theo, ta seõ tìm max cuûa haøm soá g ( x) = + vôùi x ≤ k
x +1 x n−1 + k n

 n −3

n −1  k x 2
n
1 
Ta coù g / ( x) = . 3
− 3 
2  n−1 
 (x + k ) ( x + 1) 2
n 2

n −3 3 3
n −1
g ( x) = 0 ⇔ k x
/ n 2 .( x + 1) = ( x 2 +k ) n 2

2n n −3
⇔k 3 x 3
.( x + 1) = x n−1 + k n (2)
2 2
Ñaët t = x ⇒ t ≤ k . Khi ñoù, phöông trình (2) trô ûthaønh
3 3

2 n n −3
 23  3( n2−1)
k t3 2 .  t + 1 = t + kn
 
3( n −1) 2n n 2 n n −3
⇔t 2 −k t −k t
3 2 3 2 + kn = 0
3( n −1) 2n n 2 n n −3 2
Xeùt haøm soá h(t ) = t 2 −k t −k t 3 2 3 2 + k vôùi t ≤ k
n 3

1
n −5
 2n 3 2n

Ta coù h / (t ) = .t 3 .  3(n − 1)t n − nk 3 t 2 − (n − 3)k 3 
2  
2n 3 2n
h (t ) = 0 ⇔ 3( n − 1)t − nk t − (n − 3)k
/ n 3 2 3
=0
2n 3 2n 2
Xeùt tieáp haøm soá m(t ) = 3( n − 1)t − nk t − (n − 3)k n 3 2 3
vôùi t ≤ k 3

3n 2  
1 2 n −3 2n
Ta coù m / (t ) = t .  2( n − 1)t 2 − k 3 
2  
2
 2n
 2 n−3

m (t ) = 0 ⇔ t0 = 
/ k3 

 2(n − 1) 
 
328
2n − 1
2
Do 0 < k ≤ neân t0 < k 3 . Qua t0 thì m / (t ) ñoåi daáu töø aâm sang döông neân
( n − 1) 2

 2

m(t ) nghòch bieán treân (0, t0 ] vaø ñoàng bieán treân t0 , k 3  .
 

 23  2n
 2n -1 
Ta laïi coù m(0) = 3 − n ≤ 0, m  k  = nk 3 (2 − k ) > 0  do 2 > ≥ k 
   (n -1) 2 
 
2
Neân phöông trình m(t ) = 0 coù nghieäm duy nhaát 0 < t1 < k . 3

2
⇒ Phöông trình h (t ) = 0 coù nghieäm duy nhaát 0 < t1 < k
/ 3

Baûng bieán thieân cuûa h(t )

t 0 t1 k2 3

h / (t ) _ 0 +
kn 0
h(t )

Caên cöù vaøo baûng bieán thieân, ta coù


h(t ) = 0 coù 2 nghieäm döông phaân bieät laø k 2 3 vaø t2 < t1 .

Do ñoù g / ( x) = 0 coù 2 nghieäm döông phaân bieät laø k vaø t23 2 < t13 2 < k .

Baûng bieán thieân cuûa g ( x)

x 0 t23 2 k

g / ( x) _ 0 + 0

g ( x)

Caên cöù vaøo baûng bieán thieân, ta suy ra

329
n
g ( x) ≤ max { g (0), g ( k )} = ∀x ≤ k (3)
k +1
Töø (1) vaø (3), ta suy ra ñpcm.
Baøi toaùn 66.
Cho a, b, c laø ñoä daøi ba caïnh cuûa moät tam giaùc. Chöùng minh raèng

a−b b−c c−a


+ + ≥0
b(b + c − a ) c(c + a − b) a (a + b − c)

Lôøi giaûi.
Do a, b, c laø ñoä daøi ba caïnh cuûa moät tam giaùc neân toàn taïi caùc soâ thöïc döông

x, y , z sao cho a = y + z, b = z + x, c = x + y . Khi ñoù, baát ñaúng thöùc caàn chöùng minh

töông ñöông vôùi


y−x x−z z−y
+ + ≥0
x ( z + x) z ( y + z ) y ( x + y )

Ñeán ñaây, ta coù hai caùch chöùng minh cho baát ñaúng thöùc treân
* Caùch 1.
Coù 2 tröôøng hôïp xaûy ra

+ Tröôøng hôïp 1. x ≥ y ≥ z > 0 . Khi ñoù, ta coù

y−x x−z z−y


+ + ≥0
x ( z + x) z ( y + z ) y ( x + y )
 1 1   1 1 
⇔ ( x − y)  −  + ( y − z)  − ≥0
 z ( y + z ) x ( z + x )   z ( y + z ) y ( x + y ) 
Baát ñaúng thöùc naøy ñuùng do x ≥ y ≥ z > 0.

+ Tröôøng hôïp 2. z ≥ y ≥ x > 0 . Khi ñoù, ta coù

y−x x−z z−y


+ + ≥0
x ( z + x) z ( y + z ) y ( x + y )
 1 1   1 1 
⇔ ( y − x)  −  + ( z − y)  − ≥0
 x( z + x) z ( y + z )   y( x + y ) z ( y + z) 

330
Baát ñaúng thöùc naøy ñuùng do z ≥ y ≥ x > 0 .

Toùm laïi, trong moïi tröôøng hôïp ta luoân coù


y−x x−z z−y
+ + ≥0
x ( z + x) z ( y + z ) y ( x + y )
⇒ ñpcm.

* Caùch 2.
Ta coù
y−x x−z z−y
+ + ≥0
x ( z + x) z ( y + z ) y ( x + y )
x 1 y 1 z 1 x 1 y 1 z 1
⇔ . + . + . ≥ . + . + .
y+ z z z+ x x x+ y y y+z x z+x y x+ y z

 x y z 
Ñieàu naøy ñuùng theo baát ñaúng thöùc saép xeáp laïi do  , ,  vaø
 y+ z z+ x x+ y

1 1 1
 x , y , z  laø hai daõy ngöôïc chieàu nhau.
 
Vaäy ta coù ñpcm.
Ñaúng thöùc xaûy ra khi vaø khi x = y = z ⇔ a = b = c.

Baøi toaùn 67.


Cho k ≥ a, b, c, d > 0. Chöùng minh raèng

a 4 + b 4 + c 4 + d 4 (2k − a ) 4 + (2k − b) 4 + (2k − c) 4 + (2k − d ) 4



abcd (2k − a )(2k − b)(2k − c)(2k − d )

Lôøi giaûi.
Khoâng maát tính toång quaùt, coù theå giaû söû k ≥ a ≥ b ≥ c ≥ d > 0. Khi ñoù, baát ñaúng
thöùc caàn chöùng minh töông ñöông vôùi

( a 2 − b 2 ) 2 (c 2 − d 2 ) 2 2(a 2b 2 + c 2 d 2 ) ((2k − a ) 2 − (2k − b) 2 ) 2


+ + ≥ +
abcd abcd abcd (2k − a )(2k − b)(2k − c )(2k − d )

((2k − c) 2 − (2k − d ) 2 ) 2 2((2k − a ) 2 (2k − b) 2 + (2k − c) 2 (2k − d ) 2 )


+ +
(2k − a )(2k − b)(2k − c)(2k − d ) (2k − a )(2k − b)(2k − c)(2k − d )
331
Do ñoù, ñeå chöùng minh baát ñaúng thöùc ñaõ cho, ta chæ caàn chöùng minh
(a 2 − b 2 ) 2 ((2k − a ) 2 − (2k − b) 2 ) 2
≥ (1)
abcd (2k − a )(2k − b)(2k − c )(2k − d )
(c 2 − d 2 ) 2 ((2k − c) 2 − (2k − d ) 2 ) 2
≥ (2)
abcd (2k − a )(2k − b)(2k − c)(2k − d )
2(a 2b 2 + c 2 d 2 ) 2((2k − a) 2 (2k − b) 2 + (2k − c ) 2 (2k − d ) 2 )
≥ (3)
abcd (2k − a )(2k − b)(2k − c )(2k − d )

Ta coù

(a 2 − b 2 ) 2 ((2k − a ) 2 − (2k − b) 2 ) 2

abcd (2k − a )(2k − b)(2k − c )(2k − d )
⇔ ( a − b) 2 (a + b) 2 (2 k − a )(2k − b)(2k − c)(2k − d ) ≥ ( a − b) 2 (4k − a − b) 2 abcd
⇔ ( a + b)2 (2k − a)(2k − b)(2k − c )(2k − d ) ≥ (4k − a − b) 2 abcd
Do k ≥ c ≥ d > 0 neân (2k − c)(2k − d ) ≥ cd > 0. Do ñoù, ñeå chöùng minh (1), ta chæ

caàn chöùng minh


(a + b) 2 (2k − a )(2k − b) ≥ (4k − a − b) 2 ab

a+b 
2
a +b  a −b   
2 2
a + b   a + b   a − b  
2 2 2

⇔    2k −  −   ≥  2k −     −  
 2    2   2    2    2   2  

 a − b   a+b a+b 
2 2 2

⇔    2k −  −   ≥ 0 (ñuùng do k ≥ a ≥ b > 0)
 2    2   2  
Vaäy (1) ñuùng.
Chöùng minh töông töï, ta coù (2) ñuùng.
Tieáp theo, ta seõ chöùng minh (3) ñuùng.
Ta seõ chöùng minh
ab (2k − c )(2k − d )

cd (2k − a)(2k − b)

Thaät vaäy
ab (2k − c )(2k − d )

cd (2k − a)(2k − b)

332
a(2k − a ) b(2k − b)
⇔ . ≥1
c(2k − c ) d (2k − d )
k 2 − (k − a) 2 k 2 − ( k − b) 2
⇔ 2 . ≥ 1 (ñuùng do k ≥ a ≥ b ≥ c > 0)
k − (k − c) 2 k 2 − (k − d ) 2

Vaäy
ab (2k − c )(2k − d )
≥ ≥1
cd (2k − a)(2k − b)

1
Do haøm soá f ( x) = x + ñoàng bieán treân [1, +∞) neân ta coù
x

 ab   (2k − c)(2k − d ) 
f  ≥ f  
 cd   (2k − a )(2k − b) 
Maët khaùc, ta coù

 ab  a b + c d
2 2 2 2
f  =
 cd  abcd
 (2k − c)(2k − d )  (2k − a ) 2 (2k − b)2 + (2k − c ) 2 (2k − d ) 2
f =
 (2 k − a )(2 k − b )  (2k − a )(2k − b)(2k − c)(2k − d )

Do ñoù

2( a 2b 2 + c 2 d 2 ) 2((2k − a ) 2 (2k − b) 2 + (2k − c) 2 (2k − d ) 2 )



abcd (2k − a )(2k − b)(2k − c)(2k − d )

Vaäy (3) ñuùng.


Töø ñaây, ta suy ra ñieàu phaûi chöùng minh.
Ñaúng thöùc xaûy ra khi vaø chæ khi a = b = c = d .

Baøi toaùn 68.


Cho x, y , z > 0 thoûa x + y + z = 1. Chöùng minh raèng

x 2 + xyz + y 2 + xyz + z 2 + xyz ≥ x 2 + y 2 + z 2 + xy + yz + zx + 2 3 xyz

Lôøi giaûi.
Ta coù baát ñaúng thöùc caàn chöùng minh töông ñöông vôùi

333
( ) ≥x +y
2
x 2 + xyz + y 2 + xyz + z 2 + xyz 2 2
+ z 2 + xy + yz + zx + 2 3 xyz

⇔ 2 xy ( x + yz )( y + zx) + 2 yz ( y + zx)( z + xy ) + 2 zx ( z + xy )( x + yz ) ≥
≥ ( xy + yz + zx − 3 xyz ) + 2 3xyz
Ta coù

2 xy ( x + yz )( y + zx) = 2 xy ( x ( x + y + z ) + yz )( y ( x + y + z ) + zx)
= 2( x + y ) xy ( z + x)( z + y )
= 2( x + y ) x 2 y 2 + xyz

(
≥ ( x + y ) xy + 3 xyz ) (theo bñt Bunhiacopxki)
= xy − xyz + ( x + y ) 3 xyz

Töông töï, ta coù

2 yz ( y + zx)( z + xy ) ≥ yz − xyz + ( y + z ) 3 xyz


2 zx( z + xy )( x + yz ) ≥ zx − xyz + ( z + x) 3 xyz
⇒ 2 xy( x + yz )( y + zx) + 2 yz ( y + zx)( z + xy) + 2 zx ( z + xy )( x + yz ) ≥
≥ ( xy + yz + zx − 3 xyz ) + 2 3xyz
⇒ ñpcm.

1
Ñaúng thöùc xaûy ra khi vaø chæ khi x = y = z = .
3
Baøi toaùn 69. (USAMO 1999)
a1 + a2 + ... + an ≥ n
Cho caùc soá thöïc a1 , a2 ,..., an (n > 3) thoûa  2 . Chöùng minh raèng
a1 + a2 + ... + an ≥ n
2 2 2

max{a1 , a2 ,...a n } ≥ 2.

Lôøi giaûi.
* Caùch 1.

Giaû söû ngöôïc laïi ai < 2 ∀i = 1, n . Khi ñoù, ta coù

a1 + a2 + ... + ai − 2(i − 1) < 2 ∀i = 1, n

Do ai < 2 ∀i = 1, n neân

334
(2 − ai )(2 − a j ) > 0
⇒ 4 − 2( ai + a j ) + ai a j > 0
⇔ 8 − 4(ai + a j ) + 2ai a j > 0
⇔ (22 − 4( ai + a j ) + ( ai + a j ) 2 ) + 22 > ai2 + a 2j
⇔ ( ai + a j − 2) 2 + 22 > ai2 + a 2j

Do ñoù
(a1 + a2 − 2) 2 + 2 2 > a12 + a22
(a1 + a2 + a3 − 2.2)2 + 22 > a32 + ( a1 + a2 − 2) 2
........
(a1 + a2 + ... + an − 2(n − 1))2 + 22 > an2 + ( a1 + a2 + ... + an−1 − 2( n − 2))2

Coäng n − 1 baát ñaúng thöùc treân laïi veá theo veá, ta ñöôïc
(a1 + a2 + ... + an − 2( n − 1)) 2 + 4(n − 1) > a12 + a22 + ... + an2

Do ai < 2 ∀i = 1, n vaø a1 + a2 + ... + an ≥ n neân

2 > a1 + a2 + ... + an − 2( n − 1) ≥ 2 − n

Do n ≥ 4 neân n - 2 ≥ 2. Do ñoù
n − 2 > a1 + a2 + ... + an − 2( n − 1) ≥ 2 − n
⇒ ( n − 2) 2 > (a1 + a2 + ... + an − 2( n − 1)) 2
⇒ ( n − 2) 2 + 4( n − 1) > ( a1 + a2 + ... + an − 2(n − 1)) 2 + 4( n − 1)

Do ñoù
n 2 = ( n − 2) 2 + 4( n − 1) > a12 + a22 + ... + an2

Ñieàu naøy traùi vôùi giaû thieát a12 + a22 + ... + an2 ≥ n 2 .

Vaäy ta phaûi coù max{a1 , a2 ,..., an } ≥ 2.

* Caùch 2.

Giaû söû ngöôïc laïi ai < 2 ∀i = 1, n .


n n
Ñaët bi = 2 − ai (i = 1, n), S = ∑ bi vaø T = ∑ bi2 . Theá thì, töø giaû thieát, ta coù
i =1 i =1

335
(2 − b1 ) + (2 − b2 ) + ... + (2 − bn ) ≥ n

(2 − b1 ) + (2 − b2 ) + ... + (2 − bn ) ≥ n
2 2 2 2

S ≤ n
⇔
T ≥ n − 4n + 4 S
2

Töø ñaây, ta coù


T ≥ n 2 − 4n + 4 S
= ( n − 4) n + 4 S

≥ S ( n − 4) + 4 S (do n ≥ 4)
= nS (1)

Do ai < 2 ∀i = 1, n neân bi > 0 ∀i = 1, n ⇒ bi < n ∀i = 1, n (do S ≤ n) . Do ñoù


n n
T = ∑ bi2 < ∑ nbi = nS (2)
i =1 i =1

Töø (1) vaø (2), ta suy ra maâu thuaãn. Vaäy ta phaûi coù
max{a1 , a2 ,..., an } ≥ 2 (ñpcm)

Baøi toaùn 70. (Toaùn Hoïc Tuoåi Treû 2006)


Cho caùc soá thöïc a, b, c, a1 , b1 , c1 ( aa1 ≠ 0) thoûa
2
 c c1   b b1  bc1 − b1c
 −  +  − . <0
 a a1  a a1 aa 1

Chöùng minh raèng hai phöông trình ax 2 + bx + c = 0 vaø a1 x 2 + b1 x + c1 = 0 ñeàu coù

hai nghieäm phaân bieät vaø caùc nghieäm naøy naèm xen keõ nhau khi bieåu dieãn treân truïc
soá.
Lôøi giaûi.
Khoâng maát tính toång quaùt, ta chæ caàn xeùt a = a1 = 1 laø ñuû.

Khi ñoù, baøi toaùn chuyeån veà

336
“Caùc soá thöïc b, c, b1 , c1 thoûa (c − c1 ) 2 + (b − b1 )(bc1 − b1c ) < 0 . Khi ñoù, caùc phöông

trình f ( x) = x 2 + bx + c = 0 vaø g ( x ) = x 2 + b1 x + c1 = 0 ñeàu coù hai nghieäm phaân

bieät vaø caùc nghieäm naøy naèm xen keõ nhau khi bieåu dieãn treân truïc soá.”
Ñeå chöùng minh hai phöông trình naøy coù hai nghieäm phaân bieät, ta caàn phaûi chöùng

∆ f = b − 4c > 0
2

minh 
∆ g = b1 − 4c1 > 0
2

Tröôùc heát, ta chöùng minh (b 2 − 4c)(b12 − 4c1 ) > 0

b 2b12
Giaû söû ngöôïc laïi (b 2 − 4c)(b12 − 4c1 ) ≤ 0 ⇒ b 2c1 + b12c − 2cc1 ≥ + 2cc1 . Do ñoù
4
(c − c1 ) 2 + (b − b1 )(bc1 − b1c ) = (b 2c1 + b12c − 2cc1 ) + c 2 + c12 − bb1 (c + c1 )
b 2b12
≥ + 2cc1 + c 2 + c12 − bb1 (c + c1 )
4
1
= .(bb1 − 2(c + c1 )) 2
4
≥0

Ñieàu naøy traùi vôùi giaû thieát.


Vaäy ta phaûi coù (b 2 − 4c)(b12 − 4c1 ) > 0 (*)

b 2 − 4c > 0
Tieáp theo, ta seõ chöùng minh  2 . Giaû söû ñieàu naøy khoâng ñuùng. Khi ñoù,
 1
b − 4 c1 > 0

b 2 − 4c < 0
töø (*), ta coù  2 . Do ñoù
 1
b − 4c1 < 0

(c − c1 ) 2 + (b − b1 )(bc1 − b1c ) = (c − c1 ) 2 + b 2c1 + b12c − bb1 (c + c1 )


≥ (c − c1 ) 2 + 2bb1 cc1 − bb1 (c + c1 )

( )
2
= (c − c1 ) 2 − bb1 c − c1

( c )  ( c ) − bb1 
2 2
= c− c+
 
1 1

337
( ) (4 )
2
≥ c − c1 cc1 − bb1

 b2 b2 
( )
2
≥ c − c1 4 . 1 − bb1 
 4 4 
 
=0

Ñieàu naøy traùi vôùi giaû thieát.

b − 4c > 0
2
Vaäy ta phaûi coù  2 .
 1
b − 4 c1 > 0

Töùc laø caùc phöông trình f ( x) = x 2 + bx + c = 0 vaø g ( x ) = x 2 + b1 x + c1 = 0 ñeàu coù

hai nghieäm phaân bieät.


Goïi x1 , x2 laø caùc nghieäm cuûa phöông trình f ( x) = x 2 + bx + c = 0 thì theo ñònh lyù

Viet, ta coù x1 + x2 = −b vaø x1x2 = c. Ñeå chöùng minh f ( x) vaø g ( x ) coù caùc nghieäm

naèm xen keõ nhau khi bieåu dieãn treân truïc soá, ta chæ caàn chöùng minh
g ( x1 ).g ( x2 ) < 0

Ta coù

 x1 + bx1 + c = 0  x1 = −bx1 − c  g ( x ) = (b1 − b) x1 + c1 − c


2 2

 2 ⇒ 2 ⇒ 1
 x2 + bx2 + c = 0  x2 = −bx2 − c  g ( x2 ) = (b1 − b) x2 + c1 − c

Do ñoù
g ( x1 ).g ( x2 ) = ((b1 − b) x1 + c1 − c)((b1 − b) x2 + c1 − c)
= (c1 − c) 2 + (c1 − c)(b1 − b)( x1 + x2 ) + (b1 − b) 2 x1 x2
= (c1 − c) 2 − b(c1 − c)(b1 − b) + c(b1 − b) 2
= (c1 − c) 2 + (b1 − b)(c (b1 − b) − b(c1 − c ))
= (c1 − c) 2 + (b − b1 )(bc1 − b1c ) < 0 (theo gt)
⇒ g ( x1 ).g ( x2 ) < 0

Vaäy f ( x) vaø g ( x ) coù caùc nghieäm naèm xen keõ nhau khi bieåu dieãn treân truïc soá.

Töø caùc chöùng minh treân, ta suy ra ñpcm.

338
Baøi 71.
Cho caùc soá döông a, b, c thoûa 21ab + 2bc + 8ca ≤ 12. Tìm giaù trò nhoû nhaát cuûa bieåu

thöùc
1 2 3
P= + +
a b c
Lôøi giaûi.
+ Caùch 1.
1 2 3
Ñaët x = , y = , z = , baøi toaùn chuyeån veà
a b c
x, y , z > 0 thoûa 2 x + 4 y + 7 z ≤ 2 xyz. Tìm giaù trò nhoû nhaát cuûa bieåu thöùc
P= x+ y+z

Khoâng maát tính toång quaùt, ta chæ caàn xeùt tröôøng hôïp 2 x + 4 y + 7 z = 2 xyz laø ñuû (taïi

7 2 7
sao?). Ñaët x = 7m, y = .n, z = . p thì ta coù m + n + p = mnp . Do ñoù, toàn
2 7
taïi tam giaùc nhoïn ABC sao cho m = tgA, n = tgB, p = tgC. Khi ñoù, ta coù

7
P= .(14tgA + 7 tgB + 4tgC )
14

 π
Xeùt haøm soá f ( x) = tgx vôùi x ∈  0,  .
 2
Ta coù

f / ( x) = tg 2 x + 1
f // ( x) = 2tgx( tg 2 x + 1) > 0

 π
⇒ f ( x) laø haøm loõm treân  0,  .
 2
Do ñoù, theo tính chaát haøm loõm, ta coù
 3 7  3 7  3 7
f ( A) ≥ f  arctg  + f  arctg
/
 A − arctg 
 7   7  7 

339
3 7 16  3 7
= + . A − arctg 
7 7  7 
 3 7
⇒ 14 f ( A) ≥ 6 7 + 32  A − arctg 
 7 

Töông töï, ta coù


 5 7 / 5 7  5 7
f ( B) ≥ f  arctg  + f  arctg  B − arctg 
 7   7  7 
5 7 32  5 7
= + . B − arctg 
7 7  7 
 5 7
⇒ 7 f ( B ) ≥ 5 7 + 32  B − arctg 
 7 

( ) ( )(
f (C ) ≥ f arctg 7 + f / arctg 7 C − arctg 7 )
(
= 7 + 8 C − arctg 7 )
(
⇒ 4 f (C ) ≥ 4 7 + 32 C − arctg 7 )
Do ñoù

7
P= .(14 f ( A) + 7 f ( B) + 4 f (C ))
14
7   3 7 5 7 
≥ .15 7 + 32  A + B + C − arctg − arctg − arctg 7  
14   7 7 

15 3 7 5 7
= (vì A + B + C = arctg + arctg + arctg 7 = π )
2 7 7
 3 7  1
 A = arctg a =
 7 m = 3  3
 
 5 7  5  4
Ñaúng thöùc xaûy ra khi vaø chæ khi  B = arctg ⇔  n = ⇔ b = .
 7  2  5
C = arctg 7  p = 2  3
 c = 2
 

Vaäy

340
15
min P = .
2

+ Caùch 2.
1 4 3
t a= ,b = ,c = , bài toán chuy n v
3x 5y 2z
“ x, y , z > 0 và 3 x + 5 y + 7 z ≤ 15 xyz . Tìm giá tr nh nh t c a bi u th c
1
P = .(6 x + 5 y + 4 z ). ”
2
Áp d ng b t ng AM-GM cho 15 s d ng, ta có

15 xyz ≥ 3 x + 5 y + 7 z ≥ 1515 x 3 y 5 z 7
⇒ 15 x12 y10 z 8 ≥ 1
⇔ 15 x 6 y 5 z 4 ≥ 1

i áp d ng b t ng th c AM-GM cho 15 s d ng, ta có


1 15 15
P = .(6 x + 5 y + 4 z ) ≥ .15 x 6 y 5 z 4 ≥
2 2 2
 1
a = 3

x = y = z  4
ng th c x y ra khi và ch khi  ⇔ x = y = z = 1 ⇔ b = .
15 xyz = 3 x + 5 y + 7 z  5
 3
c = 2

t lu n
15
minP = .
2

341
Baøi toaùn 72.
Cho a, b, c > 0 . Chöùng minh raèng

a abc
∑ b+c
≥2
(a + b)(b + c )(c + a )
+1
cyc

Lôøi giaûi.
Ta coù baát ñaúng thöùc caàn chöùng minh töông ñöông vôùi

∑ a ( a + b)( a + c ) ≥ 2 ( a + b + c )( ab + bc + ca )
cyc

⇔ ∑ a ( a + b)( a + c ) + 2∑ ( a + b) ab(a + c)(b + c) ≥ 4(a + b + c)(ab + bc + ca )


cyc cyc

⇔ a 3 + b3 + c 3 + 2∑ ( a + b) ab(a + c)(b + c ) ≥ 3∑ ab(a + b) + 9abc


cyc cyc

( )
2
AÙp duïng baát ñaúng thöùc Bunhiacopxki, ta coù (a + c)(b + c ) ≥ ab + c

Do ñoù, ñeå chöùng minh baát ñaúng thöùc ñaõ cho, ta chæ caàn chöùng minh

a3 + b3 + c 3 + 2∑ ( a + b) c + ab
cyc
( ) ab ≥ 3∑ ab( a + b) + 9abc
cyc

⇔ a 3 + b3 + c 3 + 2 abc .∑ ( a + b) c ≥ ∑ ab( a + b) + 9abc


cyc cyc

AÙp duïng baát ñaúng thöùc AM-GM, ta coù

2 abc .∑ (a + b) c ≥ 12abc
cyc

Do ñoù, theo baát ñaúng thöùc Schur, ta coù

a3 + b3 + c 3 + 2 abc .∑ ( a + b) c ≥ a3 + b3 + c 3 + 12abc
cyc

= (a 3 + b3 + c3 + 3abc) + 9abc
≥ ∑ ab(a + b) + 9abc
cyc

⇒ a 3 + b3 + c 3 + 2 abc .∑ ( a + b) c ≥ ∑ ab( a + b) + 9abc


cyc cyc

⇒ ñpcm.

Ñaúng thöùc xaûy ra khi vaø chæ khi a = b = c hoaëc a = b, c → 0 vaø caùc hoaùn vò.

342
Baøi toaùn 73. (Phaïm Kim Huøng)
Cho a, b, c ≥ 0 thoûa a + b + c = 1. Chöùng minh raèng

ab + bc + ca ≥ 8( a 2b 2 + b 2c 2 + c 2 a 2 )( a 2 + b 2 + c 2 + 16abc )

Lôøi giaûi.

 q, r ≥ 0

Ñaët q = ab + bc + ca, r = abc ⇒  1 theá thì theo baát ñaúng thöùc Schur, ta coù
 q ≤
3
4q − 1
r≥ . Töø caùch ñaët, ta coùù
9
a 2b 2 + b 2c 2 + c 2 a 2 = q 2 − 2r
a 2 + b 2 + c 2 = 1 − 2q

Do ñoù, baát ñaúng thöùc caàn chöùng minh trôû thaønh

q ≥ 8( q 2 − 2r )(16r + 1 − 2q)
⇔ f ( r ) = 8(2r − q 2 )(16 r + 1 − 2q ) + q ≥ 0

Ta coù f / ( r ) = 6(32r − (4q − 1)(2q + 1))

Coù 2 tröôøng hôïp xaûy ra

* Tröôøng hôïp 1. 1 ≥ 4q ⇒ f / (r ) ≥ 0 ⇒ f (r ) laø haøm ñoàng bieán ∀r ≥ 0.

4q − 1
* Tröôøng hôïp 2. 4q ≥ 1 ⇒ r ≥ ≥ 0 . Do ñoù
9

 32(4q − 1) 
f / ( r ) = 6(32r − (4q − 1)(2q + 1)) ≥ 6  − (4q − 1)(2q + 1) 
 9 
2(4q − 1)(23 − 18q)
=
3
≥0

⇒ f (r ) laø haøm ñoàng bieán ∀r ≥ 0.

Toùm laïi, trong moïi tröôøng hôïp, ta luoân coù f (r ) laø haøm ñoàng bieán ∀r ≥ 0. Do ñoù

f (r ) ≥ f (0) = q (4q − 1)2 ≥ 0


⇒ ñpcm.
343
* Nhaän xeùt.
Coù theå deã daøng chöùng minh ñöôïc 16 laø haèng soá toát nhaát cho baát ñaúng thöùc

ab + bc + ca ≥ 8( a 2b 2 + b 2c 2 + c 2 a 2 )( a 2 + b 2 + c 2 + kabc)

Baøi toaùn 74. (Voõ Quoác Baù Caån)


Cho a, b, c > 0 thoûa abc = 1. Chöùng minh raèng

a b c 3
+ 3 + 3 ≥
c +1 a +1 b +1 2
3

Lôøi giaûi.
Ta coù baát ñaúng thöùc caàn chöùng minh töông ñöông vôùi

2∑ a( a 3 + 1)(b3 + 1) ≥ 3( a 3 + 1)(b3 + 1)(c3 + 1)


cyc

⇔ 2∑ a 4b3 + 2∑ a 4 + 2∑ ab 3 + 2∑ a ≥ 3∑ a 3b3 + 3∑ a 3 + 6
cyc cyc cyc cyc cyc cyc

     
⇔  2∑ a 4b3 + ∑ ab3 − 3∑ a 3b3  +  2∑ a 4 + ∑ ab3 − 3∑ a 3  + 2  ∑ a − 3  ≥ 0
 cyc cyc cyc   cyc cyc cyc   cyc 
   
⇔ 2∑ ab 3 ( a − 1) 2 (2a + 1) +  2∑ a 4 + ∑ ab 3 − 3∑ a 3  + 2  ∑ a − 3  ≥ 0
 cyc   cyc 
cyc  cyc cyc   
Do ñoù, ñeå chöùng minh baát ñaúng thöùc ñaõ cho, ta chæ caàn chöùng minh
2∑ a 4 + ∑ ab3 − 3∑ a 3 ≥ 0 (1)
cyc cyc cyc

∑a ≥ 3 (2)
cyc

* Chöùng minh (1).


Ta coù

2∑ a 4 + ∑ ab3 − 3∑ a 3 = 2∑ a 4 + ∑ ab3 − 3 3 abc .∑ a 3


cyc cyc cyc cyc cyc cyc

≥ 2∑ a 4 + ∑ ab3 − (a + b + c ).∑ a 3 (theo AM-GM)


cyc cyc cyc

= a +b +c −a b−b c −c a
4 4 4 3 3 3

1
= .∑ (3a 4 + b 4 − 3a 3b)
4 cyc
344
1
= .∑ ( a − b) 2 (3a 2 + 2ab + b 2 )
4 cyc
≥0
⇒ (1) ñuùng.

* Chöùng minh (2).


Ta coù
1
∑ a − 3 = ∑ a − 3 3 abc = 2 . ( ) (
a + 3 b + 3 c .∑ )
2
3 3
a−3b ≥0
cyc cyc cyc

⇒ (2) ñuùng.

Töø (1) vaø (2), ta suy ra ñpcm.


Ñaúng thöùc xaûy ra khi vaø chæ khi a = b = c = 1.

Baøi toaùn 75. (Vasile Cirtoaje)


Cho a, b, c > 0 thoûa a + b + c = 3. Chöùng minh raèng

1 1 1
+ + ≥1
2ab + 1 2bc + 1 2ca 2 + 1
2 2

Lôøi giaûi.
Ta coù
1 1 1
+ + =
2ab + 1 2bc + 1 2ca 2 + 1
2 2

 1   1   1 
= − 1 +  − 1 +  − 1 + 3
 2ab + 1   2bc + 1   2ca + 1 
2 2 2

ab 2
= 3 − 2.∑
cyc 2ab + 1
2

ab 2
≥ 3 − 2.∑ (theo bñt AM-GM)
3 2 4
cyc 3 ab
2
= 3 − .∑ 3 ab 2
3 cyc
2 a + 2b
≥ 3 − .∑ (theo bñt AM-GM)
3 cyc 3

345
2
= 3 − .∑ a
3 cyc
2
= 3 − .3
3
=1
1 1 1
⇒ + + ≥ 1 (ñpcm)
2ab + 1 2bc + 1 2ca 2 + 1
2 2

Ñaúng thöùc xaûy ra khi vaø chæ khi a = b = c = 1.

Baøi toaùn 76. (Vasile Cirtoaje)


Cho a, b, c ≥ 0. Chöùng minh raèng

(a 2 − bc) b + c + (b 2 − ca ) c + a + (c 2 − ab) a + b ≥ 0

Lôøi giaûi.
Ta coù baát ñaúng thöùc caàn chöùng minh töông ñöông vôùi

∑ (2a 2 − 2bc) b+c ≥0


cyc

⇔ ∑ (( a + c )(a − b) − ( a + b)(c − a )) b + c ≥ 0
cyc

⇔ ∑ ( a + c)(a − b) b + c − ∑ (a + b)(c − a) b + c ≥ 0
cyc cyc

⇔ ∑ ( a + c)(a − b) b + c − ∑ (b + c )( a − b) a + c ≥ 0
cyc cyc

⇔ ∑ ( a − b) ( a + c )(b + c).
cyc
( a+c − b+c ≥0)
(a − b) 2 ( a + c )(b + c)
⇔∑ ≥ 0 (ñuùng)
cyc a+c + b+c
⇒ ñpcm.

Ñaúng thöùc xaûy ra khi vaø chæ khi a = b = c.

346
Baøi toaùn 77.
Cho a, b, c > 0 vaø k laø haèng soá döông cho tröôùc. Tìm giaù trò nhoû nhaát cuûa bieåu

thöùc
b+c c+ a a +b kabc
f (a, b, c) = + + +
a b c (a + b)(b + c)(c + a )

Lôøi giaûi.

Ñaët x = a 2b + b 2c + c 2 a + a 2c + c 2b + b 2 a, y = 6abc . Khi ñoù, ta coù

6x ky k
f (a, b, c) = + = 6t +
y 6x + 2 y 6t + 2

x
trong ñoù t = ≥1.
y

k
Ñaët g (t ) = 6t + vôùi t ≥ 1 , ta caàn tìm giaù trò nhoû nhaát cuûa g (t ) .
6t + 2
Coù 2 tröôøng hôïp xaûy ra

* Tröôøng hôïp 1. k ≤ 64 . Khi ñoù, ta coù


k 3(t − 1)(48t − k + 16)
g (t ) − 6 − = ≥0
8 8(3t + 1)
k
⇒ g (t ) ≥ 6 +
8
k
⇒ f ( a, b, c) ≥ 6 +
8
Ñaúng thöùc xaûy ra khi vaø chæ khi t = 1 ⇔ a = b = c.
Vaäy trong tröôøng hôïp naøy, ta coù
k
min f ( a, b, c) = 6 +
8
* Tröôøng hôïp 2. k ≥ 64 . Khi ñoù, ta coù

g / (t ) =
(
6 6t + 2 − k )( 6t + 2 + k )
(6t + 2) 2

347
k −2
g / (t ) = 0 ⇔ t = t0 =
6

Qua t0 thì g / (t ) ñoåi daáu töø aâm sang döông neân ta coù

g (t ) ≥ g (t0 ) = 2 k − 2 ∀t ≥ 1
⇒ f ( a, b, c ) ≥ 2 k − 2

Ñaúng thöùc xaûy ra khi vaø chæ khi

k −2
t=
6
⇔ a 2b + b 2 c + c 2 a + a 2c + c 2b + b 2 a = ( )
k − 2 abc (*)

Ta chæ caàn chöùng minh raèng toàn taïi boä soá döông (a, b, c) thoûa maõn heä thöùc (*) laø

baøi toaùn ñöôïc giaûi quyeát hoaøn toaøn.


Thaät vaäy, cho b = c = 1 thì heä thöùc (*) trôû thaønh
 k 
f (a ) = a 2 −  − 2a +1 = 0
 2 

k 64
Ta coù lim+ f ( a ) = 1 > 0, f (1) = 4 − ≤ 4− = 0 . Do ñoù, toàn taïi a ∈ (0,1] sao
a →0 2 2
cho f (a ) = 0 . Vaäy toàn taïi boä soá (a, b, c) thoûa maõn heä thöùc (*).

Do ñoù, trong tröôøng hôïp naøy, ta coù

min f (a, b, c) = 2 k − 2

Keát luaän
k
+ Neáu k ≤ 64 thì min f ( a, b, c) = 6 +
8
+ Neáu k ≥ 64 thì min f ( a, b, c) = 2 k − 2

348
Baøi toaùn 78. (Voõ Quoác Baù Caån)
Cho a, b, c > 0 thoûa abc = 1. Chöùng minh raèng

a b c
+ + ≥1
8c + 1
3
8a + 1
3
8b + 1
3

Lôøi giaûi.
Ta coù nhaän xeùt sau
1 1
≥ ∀x > 0 (*)
8x + 13 2 x2 + 1

Thaät vaäy

(*) ⇔ 2 x 2 + 1 ≥ 8 x 3 + 1

⇔ (2 x 2 + 1) 2 ≥ 8 x3 + 1
⇔ 4 x 2 ( x − 1) 2 ≥ 0 (ñuùng)

Vaäy (*) ñuùng.


Do ñoù
a b c a b c
+ + ≥ + +
8c 3 + 1 8a 3 + 1 8b3 + 1 2c + 1 2a + 1 2b + 1
2 2 2

Nhö vaäy, ñeå chöùng minh baát ñaúng thöùc ñaõ cho, ta chæ caàn chöùng minh
a b c
+ ≥1 +
2c + 1 2a + 1 2b + 1
2 2 2

⇔ ∑ a (2a 2 + 1)(2b 2 + 1) ≥ (2a 2 + 1)(2b 2 + 1)(2c 2 + 1)


cyc

⇔ 4∑ a 3b 2 + 2∑ a 3 + 2∑ ab 2 + ∑ a ≥ 9 + 2∑ a 2 + 4∑ a 2b 2
cyc cyc cyc cyc cyc cyc

   
⇔ 2  ∑ a 3b 2 + ∑ ab 2 − 2∑ a 2b 2  + 2  ∑ a 3 − ∑ a 2  + 2∑ a 3b 2 + ∑ a ≥ 9
 cyc cyc cyc   cyc cyc  cyc cyc

 
⇔ 2∑ ab 2 ( a − 1) 2 + 2  ∑ a 3 − ∑ a 2  + 2∑ a 3b 2 + ∑ a ≥ 9 (**)
 cyc 
cyc  cyc  cyc cyc

Laïi coù theo baát ñaúng thöùc AM-GM thì

349
2∑ a3b 2 + ∑ a ≥ 6 3 a 5b5c5 + 3 3 abc = 9
cyc cyc

Do ñoù, ñeå chöùng minh baát ñaúng thöùc (**), ta chæ caàn chöùng minh

∑ a3 − ∑ a 2 ≥ 0 (***)
cyc cyc

Nhöng ñieàu naøy hieån nhieân ñuùng vì

1  
∑ a3 − ∑ a 2 = 3 .  3∑ a3 − 3 3 abc .∑ a 2 
cyc cyc  cyc cyc 
1   
≥ . 3∑ a 3 −  ∑ a  .∑ a 2  ( theo bñt AM-GM)
3  cyc  cyc  cyc 
  
1
= .∑ (a − b) 2 (a + b)
3 cyc
≥0

Vaäy (***) ñuùng.


Töø ñaây, ta suy ra ñpcm.
Ñaúng thöùc xaûy ra khi vaø chæ khi a = b = c = 1.

* Ghi chuù.
Ngoaøi ra, ta coøn coù moät caùch khaùc ñeå chöùng minh baât ñaúng thöùc
a b c
+ + ≥1
2c + 1 2a + 1 2b + 1
2 2 2

Cuï theå nhö sau


x y
Do a, b, c > 0, abc = 1 neân toàn taïi caùc soá thöïc döông x, y , z sao cho a = ,b = ,
y z

z
c= . Khi ñoù, baát ñaúng thöùc treân trôû thaønh
x

x3 y3 z3
+ + ≥1
y (2 z 2 + x 2 ) z (2 x 2 + y 2 ) x(2 y 2 + z 2 )

AÙp duïng baát ñaúng thöùc Bunhiacopxki, ta coù

350
x3 y3 z3 x4 y4 z4
+ + = + +
y (2 z 2 + x 2 ) z (2 x 2 + y 2 ) x (2 y 2 + z 2 ) xy (2 z 2 + x 2 ) yz (2 x 2 + y 2 ) zx(2 y 2 + z 2 )
( x2 + y 2 + z 2 )2

x 3 y + y 3 z + z 3 x + 2 xyz ( x + y + z )
Do ñoù, ñeå chöùng minh baát ñaúng thöùc ñaõ cho, ta chæ caàn chöùng minh

( x2 + y 2 + z 2 )2
≥1
x 3 y + y 3 z + z 3 x + 2 xyz ( x + y + z )
   
⇔  ∑ x 4 − ∑ x3 y  + 2  ∑ x 2 y 2 − ∑ x 2 yz  ≥ 0
 cyc   cyc 
 cyc   cyc 
1
⇔ .∑ ( x − y ) 2 (3 x 2 + 2 xy + y 2 ) + ∑ z 2 ( x − y ) 2 ≥ 0 (ñuùng)
4 cyc cyc

⇒ ñpcm.

Baøi toaùn 79. (Ñinh Ngoïc An)


Cho a, b, c ≥ 0 thoûa a + b + c = ab + bc + ca . Tìm giaù trò nhoû nhaát vaø giaù trò lôùn nhaát

cuûa bieåu thöùc


1 1 1
S= + +
a+b b+c c+a
Lôøi giaûi.
3
Ta seõ chöùng minh raèng max S = vaø khoâng coù min S .
2
Thaät vaäy, töø giaû thieát, ta coù
ab + bc + ca  1 1 1  1  ab bc ca 
S= . + +  = 1+ . + + 
a +b +c  a +b b+c c +a  a+b+c a+b b+c c+a
Do ñoù S > 1 .
a
Cho c = 0, b = ( a > 1) thì ta cuõng coù a + b + c = ab + bc + ca . Khi ñoù, ta coù
a −1
a −1
S = 1+
a
⇒ lim+ S = 1.
a→1

351
Vaäy khoâng toàn taïi min S .
Maët khaùc, aùp duïng baát ñaúng thöùc AM-GM, ta coù
1  ab bc ca 
S =1+ . + + 
a+b+c a+b b+c c+a
1 1 1 2 
1  .( a + b ) 2
.(b + c ) 2
.( c + a ) 
≤1+ . 4 +4 +4 
a+b+c  a+b b+c c+a 
 
3
=
2
Ñaúng thöùc xaûy ra chaúng haïn khi a = b = c = 1.
3
Vaäy max S = .
2
Baøi toaùn 80. (Ñinh Ngoïc An)
Cho a, b, c, d ≥ 0 thoûa ab + bc + cd + da = 1 . Tìm giaù trò nhoû nhaát cuûa bieåu thöùc

f (a, b, c, d ) = a 2 + b 2 + c 2 + d 2 + 2abcd

Lôøi giaûi.
Ta coù
ab + bc + cd + da = 1 ⇔ ( a + c)(b + d ) = 1

Neáu ac ≥ 1 thì deã thaáy f (a, b, c, d ) ≥ 2 .

Neáu ac ≤ 1 . Khi ñoù, ta coù

 b + d b + d  (b − d ) (1 − ac)
2
f (a, b, c, d ) − f  a, , c, = ≥0
 2 2  2
 b+d b+d 
⇒ f ( a, b, c, d ) ≥ f  a, , c, 
 2 2 

(b + d )2  b+d b+d 
Baây giôø, neáu ≥ 1 thì deã thaáy f  a, , c,  ≥ 2.
4  2 2 

(b + d )2
Do ñoù f (a, b, c, d ) ≥ 2 . Neáu ≤ 1 thì baèng laäp luaän töông töï nhö treân, ta coù
4

352
 b+d b+d  a+c b+d a+c b+d 
f  a, , c, ≥ f  , , , 
 2 2   2 2 2 2 

Do ñoù
 b+d b+d 
f (a, b, c, d ) ≥ f  a, , c, 
 2 2 
a+c b+d a+c b+d 
≥ f
2 
, , ,
 2 2 2
(a + c) 2 (b + d ) 2 ( a + c) 2 (b + d ) 2
= + +
2 2 8
1
≥1+
8
9
=
8
9
Toùm laïi, trong moïi tröôøng hôïp, ta luoân coù f (a, b, c, d ) ≥ .
8
1
Ñaúng thöùc xaûy ra khi vaø chæ khi a = b = c = d = .
2
9
Vaäy min f ( a, b, c, d ) = .
8
Baøi toaùn 81. (Ñinh Ngoïc An)
Cho a, b, c, d ≥ 0 thoûa ab + bc + cd + da + ac + bd = 1 . Tìm giaù trò nhoû nhaát cuûa

bieåûu thöùc

f (a, b, c, d ) = a 2 + b 2 + c 2 + d 2 + 2abcd

Lôøi giaûi.
Xeùt soá thöïc khoâng aâm y thoûa maõn

y 2 + ab + 2 y ( a + b) = 1 ⇔ ( a + b)(c + d − 2 y ) = y 2 − cd (*)

(chuù yù raèng y luoân luoân toàn taïi)

c+d c+d
Khi ñoù, ta phaûi coù y ≤ . Thaät vaäy, giaû söû ngöôïc laïi y > . Khi ñoù, ta coù
2 2

353
2
c+d 
y + ab + 2 y ( a + b) > 
2
 + ab + ( a + b)(c + d )
 2 
≥ cd + ab + (a + b)(c + d )
= ab + bc + cd + da + ac + bd
=1

Ñieàu naøy maâu thuaãn vì y 2 + ab + 2 y ( a + b) = 1 .

c+d
Vaäy, ta phaûi coù y ≤ . Do ñoù, töø (*), ta suy ra ñöôïc y 2 ≥ cd . Khi ñoù, ta coù
2
f (a, b, c, d ) − f ( a, b, y , y ) = (c − d ) 2 + 2(1 − ab)( y 2 − cd ) ≥ 0
⇒ f ( a, b, c, d ) ≥ f ( a, b, y, y )

Baèng laäp luaän töông töï, ta daãn ñeán

f (a, b, c, d ) ≥ f ( x, x, y , y ) = 2( x 2 + y 2 + x 2 y 2 )

Vôùi x, y ≥ 0 thoûa maõn x 2 + y 2 + 4 xy = 1.

Neân
13
f (a, b, c, d ) ≥ f ( x, x, y , y ) = 2( x 2 + y 2 + x 2 y 2 ) ≥
18
1
Ñaúng thöùc xaûy ra khi vaø chæ khi a = b = c = d = .
6
13
Vaäy min f (a, b, c, d ) = .
18
Baøi toaùn 82.
Cho x, y , z > 0 thoûa xy + yz + zx + xyz = 4 . Tìm haèng soá k toát nhaát cho baát ñaúng

thöùc

x 2 + y 2 + z 2 + 3k ≥ (k + 1)( x + y + z )

Lôøi giaûi.

Cho x = y = 2, z = 2 − 1 , ta suy ra ñöôïc k ≤ 2 2 + 1 . Ta seõ chöùng minh ñaây laø

giaù trò caàn tìm, töùc laø chöùng minh

354
(
x2 + y2 + z 2 + 3 2 2 + 1 ≥ 2 ) ( )
2 + 1 (x + y + z) (*)

Töø giaû thieát xy + yz + zx + xyz = 4 , ta suy ra ñöôïc

1 1 1
+ + =1
x+2 y+2 z+2

m + n + p = 1
1 1 1 
Do ñoù, ta coù theå ñaët m = ,n = ,p= ⇒ 1 ⇒ m, n, p laø
x+2 y+2 z + 2 0 < m, n, p <
 2
ñoä daøi ba caïnh cuûa moät tam giaùc. Do ñoù, toàn taïi caùc soá thöïc döông a, b, c sao cho

m = b + c, n = c + a, p = a + b. Khi ñoù, töø caùch ñaët, ta coù

1 − 2m n + p − m 2a
x= = =
m m b+c
1 − 2n p + m − n 2b
y= = =
n n c+a
1− 2 p m + n − p 2c
z= = =
p p a+b

Baát ñaúng thöùc (*) trôû thaønh


4a 2
(
∑ (b + c)2 + 3 2 2 + 1 ≥ 2
cyc
) ( 2 + 1 .∑ ) 2a
cyc b + c

 4 2 +1 a ( ) 
⇔ ∑
 (b
4a 2
+ c ) 2

b + c
+ 2 2 +1 ( ) ≥0

 
cyc

⇔∑
4a 2 − 4 ( ) (
2 + 1 a (b + c ) + 2 2 + 1 (b + c )2 ) ≥0
cyc (b + c) 2

⇔∑
( 2a − ( 2 ) (
2 + 1 b − 2 2 + 1 c ( a − b) )) −
cyc (b + c) 2

−∑
( 2a − ( 2 ) (
2 + 1 b − 2 2 + 1 c (c − a ) )) ≥0
cyc (b + c) 2

355
⇔∑
( 2a − ( 2 ) (
2 + 1 b − 2 2 + 1 c ( a − b) )) −
cyc (b + c) 2

−∑
( 2b − ( 2 ) ( ))
2 + 1 c − 2 2 + 1 a ( a − b)
≥0
cyc (a + c) 2

( ( ) (
⇔ ∑ ( a 2 − b 2 ) 2 2a 2 + 2b 2 + 1 − 2 2 c 2 + 1 − 2 2 ab + 3 − 2 2 c (a + b) ≥ 0
cyc
) ( ) )
Ñaët

( ) ( ) (
S a 2 = 2b 2 + 2c 2 + 1 − 2 2 a 2 + 1 − 2 2 bc + 3 − 2 2 a (b + c))
Sb2 = 2c 2 + 2a 2 + (1 − 2 2 ) b + (1 − 2 2 ) ca + ( 3 − 2 2 ) b(c + a )
2

Sc 2 = 2a 2 + 2b 2 + (1 − 2 2 ) c + (1 − 2 2 ) ab + ( 3 − 2 2 ) c( a + b)
2

Khi ñoù, baát ñaúng thöùc caàn chöùng minh trôû thaønh

S a 2 (b 2 − c 2 ) 2 + Sb2 (c 2 − a 2 ) 2 + Sc 2 (a 2 − b 2 )2 ≥ 0

Khoâng maát tính toång quaùt, ta coù theå giaû söû a ≥ b ≥ c > 0. Khi ñoù, ta coù

( ) ( ) ( )
Sb2 = 2c 2 + 2a 2 + 1 − 2 2 b 2 + 1 − 2 2 ca + 3 − 2 2 b(c + a)

≥ 2c + 2b + (1 − 2 2 ) b + (1 − 2 2 ) cb + ( 3 − 2 2 ) b(c + b)
2 2 2

= 2 ( 2 − 1) b − 4 ( 2 − 1) bc + 2c
2
2 2

(( ) )
2
=2 2 −1 b − c ≥0

(
) ( ) ( )
Sc 2 = 2a 2 + 2b 2 + 1 − 2 2 c 2 + 1 − 2 2 ab + 3 − 2 2 c( a + b)

≥ 4ab + (1 − 2 2 ) c + (1 − 2 2 ) ab + ( 3 − 2 2 ) c (a + b)
2

= ( 5 − 2 2 ) ab + (1 − 2 2 ) c + ( 3 − 2 2 ) c( a + b)
2

≥ ( 5 − 2 2 ) c + (1 − 2 2 ) c + 2 ( 3 − 2 2 ) c
2 2 2

= ( 6 − 4 2 ) c + 2 (3 − 2 2 ) c
2 2

= 4 ( 2 − 1) c
2
2

≥0
356
( ) ( ) ( )
2
S a 2 + Sb2 = 2 − 1 (a 2 + b 2 ) + 2 2 − 1 ab − 4 2 − 1 c( a + b) + 4c 2

( ) ( )
2
= 2 − 1 (a + b) 2 − 4 2 − 1 c (a + b) + 4c 2

(( ) )
2
= 2 − 1 (a + b) − 2c
≥0

Do ñoù
S a 2 (b 2 − c 2 ) 2 + Sb2 (c 2 − a 2 ) 2 + Sc 2 (a 2 − b 2 ) 2 ≥ ( Sa 2 + Sb2 )(b 2 − c 2 ) 2 ≥ 0
⇒ (*) ñuùng.

Vaäy kmax = 2 2 + 1 .

Baøi toaùn 83. (Voõ Quoác Baù Caån)


Cho tam giaùc ABC . Chöùng minh raèng

 A  A  B  B  C  C
1 − sin 1 + cos  1 − sin 1 + cos  1 − sin  1 + cos 
 2  2 
+
2  2 
+
2  2
≥ 2+ 3
A A B  B C  C
sin .1 + sin  sin . 1 + sin  sin . 1 + sin 
2  2 2  2 2  2

Lôøi giaûi.
AÙp duïng baát ñaúng thöùc Bunhiacopxki, ta coù
2
 A  A   A  A
 1 − sin  1 + cos   ∑  1 − sin  1 + cos 
 2  2  cyc  2  2 
∑ A A

A A  A  A
cyc
sin 1 + sin 
2 2
∑ sin 1 − sin 1 + cos  1 + sin 
2 2  2  2
cyc

AÙp duïng baát ñaúng thöùc AM-GM, ta coù


2
 A A
A A   sin 2 + 1 − sin 2  1
sin 1 − sin  ≤   =
2 2  2  4
 

Töông töï, ta coù

357
B B 1
sin 1 − sin  ≤
2 2 4
C C 1
sin 1 − sin  ≤
2 2 4

A A  A  A
⇒ ∑ sin 1 − sin 1 + cos 1 + sin  ≤
cyc 2 2  2  2
1  A  A
≤ .∑  1 + cos  1 + sin 
4 cyc  2  2
1  A A 1 
= . 3 + ∑ sin + ∑ cos + .∑ sin A 
4  cyc 2 cyc 2 2 cyc 
A 3 A 3 3 3 3
Chuù yù raèng ∑
cyc
sin ≤ , ∑ cos ≤
2 2 cyc 2
, ∑ sin A ≤
2 cyc 2

A A  A  A 1  3 3 3 3 3
⇒ ∑ sin 1 − sin 1 + cos 1 + sin  ≤ .  3 + + + 
cyc 2 2  2  2 4  2 2 4 

=
(
9 2+ 3 )
16
1 16
⇒ ≥
A A 
∑ sin 2 1 − sin 2 
A  A (
1 + cos  1 + sin  9 2 + 3 )
cyc  2  2
2
 A  A  16   1 − sin A 1 + cos A  
1 − sin  1 + cos   ∑  
 2  2 
⇒ ∑  ≥ 
2 2 cyc

cyc A
sin 1 + sin 
A (
9 2+ 3 )
2 2

Do ñoù, ñeå chöùng minh baát ñaúng thöùc ñaõ cho, ta chæ caàn chöùng minh
2
  A  A 
16  ∑ 1 − sin 1 + cos  
 cyc  2  2 
≥2+ 3
9 2+ 3( )

358
 A  A 3 3 3
⇔ ∑ 1 − sin  1 + cos  ≥ +
cyc  2  2 2 4
A  A 1  3 3 3
⇔ −∑ sin + ∑  cos − .sin A  ≥ − +
cyc 2 cyc  2 2  2 4

A 3
Do ∑
cyc
sin ≤
2 2
neân

A  A 1  3  A 1 
−∑ sin + ∑  cos − .sin A  ≥ − + ∑  cos − .sin A 
cyc 2 cyc  2 2  2 cyc  2 2 

Do ñoù, ta chæ caàn chöùng minh

 A 1  3 3
∑  cos − .sin A  ≥
cyc  2 2  4

1 π 3  π
Xeùt haøm soá f ( x) = cos x − .sin 2 x + x − − vôùi x ∈  0; 
2 6 4  2
π
Ta coù f / ( x) = − sin x + 1 − cos2 x = sin x.(2sin x − 1), f / ( x ) = 0 ⇔ x = = x0
6
Qua x0 thì f / ( x) ñoåi daáu töø aâm sang döông neân

π   π
f ( x) ≥ f   = 0 ∀x ∈  0, 
6  2
A B C  π
Do , , ∈  0,  neân
2 2 2  2

 A
f  ≥0
2

A 1 A π 3
⇒ cos − .sin A + − − ≥0
2 2 2 6 4

A 1 π A 3
⇒ cos − .sin A ≥  −  +
2 2 6 2 4
Töông töï, ta coù

359
B 1 π B  3
cos − .sin B ≥  −  +
2 2 6 2 4
C 1 π C  3
cos − .sin C ≥  −  +
2 2 6 2 4
Do ñoù

 A 1   π A 3 3 3
∑  cos − .sin A  ≥ ∑   −  +
cyc  2 2  cyc  6 2 
=
4  4

⇒ ñpcm.
π
Ñaúng thöùc xaûy ra khi vaø chæ khi A = B = C = .
3
Baøi toaùn 84.
Cho a, b, c > 0 thoûa a + b + c = 1. Chöùng minh raèng
2 2 2
1  1  1  8(a 2 + b 2 + c 2 )2
 − 2 +  − 2  +  − 2 ≥
a  b  c  (1 − a )(1 − b)(1 − c )
Lôøi giaûi.
1
Ñaët x = a 2 + b2 + c 2 . Khi ñoù, deã thaáy ≤ x < 1. Do ñoù
3
( x − 1)(3 x − 1) ≤ 0
⇒ 4 x − 1 ≥ 3x2
Ta laïi coù

a 2b 2 + b 2c 2 + c 2 a 2 ≥ abc (a + b + c) = abc

Do ñoù

(4 x − 1)( a 2b 2 + b 2 c 2 + c 2 a 2 ) ≥ 3abcx 2

Maët khaùc, ta laïi coù

4 x − 1 = (b + c − a) 2 + (c + a − b)2 + ( a + b − c) 2

AÙp duïng baát ñaúng thöùc Chebyshev, ta coù

3((b + c − a) 2 b 2 c 2 + (c + a − b) 2 c 2 a 2 + ( a + b − c ) 2 a 2b 2 ) ≥

360
≥ ((b + c − a )2 + (c + a − b) 2 + ( a + b − c) 2 )(b 2 c 2 + c 2 a 2 + a 2b 2 )

Do ñoù, ta coù

((b + c − a ) 2 b 2c 2 + (c + a − b) 2 c 2 a 2 + (a + b − c) 2 a 2b 2 ) ≥ abc(a 2 + b 2 + c 2 )
⇔ ((1 − 2a ) 2 b 2c 2 + (1 − 2b) 2 c 2 a 2 + (1 − 2c )2 a 2b 2 ) ≥ abc( a 2 + b 2 + c 2 )

AÙp duïng baát ñaúng thöùc AM-GM, ta coù


(a + b)(b + c)(c + a) ≥ 8abc
⇔ (1 − a )(1 − b)(1 − c ) ≥ 8abc

Do ñoù
((1 − 2a) 2 b 2 c 2 + (1 − 2b) 2 c 2 a 2 + (1 − 2c) 2 a 2b 2 )(1 − a)(1 − b)(1 − c) ≥
≥ 8a 2b 2 c 2 (a 2 + b 2 + c 2 )
2 2 2
1  1  1  8(a 2 + b 2 + c 2 )2
⇔  − 2 +  − 2 +  − 2 ≥
a  b  c  (1 − a )(1 − b)(1 − c )
⇒ ñpcm.

1
Ñaúng thöùc xaûy ra khi vaø chæ khi a = b = c = .
3
Baøi toaùn 85. (Voõ Quoác Baù Caån)
Chöùng minh raèng vôùi moïi soá döông a, b, c ta ñeàu coù

24(a 2 + b 2 + c 2 )
3( a 3 + b3 + c3 ) − + 36 ≥ ab + bc + ca + ∑ ab( a + b)
a+b+c cyc

Lôøi giaûi.
Tröôùc heát xin ñöôïc nhaéc laïi khoâng chöùng minh keát quaû quen thuoäc sau
Xeùt ba daõy (an ), (bn ),(cn ) ñöôïc xaùc ñònh bôûi

a0 = a, b0 = b, c0 = c
a2 n + b2 n
c2 n +1 = c2 n , a2 n+1 = b2 n +1 = , ∀n ∈ N
2
a +c
a2 n+ 2 = b2 n +1 , b2 n+ 2 = c2 n +2 = 2 n +1 2 n +1 , ∀n ∈ N
2
Khi ñoù, ta coù
361
a+b+c
lim an = lim bn = lim cn = =t
n→∞ n→∞ n→∞ 3
Trôû laïi baøi toaùn cuûa ta
Ñaët

24( a 2 + b2 + c 2 )
f (a, b, c) = 3(a + b + c ) −
3 3 3
− ab − bc − ca − ∑ ab( a + b) + 36
a+b+c cyc

Ta seõ chöùng minh

 a+b a+b  
f (a, b, c) ≥ min  f  , , c  , f (0, a + b, c) 
  2 2  

 a+b a+b  
Thaät vaäy, giaû söû ngöôïc laïi f (a, b, c) < min  f  , , c  , f (0, a + b, c)  . Khi
  2 2  
ñoù, ta coù
 a+b a+b 
 f (a, b, c ) < f  , ,c 
  2 2 
 f (a, b, c ) < f (0, a + b, c)

Ta coù
 a+b a+b 
f (a, b, c) < f  , ,c 
 2 2 
 9( a + b) 12 a + b +1 c 
⇔ ( a − b) 2  − + − <0
 4 a+b+c 4 2
48
⇒ 10a + 10b − 2c + 1 − <0 (1)
a+b+c
f (a, b, c) < f (0, a + b, c)
 48 
⇔ ab  −10 a − 10b + 2c − 1 + <0
 a+b+c
48
⇔ 10a + 10b − 2c + 1 − >0 (2)
a+b+c
Töø (1) vaø (2), ta suy ra maâu thuaãn. Vaäy ta phaûi coù

 a+b a+b  
f (a, b, c) ≥ min  f  , , c  , f (0, a + b, c) 
  2 2  
362
Tieáp theo, ta seõ chöùng minh

  a+b a+b  
min  f  , , c  , f (0, a + b, c )  ≥ 0
  2 2  
Tröôùc heát, ta chöùng minh
f (0, a + b, c) ≥ 0 (3)

Baèng laäp luaän töông töï nhö treân, ta coù

  a+b+c a+b+c 
f (0, a + b, c) ≥ min  f  0, ,  , f (0,0, a + b + c) 
  2 2  
Ta laïi coù
3 2
 a+b+c a+b+c  a+b+c  a+b+c  a+b+c
f  0, ,  = 4  −  − 24   + 36 ≥ 0
 2 2   2   2   2 
f (0,0, a + b + c) = 6(( a + b + c)3 − 4(a + b + c) + 6) ≥ 0
Do ñoù
f (0, a + b, c) ≥ 0

Vaäy (3) ñuùng.


Baây giôø, ta seõ chöùng minh

a+b a+b 
f , ,c  ≥ 0 (4)
 2 2 
Töø (3) vaø keát quaû treân, ta suy ra ñöôïc
a+b a+b 
f , , c  = f ( a1 , b1 , c1 )
 2 2 
≥ min{0, f (a2 , b2 , c2 )}
≥ ...
≥ min{0, f (an , bn , cn )} ∀n ∈ N

Do f (a, b, c) lieân tuïc neân

a+b a+b 
f
 2
,
2  n→∞
{ }
, c  ≥ min 0, lim f ( an , bn , cn ) = min{0, f (t , t , t )}

363
a+b+c
Trong ñoù t = .
3
Ta laïi coù

f (t , t , t ) = 3t 3 − 3t 2 − 24t + 36 = 3(t − 2) 2 (t + 3) ≥ 0

Do ñoù

 a+b a +b 
f , ,c ≥ 0
 2 2 
Vaäy (4) ñuùng.
Töø ñaây, ta suy ra ñpcm.
Ñaúng thöùc xaûy ra khi vaø chæ khi a = b = c = 2.

Baøi toaùn 86.


Cho a, b, c > 0. Chöùng minh raèng
2
 a 2 b 2 c2   a c b
2 + +  + 3 ≥  + + 
 bc ca ab   c b a

Lôøi giaûi.
* Caùch 1.
AÙp duïng baát ñaúng thöùc Bunhiacopxki, ta coù
2
 a c b a c b
 + +  ≤ 3 + + 
 c b a c b a

Do ñoù, ñeå chöùng minh baát ñaúng thöùc ñaõ cho, ta chæ caàn chöùng minh
 a 2 b2 c 2  a c b
2  + +  + 3 ≥ 3 + + 
 bc ca ab  c b a
⇔ 2( a 3 + b3 + c3 ) + 3abc ≥ 3(a 2b + b 2c + c 2 a )

AÙp duïng baát ñaúng thöùc AM-GM, ta coù


a3 + ab 2 ≥ 2a 2b
b3 + bc 2 ≥ 2b 2 c
c 2 + ca 2 ≥ 2c 2 a
364
⇒ a 3 + b3 + c 3 + ab 2 + bc 2 + ca 2 ≥ 2( a 2b + b 2 c + c 2 a) (1)

Maët khaùc, theo baát ñaúng thöùc Schur thì

a3 + b3 + c 3 + 3abc ≥ a 2b + b 2c + c 2 a + ab 2 + bc 2 + ca 2 (2)

Töø (1) vaø (2), ta suy ra ñöôïc

2(a 3 + b3 + c 3 ) + ab 2 + bc 2 + ca 2 + 3abc ≥ 3( a 2b + b 2 c + c 2 a ) + ab 2 + bc 2 + ca 2

⇔ 2( a 3 + b3 + c3 ) + 3abc ≥ 3(a 2b + b 2c + c 2 a )
⇒ ñpcm.

Ñaúng thöùc xaûy ra khi vaø chæ khi a = b = c.

* Caùch 2.
AÙp duïng baát ñaúng thöùc AM-GM, ta coù

a2 c2 a
2. + +3≥ 6
bc ab b
b2 a 2 b
2. + +3≥6
ca bc c
c2 b2 c
2. + + 3 ≥ 6
ab ca a

 a 2 b2 c 2   a b c
⇒ 3 + +  + 9 ≥ 6  + + 
 bc ca ab   b c a

 a 2 b2 c 2   a b c
⇔  + +  + 3 ≥ 2 + +  (3)
 bc ca ab   b c a 
Laïi aùp duïng baát ñaúng thöùc AM-GM, ta coù

a 2 b2 a
2. + ≥ 3.
bc ca c
2 2
b c b
2. + ≥ 3.
ca ab a
c2 a 2 c
2. + ≥ 3.
ab bc b

365
 a 2 b2 c2  a c b
⇒ 3 + +  ≥ 3  + + 
 bc ca ab  c b a
a 2 b 2 c2 a c b
⇔ + + ≥ + + (4)
bc ca ab c b a
Töø (3) vaø (4), ta suy ra ñöôïc

 a 2 b 2 c2  a c b  a b c
2 + +  + 3 ≥ + + + 2 + + 
 bc ca ab  c b a  b c a
2
 a 2 b2 c 2   a c b
⇔ 2 + +  + 3 ≥  + +  (ñpcm )
 bc ca ab   c b a 
Ñaúng thöùc xaûy ra khi vaø chæ khi a = b = c.

Baøi toaùn 87. (Phaïm Kim Huøng)


Cho a, b, c laø ñoä daøi ba caïnh cuûa moät tam giaùc. Chöùng minh raèng

 a2 b2 c2  a 2 c2 b2
2 + +  ≥ a + b + c + + +
 b c a c b a

Lôøi giaûi.
Ta coù 2 Boå ñeà sau

Boå ñeà 1. (IMO 1983)


a, b, c laø ñoä daøi ba caïnh cuûa moät tam giaùc. Khi ñoù, ta coù

a 2 b2 c 2 ab bc ca
+ + ≥ + +
b c a c a b
Chöùng minh.
Ta coù a, b, c laø ñoä daøi ba caïnh cuûa moät tam giaùc neân toàn taïi caùc soá döông x, y , z

sao cho a = y + z , b = z + x, c = x + y. Khi ñoù, ta coù

a 2 b2 c 2 ab bc ca
+ + ≥ + +
b c a c a b
⇔ a c + c b + b a ≥ a b + b 2c 2 + c 2 a 2
3 3 3 2 2

⇔ x3 y + y 3 z + z 3 x ≥ x 2 yz + xy 2 z + xyz 2

366
x2 z 2 y2
⇔ + + ≥ x+y+z
z y x
( x − y ) 2 ( y − z ) 2 ( z − x) 2
⇔ + + ≥ 0 (ñuùng)
x y z

Boå ñeà 1 ñöôïc chöùng minh hoaøn toaøn.


Ñaúng thöùc xaûy ra khi vaø chæ khi x = y = z ⇔ a = b = c.

Boå ñeà 2.
a, b, c laø ñoä daøi ba caïnh cuûa moät tam giaùc. Khi ñoù, ta coù

a b c a c b
3 + +  ≥ 2  + +  + 3
b c a c b a
Chöùng minh.
Ta coù
a b c a c b
3 + +  ≥ 2  + +  + 3
b c a c b a
a b c  a c b 
⇔ 3 + + − 3  ≥ 2  + + − 3 
b c a  c b a 
⇔ ∑ (5a − 5b + 3c )( a − b)2 ≥ 0
cyc

Ñaët S a = 5b − 5c + 3a, Sb = 5c − 5a + 3b, Sc = 5a − 5b + 3c .

Baát ñaúng thöùc caàn chöùng minh töông ñöông vôùi

S a (b − c ) 2 + Sb (c − a ) 2 + Sc (a − b) 2 ≥ 0

+ Tröôøng hôïp 1. a ≤ b ≤ c . Khi ñoù, ta coù Sb ≥ 0 vaø

S a + Sb = 8b − 2a > 0 (do b ≥ a )
S c + Sb = 8c − 2b > 0

Do ñoù

S a (b − c )2 + Sa (c − a) 2 + Sc (a − b) 2 ≥ ( S a + Sb )(b − c) 2 + ( Sc + Sb )( a − b)2 ≥ 0
⇒ ñpcm.

367
+ Tröôøng hôïp 2. a ≥ b ≥ c . Khi ñoù ta coù S a , S c ≥ 0 . Do ñoù neáu Sb ≥ 0 thì ta coù

ngay ñpcm, vì vaäy ta chæ caàn xeùt tröôøng hôïp Sb ≤ 0 laø ñuû.

+ Tröôøng hôïp 2.1. a + ( )


3 − 1 c ≤ 3b ⇔ a − c ≤ 3(b − c )

Ta coù
S a + 3Sb = 14b + 10c − 12a ≥ 12(b + c − a ) > 0

Do ñoù

S a (b − c ) 2 + Sa (c − a) 2 + Sc (a − b) 2 ≥ ( S a + 3Sb )(b − c) 2 ≥ 0

⇒ ñpcm.

+ Tröôøng hôïp 2.2. a + ( )


3 − 1 c ≥ 3b ⇔ a − b ≥ ( )
3 − 1 (b − c )

3b
+ Tröôøng hôïp 2.2.1. a ≥
2
Ta coù
S a + 2 Sb = 11b + 5c − 7 a ≥ 8(b + c ) − 7 a > 0
 3b 
Sc + 2Sb = b + 13c − 5a > b + 13( a − b) − 5a = 8  a −  ≥ 0
 2

Do ñoù

S a (b − c)2 + S a (c − a) 2 + Sc (a − b) 2 ≥ ( S a + 2 Sb )(b − c) 2 + ( Sc + 2 Sb )( a − b) 2 ≥ 0
⇒ ñpcm.
3b
+ Tröôøng hôïp 2.2.2. a ≤
2
a
+ Tröôøng hôïp 2.2.2.1. a + c ≥ 2b ⇒ c ≥
3
Ta coù
S a + 2 Sb = 11b + 5c − 7 a ≥ 8(b + c ) − 7 a > 0
2a a
Sc + 2Sb = b + 13c − 5a ≥ + 13. − 5a = 0
3 3
Do ñoù
368
S a (b − c)2 + S a (c − a) 2 + Sc (a − b) 2 ≥ ( S a + 2 Sb )(b − c) 2 + ( Sc + 2 Sb )( a − b) 2 ≥ 0
⇒ ñpcm.
+ Tröôøng hôïp 2.2.2.2. a + c ≤ 2b ⇔ a − c ≤ 2(b − c )

Ta coù

( ) ( )
2 2
S a + 4 Sb + 3 − 1 Sc = (5a − 5b + 3c) 3 − 1 + 17b + 15c − 17a

( 3 −1 c )
Do a + ( )
3 − 1 c ≥ 3b neân b ≤
a
3
+
3

Suy ra

5a − 5b + 3c ≥ 5a −
5a 5

( 3 −1 c ) + 3c
3 3

=
5 ( 3 −1 a ) + (5 − 2 3 ) c
3 3

>
5 ( 3 −1 a )
3
Do ñoù

( )
3
3 −1 a
( )
2 5
S a + 4 Sb + 3 − 1 Sc > + 17b + 15c − 17 a
3

( )
3
5 3 −1 a
≥ + 16 ( b + c ) − 17 a
3

( )
3
5 3 −1 a
> −a >0
3
Do ñoù

S a (b − c ) 2 + Sb (c − a ) 2 + Sc ( a − b) 2 ≥  Sa + 4Sb + ( )
3 − 1 Sc  (b − c ) 2 ≥ 0
2

 
⇒ ñpcm.
Boå ñeà 2 ñöôïc chöùng minh hoaøn toaøn.

369
Ñaúng thöùc xaûy ra khi vaø chæ khi a = b = c .
Trôû laïi baøi toaùn cuûa ta
Theo keát quaû Boå ñeà 2, ta coù

a b c a c b
3 + +  ≥ 2  + +  + 3
b c a c b a

a b c a c b
⇒ 3( a + b + c)  + +  ≥ 2( a + b + c )  + +  + 3( a + b + c )
b c a c b a
 a 2 b2 c 2  ab bc ca  a 2 c 2 b2 
⇔ 3 + +  + + + ≥ 2  + +  + 2( a + b + c )
 b c a c a b  c b a

Maët khaùc, theo Boå ñeà 1 thì

a 2 b2 c 2 ab bc ca
+ + ≥ + +
b c a c a b
Do ñoù
 a2 b2 c2   a 2 b 2 c 2  ab bc ca
4  + +  ≥ 3 + +  + + +
 b c a   b c a  c a b
 a 2 c2 b2 
≥ 2  + +  + 2(a + b + c)
 c b a 
 a 2 b2 c 2  a2 c 2 b2
⇒ 2 + +  ≥ a + b + c + + + (ñpcm )
 b c a c b a

Ñaúng thöùc xaûy ra khi vaø chæ khi a = b = c.

Baøi toaùn 88. (Voõ Quoác Baù Caån)


Cho 2 soá khoâng aâm a, b. Chöùng minh raèng
a( a − b)( a − 1) + b(b − a )(b − 1) + (1 − a)(1 − b) ≥ 0

Lôøi giaûi.
Ta coù baát ñaúng thöùc caàn chöùng minh töông ñöông vôùi

(a − b) 2 ( a + b − 1) + ( a − 1)(b − 1) ≥ 0

Khoâng maát tính toång quaùt, ta coù theå giaû söû a ≥ b ≥ 0.

370
Coù 3 tröôøng hôïp xaûy ra
+ Tröôøng hôïp 1. a ≥ b ≥ 1. Khi ñoù, hieån nhieân ta coù

(a − b) 2 ( a + b − 1) + ( a − 1)(b − 1) ≥ 0

+ Tröôøng hôïp 2. 1 ≥ a ≥ b ≥ 0.

+ Tröôøng hôïp 2.1. a + b ≥ 1. Khi ñoù, hieån nhieân ta coù

(a − b) 2 ( a + b − 1) + ( a − 1)(b − 1) ≥ 0

+ Tröôøng hôïp 2.2. 1 ≥ a + b ≥ 0. Khi ñoù, ta coù

(a − b) 2 ( a + b − 1) + (a − 1)(b − 1) = ( a − b)2 ( a + b − 1) + 1 − a − b + ab
= (1 − a − b)(1 − ( a − b) 2 ) + ab
≥ 0 ( do 1 ≥ a ≥ b ≥ 0, a + b ≤ 1)

+ Tröôøng hôïp 3. a ≥ 1 ≥ b ≥ 0.

Xeùt haøm soá f (a ) = (a − b) 2 ( a + b − 1) + ( a − 1)(b − 1) vôùi a ≥ 1.

Ta coù
f / ( a) = 3a 2 − 2a − 1 − b 2 + 3b − 2ab
f // (a ) = 6a − 2 − 2b > 0

⇒ f / ( a ) laø haøm ñoàng bieán treân [1, +∞) .

⇒ f / ( a) ≥ f / (1) = b(1 − b) ≥ 0 ∀a ≥ 1

⇒ f (a ) laø haøm ñoàng bieán treân [1, +∞) .

⇒ f (a ) ≥ f (1) = b(1 − b)2 ≥ 0 ∀a ≥ 1.

Do ñoù

(a − b) 2 ( a + b − 1) + ( a − 1)(b − 1) ≥ 0

Toùm laïi, trong moïi tröôøng hôïp, ta luoân coù

(a − b) 2 ( a + b − 1) + ( a − 1)(b − 1) ≥ 0 (ñpcm)

Ñaúng thöùc xaûy ra khi vaø chæ khi (a, b) = (1, 0),(1,1).

371
Baøi toaùn 89. (Voõ Quoác Baù Caån)
Cho caùc soá döông a, b, c. Chöùng minh raèng

2(1 + a 3 )(1 + b3 )(1 + c3 ) ≥ (1 + abc)(1 + a 2 )(1 + b 2 )(1 + c 2 )

Lôøi giaûi.
Ta coù baát ñaúng thöùc caàn chöùng minh töông ñöông vôùi

23 (1 + a 3 )3 (1 + b3 )3 (1 + c 3 )3 ≥ (1 + abc)3 (1 + a 2 )3 (1 + b 2 )3 (1 + c 2 )3

AÙp duïng baát ñaúng thöùc AM-GM, ta coù


(1 + a 3 )(1 + b3 )(1 + c 3 ) = 1 + ( a 3 + b3 + c3 ) + ( a 3b3 + b3c 3 + c 3a3 ) + a3b3 c3
≥ 1 + 3abc + 3a 2b 2c 2 + a 3b 3c3
= (1 + abc )3

Do ñoù, ñeå chöùng minh baát ñaúng thöùc ñaõ cho, ta chæ caàn chöùng minh

23 (1 + a 3 )2 (1 + b3 )2 (1 + c 3 ) 2 ≥ (1 + a 2 )3 (1 + b2 )3 (1 + c 2 )3

AÙp duïng baát ñaúng thöùc Chebyshev, ta coù


2(1 + a 3 ) 2 = 2(1 + a 3 )(1 + a 3 )
≥ (1 + a 3 )(1 + a 2 )(1 + a)
= (1 + a 2 )((1 + a 3 )(1 + a ) − (1 + a 2 ) 2 ) + (1 + a 2 )3
= (1 + a 2 )a (a − 1) 2 + (1 + a 2 )3
≥ (1 + a 2 )3
⇒ 2(1 + a 3 ) 2 ≥ (1 + a 2 )3 > 0

Töông töï, ta coù


2(1 + b3 )2 ≥ (1 + b 2 )3 > 0
2(1 + c3 ) 2 ≥ (1 + c 2 )3 > 0

Do ñoù

23 (1 + a 3 )2 (1 + b3 )2 (1 + c 3 ) 2 ≥ (1 + a 2 )3 (1 + b2 )3 (1 + c 2 )3

⇒ ñpcm.
Ñaúng thöùc xaûy ra khi vaø chæ khi a = b = c = 1.

372
Baøi toaùn 90. (Voõ Quoác Baù Caån)
Cho caùc soá khoâng aâm a, b, c thoûa a + b + c = 1 . Tìm haèng soá k > 0 lôùn nhaát sao

cho baát ñaúng thöùc sau ñuùng

a 2 + kb b 2 + kc c 2 + ka 3k + 1
+ + ≥
b+c c+a a+b 2
Lôøi giaûi.
Ta coù baát ñaúng thöùc ñaõ cho töông ñöông vôùi
 2a 2   2a 
 ∑ − 1 + k  ∑ − 3  ≥ 0
 cyc b + c   cyc a + b 
(a − b) 2 ( a + b + c ) k ( a − b)(b − c)(c − a )
⇔∑ − ≥0
cyc (a + c)(b + c ) ( a + b)(b + c)(c + a )
3( a − b) 2 3k ( a − b)(b − c)(c − a )
⇔∑ − ≥0
cyc (a + c)(b + c ) ( a + b)(b + c)(c + a)
k ∑ (a − b)3
3( a − b)2
⇔∑ − cyc
≥0
cyc (a + c)(b + c ) (a + b)(b + c)(c + a)
⇔ ∑ ( a − b) 2 ((3 − k )a + (3 + k )b) ≥ 0 (*)
cyc

Khoâng maát tính toång quaùt, ta coù coù theå giaû söû a = min{a, b, c} .

Ñaët b = a + x, c = a + y ( x, y ≥ 0) . Khi ñoù, ta coù

(*) ⇔ x 2 ((3 − k ) a + (3 + k )(a + x)) + ( x − y )2 ((3 − k )( a + x) + (3 + k )(a + y )) +


+ y 2 ((3 − k )( a + y ) + (3 + k ) a) ≥ 0
⇔ 12( x 2 − xy + y 2 )a + 6 x 3 + 3( k − 1) x 2 y − 3(k + 1) xy 2 + 6 y 3 ≥ 0 ∀a, x, y ≥ 0
⇔ 2 x3 + (k − 1) x 2 y − ( k + 1) xy 2 + 2 y 3 ≥ 0 ∀x, y ≥ 0
⇔ 2t 3 + (k − 1)t 2 − (k + 1)t + 2 ≥ 0 ∀t ≥ 0
⇔ 2t 3 − t 2 − t + 2 ≥ kt (1 − t ) ∀t ≥ 0
⇔ 2t 3 − t 2 − t + 2 ≥ kt (1 − t ) ∀t ∈ (0,1)
2t 3 − t 2 − t + 2
⇔k≤ = f (t ) ∀t ∈ (0,1)
t (1 − t )
Ta coù
373
2(t 4 − 2t 3 + t 2 − 2t + 1)
f / (t ) = −
t 2 (1 − t ) 2

 1+ 2 − 2 2 −1
t1 = (nhaän)
f (t ) = 0 ⇔
/  2

t = 1 + 2 + 2 2 − 1 ( loaïi)
 2 2

Qua t1 thì f / (t ) ñoåi daáu töø aâm sang döông neân

 1+ 2 − 2 2 −1  4 2 − 2 2 2 −1
f (t ) ≥ f   = − 2 −1+ ∀t ∈ (0,1)
 2  − −
  2 2 1 1

Do ñoù

4 2 − 2 2 2 −1
k ≤ − 2 −1+
2 2 −1 −1

Qua caùc laäp luaän treân, ta suy ra ñöôïc

4 2 − 2 2 2 −1
kmax = − 2 − 1 + .
2 2 −1 −1

Baøi toaùn 91. (Traàn Tuaán Anh)


Cho caùc soá khoâng aâm a, b, c thoûa a + b + c = 1. Tìm giaù trò lôùn nhaát vaø giaù trò nhoû

nhaát cuûa bieåu thöùc

P ( a , b , c ) = a (b − c ) 3 + b ( c − a ) 3 + c ( a − b ) 3

Lôøi giaûi.

+ Caùch 1.
Khoâng maát tính toång quaùt, ta coù theå giaû söû b laø soá haïng naèm giöõa a vaø c .
Ñeå tìm giaù trò lôùn nhaát vaø giaù trò nhoû nhaát cuûa P , tröôùc heát ta seõ tìm giaù trò nhoû

nhaát cuûa haøm soá f (b ) = (1 − b)b3 − b(1 − b)3 = −2b3 + 3b 2 − b vôùi 0 ≤ b ≤ 1 .

Ta coù

374
f / (b) = −(6b 2 − 6b + 1)
 3− 3
b1 =
6
f / (b) = 0 ⇔ 
 3+ 3
b2 =
 6

Baûng bieán thieân cuûa f (b)

3− 3 3+ 3
b 0 1
6 6

f / (b ) − 0 + 0 −

f (b)

Caên cöù vaøo baûng bieán thieân, ta suy ra ñöôïc


  3 − 3  
min f (b) = min  f (1), f  
0≤b≤1
  6  

 3− 3  3
Ta laïi coù f (1) = 0, f  =− . Do ñoù
 6  18

3− 3  3
min f (b) = f   =−
0≤b≤1
 6  18

Tieáp theo, ta seõ chöùng minh


P (a + c, b,0) ≤ P( a, b, c ) ≤ P (a + c,0, b)

Coù 2 tröôøng hôïp xaûy ra


* Tröôøng hôïp 1. 1 ≥ a ≥ b ≥ c ≥ 0.

Xeùt haøm soá g ( a ) = a (b − c)3 + b(c − a )3 + c (a − b)3 + b( a + c )3 − b3 (a + c )

Ta coù
g / (a ) = (b − c )3 − 3b(a − c) 2 + 3c( a − b)2 + 3b(a + c ) 2 − b3
= 12abc − b3 + (b − c )3 + 3c (a − b) 2
375
≥ 12b 2c − b3 + (b − c)3
= 9b 2 c + 3bc 2 − c 3
≥0

⇒ g ( a) laø haøm ñoàng bieán.

⇒ g ( a) ≥ g (b) = b(b + c )3 − b3 (b + c ) = bc(b + c)(2b + c ) ≥ 0

⇒ a (b − c)3 + b(c − a )3 + c( a − b)3 ≥ b 3 ( a + c ) − b( a + c )3


⇒ P (a, b, c) ≥ P ( a + c, b,0)

Xeùt tieáp haøm soá h(a ) = b(a + c)3 − b3 ( a + c ) − a (b − c )3 − b(c − a )3 − c( a − b)3

Ta coù

h / ( a) = 3b(a + c) 2 − b3 + 3b( a − c ) 2 − (b − c)3 − 3c( a − b)2 ≥ 0

⇒ h( a ) laø haøm ñoàng bieán.

⇒ h( a ) ≥ h(b) = b(b + c)3 − b 3 (b + c) = bc (b + c )(2b + c) ≥ 0


⇒ 3b( a + c )2 − b3 + 3b(a − c) 2 − (b − c)3 − 3c (a − b) 2 ≥ 0
⇒ b(a + c)3 − b3 ( a + c ) ≥ a (b − c)3 + b(c − a )3 + c (a − b)3
⇒ P (a + c, 0, b) ≥ P ( a, b, c )

Vaäy trong tröôøng hôïp naøy, ta coù


P (a + c, b,0) ≤ P( a, b, c ) ≤ P (a + c,0, b)

* Tröôøng hôïp 2. 1 ≥ c ≥ b ≥ a ≥ 0.

Xeùt haøm soá k (c) = a (b − c )3 + b(c − a)3 + c(a − b)3 + b(a + c)3 − b3 ( a + c)

Ta coù

k / (c) = 3b(c − a ) 2 + 3b(c + a) 2 − 3a (c − b) 2 − b3 − (b − a)3 ≥ 0


⇒ k (c) laø haøm ñoàng bieán.
⇒ k (c) ≥ k (b) = b(b + a )3 − b3 (b + a ) = ab(a + b)(a + 2b) ≥ 0
⇒ a (b − c)3 + b(c − a )3 + c( a − b)3 + b( a + c )3 − b3 (a + c) ≥ 0
⇒ a (b − c)3 + b(c − a )3 + c( a − b)3 ≥ b3 ( a + c ) − b( a + c)3
⇒ P (a, b, c) ≥ P( a + c, b, 0)

Xeùt tieáp haøm soá m(c) = b( a + c )3 − b3 (a + c) − a (b − c)3 − b(c − a )3 − c (a − b)3


376
Ta coù

m / (c) = 3b(c + a ) 2 + 3a(c − b)2 − 3b(c − a ) 2 − b3 + (b − a )3

= 12abc + 3a(c − b) 2 − b3 + (b − a )3

≥ 12ab 2 − b3 + (b − a)3
= 3a 2b + 9ab 2 − a 3
≥0
⇒ m(c) laø haøm ñoàng bieán.
⇒ m(c) ≥ m(b) = b( a + b)3 − b3 (a + b) = ab( a + b)( a + 2b) ≥ 0
⇒ b(a + c)3 − b3 ( a + c) − a (b − c )3 − b(c − a)3 − c (a − b)3 ≥ 0
⇒ b(a + c)3 − b3 ( a + c) ≥ a (b − c )3 + b(c − a )3 + c( a − b) 3
⇒ P (a + c, 0, b) ≥ P (a, b, c)

Vaäy trong tröôøng hôïp naøy, ta cuõng coù


P (a + c, b,0) ≤ P( a, b, c ) ≤ P (a + c,0, b)

Toùm laïi, trong moïi tröôøng hôïp, ta luoân coù


P (a + c, b,0) ≤ P( a, b, c ) ≤ P (a + c,0, b)

Maët khaùc, ta laïi coù

P (a + c, b,0) = b3 (a + c) − (a + c)3 b = b3 (1 − b) − (1 − b)3 b = f (b)

P (a + c,0, b) = ( a + c)3 b − b3 (a + c) = (1 − b)3 b − b3 (1 − b) = − f (b)

Do ñoù, theo keát quaû treân, ta coù

3
P (a + c, b,0) ≥ −
18
3
P (a + c,0, b) ≤
18
Nhö vaäy, ta coù

3
P (a, b, c) ≥ − (1)
18
3
P (a, b, c) ≤ (2)
18

377
3+ 3 3− 3
Ñaúng thöùc ôû (1) xaûy ra chaúng haïn khi a = ,b = , c = 0.
6 6

3+ 3 3− 3
Ñaúng thöùc ôû (2) xaûy ra chaúng haïn khi a = , b = 0, c = .
6 6
Vaäy

3
max P (a, b, c ) =
18
3
min P( a, b, c) = −
18
+ Caùch 2.
Khoâng maát tính toång quaùt, ta coù theå giaû söû b laø soá haïng naèm giöõa a vaø c .
Ta coù

P (a, b, c ) = a (b − c)3 + b(c − a )3 + c( a − b)3


= ab 3 + bc 3 + ca 3 − a 3b − b3c − c 3a
= ( a − b)(b − c)(c − a )(a + b + c)
= ( a − b)(b − c)(c − a )

Coù 2 tröôøng hôïp xaûy ra

* Tröôøng hôïp 1. 1 ≥ a ≥ b ≥ c ≥ 0 . Khi ñoù, ta coù P ( a, b, c ) ≤ 0 .

AÙp duïng baát ñaúng thöùc AM-GM, ta coù


P (a, b, c) = −( a − b)(b − c)( a − c)
a −b b−c a −c
= −4. . .
2 3 −1 3 + 1
3
 a −b b−c a −c 
 2 + 3 −1 + 3 +1 
≥ −4  
 3 
 
 
3
 ( a + b) 3 
 −c 3 
= −4  2 
 3 
 
 
378
3
= .(3c − 1)3
18
3
≥−
18
 3+ 3
a =
a + b + c = 1  6

a − b b − c a−c  3− 3
Ñaúng thöùc xaûy ra khi vaø chæ khi  = = ≥ 0 ⇔ b =
 2 3 −1 3 +1  6
c = 0 c = 0



* Tröôøng hôïp 2. 1 ≥ c ≥ b ≥ a ≥ 0. Khi ñoù, deã thaáy P (a, b, c) ≥ 0.

AÙp duïng baát ñaúng thöùc AM-GM, ta coù


P (a, b, c) = (c − b)(b − a )(c − a )
c−b b−a c−a
= 4. . .
2 3 −1 3 +1
 c−b b−a c−a 
3

 2 + 3 −1 + 3 + 1 
≤ 4 
 3 
 
 
3
 (c + b) 3 
 −a 3
= 4 2 
 3 
 
 
3
= .(1 − 3a )3
18
3

18

379
 3+ 3
c =
a + b + c = 1  6
 
c − b b − a c−a 3− 3
Ñaúng thöùc xaûy ra khi vaø chæ khi  = = ≥ 0 ⇔ b =
 2 3 −1 3 +1  6
a = 0 a = 0



Töø caùc chöùng minh treân, ta suy ra ñöôïc

3
max P (a, b, c ) =
18
3
min P( a, b, c) = −
18
Baøi toaùn 92. (Phaïm Vaên Thuaän)
Cho caùc soá khoâng aâm a, b, c thoûa a + b + c = 1. Tuøy theo giaù trò cuûa n ∈ N , tìm giaù

trò lôùn nhaát vaø giaù trò nhoû nhaát cuûa bieåu thöùc

P ( a , b, c ) = a ( b − c ) n + b ( c − a ) n + c ( a − b ) n

Lôøi giaûi.
+ n = 0 ⇒ P ( a , b, c ) = 1
+ n = 1 ⇒ P (a, b, c) = 0

+ Xeùt n ≥ 2

a) n leû ⇒ n ≥ 3 .
Ta seõ chöùng minh
P (a + c, b,0) ≤ P( a, b, c ) ≤ P (a + c,0, b)

Coù 2 tröôøng hôïp xaûy ra


* Tröôøng hôïp 1. 1 ≥ a ≥ b ≥ c ≥ 0.

Xeùt haøm soá g ( a ) = ( a + c ) n b − (a + c)b n − a (b − c) n − b(c − a )n − c (a − b) n

Ta coù
g / (a ) = nb(a + c) n −1 − b n − (b − c )n + nb( a − c ) n−1 − nc (a − b) n −1
= nb(a + c) n−1 − b n − (b − c) n + n(b(a − c) n −1 − c( a − b) n−1 )
380
≥ nb(a + c) n−1 − b n − (b − c) n
≥0
⇒ g ( a) laø haøm ñoàng bieán.
⇒ g ( a) ≥ g (b) = (b + c) n b − (b + c)b n ≥ 0
⇒ ( a + c ) n b − (a + c)b n ≥ a (b − c) n + b(c − a )n + c (a − b) n
⇒ P (a + c, 0, b) ≥ P (a , b, c ) (1)

Xeùt tieáp haøm soá h(a ) = (a + c)n b − ( a + c)b n + a (b − c) n + b(c − a )n + c (a − b) n

Ta coù
h / ( a) = nb( a + c) n −1 − b n + (b − c) n − nb( a − c ) n−1 + nc (a − b) n−1
= nb((a + c) n −1 − ( a − c )n −1 ) − b n + (b − c) n + nc( a − b)n −1
≥ nb((b + c) n−1 − (b − c )n −1 ) − b n + (b − c) n
n −3 n−1 n−1
2 2 2
= 2nb ∑ Cn2−l +11b n −2l −2c 2l +1 − ∑ Cn2l +1b n− 2l −1c 2l +1 + ∑ Cn2l b n−2l c 2l
l =0 l =0 l =1
n −3 n −1
2 2
= ∑ b n−2l −1c 2l +1 (2nCn2−l +11 − Cn2l +1 ) + ∑ Cn2l b n −2l c 2l − c n
l =0 l =1

≥ 0 (do 2nCn2−l 1+1 − Cn2l +1 ≥ 0 ∀l < n)

⇒ h( a ) laø haøm ñoàng bieán.


⇒ h( a ) ≥ h(b) = (b + c )n b − (b + c)b n ≥ 0
⇒ a (b − c) n + b(c − a) n + c( a − b)n ≥ (a + c)b n − ( a + c) n b
⇒ P (a, b, c) ≥ P ( a + c, b,0) (2)

Töø (1) vaø (2), ta suy ra trong tröôøng hôïp naøy, ta coù
P (a + c, b,0) ≤ P( a, b, c ) ≤ P (a + c,0, b)

* Tröôøng hôïp 2. 1 ≥ c ≥ b ≥ a ≥ 0.

Xeùt haøm soá k (c) = ( a + c )n b − ( a + c )b n − a(b − c) n − b(c − a )n − c (a − b) n

Ta coù
k / (c ) = nb(c + a ) n−1 − b n + na (c − b) n−1 − nb(c − a ) n−1 + (b − a) n
= nb((c + a) n −1 − (c − a )n −1 ) − b n + (b − a )n + na (c − b) n−1

381
≥ nb((b + a) n −1 − (b − a) n −1 ) − b n + (b − a) n
n −3 n −1 n −1
2 2 2
= 2nb ∑ Cn2−l +11b n−2l −2 a 2 l +1 − ∑ Cn2 l +1b n −2l −1a 2l +1 + ∑ Cn2l b n −2l a 2l
l =0 l =0 l =1
n −3 n−1
2 2
= ∑ b n−2 l −1a 2l +1 (2nCn2−l 1+1 − Cn2l +1 ) + ∑ Cn2l b n− 2l a 2l − a n
l =0 l =1

≥ 0 (do 2nCn2−l +11 − Cn2l +1 ≥ 0 ∀l < n)


⇒ k (c) laø haøm ñoàng bieán.
⇒ k (c) ≥ k (b) = (b + a) n b − (b + a )b n ≥ 0
⇒ ( a + c) n b − (a + c)b n ≥ a(b − c) n + b(c − a )n + c(a − b) n
⇒ P (a + c, 0, b) ≥ P ( a, b, c ) (3)

Xeùt tieáp haøm soá m(c) = ( a + c)n b − ( a + c)b n + a(b − c )n + b(c − a )n + c( a − b) n

Ta coù
m / (c) = nb(c + a ) n−1 − b n − na (c − b)n −1 + nb(c − a )n −1 − (b − a) n
= (nb(c + a )n −1 − b n − (b − a ) n ) + n(b(c − a ) n −1 − a (c − b)n −1 )
≥0
⇒ m(c) laø haøm ñoàng bieán.
⇒ m(c) ≥ m(b) = (b + a ) n b − (b + a)b n ≥ 0
⇒ a (b − c) n + b(c − a) n + c( a − b)n ≥ −( a + c) n b + ( a + c )b n
⇒ P (a, b, c) ≥ P ( a + c, b,0) (4)

Töø (3) vaø (4), ta suy ra trong tröôøng hôïp naøy, ta cuõng coù
P (a + c, b,0) ≤ P( a, b, c ) ≤ P (a + c,0, b)

Toùm laïi, trong moïi tröôøng hôïp, ta luoân coù


P (a + c, b,0) ≤ P( a, b, c ) ≤ P (a + c,0, b)

tn − t
Xeùt haøm soá f (t ) = vôùi t > 0.
(t + 1) n+1

Ta coù

−t n + nt n−1 + nt − 1
f / (t ) =
(t + 1) n +2

382
f / (t ) = 0 ⇔ −t n + nt n −1 + nt − 1 = 0

1
Deã thaáy neáu t0 > 0 laø moät nghieäm cuûa phöông trình f / (t ) = 0 thì cuõng laø
t0

nghieäm cuûa phöông trình f / (t ) = 0 . Do ñoù, ta chæ caàn tìm nghieäm cuûa phöông

trình f / (t ) = 0 treân (0,1] laø ñuû.

Xeùt haøm soá ϕ (t ) = −t n + nt n−1 + nt − 1 vôùi t ∈ (0,1].

Ta coù
ϕ / (t ) = n(1 − t n −1 ) + n(n − 1)t n −2 > 0 ∀t ∈ (0,1]
⇒ ϕ (t ) laø haøm ñoàng bieán treân (0,1].

Ta laïi coù lim+ ϕ (t ) = −1 < 0,ϕ (1) = 2( n − 1) > 0 neân toàn taïi duy nhaát t1 ∈ (0,1] sao
t →0

cho ϕ (t1 ) = 0.

1
Do ñoù, phöông trình f / (t ) = 0 chæ coù 2 nghieäm phaân bieät laø t1 vaø .
t1

1
Qua t1 thì f / (t ) ñoåi daáu töø aâm sang döông, qua thì f / (t ) ñoåi daáu töø döông
t1

sang aâm neân

  1  
f (t ) ≤ max  lim+ f (t ), f    ∀t > 0
t →0  t1  

Ta laïi coù neáu b > 0 thì


n
a+c  a+c
( a + c ) b − ( a + c )b
n n   − 
P (a + c,0, b) = (a + c) b − ( a + c)b =
n n
=  b   b 
(a + b + c) n+1 a+c 
n+1

 + 1
 b 
a+c
= f 
 b 
P (a + c, b,0) = −( a + c ) n b + (a + c)b n

383
(a + c) n b − ( a + c)b n
=−
(a + b + c ) n+1
n
 a+c a+c
  − 
=−  b   b 
n +1
 a+c 
 + 1
 b 
a+c
=−f  
 b 
Neân theo treân, ta coù

  1     1  
P (a + c,0, b) ≤ max  lim+ f (t ), f    = max 0, f   
t →0  t1     t1  
  1     1  
P (a + c,0, b) ≥ − max  lim+ f (t ), f    = − max 0, f   
t →0  t1     t1  

Ngoaøi ra, neáu b = 0 thì ta coù P ( a + c, b,0) = P ( a + c,0, b) = 0 neân ta luoân coù

  1     1  
P (a + c,0, b) ≤ max  lim+ f (t ), f    = max 0, f  
t →0  t1     t1  
  1     1  
P (a + c,0, b) ≥ − max  lim+ f (t ), f    = − max 0, f  
t →0  t1     t1  

Do ñoù, ta coù

  1     1  
− max 0, f    ≤ P (a, b, c) ≤ max 0, f   
  t1     t1  

Deã thaáy raèng ñaúng thöùc luoân xaûy ra neân ta coù

  1  
min P ( a, b, c ) = − max 0, f  
  t1  
  1  
max P (a, b, c) = max 0, f  
  t1  

384
b) n chaün ⇒ n ≥ 2.
Khi ñoù, deã thaáy min P ( a, b, c) = 0 vaø P (a, b, c) laø moät bieåu thöùc ñoái xöùng vôùi a, b

vaø c neân khoâng maát tính toång quaùt, ta coù theå giaû söû 1 ≥ a ≥ b ≥ c ≥ 0.
Ta seõ chöùng minh
P (a, b, c) ≤ P( a + c, 0, b)

Xeùt haøm soá u (a ) = (a + c)n b − ( a + c)b n − a (b − c) n − b(c − a )n − c( a − b) n

Ta coù
u / ( a) = nb( a + c )n −1 − b n − (b − c )n + nb( a − c ) n−1 − nc(a − b) n −1
= nb(a + c) n−1 − b n − (b − c) n + n(b(a − c) n−1 − c( a − b)n −1 )
≥ nb(a + c) n−1 − b n − (b − c) n
≥0
⇒ u ( a ) laø haøm ñoàng bieán.
⇒ u ( a ) ≥ u (b) = (b + c ) n b − (b + c )b n ≥ 0
⇒ ( a + c ) n b − (a + c)b n ≥ a(b − c) n + b(c − a ) n + c (a − b) n
⇒ P (a + c, 0, b) ≥ P ( a, b, c )

Theo treân, ta laïi coù

  1     1  
P (a + c,0, b) ≤ max  lim+ f (t ), f    = max 0, f  
t →0  t1     t1  

Do ñoù
  1  
P (a, b, c) ≤ max 0, f  
  t1  

Ngoaøi ra, ta cuõng deã thaáy ñaúng thöùc luoân xaûy ra neân

  1  
max P (a, b, c) = max 0, f  
  t1  

Keát luaän
+ n = 0 ⇒ P ( a , b, c ) = 1
+ n = 1 ⇒ P( a, b, c) = 0

385
  1  
+ n ≥ 2 ⇒ max P( a, b, c) = max 0, f   0
  t1  

  1  
* n leû ⇒ min P ( a, b, c ) = − max 0, f  
  t1  
* n chaün ⇒ min P (a , b, c ) =

tn − t
trong ñoù f (t ) = vaø t1 laø nghieäm döông duy nhaát thuoäc (0,1] cuûa phöông
(t + 1) n+1

trình −t n + nt n−1 + nt − 1 = 0.

Baøi toaùn 93. (Vietnam TST 1996)


Cho caùc soá thöïc a, b, c baát kyø. Chöùng minh raèng

4
F ( a, b, c ) = ( a + b)4 + (b + c )4 + (c + a ) 4 − .(a 4 + b 4 + c 4 ) ≥ 0
7
Lôøi giaûi.
Ta coù
 b+c b+c
F ( a , b, c ) − F  a , , =
 2 2 
4
= ( a + b)4 + (b + c) 4 + (c + a) 4 − .(a 4 + b 4 + c 4 ) −
7
4  4 b+c 
4 4
 b+c
− 2 a +  − (b + c) + . a + 2 
4
 
 2  7   2  

b + c  4  (b + c ) 
4 4

= ( a + b ) + (c + a ) − 2  a +
4 4
 +  − b 4
− c 4

 2  7 8 
 
3  4 4 (b + c) 4 
= a(4b + 4c − (b + c) ) + 3a (2b + 2c − (b + c) ) + .  b + c −
3 3 3 2 2 2 2

7  8 
3
= 3a (b + c)(b − c )2 + 3a 2 (b − c) 2 + .(b − c) 2 (7b 2 + 7c 2 + 10bc)
56
3
= 3a (a + b + c)(b − c) 2 + .(b − c) 2 (7b 2 + 7c 2 + 10bc)
56

386
Soá haïng cuoái cuøng luoân luoân khoâng aâm. Neáu a, b, c cuøng daáy thì baát ñaúng thöùc

caàn chöùng minh laø hieån nhieân. Neáu a, b, c khoâng cuøng daáu thì phaûi coù ít nhaát moät

trong ba soá a, b, c cuøng daáu vôùi a + b + c . Khoâng maát tính toång quaùt, giaû söû ñoù laø

 b+c b+c
a . Töø ñaúng thöùc treân ta suy ra F ( a, b, c ) ≥ F  a, ,  . Nhö vaäy, ta chæ caàn
 2 2 

chöùng minh
F ( a , b , b ) ≥ 0 ∀a , b ∈ R
4
⇔ 2( a + b)4 + (2b) 4 − .(a 4 + 2b 4 ) ≥ 0
7
Neáu b = 0 thì baát ñaúng thöùc laø hieån nhieân. Neáu b ≠ 0 , chia hai veá cua baát ñaúng
a
thöùc cho b 4 roài ñaët x = thì ta ñöôïc baát ñaúng thöùc töông ñöông
b
4
2( x + 1)4 + 16 − .( x 4 + 2) ≥ 0
7
Baát ñaúng thöùc cuoái cuøng coù theå chöùng minh nhö sau
4
Xeùt haøm soá f ( x) = 2( x + 1)4 + 16 − .( x 4 + 2)
7
Ta coù
16 3
f / ( x ) = 8( x + 1)3 − .x
7

2
f / ( x) = 0 ⇔ x + 1 = 3 .x ⇔ x = −2.9294.
7
f min = f ( −2.9294) = 0.4924 > 0

(Caùc phaàn tính toaùn cuoái ñöôïc tính vôùi ñoä chính xaùc tôùi 4 chöõ soá sau daáu phaåy. Do
f min tính ñöôïc laø 0.4924 neân neáu tính caû sai soá tuyeät ñoái thì giaûtò chính xaùc cuûa

f min vaãn laø moät soá döông. Vì ñaây laø moät baát ñaúng thöùc raát chaët neân khoâng theå

387
4 16
traùnh ñöôïc caùc tính tính toaùn vôùi soá leû treân ñaây. Chaúng haïn neáu thay baèng
7 27
4
ñeå xmin = −3 thì f min
*
coù giaù trò aâm! ÔÛ ñaây f * ( x ) = 2( x + 1)4 + 16 − .( x 4 + 2) .)
7

* Chuù yù.
Ta coù theå ñöa baøi toaùn veà chöùng minh F ( a, b, b) ≥ 0 ∀a, b ∈ R baèng caùch söû duïng

Boå ñeà sau


Boå ñeà. ∀a, b, c ∈ R thì toàn taïi caùc soá thöïc x0 , y0 , x1, y1 sao cho

p = a + b + c = 2 x0 + y0 = 2 x1 + y1
q = ab + bc + ca = x02 + 2 x0 y0 = x12 + 2 x1 y1
x02 y0 ≤ r = abc ≤ x12 y1

Ngoaøi ra, neáu a, b, c ≥ 0 thì x0 , x1, y1 ≥ 0 . Trong ñoù

+ Neáu p 2 ≥ 4q thì y0 ≤ 0
+ Neáu p 2 ≤ 4q thì y0 ≥ 0

Baøi toaùn 94. (Phaïm Kim Huøng)


Cho caùc soá khoâng aâm a, b, c thoûa a + b + c = 3.Tìm giaù trò lôùn nhaát cuûa bieåu thöùc

f (a, b, c) = a k (b + c ) + b k (c + a ) + c k ( a + b)

trong ñoù k > 0 laø haèng soá cho tröôùc.


Lôøi giaûi.
Khoâng maát tính toång quaùt, ta coù theå giaû söû a ≥ b ≥ c ≥ 0.
Coù 3 tröôøng hôïp xaûy ra
* Tröôøng hôïp 1. 1 > k > 0. Khi ñoù, aùp duïng baát ñaúng thöùc Holder, ta coù

f (a, b, c) = a k (b + c) + b k (c + a ) + c k ( a + b)
= a k (3 − a) + b k (3 − b) + c k (3 − c )
= 3(a k + b k + c k ) − (a k +1 + b k +1 + c k +1 )
( a + b + c) k ( a + b + c) k +1
≤ 3. − =6
3k −1 3k
388
Ñaúng thöùc xaûy ra khi vaø chæ khi a = b = c = 1.

* Tröôøng hôïp 2. k > 2 . Khi ñoù, ta seõ chöùng minh


f (a, b, c) ≤ f (a, b + c,0)
⇔ a k (b + c ) + b k (c + a ) + c k ( a + b) ≤ a k (b + c ) + (b + c) k a
⇔ b k (c + a ) + c k ( a + b) ≤ (b + c) k a
⇔ ((b + c) k − b k − c k ) a ≥ b k c + bc k

Do k > 2 neân

(b + c )k − b k − c k = (b + c) k −1 (b + c) − b k − c k
≥ (b k −1 + c k −1 )(b + c) − b k − c k
= b k −1c + bc k −1
⇒ ((b + c) k − b k − c k ) a ≥ (b k −1c + bc k −1 )a ≥ b k c + bc k (do a ≥ b ≥ c ≥ 0)
⇒ f ( a, b, c) ≤ f (a, b + c, 0)

Tieáp theo, ta seõ tìm giaù trò lôùn nhaát cuûa bieåu thöùc

f (a, b, 0) = a k b + b k a

trong ñoù a, b ≥ 0 thoûa a + b = 3.

Khoâng maát tính toång quaùt, ta coù theå giaû söû a ≥ b ≥ 0 ⇒ a > 0. Khi ñoù, ta coù

a k b + bk a
k +1
f (a, b, 0) = a b + b a = 3 .
k k
k +1
= 3k +1 g (t )
( a + b)

tk + t b
trong ñoù g (t ) = k +1
vaø t = ≤ 1.
(t + 1) a

Ta coù
−t k + kt k −1 − kt + 1
g (t ) =
/
(t + 1) k +2
g / (t ) = 0 ⇔ −t k + kt k −1 − kt + 1 = 0 (*)

Deã thaáy 1 laø moät nghieäm cuûa phöông trình (*). Goïi α 0 , α1 ,...,α m laø taát caû caùc

nghieäm thuoäc [0,1) neáu coù phöông trình (*). Khi ñoù, deã thaáy
g (t ) ≤ max{g (1), g (α i )} ∀t ∈ [0,1]

389
⇒ f ( a, b,0) ≤ 3k +1 max{g (1), g (α i )}

Ngoaøi ra, deã thaáy ñaúng thöùc luoân xaûy ra.


* Tröôøng hôïp 3. 2 ≥ k ≥ 1.
Ñaët a + b = 2t , a − b = 2u ⇒ t ≥ u ≥ 0, t ≥ c ≥ 0 . Khi ñoù, ta coù

f (a, b, c) = (t + u ) k (t − u + c ) + (t − u ) k (t + u + c ) + 2c k t = h(u )

Ta coù
h / (u ) = k (t + u ) k −1 (t − u + c ) − (t + u ) k − k (t − u ) k −1 (t + u + c) + (t − u ) k
= (t + u ) k −1 (( k − 1)t − (k + 1)u + kc ) − (t − u ) k −1 (( k − 1)t + (k + 1)u + kc )

t = u
Neáu  thì ta coù h / (u ) ≤ 0 .
 (k − 1)t − (k + 1)u + kc ≤ 0

t > u
Neáu  thì do 2 ≥ k ≥ 1 neân (t + u )k −1 ≤ (t + u )(t − u )k −2 . Do
(k − 1)t − (k + 1)u + kc > 0

ñoù, ta coù
h / (u ) ≤ (t + u )(t − u ) k −2 ((k − 1)t − ( k + 1)u + kc) −
− (t − u ) k −1 (( k − 1)t + ( k + 1)u + kc)
= (t − u ) k −2 ((t + u )(( k − 1)t − (k + 1)u + kc ) − (t − u )((k − 1)t + (k + 1)u + kc))
= −2u (t − u ) k −2 (2t − kc)
≤0
Toùm laïi, ta luoân coù h / (u ) ≤ 0 ⇒ h(u ) laø haøm nghòch bieán treân [0,+∞). Do ñoù
f (a, b, c) = h(u ) ≤ h(0) = f (t , t , c)

Baây giôø, ta seõ tìm giaù trò lôùn nhaát cuûa bieåu thöùc

f (t , t , c) = 2t k (t + c ) + 2tc k

trong ñoù t ≥ c ≥ 0 thoûa 2t + c = 3.


Ta coù

t k (t + c) + tc k
k +1
f (t , t , c) = 2t (t + c) + 2tc = 2.3 .
k k
k +1
= 2.3k +1ϕ ( x)
(2t + c)

390
xk + x + 1 c
trong ñoù ϕ ( x) = vaø x = ≤ 1.
( x + 2) k +1 t

Ta coù
− x k + 2kx k −1 − kx + 1 − k
ϕ / ( x) =
( x + 2) k + 2
ϕ / ( x) = 0 ⇔ − x k + 2kx k −1 − kx + 1 − k = 0 (**)

Deã thaáy 1 laø moät nghieäm cuûa phöông trình (**). Goïi β 0 , β1 ,..., β n laø taát caû caùc

nghieäm thuoäc [0,1) neáu coù phöông trình (**). Khi ñoù, deã thaáy

ϕ ( x) ≤ max{ϕ (0), ϕ (1),ϕ ( β i )} ∀x ∈ [0,1]


⇒ f (t , t , c ) ≤ 2.3k +1 max{ϕ (0),ϕ (1),ϕ ( βi )}

Ngoaøi ra, deã thaáy ñaúng thöùc luoân xaûy ra.


Keát luaän
+ 1 > k > 0 ⇒ max f (a, b, c) = 6
+ 2 ≥ k ≥ 1 ⇒ max f (a, b, c) = 2.3k +1 max{ϕ (0), ϕ (1), ϕ ( βi )}
+ k > 2 ⇒ max f (a, b, c) = 3k +1 max{g (1), g (α i )}

trong ñoù

xk + x + 1
+ ϕ ( x) = vaø β 0 , β1 ,..., β n laø taát caû caùc nghieäm thuoäc [0,1) neáu coù
( x + 2) k +1

phöông trình − x k + 2kx k −1 − kx + 1 − k = 0 .

tk + t
+ g (t ) = vaø α 0 , α1 ,...,α m laø taát caû caùc nghieäm thuoäc [0,1) neáu coù
(t + 1) k +1

phöông trình − x k + 2kx k −1 − kx + 1 − k = 0 .

391
Baøi toaùn 95. (Voõ Quoác Baù Caån)
Chöùng minh raèng vôùi moïi soá döông x, y , z thoûa xy + yz + zx = 1 ta luoân coù baát

ñaúng thöùc

x2 y2 z 2
+ + − 2( x 2 + y 2 + z 2 ) ≥ 3 − 2
y z x

Lôøi giaûi.
Ta coù Boå ñeà sau
x + y + z ≥ 0
Boå ñeà. x, y , z laø caùc soá thöïc thoûa  . Khi ñoù, ta coù
 xy + yz + zx ≥ 0

x(b − c) 2 + y (c − a ) 2 + z ( a − b)2 ≥ 0 ∀a, b, c ∈ R

Boå ñeà treân chöùng minh raát ñôn giaûn (chæ caàn duøng tam thöùc baäc hai laø ñöôïc) neân
ôû ñaây ta khoâng nhaéc laïi chöùng minh cuûa noù.
Ta coù baát ñaúng thöùc caàn chöùng minh töông ñöông vôùi
 x2 y 2 z 2 
 + + − x − y − z  + x + y + z − 3 ≥ 2( x 2 + y 2 + z 2 − 1)
 y z x 
( x − y)2 1 ( x − y )2
⇔∑ + .∑ ≥ ∑ ( x − y)2
cyc y 2 cyc x + y + z + 3 cyc
 
1 1
⇔ ∑ ( x − y) 2
+ − 1 ≥ 0
cyc 
 (
y 2 x+ y+ z+ 3 
 )
Ñaët
1 1
Sx = + −1
(
z 2 x+ y+ z+ 3 )
1 1
Sy = + −1
(
x 2 x+ y+ z+ 3 )
1 1
Sz = + −1
(
y 2 x+ y+ z+ 3 )
Khi ñoù, baát ñaúng thöùc caàn chöùng minh töông ñöông vôùi
392
S x ( y − z ) 2 + S y ( z − x ) 2 + S z ( x − y )2 ≥ 0

Ta coù
1 1 1 3
S x + Sy + Sz = + + −3+
x y z 2 x+ y+ z+ 3 ( )
xy + yz + zx 3
= −3+
xyz 2 x+ y+z+ 3 ( )
1 3
= −3+
xyz (
2 x+ y+z+ 3 )
3 3 3
≥ −3+
( xy + yz +
3
zx) 2 (
2 x+ y+ z+ 3 )
3
= 3 3 −3+
(
2 x+ y+z+ 3 )
>0

1
Ñaët t = . Khi ñoù, ta coù
(
2 x+ y+ z+ 3 )
 1  1   1  1 
S x S y + S y S z + S z S x =  t + − 1 t + − 1 +  t + − 1  t + − 1 +
 x  y   y  z 
 1  1 
+  t + − 1 t + − 1
 z  x 
1 1 1  1 1 1 1 1 1
= 3t 2 + 2  + + − 3  t + + + − 2  + +  + 3
x y z  xy yz zx x y z
1 1 1 1 1 1
> + + − 2 + +  + 3
xy yz zx x y z
x + y + z + 3xyz − 2
=
xyz
Ta chöùng minh
x + y + z + 3 xyz − 2
≥0
xyz
⇔ x + y + z + 3xyz − 2 ≥ 0 (*)
393
Neáu x + y + z ≥ 2 thì (*) hieån nhieân ñuùng.

Neáu x + y + z ≤ 2 , ñaët p = x + y + z ⇒ 2 ≥ p ≥ 3. Theá thì theo baát ñaúng thöùc

Schur, ta coù

4 p − p3
xyz ≥
9
Do ñoù

4 p − p 3 − p 3 + 7 p − 6 (2 − p )( p − 1)( p + 3)
p + 3 xyz − 2 ≥ p − 2 + = = ≥0
3 3 3
⇒ (*) ñuùng.

 S x + S y + S z > 0
Vaäy ta coù  neân theo Boå ñeà treân, ta coù
 x y
S S + S S
y z + S S
z x > 0

S x ( y − z ) 2 + S y ( z − x ) 2 + S z ( x − y )2 ≥ 0 (ñpcm)

1
Ñaúng thöùc xaûy ra khi vaø chæ khi x = y = z = .
3

Baøi toaùn 96.

Cho a, b, c, d ≥ 0 thoûa a 2 + b 2 + c 2 + d 2 = 1. Tìm giaù trò nhoû nhaát cuûa bieåu thöùc

a b c d
P= + + +
1 + bcd 1 + cda 1 + dab 1 + abc
Lôøi giaûi.
AÙp duïng baát ñaúng thöùc Bunhiacopxki, ta coù

a2 b2 c2 d2
P= + + +
a + abcd b + abcd c + abcd d + abcd
( a + b + c + d )2

a + b + c + d + 4abcd
1 + 2( ab + ac + ad + bc + bd + cd )
=
a + b + c + d + 4abcd
Ta laïi coù
1 + 2( ab + ac + ad + bc + bd + cd ) − (a + b + c + d + 4abcd ) =

394
= (1 − a)(1 − b )(1 − c )(1 − d ) + ( ab + ac + ad + bc + bd + cd ) +
+ ( abc + bcd + cda + dab) − 5abcd
≥ ( ab + ac + ad + bc + bd + cd ) + ( abc + bcd + cda + dab) − 5abcd
≥0
⇒ 1 + 2(ab + ac + ad + bc + bd + cd ) ≥ a + b + c + d + 4abcd
1 + 2( ab + ac + ad + bc + bd + cd )
⇒ ≥1
a + b + c + d + 4abcd
⇒ P ≥1
Ñaúng thöùc xaûy ra khi vaø chæ khi (a, b, c, d ) = (1,0,0,0).

Vaäy
min P = 1.
Baøi toaùn 97. (Vasile Cirtoaje)
Chöùng minh raèng vôùi moïi soá thöïc a, b, c ta luoân coù baát ñaúng thöùc

(a 2 + b 2 + c 2 ) 2 ≥ 3(a 3b + b 3c + c3a )

Lôøi giaûi.
* Caùch 1.
Khoâng maát tính toång quaùt, ta coù theå giaû söû a = min{a, b, c}.

Ñaët b = a + p, c = a + q ( p, q ≥ 0) . Khi ñoù, ta coù

(a 2 + b 2 + c 2 )2 ≥ 3(a 3b + b3c + c 3a )
⇔ f ( a ) = ( p 2 − pq + q 2 )a 2 − ( p 3 − 5 p 2 q + 4 pq 2 + q 3 ) +
+ ( p 4 − 3 p 3q + 2 p 2 q 2 + q 4 ) ≥ 0

Ta coù
∆ f = −3( p 3 − p 2 q − 2 pq 2 + q 3 )2 ≤ 0
⇒ f (a) ≥ 0
⇒ ñpcm.

* Caùch 2.
Ta coù

395
1
(a 2 + b 2 + c 2 ) 2 − 3( a 3b + b3c + c3 a) = .∑ ( a 2 − 2ab + bc − c 2 + ca ) 2 ≥ 0
2 cyc
⇒ ( a 2 + b 2 + c 2 ) 2 ≥ 3( a 3b + b3c + c3 a) (ñpcm )

* Caùch 3.
Ta coù
1
(a 2 + b 2 + c 2 )2 − 3( a 3b + b3c + c3 a) = .∑ (2a 2 − b 2 − c 2 + 3bc − 3ab) 2 ≥ 0
6 cyc
⇒ ( a 2 + b 2 + c 2 ) 2 ≥ 3( a 3b + b 3c + c3 a) (ñpcm )

a = b = c
Ñaúng thöùc xaûy ra khi vaø chæ khi 
 a : b : c = sin 2 4π : sin 2 2π : sin 2 π
 7 7 7

* Nhaän xeùt.
Ñaây laø moät baát ñaúng thöùc maïnh vaø coù nhieàu öùng duïng. Sau ñaây laø moät soá öùng
duïng cuûa noù

+ ÖÙng duïng 1. (Vasile Cirtoaje)


Cho a, b, c > 0 thoûa a + b + c = 3. Chöùng minh raèng

a b c 3
+ + ≥
ab + 1 bc + 1 ca + 1 2
Lôøi giaûi.
Ta coù
a  a 
∑ ab + 1 = ∑  ab + 1 − a  + a + b + c
cyc cyc

 a 
= 3 + ∑ − a
cyc  ab + 1 
a 2b
= 3−∑
cyc ab + 1

a 2b
≥ 3− ∑ ( theo bñt AM-GM )
cyc 2 ab

396
1
= 3 − .∑ a 3 2b1 2
2 cyc

Theo treân, ta coù


1
∑ a3 2b1 2 ≤ 3 .(a + b + c)2 = 3
cyc

Do ñoù
a 1 1 3
∑ ab + 1 ≥ 3 − 2 .∑ a3 2b1 2 ≥ 3 − 2 .3 = 2
cyc cyc

⇒ ñpcm.

Ñaúng thöùc xaûy ra khi vaø chæ khi a = b = c = 1.

+ ÖÙng duïng 2.

Cho caùc soá khoâng aâm a, b, c, x thoûa a 2 + b2 + c 2 = 1 . Chöùng minh raèng

a2 b2 c2 3
+ + ≥
1 + xab 1 + xbc 1 + xca 3 + x
Lôøi giaûi.
AÙp duïng baát ñaúng thöùc AM-GM, ta coù
2
a2  3  6a 2
+ a (1 + xab) 
2
 ≥
1 + xab 3+ x  3+ x
2
b2  3  6b 2
+ b (1 + xbc) 
2
 ≥
1 + xbc  3+ x  3+ x
2
c2  3  6c 2
+ c 2 (1 + xca )   ≥
1 + xca  3+ x  3+ x
2 2
a2 b2 c2 6  3   3  3
⇒ + + ≥ −  − x  (a b + b c + c a)
3 3
1 + xab 1 + xbc 1 + xca 3 + x  3 + x  3+ x 
Theo treân, ta coù
1 1
a3b + b3c + c 3a ≤ .( a 2 + b 2 + c 2 ) 2 =
3 3
Do ñoù

397
2 2
a2 b2 c2 6  3   3  3
+ + ≥ −  − x  (a b + b c + c a )
3 3
1 + xab 1 + xbc 1 + xca 3 + x  3 + x  3+ x 
2 2
6  3   3  1
≥ −  − x  .
3+ x  3+ x   3+ x  3
3
=
3+ x
2 2 2
a b c 3
⇒ + + ≥ (ñpcm )
1 + xab 1 + xbc 1 + xca 3 + x
1
Ñaúng thöùc xaûy ra khi vaø chæ khi a = b = c = .
3

Baøi toaùn 98. (Komal)


Cho caùc soá döông a, b, c thoûa abc = 1. Chöùng minh raèng

1 1 1 3  1 1 1 2
+ + − ≥  2 + 2 + 2 . 2
a b c a+b+c a b c  a + b2 + c 2

Lôøi giaûi.
Ta coù baát ñaúng thöùc caàn chöùng minh töông ñöông vôùi

3abc 2(a 2b 2 + b 2c 2 + c 2 a 2 )
ab + bc + ca − ≥
a+b+c a 2 + b2 + c 2
Do caû hai veá cuûa baát ñaúng thöùc naøy ñoàng baäc neân khoâng maát tính toång quaùt, ta coù
1
theå giaû söû a + b + c = 1 . Ñaët q = ab + bc + ca, r = abc ⇒ ≥ q ≥ 0, r ≥ 0. Khi ñoù, ta
3
coù baát ñaúng thöùc caàn chöùng minh töông ñöông vôùi
2(q 2 − 2r )
q − 3r ≥
1 − 2q
⇔ r (6q + 1) + q (1 − 4q ) ≥ 0

Neáu 1 ≥ 4q thì baát ñaúng thöùc treân hieån nhieân ñuùng.

4q − 1
Neáu 4q ≥ 1 thì theo baát ñaúng thöùc Schur, ta coù r ≥ ≥ 0. Do ñoù
9

398
(4q − 1)(6q + 1) (4q − 1)(1 − 3q )
r (6q + 1) + q (1 − 4q ) ≥ + q (1 − 4q ) = ≥0
9 9
⇒ ñpcm.

Baøi toaùn 99. (Nguyeãn Anh Cöôøng)


Cho caùc soá döông x, y , z thoûa x + y + z = 1. Chöùng minh raèng

x y z
+ + ≥2
( x + y )( x + z ) + yz ( y + z )( y + x) + zx ( z + x )( z + y ) + xy

Lôøi giaûi.
Ta coù baát ñaúng thöùc caàn chöùng töông ñöông vôùi

x
∑ ( x + y )( x + z ) + yz
≥2
cyc

x
⇔∑ ≥2
cyc x( x + y + z ) + yz + yz
1
⇔∑ ≥2
cyc yz yz
1+ +
x x

yz zx xy
Ñaët m = ,n = ,p= thì ta coù m, n, p > 0 vaø mn + np + pm = 1. Khi ñoù,
x y z

ta coù baát ñaúng thöùc caàn chöùng minh töông ñöông vôùi
1
∑ ≥2
cyc m +1 + m
2

⇔∑ m2 + 1 − m ≥ 2
cyc

Ñaët a = m 2 + 1 − m, b = n 2 + 1 − n, c = p 2 + 1 − p thì ta coù 1 > a, b, c > 0 vaø

1 − a2
m=
2a
1 − b2
n=
2b

399
1 − c2
p=
2c
(1 − a 2 )(1 − b 2 )
⇒∑ = mn + np + pm = 1
cyc 4ab
⇔ ∑ c (1 − a 2 )(1 − b 2 ) = 4abc
cyc

⇔ ( a + b + c ) − ∑ ab( a + b) + abc( ab + bc + ca ) = 4abc


cyc

⇔ ( a + b + c − abc)(1 − ab − bc − ca ) = 0 (*)

Do 1 > a, b, c > 0 neân a + b + c − abc > 0 . Do ñoù


(*) ⇔ ab + bc + ca = 1

Do ñoù, ñeå chöùng minh baát ñaúng thöùc ñaõ cho, ta chæ caàn chöùng minh

a+ b+ c ≥2 (**)

trong ñoù a, b, c > 0 thoûa ab + bc + ca = 1 .

Ta coù

( )
4
(**) ⇔ a+ b+ c ≥ 16( ab + bc + ca ) (***)

Do caû 2 veá cuûa baát ñaúng thöùc treân ñoàng baäc neân khoâng maát tính toång quaùt, ta coù

theå giaû söû a + b + c = 1.


1
Ñaët q = ab + bc + ca , r = abc ⇒ ≥ q ≥ 0, r ≥ 0. Khi ñoù, ta coù
3
(***) ⇔ 1 ≥ 16(q 2 − 2r )
⇔ 32r + (1 − 4q)(1 + 4q ) ≥ 0

Neáu 1 ≥ 4q thì baát ñaúng thöùc treân hieån nhieân ñuùng.

4q − 1
Neáu 4q ≥ 1 thì theo baát ñaúng thöùc Schur, ta coù r ≥ ≥ 0. Do ñoù
9
32(4q − 1) (4q − 1)(23 − 36q)
32r + (1 − 4q )(1 + 4q ) ≥ + (1 − 4q)(1 + 4q ) = ≥0
9 9
⇒ (***) ñuùng.

⇒ ñpcm.
400
Baøi toaùn 100. (Phaïm Kim Huøng, Voõ Quoác Baù Caån)
Cho a, b, c ≥ 0 thoûa a + b + c = 1. Tìm ñieàu kieän caàn vaø ñuû vôùi a, b, c ñeå baát ñaúng

thöùc sau ñuùng

(a 2 + b 2 + c 2 )(8a 2b 2 + 8b 2 c 2 + 8c 2 a 2 + 19 abc) ≥ ab + bc + ca

Lôøi giaûi.

Ta seõ chöùng minh raèng ñieàu kieän caàn vaø ñuû ñeå baát ñaúng thöùc treân ñuùng laø a, b

vaø c laø ñoä daøi ba caïnh cuûa moät tam giaùc (coù theå suy bieán).

+ Ñieàu kieän caàn.

Giaû söû a , b , c khoâng laø ñoä daøi ba caïnh cuûa moät tam giaùc (coù theå suy bieán).

Khi ñoù, cho c = 0, a, b > 0 thì baát ñaúng thöùc treân trôû thaønh

8( a 2 + b 2 ) a 2b 2 ≥ ab
⇔ 8(a 2 + b 2 )ab ≥ 1 (*)

Cho a = 1, b → 0+ thì ta coù lim+ 8ab( a 2 + b 2 ) = 0 < 1 neân (*) khoâng ñuùng.
b →0

Vaäy ta phaûi coù a , b , c laø ñoä daøi ba caïnh cuûa moät tam giaùc (coù theå suy bieán).

+ Ñieàu kieän ñuû.

Giaû söû a , b , c laø ñoä daøi ba caïnh cuûa moät tam giaùc (coù theå suy bieán). Khi ñoù,

ta seõ chöùng minh

(a 2 + b 2 + c 2 )(8a 2b 2 + 8b 2 c 2 + 8c 2 a 2 + 19 abc) ≥ ab + bc + ca

1
Ñaët q = ab + bc + ca, r = abc ⇒ ≥ q ≥ 0, r ≥ 0. Do a , b , c laø ñoä daøi ba caïnh
3
cuûa moät tam giaùc (coù theå suy bieán) neân

4q − 1 = ( a+ b+ c )( a+ b− c )( b+ c− a )( )
c+ a− b >0
1 1
⇒ ≥q≥
3 4

401
4q − 1
Do ñoù, theo baát ñaúng thöùc Schur, ta coù r ≥ ≥ 0.
9
Baát ñaúng thöùc caàn chöùng minh töông ñöông vôùi

(1 − 2q )(19r + 8(q 2 − 2r )) ≥ q

⇔ (1 − 2q )(3r + 8q 2 ) ≥ q (**)

Ta coù

 4q − 1 
(1 − 2q )(3r + 8q 2 ) − q ≥ (1 − 2q )  + 8q 2  − q
 3 
(4q − 1)(1 − 3q )(4q + 1)
=
3
≥0

Do ñoù (**) ñuùng.

Vaäy ñieàu kieän caàn vaø ñuû ñeå baát ñaúng thöùc ñaõ cho ñuùng laø a , b , c laø ñoä daøi ba

caïnh cuûa moät tam giaùc (coù theå suy bieán).

Baøi toaùn 101. (Titu Andreescu)


Cho caùc soá thöïc a, b thoûa 3( a + b) ≥ 2 ab + 1 . Chöùng minh raèng

9(a 3 + b3 ) ≥ a3b3 + 1

Lôøi giaûi.
Ñaët S = a + b, P = ab thì töø giaû thieát, ta coù 3S ≥ 2 P + 1 (*). Baát ñaúng thöùc caàn

chöùng minh trôû thaønh

9 S ( S 2 − 3P ) ≥ P + 1 ( P 2 − P + 1)

Coù 2 tröôøng hôïp xaûy ra

7 −3 5 7+3 5
* Tröôøng hôïp 1. P ≤ ∨P≥ . Khi ñoù, töø (*), ta coù
2 2
 4( P + 1) 2  2 P + 1 (4 P 2 − 19 P + 4)
9 S ( S − 3P ) ≥ 6 P + 1 
2
− 3P  =
 9  3

402
Do ñoù, ñeå chöùng minh baát ñaúng thöùc ñaõ cho, ta chæ caàn chöùng minh
2(4 P 2 − 19 P + 4) ≥ 3( P 2 − P + 1)
 7 − 3 5  7+3 5 
⇔ 5 P −  P −  ≥ 0 (ñuùng)
 2  2 

7−3 5 7+3 5
* Tröôøng hôïp 2. ≤P≤ . Töø ñaây, ta coù a, b cuøng daáu, töø ñoù töø giaû
2 2
3S − 2
thieát, ta suy ra ñöôïc a, b > 0. Khi ñoù, (*) trôû thaønh 3S ≥ 2( P + 1) ⇒ 0 < P ≤ .
2
Baát ñaúng thöùc caàn chöùng minh trôû thaønh

9 S ( S 2 − 3P ) ≥ P 3 + 1
⇔ 9S 3 − 27 PS − P 3 − 1 ≥ 0
S 2 3S − 2
+ Tröôøng hôïp 2.1. ≥ ⇔ S 2 − 6S + 4 ≥ 0. Khi ñoù, ta coù
4 2

27 S (3S − 2) (3S − 2)3


9 S − 27 PS − P − 1 ≥ 9S −
3 3 3
− −1
2 8
45S ( S 2 − 6 S + 4)
=
8
≥0

S 2 3S − 2
( ) ( )
3 3
+ Tröôøng hôïp 2.2. ≤ ⇔ 3− 5 ≤ S 3 ≤ 3 + 5 . Khi ñoù, ta coù
4 2

27 S 3 S 6
9 S 3 − 27 PS − P 3 − 1 ≥ 9S 3 − − −1
4 64

(
 3+ 5
) ( )
− S 3   S 3 − 3 − 5 
3 3

=  
64
≥0

Toùm laïi, trong moïi tröôøng hôïp, ta luoân coù 9 S ( S 2 − 3P ) ≥ P 3 + 1 (ñpcm)

 3− 5 3− 5   3+ 5 3+ 5 
Ñaúng thöùc xaûy ra khi vaø chæ khi (a, b) =  , , , .
 2 2   2 2 

403
Baøi toaùn 102. (Phaïm Kim Huøng)
Cho caùc soá khoâng aâm a, b, c. Chöùng minh raèng

a3 b3 c3 a+b+c
+ 2 2+ 2 ≥
2a + b
2 2
2b + c 2c + a 2
3
Lôøi giaûi.
Ta coù baát ñaúng thöùc caàn chöùng minh töông ñöông vôùi
2b − a 2c − b 2a − c
.(a − b) 2 + 2 2 .(b − c) 2 + 2 .(c − a ) 2 ≥ 0
2a + b
2 2
2b + c 2c + a 2

Coù 2 tröôøng hôïp xaûy ra


+ Tröôøng hôïp 1. a ≥ b ≥ c ≥ 0 . Khi ñoù, ta coù
4b c
− ≥0
2a 2 + b 2 2c 2 + a 2
−2 a 2a
+ 2 ≥0
2a + b
2 2
2c + a 2
4b − 2a 2a − c
⇒ 2 + 2 ≥0
2a + b 2
2c + a 2
4b − 2a 2a − c
⇒ .( a − b)2 + 2 .(c − a) 2 ≥ 0 (1)
2a + b
2 2
2c + a 2

(4c − 2b)b 2 (2a − c) a 2


+ ≥0
2b 2 + c 2 2c 2 + a 2
4c − 2b 2a − c
⇒ .(b − c) 2 + 2 .(c − a )2 ≥ 0 (2)
2b + c
2 2
2c + a 2

Coäng caùc baát ñaúng thöùc (1) vaø (2) veá theo veá roâi chia caû hai veá cho 2, ta ñöôïc
2b − a 2c − b 2a − c
.(a − b) 2 + 2 2 .(b − c) 2 + 2 .(c − a ) 2 ≥ 0
2a + b
2 2
2b + c 2c + a 2

+ Tröôøng hôïp 2. c ≥ b ≥ a ≥ 0.

+ Tröôøng hôïp 2.1. 2b ≥ c + a. Khi ñoù, ta seõ chöùng minh


2b − a 4(2a − c)
+ 2 ≥0
2a + b
2 2
2c + a 2

404
Thaät vaäy, deã thaáy veá traùi laø haøm taêng cuûa c neân ta chæ caàn chöùng minh khi c = b,

töùc laø chöùng minh


2b − a 4(2a − b)
+ 2 ≥0
2a + b
2 2
2b + a 2
⇔ 4b3 + 2a 2b − 2ab 2 + 16a 3 − 8a 2b + 8ab 2 − 4b3 ≥ 0
⇔ 3a (5a 2 − 2ab + 2b 2 ) ≥ 0 (ñuùng)

2b − a 4(2a − c)
Do ñoù + ≥0
2a 2 + b 2 2c 2 + a 2
Vaäy
2b − a 2c − b 2a − c
.(a − b) 2 + 2 2 .(b − c )2 + 2 .(c − a) 2 ≥
2a + b
2 2
2b + c 2c + a 2

2b − a 2a − c
≥ .(c − a ) 2 + 2 .(c − a ) 2
4(2a + b )
2 2
2c + a 2

≥0
+ Tröôøng hôïp 2.2. 2b ≤ c + a. Khi ñoù, ta seõ chöùng minh
2b − a 6a − 3c
+ 2 ≥0 (3)
2a + b
2 2
2c + a 2
Thaät vaäy, deã thaáy veá traùi laø haøm taêng cuûa c neân chæ caàn chöùng khi c = 2b − a . Baát
ñaúng thöùc (3) trôû thaønh
2b − a 9a − 6b
+ 2 ≥0
2a + b 8b + 3a 2 − 8ab
2 2

⇔ 10b3 − 15b 2 a + 2a 2b + 15a 3 ≥ 0 (ñuùng)

Tieáp theo, ta seõ chöùng minh


2c − b 3 2a − c
+ . 2 ≥0 (4)
2b + c
2 2
2 2c + a 2
Thaät vaäy, vì veá traùi laø haøm giaûm theo a neân ta chæ caàn chöùng minh khi a = b , baát
ñaúng thöùc trôû thaønh
4c − 2b 6b − 3c
+ ≥0
2b 2 + c 2 2c 2 + b 2
⇔ 5c3 + 2c 2b − 2b 2c + 10b3 ≥ 0 (ñuùng)
405
Neáu c ≤ 2a thì ta coù baát ñaúng thöùc caàn chöùng minh ñuùng. Neáu c ≥ 2a thì töø 2 baát
3
ñaúng thöùc treân, vôùi chuù yù raèng (c − a ) 2 ≤ 3(b − a ) 2 + .(c − b)2 , ta coù
2
2b − a 2c − b 2a − c
.(a − b) 2 + 2 2 .(b − c) 2 + 2 .(c − a ) 2 ≥
2a + b
2 2
2b + c 2c + a 2

 2b − a 3(2a − c)   2c − b 3 2a − c 
≥ 2 + 2 2 
.(b − a ) 2 +  2 2 + . 2 2 
.(c − b) 2
 2a + b 2c + a   2b + c 2 2c + a 
2

≥0
Toùm laïi, ta luoân coù
2b − a 2c − b 2a − c
.( a − b ) 2
+ .(b − c ) 2
+ .(c − a) 2 ≥ 0 (ñpcm)
2a + b
2 2
2b + c
2 2
2c + a
2 2

Ñaúng thöùc xaûy ra khi vaø chæ khi a = b = c.

Baøi toaùn 103. (Voõ Quoác Baù Caån)


n n
1
Cho n soá thöïc a1 , a2 ,..., an > 0 thoûa ∑ ai = ∑ . Chöùng minh raèng
i =1 i =1 ai

n n
3∑ ai ≥ ∑ ai2 + 8
i =1 i =1

Lôøi giaûi.
Ta coù
n n
1
∑ ai = ∑
i =1 i =1 ai
n
ai2 − 1
⇔∑ =0
i =1 ai

Baát ñaúng thöùc caàn chöùng minh töông ñöông vôùi


n
8( ai2 − 1)
∑ ≥0
i =1 3ai + ai2 +8

406
ai2 − 1
n
ai
⇔∑ ≥0
i =1 8
3 + 1+ 2
ai

Khoâng maát tính toång quaùt, ta coù theå giaû söû a1 ≥ a2 ≥ ... ≥ an > 0.

 a12 − 1 a22 − 1 an2 − 1


 ≥ ≥ ... ≥
 a1 a2 an
Khi ñoù, deã thaáy  1 1 1 neân theo baát ñaúng
 ≥ ≥ ... ≥
8 8 8
3 + 1 + 2 3 + 1 + 2 3 + 1+ 2
 a1 a2 an

thöùc Chebyshev, ta coù

ai2 − 1  
 
n
1  n ai2 − 1   n
ai 1 
∑ 8
≥ . ∑  ∑
n  i =1 ai   i =1 8 
=0
i =1
3 + 1+ 2  3 + 1 + 2 
ai  ai 
⇒ ñpcm.

Ñaúng thöùc xaûy ra khi vaø chæ khi a1 = a2 = ... = an = 1.

Baøi toaùn 104.


Cho caùc soá khoâng aâm a, b, c. Chöùng minh raèng
3
b+c 
a + b + c − 3abc ≥ 2 
3 3 3
− a
 2 
Lôøi giaûi.
Ñaët
3
 b+c
f (a, b, c) = a + b + c − 3abc + 2  a −
3 3 3

 2 

Khi ñoù, ta caàn chöùng minh


f (a, b, c) ≥ 0

Tröôùc heát, ta chöùng minh


407
 b+c b+c
f ( a , b, c ) ≥ f  a , ,  (*)
 2 2 

Thaät vaäy
3
 b+c
(*) ⇔ a + b + c − 3abc + 2  a −
3 3 3
 ≥
 2 
(b + c )3
2 3
b+c  b+c
≥a +
3
− 3a.  + 2 a − 
4  2   2 
4b3 + 4c 3 − (b + c)3 3a ((b + c) 2 − 4bc )
⇔ + ≥0
4 4
3(b − c) 2 (b + c ) 3a (b − c) 2
⇔ + ≥ 0 (ñuùng)
4 4
Vaäy (*) ñuùng.
Tieáp theo, ta seõ chöùng minh
f (a, t , t ) ≥ 0 (**)

b+c
trong ñoù t = .
2
Ta coù
(**) ⇔ a 3 + 2t 3 − 3at 2 + 2( a − t )3 ≥ 0
⇔ ( a − t ) 2 ( a + 2t ) + 2( a − t )3 ≥ 0
⇔ 3a( a − t ) 2 ≥ 0 (ñuùng)

Töø (*) vaø (**), ta suy ra ñieàu phaûi chöùng minh.


Ñaúng thöùc xaûy ra khi vaø chæ khi a = b = c hoaëc a = 0, b = c.

408
Baøi toaùn 105.
Chöùng minh raèng vôùi moïi soá döông a, b, c thì

a b c a+ b+ c
a) + + ≥
a+b b+c c+a 2
a3 b3 c3 a+ b+ c
b) + + ≥
a 2 + ab + b 2 b 2 + bc + c 2 c 2 + ca + a 2 3

Lôøi giaûi.

a) Ñaët a = x 2 , b = y 2 , c = z 2 ( x, y , z > 0) . Baát ñaúng thöùc caàn chöùng minh trôû thaønh

x2 y2 z2 x+ y+ z
+ + ≥
x +y
2 2
y +z2 2
z +x
2 2 2
2
 x2 y2 z2   x + y + z 2
⇔ + +  ≥ 
 x2 + y 2 y +z
2 2 2 2 
z +x   2 

2x4 4x2 y 2
⇔∑ 2 +∑ ≥ ( x + y + z)2
cyc x + y
2
cyc ( x + y )( y + z )
2 2 2 2

x4 y4
Löu yù raèng ∑ x2 + y 2 ∑ x2 + y2 = 0

cyc cyc

Do ñoù, baát ñaúng thöùc caàn chöùng minh töông ñöông vôùi

x4 + y 4 4 x2 y2
∑ x 2 + y 2 + ∑ 2 2 2 2 ≥ ( x + y + z )2
cyc cyc ( x + y )( y + z )

 x2 y 2 y2z2 z 2 x2   1 1 1 
Deã thaáy  , ,  vaø  , ,  laø
 x2 + y 2 y2 + z 2 z 2 + x2   x2 + y 2 y2 + z 2 z 2 + x2 
   

2 daõy ñôn ñieäu ngöôïc chieàu nhau neân theo baát ñaúng thöùc saép xeáp laïi, ta coù

x2 y2 x2 y2
∑ ≥∑ 2
( x 2 + y 2 )( y 2 + z 2 ) cyc x + y
2
cyc

Do ñoù, ñeå chöùng minh baát ñaúng thöùc ñaõ cho, ta chæ caàn chöùng minh

409
x4 + y 4 4x2 y 2
∑ x 2 + y 2 ∑ x 2 + y 2 ≥ ( x + y + z)2
+
cyc cyc

2 x2 y 2
⇔ ∑x +∑ 2 2
≥ 2( xy + yz + zx)
cyc x + y
2
cyc

1 xy ( x − y ) 2
⇔ .∑ ( x − y ) 2 ≥ ∑ 2
2 cyc cyc x + y2

( x − y)4
⇔∑ ≥ 0 (ñuùng)
cyc x2 + y2

⇒ ñpcm.

Ñaúng thöùc xaûy ra khi vaø chæ khi a = b = c.

b) Ñaët a = x 2 , b = y 2 , c = z 2 ( x, y , z > 0) . Baát ñaúng thöùc caàn chöùng minh trôû thaønh

x3 y3 z3 x+ y+z
+ + ≥
x4 + x2 y 2 + y4 y 4 + y2 z 2 + z 4 z 4 + z2 x2 + x4 3
2
 x3 y3 z3   x + y + z 2
⇔ + +  ≥ 
 x 4 + x2 y 2 + y 4 y4 + y2z2 + z4 z 4 + z 2 x 2 + x4   3 

x6 x3 y 3
⇔∑ + 2 ∑ 4 2 2 4 4 22 4 ≥
cyc x4 + x2 y 2 + y4 cyc ( x + x y + y )( y + y z + z )
x 2 + y 2 + z 2 + 2 xy + 2 yz + 2 zx

3
x6 y6
Löu yù raèng ∑ x 4 + x2 y 2 + y 4 ∑ x 4 + x 2 y 2 + y 4 = 0

cyc cyc

Do ñoù, baát ñaúng thöùc caàn chöùng minh töông ñöông vôùi
1 x6 + y 6 x3 y 3
.∑ 4 + 2∑ 4 2 2 4 4 22 4 ≥
2 cyc x + x 2 y 2 + y 4 cyc ( x + x y + y )( y + y z + z )
x 2 + y 2 + z 2 + 2 xy + 2 yz + 2 zx

3

410
x3 y3 1  2 3( x 6 + y 6 ) 
⇔ 6∑ ≥ .∑  x + y + 4 xy − 4
2

cyc ( x 4 + x 2 y 2 + y 4 )( y 4 + y 2 z 2 + z 4 ) 2 cyc  x + x2 y 2 + y4 
6 x3 y 3 6 x3 y 3 − ( x − y ) 4 ( x + y ) 2
⇔∑ ≥∑
cyc ( x 4 + x 2 y 2 + y 4 )( y 4 + y 2 z 2 + z 4 ) cyc x4 + x2 y 2 + y4

 x3 y3 y3 z 3 z 3 x3 
Maët khaùc, deã thaáy  , ,  vaø
 ( x 4 + x2 y 2 + y 4 ) ( y 4 + y 2 z 2 + z 4 ) ( z 4 + z 2 x2 + z 4 ) 
 

 1 1 1 
 , ,  laø hai daõy ñôn ñieäu
 ( x 4 + x2 y 2 + y 4 ) ( y 4 + y 2 z 2 + z 4 ) ( z 4 + z 2 x2 + z 4 ) 
 

ngöôïc chieàu nhau neân theo baát ñaúng thöùc saép xeáp laïi, ta coù

x3 y3 x3 y 3
∑ ≥∑ 4
( x 4 + x 2 y 2 + y 4 )( y 4 + y 2 z 2 + z 4 ) cyc x + x y + y
2 2 4
cyc

Töø ñaây, ta suy ra ñieàu phaûi chöùng minh.


Ñaúng thöùc xaûy ra khi vaø chæ khi a = b = c.

Baøi toaùn 106. (Phan Thaønh Vieät)


Cho caùc soá khoâng aâm a, b, c. Chöùng minh raèng

a4 b4 c4 a 3 + b 3 + c3
+ + ≥
a 2 + ab + b 2 b 2 + bc + c 2 c 2 + ca + a 2 a+b+c
Lôøi giaûi.
Ta coù
 
a 3 + b3 + c 3
( a + b + c )  ∑
 cyc a 2
− ∑ ab  + 3abc
=  cyc  =
3abc
+ ∑ a 2 − ∑ ab
a+b+c a+b+c a + b + c cyc cyc

Do ñoù, baát ñaúng thöùc caàn chöùng minh töông ñöông vôùi

a4 3abc
∑ a 2 + ab + b2 − ∑ a 2 + ∑ ab ≥ a + b + c
cyc cyc cyc

411
 a4  3abc
⇔ ∑ 2 − a 2
+ ab ≥
cyc  a + ab + b  a+b+c
2

ab3 3abc
⇔∑ ≥
cyc a + ab + b
2 2
a+b+c

AÙp duïng baát ñaúng thöùc Bunhiacopxki, ta coù

 ab3  a 2 + ab + b 2 
 ∑ 2 2  ∑  ≥ ( a + b + c)
2

 cyc a + ab + b  cyc ab 

ab3 (a + b + c) 2
⇒∑ ≥
a 2 + ab + b 2 a 2 + b2
cyc
3+ ∑
cyc ab

Do ñoù, ñeå chöùng minh baát ñaúng thöùc ñaõ cho, ta chæ caàn chöùng minh

( a + b + c )2 3abc

a +b
2 2
a+b+c
3+∑
cyc ab
⇔ ( a + b + c )3 ≥ 9abc + 3∑ c (a 2 + b 2 )
cyc

⇔ a + b + c ≥ 3abc (ñuùng theo bñt AM-GM)


3 3 3

⇒ ñpcm.

Ñaúng thöùc xaûy ra khi vaø chæ khi a = b = c.

Baøi toaùn 107.


Chöùng minh raèng vôùi moïi soá döông a, b, c thoûa abc = 1 ta coù baát ñaúng thöùc

a2 b2 c2 1
+ + ≥
(2a + b)(1 + ab) (2b + c )(1 + bc) (2c + a )(1 + ca ) 2

Lôøi giaûi.
a, b, c > 0 x y z
Do  neân toàn taïi caùc soá döông x, y , z sao cho a = , b = vaø c = .
abc = 1 y z x

Khi ñoù, baát ñaúng thöùc caàn chöùng minh trôû thaønh

412
x2 y2 1
∑ z ( z 2 + 2 xy)( x + y) ≥ 2
cyc

AÙp duïng baát ñaúng thöùc Bunhiacopxki, ta coù


x2 y 2 1 x4 y 4
∑ z ( z 2 + 2 xy )( x + y ) = xyz .∑ xy( z 2 + 2 xy)( x + y)
cyc cyc

1 ( x 2 y 2 + y 2 z 2 + z 2 x 2 )2
≥ .
xyz ∑ xy ( z 2 + 2 xy )( x + y )
cyc

1 ( x2 y 2 + y 2 z 2 + z 2 x2 )2
= .
xyz 2 xyz ( xy + yz + zx) + 2∑ x 2 y 2 ( x + y )
cyc

( x2 y 2 + y 2 z 2 + z 2 x2 )2
=
2 xyz ( x 2 y 2 + y 2 z 2 + z 2 x 2 )( x + y + z )
x2 y 2 + y 2 z 2 + z 2 x2
=
2 xyz ( x + y + z )
1

2
⇒ ñpcm.

Baøi toaùn 108. (Vasile Cirtoaje)


Cho caùc soá thöïc a, b, c. Chöùng minh raèng

3(1 − a + a 2 )(1 − b + b 2 )(1 − c + c 2 ) ≥ 1 + abc + a 2b 2c 2

Lôøi giaûi.
Söû duïng ñaúng thöùc

2(1 − a + a 2 )(1 − b + b 2 ) = 1 + a 2b 2 + ( a − b) 2 + (1 − a) 2 (1 − b) 2

Ta coù

2(1 − a + a 2 )(1 − b + b 2 ) ≥ 1 + a 2b 2

Do ñoù, ñeå chöùng minh baát ñaúng thöùc ñaõ cho, ta chæ caàn chöùng minh
3(1 + a 2b 2 )(1 − c + c 2 ) ≥ 2(1 + abc + a 2b 2c 2 )
⇔ f (c) = (3 + a 2b 2 )c 2 − (3 + 2ab + 3a 2b 2 )c + 1 + 3a 2b 2 ≥ 0

413
Ta coù

∆ f = −3(1 − ab) 4 ≤ 0

Do ñoù
f (c ) ≥ 0

Ñaúng thöùc xaûy ra khi vaø chæ khi a = b = c = 1.

Baøi toaùn 109. (Vasile Cirtoaje)

Chöùng minh raèng vôùi moïi soá khoâng aâm a, b, c, d thoûa a 2 − ab + b 2 = c 2 − cd + d 2 ta

coù baát ñaúng thöùc


(a + b)(c + d ) ≥ 2(ab + cd )

Lôøi giaûi.
Ñaët f (a, b, c, d ) = (a + b)(c + d ) − 2(ab + cd )

Khoâng maát tính toång quaùt, ta coù theå giaû söû c + d ≥ a + b ≥ 0 . Khi ñoù, ta coù

(
f (a, b, c, d ) − f a, b, c 2 − cd + d 2 , c 2 − cd + d 2 = )
( )
= ( a + b) c + d − 2 c 2 − cd + d 2 + 2(c − d ) 2

 3( a + b) 
= (c − d ) 2  2 − 
 c + d + 2 c 2 − cd + d 2 
≥ 0 (do c + d ≥ a + b ≥ 0)

Do ñoù

(
f (a, b, c, d ) ≥ f a, b, c 2 − cd + d 2 , c 2 − cd + d 2 )
= f ( a, b, a 2 − ab + b 2 , a 2 − ab + b 2 ) (1)

Tieáp theo, ta seõ chöùng minh

(
f a, b, a 2 − ab + b 2 , a 2 − ab + b 2 ≥ 0 ) (2)

Thaät vaäy

(2) ⇔ 2(a + b) a 2 − ab + b 2 ≥ 2(a 2 + b 2 )


414
⇔ ( a + b)2 (a 2 − ab + b 2 ) ≥ ( a 2 + b 2 ) 2
⇔ ab( a − b)2 ≥ 0 (ñuùng)

Töø (1) vaø (2), ta suy ra ñieàu phaûi chöùng minh.


Baøi toaùn 110. (Phaïm Kim Huøng)
Cho caùc soá döông a, b, c. Chöùng minh raèng
2 2 2
 b2   c2   c 2  12( a 3 + b3 + c3 )
a +  + b +  +  a +  ≥ .
 c   a  b  a+b+c

Lôøi giaûi.
Ta coù
2 2 2
 b2   c2   c 2  12(a 3 + b3 + c 3 )
 a + +
  b + +
  a +  ≥
 c   a   b  a+b+c
ab 2 a 4 12(a 3 + b3 + c 3 )
⇔ ∑ a + 2∑2
+∑ 2 ≥
cyc cyc c cyc b a+b+c
 ab 2   a4 
⇔ 2∑  + ac − 2ab  + ∑  2 + b 2 − 2a 2  +
cyc  c  cyc  b 
  12(a 3 + b3 + c 3 )
+ 2  ∑ ab − ∑ a 2  ≥ − 4(a 2 + b 2 + c 2 )
 cyc  a+b+c
 cyc 
 b2 4a 2( a + b) 
⇔ ∑ (b − c) 2  2 + + − 4 ≥ 0
cyc c a+b+c c 
Ñaët

b2 4a 2(a + b)
Sa = + + −4
c2 a + b + c c
c2 4b 2(b + c )
Sb = 2 + + −4
a a+b+c a
a2 4c 2(c + a )
Sc = 2 + + −4
b a+b+c b
Baát ñaúng thöùc caàn chöùng minh töông ñöông vôùi

S a (b − c ) 2 + Sb (c − a ) 2 + Sc (a − b) 2 ≥ 0

415
Coù 2 tröôøng hôïp xaûy ra
+ Tröôøng hôïp 1. c ≥ b ≥ a > 0. Khi ñoù, ta coù Sb ≥ 0.

Ta coù
b 2 c 2 4(a + b) 2( a + b) 2(b + c)
S a + Sb = + + + + −8 ≥ 0
c2 a 2 a + b + c c a

b2 c 2 2a 2c 2b
Vì + ≥ 2, + ≥ 4, ≥2
c2 a 2 c a a

a 2 c 2 4(b + c ) 2( a + c) 2(b + c)
Sc + Sb = + + + + −8 ≥ 0
b2 a 2 a + b + c b a

a 2 c2 2a 2b 2c
Vì 2 + 2 ≥ 2, + ≥ 4, ≥ 2
b a b a a
Do ñoù

S a (b − c )2 + Sb (c − a ) 2 + Sc ( a − b)2 ≥ ( S a + Sb )(b − c )2 + ( Sc + Sb )(a − b) 2 ≥ 0

+ Tröôøng hôïp 2. a ≥ b ≥ c > 0. Khi ñoù, ta coù S a ≥ 1, Sc ≥ −1.

Ta coù

b 2 2c 2 8b + 4a 2( a + b) 4(b + c)
S a + 2 Sb = 2 + 2 + + + − 12 ≥ 0
c a a+b+c c a
4a + 8b 2a 2b 2c 2a
Vì ≥ 4, + ≥ 4, + ≥4
a+b+c c a a c

b 2 4c 2 16b + 4a 2( a + b) 8(b + c)
S a + 4 Sb = 2 + 2 + + + − 20
c a a +b +c c a
b 2 4c 2 8b + 4a 2( a + b) 8(b + c )
≥ 2+ 2 + + + − 16 = f (b)
c a a+b+c c a
Deã daøng kieåm tra f (b) laø haøm ñoàng bieán. Do ñoù

4c 2 16c 2 a
f (b) ≥ f (c) = 2 + + − 9 ≥ 2 32 − 9 > 1
a a c
+ Khaû naêng 2.1. a + c ≤ 2b ⇔ 2(b − c) ≥ a − c ≥ 0 ∧ b − c ≥ a − b ≥ 0.

Neáu Sb ≥ 0 thì ta coù ngay ñpcm. Neáu Sb ≤ 0 , thì


416
S a (b − c ) 2 + Sb (c − a ) 2 + Sc ( a − b) 2 ≥ ( Sa + 4Sb − 1)(b − c) 2 ≥ 0

+ Khaû naêng 2.2. a + c ≥ 2b. . Khi ñoù, ta seõ chöùng minh S c + 2 Sb ≥ 0. Thaät vaäy,

ta coù

a 2 2c 2 8b + 4c 2(a + c) 4(b + c )
Sc + 2Sb = + + + + − 12 = g (c)
b2 a 2 a + b + c b a
+ Khaû naêng 2.2.1. a ≥ 2b . Khi ñoù, do g (c ) laø haøm taêng neân

a2 8b 4b 2a
g (c) ≥ g (0) = 2 + + + − 12 ≥ 0
b a+b a b

a 9b a 4b a2 a −b 1
Vì + ≥ 5, + ≥ 4, 2 + ≥ 6, ≥−
b a+b b a b b a+b 3
+ Khaû naêng 2.2.2. a ≤ 2b . Khi ñoù, do g (c ) laø haøm taêng neân

a 2 8b 2 4b 4a 14
g (c) ≥ g (2b − a ) = 2 + 2 + − − ≥ 0 (do 2b ≥ a ≥ b)
b a a 3b 3
Vaäy

S a (b − c )2 + Sb (c − a ) 2 + Sc ( a − b)2 ≥ ( S a + 2 Sb )(b − c )2 + ( Sc + 2 Sb )(a − b) 2 ≥ 0


Toùm laïi, trong moïi tröôøng hôïp, ta luoân coù

S a (b − c ) 2 + Sb (c − a ) 2 + Sc ( a − b) 2 ≥ 0 (ñpcm)

Ñaúng thöùc xaûy ra khi vaø chæ khi a = b = c.


Baøi toaùn 111.
Chöùng minh raèng vôùi moïi soá döông a, b, c, d ta coù baát ñaúng thöùc

b( a + c ) c(b + d ) d (c + a ) a (d + b)
+ + + ≥4
c( a + b) d (b + c ) a (c + d ) b( d + a )

Lôøi giaûi.
Ta coù baát ñaúng thöùc caàn chöùng minh töông ñöông vôùi

417
 b d   c a 
(a + c)  +  + (b + d )  + ≥4
 c ( a + b ) a (c + d )   d (b + c ) b ( d + a ) 
 a+c b+d 
⇔ ( abc + abd + acd + bcd )  + ≥4
 ac (a + b)(c + d ) bd (b + c)(d + a ) 
 1 1 1 1 
 + + 
1 1 1 1
⇔  + + +  a c + b d ≥4
 a b c d   1 + 1  1 + 1   1 + 1  1 + 1  
  a b  c d   b c  d a  
     

AÙp duïng baát ñaúng thöùc AM-GM, ta coù

 1 1 1 1 
 + + 
 1 1 1 1 
 + + +  1 1 1 1 + 1 1 1 1 ≥
a c b d
 a b c d    +  +   +  +  
  a b  c d   b c  d a  
     
 1 1 1 1 
 + + 
 1 1 1 1  a c b d 
≥ 4 + + +  +

a b c d  1 1 1 1
2
1 1 1 1
2 
 + + +   + + +  
  a b c d  a b c d 
=4

⇒ ñpcm.
Baøi toaùn 112. (Voõ Quoác Baù Caån)
Chöùng minh raèng vôùi moïi soá thöïc döông a, b, c ta coù baát ñaúng thöùc

1 1 1 2 2 2 3 3 3
+ + + + + ≥ + +
3a 3b 3c a + 2b b + 2c c + 2a 2a + b 2b + c 2c + a
Lôøi giaûi.
Tröôùc heát, ta chöùng minh Boå ñeà sau
Boå ñeà. Vôùi moïi soá thöïc döông x, y , z ta coù baát ñaúng thöùc

x 3 + y 3 + z 3 + 2( xy 2 + yz 2 + zx 2 ) ≥ 3( x 2 y + y 2 z + z 2 x )

Chöùng minh.
Ta coù

418
x 3 + y 3 + z 3 + 2( xy 2 + yz 2 + zx 2 ) ≥ 3( x 2 y + y 2 z + z 2 x)
⇔ 3( x 3 + y 3 + z 3 ) + 6( xy 2 + yz 2 + zx 2 ) ≥ 9( x 2 y + y 2 z + z 2 x)
 
⇔ ∑ (2 x3 + y 3 − 3 x 2 y ) + 6  ∑ xy 2 − ∑ x 2 y  ≥ 0
cyc  cyc cyc 
⇔ ∑ ( x − y ) 2 (2 x + y ) + 6( x − y )( y − z)( z − x) ≥ 0
cyc

⇔ ∑ ( x − y ) 2 (2 x + y ) + 2∑ ( x − y )3 ≥ 0
cyc cyc

⇔ ∑ ( x − y ) 2 (4 x − y ) ≥ 0
cyc

Ñaët S x = 4 y − z , S y = 4 z − x, S z = 4 x − y

Baát ñaúng thöùc caàn chöùng minh trôû thaønh S x ( y − z) 2 + S y ( z − x) 2 + S z ( x − y ) 2 ≥ 0

Coù 2 tröôøng hôïp xaûy ra

* Tröôøng hôïp 1. x ≤ y ≤ z. Khi ñoù, ta coù S y ≥ 0 . Ta laïi coù

S y + S z = 4 z − y + 3x ≥ 0
S y + S x = 3z + 4 y − x ≥ 0

Chuù yù raèng ( z − x) 2 ≥ ( x − y )2 + ( y − z ) 2 neân ta coù

S x ( y − z) 2 + S y ( z − x)2 + S z ( x − y )2 ≥ ( S x + S y )( y − z )2 + ( S y + S z )( x − y ) 2 ≥ 0
* Tröôøng hôïp 2. x ≥ y ≥ z ⇒ S x , S z ≥ 0. Neáu S y ≥ 0 thì ta coù ngay ñpcm, do ñoù ta

chæ caàn xeùt tröôøng hôïp S y ≤ 0 laø ñuû.

+ Tröôøng hôïp 2.1. 2 y ≥ x + z ⇒ 2 y ≥ x. Khi ñoù, ta coù


S x + 2S y = 4 y − 2 x + 7 z ≥ 0
S z + 2S y = 2 x − y + 8 z ≥ 0

Maët khaùc theo baát ñaúng thöùc Bunhiacopxki thì ( z − x) 2 ≤ 2( x − y ) 2 + 2( y − z ) 2 .

Do ñoù

S x ( y − z) 2 + S y ( z − x)2 + S z ( x − y )2 ≥ ( S x + 2 S y )( y − z )2 + (2 S y + S z )( x − y ) 2 ≥ 0
+ Tröôøng hôïp 2.2. x + z ≥ 2 y ⇔ 2( x − y ) ≥ x − z ≥ 0.

419
+ Tröôøng hôïp 2.2.1. ( )
3 − 1 x + z ≥ 3 y ⇔ 3( x − y ) ≥ x − z ≥ 0. Khi ñoù,

ta coù
S z + 3S y = x − y + 12 z ≥ 0

Do ñoù

S x ( y − z) 2 + S y ( z − x) 2 + S z ( x − y ) 2 ≥ (3S y + S z )( x − y )2 ≥ 0

+ Tröôøng hôïp 2.2.2. ( )


3 −1 x + z ≤ 3y ⇔ y − z ≥ ( )
3 − 1 ( x − y ) ≥ 0.

Khi ñoù, ta coù

( ) ( ) ( )
2
Sx 3 − 1 + 4S y + S z = 15 − 8 3 y + 2 6 + 3 z ≥ 0

Do ñoù

S x ( y − z ) 2 + S y ( z − x )2 + S z ( x − y ) 2 ≥  S x ( )
3 − 1 + 4 S y + S z  ( x − y ) 2 ≥ 0
2

 
Toùm laïi, trong moïi tröôøng hôïp, ta luoân coù

S x ( y − z)2 + S y ( z − x) 2 + S z ( x − y )2 ≥ 0

Boå ñeà ñöôïc chöùng minh hoaøn toaøn.


Ñaúng thöùc xaûy ra khi vaø chæ khi x = y = z.

Trôû laïi baøi toaùn cuûa ta

AÙp duïng Boå ñeà treân vôùi x = t a , y = t b , z = t c (t > 0) , ta ñöôïc

t 3a + t 3b + t 3c + 2(t a +2 b + t b + 2c + t c + 2 a ) ≥ 3(t 2 a +b + t 2b +c + t 2 c + a ) ∀t > 0


⇒ t 3a −1 + t 3b−1 + t 3c −1 + 2(t a + 2b−1 + t b+ 2 c −1 + t c + 2 a −1 )
≥ 3(t 2 a +b −1 + t 2b +c −1 + t 2 c +a −1 ) ∀t > 0
1
⇒ ∫ (t 3a −1 + t 3b −1 + t 3c −1 + 2(t a +2b −1 + t b +2 c −1 + t c +2 a −1 ))dt
0
1
≥ ∫ 3(t 2 a +b −1 + t 2b +c −1 + t 2 c +a −1 )dt
0

420
1 1 1 2 2 2 3 3 3
⇔ + + + + + ≥ + +
3a 3b 3c a + 2b b + 2c c + 2a 2a + b 2b + c 2c + a
⇒ ñpcm.

Ñaúng thöùc xaûy ra khi vaø chæ khi a = b = c.

Baøi toaùn 113.


Chöùng minh raèng vôùi moïi soá thöïc döông a, b, c, d ta coù baát ñaúng thöùc

a −b b−c c−d d −a
+ + + ≥0
a + 2b + c b + 2c + d c + 2d + a d + 2a + b
Lôøi giaûi.
Ta coù baát ñaúng thöùc caàn chöùng minh töông ñöông vôùi
2a − 2b 2b − 2c 2c − 2d 2d − 2a
+ + + ≥0
a + 2b + c b + 2c + d c + 2d + a d + 2a + b
 2a − 2b   2b − 2c 
⇔ + 1 +  + 1 +
 a + 2b + c   b + 2c + d 
 2c − 2d   2d − 2a 
+ + 1 +  + 1 ≥ 4
 c + 2d + a   d + 2a + b 
3a + c 3b + d 3c + a 3d + b
⇔ + + + ≥4
a + 2b + c b + 2c + d c + 2d + a d + 2a + b
 a b c d 
⇔ 2 + + + +
 a + 2b + c b + 2c + d c + 2d + a d + 2a + b 
 a+c b+d c+a d +b 
+ + + + ≥4
 a + 2b + c b + 2c + d c + 2d + a d + 2a + b 
AÙp duïng baát ñaúng thöùc Bunhiacopxki, ta coù
a b c d
+ + + =
a + 2b + c b + 2c + d c + 2d + a d + 2a + b
a2 b2 c2 d2
= + + +
a( a + 2b + c ) b(b + 2c + d ) c (c + 2d + a ) d (d + 2a + b)
(a + b + c + d ) 2

a( a + 2b + c ) + b(b + 2c + d ) + c (c + 2d + a ) + d (d + 2a + b)
(a + b + c + d )2
=
(a + b + c + d )2
=1
421
Do ñoù

 a b c d 
2 + + + ≥2 (1)
 a + 2b + c b + 2c + d c + 2d + a d + 2a + b 
Maët khaùc, aùp duïng baát ñaúng thöùc AM-GM, ta laïi coù
a+c b+d c+a d +b
+ + + =
a + 2b + c b + 2c + d c + 2d + a d + 2a + b
 1 1   1 1 
= (a + c)  +  + (b + d )  + 
 a + 2b + c c + 2d + a   b + 2c + d d + 2a + b 
4 4
≥ ( a + c). + (b + d ).
( a + 2b + c) + (c + 2d + a ) (b + 2c + d ) + ( d + 2a + b)
=2 (2)

Töø (1) vaø (2), ta suy ra ñieàu phaûi chöùng minh.


Ñaúng thöùc xaûy ra khi vaø chæ khi a = c, b = d .

Baøi toaùn 114.


Cho caùc soá döông a, b, c. Chöùng minh raèng

a2 b2 c2
+ + ≥1
a 2 + ab + b 2 b 2 + bc + c 2 c 2 + ca + a 2
Lôøi giaûi.
Do caû 2 veá cuûa baát ñaúng thöùc ñaõ cho ñoàng baäc neân khoâng maát tính toång quaùt, ta
y z x
coù theå giaû söû abc = 1 . Ñaët a = ,b = ,c = ( x, y, z > 0) . Khi ñoù, baát ñaúng thöùc
x y z

caàn chöùng minh trôû thaønh

x4
∑ x 4 + x2 yz + y 2 z 2 ≥ 1
cyc

AÙp duïng baát ñaúng thöùc Bunhiacopxki, ta coù

x4 ( x 2 + y 2 + z 2 )2
∑ x 4 + x2 yz + y 2 z 2 x 4 + y 4 + z 4 + x2 y 2 + y 2 z 2 + z 2 x2 + xyz ( x + y + z )

cyc

Do ñoù, ñeå chöùng minh baát ñaúng thöùc ñaõ cho, ta chæ caàn chöùng minh

422
( x 2 + y 2 + z 2 ) 2 ≥ x 4 + y 4 + z 4 + x 2 y 2 + y 2 z 2 + z 2 x 2 + xyz ( x + y + z )
1
⇔ .∑ z 2 ( x − y ) 2 ≥ 0 (ñuùng)
2 cyc

⇒ ñpcm.
Ñaúng thöùc xaûy ra khi vaø chæ khi a = b = c.

Baøi toaùn 115.


Chöùng minh raèng vôùi moïi soá thöïc döông a, b, c ta coù baát ñaúng thöùc

a 2 − bc b2 − ca c 2 − ab
+ + ≥0
7 a + 2b + 2c
2 2 2
7b + 2c + 2a
2 2 2
7c + 2a + 2b
2 2 2

Lôøi giaûi.
Ta coù baát ñaúng thöùc caàn chöùng minh töông ñöông vôùi

2a 2 − 2bc
∑ ≥0
cyc 7 a 2 + 2b2 + 2c 2
(a − b)(a + c) − (c − a )(a + b)
⇔∑ ≥0
cyc 7 a 2 + 2b 2 + 2c 2
(a − b)(a + c) (c − a )(a + b)
⇔∑ −∑ ≥0
cyc 7 a + 2b + 2c
2 2 2
cyc 7 a 2 + 2b 2 + 2c 2
 a+c b+c 
⇔ ∑ ( a − b)  − ≥0
 7 a + 2b + 2c 7b 2 + 2c 2 + 2a 2 
2 2 2
cyc

⇔ ∑ Sc (a − b) 2 ≥ 0
cyc

Trong ñoù

(2(b + c )(b 2 + c 2 ) + 4a 3 + 4a (b 2 + c 2 ) − 3a 2 (b + c) − 10abc) 2b 2 + 2c 2 + 7 a 2


Sa =
(a + b) 7c 2 + 2a 2 + 2b 2 + (a + c) 7b 2 + 2c 2 + 2a 2
(2(c + a )(c 2 + a 2 ) + 4b3 + 4b(c 2 + a 2 ) − 3b 2 (c + a ) − 10abc) 2c 2 + 2a 2 + 7b 2
Sb =
(b + c) 7 a 2 + 2b 2 + 2c 2 + ( a + b) 7c 2 + 2a 2 + 2b 2
(2( a + b)( a 2 + b 2 ) + 4c 3 + 4c (a 2 + b 2 ) − 3c 2 (a + b) − 10abc) 2a 2 + 2b 2 + 7c 2
Sc =
( a + c ) 7b 2 + 2c 2 + 2a 2 + (b + c) 7a 2 + 2b 2 + 2c 2

423
AÙp duïng baát ñaúng thöùc AM-GM vaø baát ñaúng thöùc Bunhiacopxki, ta coù
2(a + b)(a 2 + b 2 ) + 4c3 + 4c (a 2 + b 2 ) − 3c 2 (a + b) − 10abc ≥
3 2 2
 a+b a+b 2a+b  a+b
≥ 8  + 4c + 8c 
3
 − 6c   − 10c  
 2   2   2   2 
  a + b 2 a+b 
= ( a + b + 2c )  4  − + 2

  2   
5c 2c
  2  
≥0
Do ñoù S c ≥ 0 . Töông töï, ta coù S a , Sb ≥ 0.

Vaäy

S a (b − c ) 2 + Sb (c − a ) 2 + Sc ( a − b) 2 ≥ 0 (ñpcm)

Baøi toaùn 116. (Voõ Quoác Baù Caån)


Chöùng minh raèng vôùi moïi soá thöïc döông a, b, c ta coù baát ñaúng thöùc

a b c
(a + b + c) 2  + +  ≥ 9(a 2 + b 2 + c 2 )
b c a
Lôøi giaûi.
Ta coù baát ñaúng thöùc caàn chöùng minh töông ñöông vôùi

a3 a 2b ab 2
∑ b + ∑ c + 3∑ ab + 2∑ a + 2∑ c ≥ 9∑ a 2
2

cyc cyc cyc cyc cyc cyc

a3 a 2b ab 2
⇔∑ +∑ + 2∑ ≥ 7∑ a 2 − 3∑ ab
cyc b cyc c cyc c cyc cyc

 a3 2  a 2b 
⇔ ∑  + ab − 2a  + ∑  + bc − 2ab  +
cyc  b  cyc  c 
 ab 2 
+ 2∑  + ac − 2ab  ≥ 5∑ a 2 − 5∑ ab
cyc  c  cyc cyc

a( a − b) 2 b(c − a) 2 a (b − c) 2 5
⇔∑ +∑ + 2∑ ≥ .∑ ( a − b)2
cyc b cyc c cyc c 2 cyc
⇔ ∑ S a (b − c )2 ≥ 0
cyc

424
Trong ñoù
b a 2a 5
Sa = + + −
c b c 2
c b 2b 5
Sb = + + −
a c a 2
a c 2c 5
Sc = + + −
b a b 2
Coù 2 tröôøng hôïp xaûy ra
+ Tröôøng hôïp 1. a ≥ b ≥ c > 0. Khi ñoù, ta coù S a ≥ 0.

+ Tröôøng hôïp 1.1. Sb ≥ 0 . Khi ñoù, ta seõ chöùng minh

Sb + Sc ≥ 0 (1)

Thaät vaäy
 a 2b   b 2c  2c
(*) ⇔  +  +  +  + ≥5
b a  c b  a
 a 2b   b 2c   b 2c 
⇔  + + + + + ≥ 5
 b a   2c b   2c a 
AÙp duïng baát ñaúng thöùc AM-GM, ta coù
b 2c
+ ≥2
2c b
 a 2b   b 2c  a 2b b
 b + a  +  2c + a  ≥ b + a + 2 a
   
 a 2b   a b b
= + + + + 
 2b a   2b a a
3
≥ 2+ 3
2

Do ñoù

 a 2b   b 2c   b 2c  3
 + + + + +  ≥ 4+ 3 > 5
 b a   2c b   2c a  2

Vaäy (1) ñuùng.


Do ñoù
425
S a (b − c ) 2 + Sb (c − a ) 2 + Sc ( a − b) 2 ≥ ( Sb + Sc )( a − b)2 ≥ 0

+ Tröôøng hôïp 1.2. Sb ≤ 0. Khi ñoù, ta seõ chöùng minh

S a + 2 Sb ≥ 0 (2)
Sc + 2Sb ≥ 0 (3)

Thaät vaäy, ta coù

 a 4b   a c  3b 15
S a + 2 Sb =  +  + 2  +  + −
b a  c a c 2
15
≥ 4+ 4+ 3−
2
>0

⇒ (2) ñuùng.

 a 4b   b c  3c 15
Sc + 2Sb =  +  + 2  +  + −
b a  c b a 2
15
≥ 4+ 4+0−
2
>0

⇒ (3) ñuùng.

Chuù yù raèng (a − c) 2 ≤ 2(a − b) 2 + 2(b − c) 2

Do ñoù

S a (b − c )2 + Sb (c − a ) 2 + Sc ( a − b)2 ≥ (2Sb + Sc )( a − b) 2 + ( Sa + 2Sb )(b − c ) 2 ≥ 0


+ Tröôøng hôïp 2. c ≥ b ≥ a > 0 . Khi ñoù, ta coù Sb > 0 . Theo (1), ta coù Sb + Sc ≥ 0

Ta seõ chöùng minh


S a + Sb ≥ 0 (4)

Thaät vaäy

 2b a   c 2a  2b
S a + Sb =  +  +  + + −5
 a b a c  c
≥ 3+ 2 2 + 0 −5
>0

426
⇒ (4) ñuùng.
Do ñoù

S a (b − c ) 2 + Sb (c − a ) 2 + Sc ( a − b) 2 ≥ ( Sb + Sc )( a − b)2 + ( S a + Sb )(b − c) 2 ≥ 0

Toùm laïi, trong moïi tröôøng hôïp, ta luoân coù

S a (b − c ) 2 + Sb (c − a ) 2 + Sc (a − b) 2 ≥ 0

⇒ ñpcm.
Ñaúng thöùc xaûy ra khi vaø chæ khi a = b = c.

Baøi toaùn 117.


Cho caùc soá thöïc döông a, b, c thoûa abc = 1 . Chöùng minh raèng

 1 1 1  16
4 + + 2 
≤1+
 a(1 + bc) b(1 + ca )
2 2
c(1 + ab)  (1 + ab)(1 + bc)(1 + ca)

Lôøi giaûi.
Ta coù baát ñaúng thöùc caàn chöùng minh töông ñöông vôùi
1 16
4∑ ≤1+
cyc a (1 + bc ) 2
(1 + ab)(1 + bc)(1 + ca )
a2 16abc
⇔ 4∑ ≤ 1 +
cyc a( a + abc) 2 (a + abc)(b + abc)(c + abc)
a 16
⇔ 4∑ ≤1+
cyc (a + 1) 2
( a + 1)(b + 1)(c + 1)

2 2 2
Ñaët x = − 1, y = − 1, z = − 1 thì ta coù
a +1 b +1 c +1
(1 − x )(1 − y )(1 − z ) = (1 + x)(1 + y )(1 + z ) ⇒ x + y + z + xyz = 0

Baát ñaúng thöùc caàn chöùng minh trôû thaønh


(1 − x )(1 + x) + (1 − y )(1 + y ) + (1 − z )(1 + z ) ≤ 1 + 2(1 + x )(1 + y )(1 + z )
⇔ x 2 + y 2 + z 2 + 2( xy + yz + zx ) + 2( x + y + z + xyz ) ≥ 0
⇔ ( x + y + z ) 2 ≥ 0 (ñuùng)

⇒ ñpcm.

427
Baøi toaùn 118. (Phaïm Vaên Thuaän)

Cho caùc soá khoâng aâm a, b, c thoûa a 2 + b 2 + c 2 = 1. Chöùng minh raèng

a3 b3 c3
+ + ≥ 2
b 2 − bc + c 2 c 2 − ca + a 2 a 2 − ab + b 2
Lôøi giaûi.
Ta coù baát ñaúng thöùc caàn chöùng minh töông ñöông vôùi
a3
∑ b2 − bc + c 2 ≥ 2
cyc

a3
⇔ ∑ 3 3 .(b + c) ≥ 2
cyc b + c

⎛ a3 ⎞
⇔ ∑ ⎜ 3 3 .(b + c) + b + c ⎟ ≥ 2( a + b + c) + 2
cyc ⎝ b + c ⎠
a 3 + b3 + c 3
⇔∑ .(b + c) ≥ 2(a + b + c) + 2
cyc b3 + c3
1
⇔ (a 3 + b3 + c3 ).∑ ≥ 2(a + b + c) + 2
cyc a − ab + b 2
2

AÙp duïng baát ñaúng thöùc AM-GM, ta coù


1 9
∑ a 2 − ab + b2 ≥ 2(a 2 + b2 + c 2 ) − ab − bc − ca
cyc

Do ñoù, ñeå chöùng minh baát ñaúng thöùc ñaõ cho, ta chæ caàn chöùng minh
9(a 3 + b3 + c3 )
≥ 2 + 2(a + b + c) (*)
2(a 2 + b 2 + c 2 ) − ab − bc − ca

Ñaët p = a + b + c, q = ab + bc + ca, r = abc ≥ 0 ⇒ 0 ≤ q ≤ 1, p = 1 + 2q .

Khi ñoù

(
(*) ⇔ 9( p (1 − q) + 3r ) ≥ 2 p + 2 (2 − q) )
⇔ 9 p − 9 pq + 27 r ≥ 4 p − 2 pq − 2q + 2 2
⇔ 5 p − 7 pq + 2q + 27r ≥ 2 2 (**)

427
Coù 2 tröôøng hôïp xaûy ra
+ Tröôøng hôïp 1. 2 q ≤ 1.

Khi ñoù

(**) ⇔ f (q ) = 5 2q + 1 − 7 q 2q + 1 + 2q + 27 r ≥ 2 2

Ta coù
5 7q
f / (q) = − 7 2q + 1 − + 2
2q + 1 2q + 1

2(2q + 1) − (21q + 2)
=
2q + 1

2(1 + 1) − 21q − 2

2q + 1
21q
=−
2q + 1
≤0

⇒ f (q ) laø haøm nghòch bieán.


⎛1⎞
⇒ f (q ) ≥ f ⎜ ⎟ = 2 2 + 27 r ≥ 2 2
⎝2⎠
⇒ (**) ñuùng.

+ Tröôøng hôïp 2. 2 q ≥ 1. Khi ñoù, theo baát ñaúng thöùc Schur, ta coù

4 pq − p 3 p (4q − p 2 ) p (2q − 1)
r≥ = = ≥0
9 9 9
Do ñoù, ñeå chöùng minh (**), ta chæ caàn chöùng minh

5 p − 7 pq + 2q + 3 p (2q − 1) ≥ 2 2
⇔ 2 p − pq + 2q ≥ 2 2
⇔ g ( q ) = 2 2q + 1 − q 2q + 1 + 2q ≥ 2 2

Ta coù
2 q
g / (q) = − 2q + 1 − + 2
2q + 1 2q + 1
428
2(2q + 1) − 3q + 1
=
2q + 1
2(1 + 1) − 3q + 1

2q + 1
3(1 − q)
=
2q + 1
≥ 0 (do q ≤ 1)

⇒ g (q ) laø haøm ñoàng bieán.


⎛1⎞
⇒ g (q) ≥ g ⎜ ⎟ = 2 2
⎝2⎠
⇒ (**) ñuùng.

Toùm laïi, trong moïi tröôøng hôïp, ta luoân coù


a3 b3 c3
+ + ≥ 2 (ñpcm)
b 2 − bc + c 2 c 2 − ca + a 2 a 2 − ab + b 2
⎛ 1 1 ⎞
Ñaúng thöùc xaûy ra khi vaø chæ khi (a, b, c) = ⎜ , ,0 ⎟ .
⎝ 2 2 ⎠

Baøi toaùn 119. (Belarus 1998)


Chöùng minh raèng vôùi moïi soá döông a, b, c ta coù baát ñaúng thöùc

a b c a+b b+c
+ + ≥ + +1
b c a b+c a+b
Lôøi giaûi.
+ Caùch 1.
Ta coù baát ñaúng thöùc caàn chöùng minh töông ñöông vôùi
ab 2 + bc 2 + ca 2 − 3abc a + b b + c
≥ + −2
abc b+c a+b
⎛1 1 1 ⎞
∑ (a − b)2 ⎜⎝ 6 .a − 6 .b + 2 .c ⎟⎠ (c − a ) 2
⇔ ≥
cyc

abc (a + b)(b + c)

429
⎛ a − b + 3c ⎞ 2 ⎛ b − c + 3a ⎞
⇔ ( a − b) 2 ⎜ ⎟ + (b − c) ⎜ ⎟+
⎝ abc ⎠ ⎝ abc ⎠
⎛ c − a + 3b 6 ⎞
+ (c − a ) 2 ⎜ − ⎟≥0
⎝ abc (a + b)(b + c ) ⎠

Ñaët
b − c + 3a
Sa =
abc
c − a + 3b 6
Sb = −
abc (a + b)(b + c)
a − b + 3c
Sc =
abc
Baát ñaúng thöùc caàn chöùng minh trôû thaønh S a (b − c) 2 + Sb (c − a ) 2 + Sc (a − b) 2 ≥ 0

Coù 2 tröôøng hôïp xaûy ra


+ Tröôøng hôïp 1. a ≥ c > 0.

+ Tröôøng hôïp 1.1. a ≥ b ≥ c > 0. Khi ñoù, ta coù Sa , Sc ≥ 0.

+ Tröôøng hôïp 1.1.1. b + c ≥ a . Khi ñoù, ta coù


c − a + 3b 6
Sb = −
abc (a + b)(b + c)
(b + c − a ) + 2b 6
= −
abc (a + b)(b + c)
2 6
≥ −
ac (a + b)(b + c)
2(ab + b(b + c) − 2ac)
= ≥0
ac(a + b)(b + c)

Do ñoù

S a (b − c ) 2 + S b ( c − a ) 2 + S c ( a − b ) 2 ≥ 0

+ Tröôøng hôïp 1.1.2. a ≥ b + c .


a b c a+b b+c
Khi ñoù, xeùt haøm soá f (a ) = + + − − − 1 vôùi a ≥ b + c
b c a b+c a+b
Ta coù
430
1 c 1 b+c
f / (a) = − 2− +
b a b + c ( a + b) 2
c c b+c
= − 2+
b(b + c) a ( a + b) 2
>0
⇒ f (a ) laø haøm ñoàng bieán.
b+c b c 2b + c b + c
⇒ f (a ) ≥ f (b + c) = + + − − −1
b c b + c b + c 2b + c
b c 2b b+c
= + − −
c b b + c 2b + c
Ta laïi coù
f (b + c) > 0 (*)

Thaät vaäy
(*) ⇔ (b 2 + c 2 )(b + c)(2b + c) − 2b 2c(2b + c) − bc(b + c) 2 > 0
⇔ (b − c) 2 (2b + c) + b 2c(b − c) + 2bc3 > 0 (ñuùng do b ≥ c > 0)

⇒ (*) ñuùng.
⇒ f (a) > 0

a b c a+b b+c
⇒ + + > + +1
b c a b+c a+b
+ Tröôøng hôïp 1.2. a ≥ c ≥ b > 0. Khi ñoù, ta coù Sa , Sc ≥ 0.

Ta coù
2b + 4c 6
Sb + S c = −
abc (a + b)(b + c)
4c 6
≥ −
abc (a + b)(b + c)
2(2( a + b)(b + c) − 3ab)
=
ab(a + b)(b + c)
2(2b 2 + 2bc + 2ac − ab)
=
ab( a + b)(b + c)
≥0

431
Do a ≥ c ≥ b > 0 neân (a − b)2 ≥ (a − c)2 . Do ñoù

S a (b − c) 2 + Sb (c − a ) 2 + Sc ( a − b) 2 ≥ (c − a ) 2 ( Sb + Sc ) ≥ 0

+ Tröôøng hôïp 1.3. b ≥ a ≥ c > 0. Khi ñoù, ta coù


b − c + 3a
Sa = ≥0
abc
c − a + 3b 6
Sb = −
abc (a + b)(b + c)
2 6
≥ −
ac ( a + b)(b + c)
2(b 2 + ab + bc − 2ac)
=
ac(a + b)(b + c)
≥0
4 a + 2c
S a + Sc = ≥0
abc

Do b ≥ a ≥ c > 0 neân (b − c)2 ≥ (a − b)2 . Do ñoù

S a (b − c) 2 + Sb (c − a ) 2 + Sc ( a − b) 2 ≥ (a − b) 2 ( S a + Sc ) ≥ 0

+ Tröôøng hôïp 2. c ≥ a > 0.

+ Tröôøng hôïp 2.1. c ≥ b ≥ a > 0. Khi ñoù, ta coù


a − b + 3c
Sc = ≥0
abc
c − a + 3b 6
Sb = −
abc (a + b)(b + c)
c + 2b 6
≥ −
abc (a + b)(b + c)
3 6
≥ −
ac ( a + b)(b + c)
3(b 2 + ab + bc − ac)
=
ac( a + b)(b + c)
≥0

432
2a + 4b 6
S a + Sb = −
abc ( a + b)(b + c)
3 6
≥ −
ac (a + b)(b + c)
3(b 2 + ab + bc − ac)
= ≥0
ac(a + b)(b + c)

Do c ≥ b ≥ a > 0 neân (c − a)2 ≥ (b − c)2 . Do ñoù

S a (b − c) 2 + Sb (c − a ) 2 + Sc ( a − b) 2 ≥ (b − c) 2 ( S a + Sb ) ≥ 0

+ Tröôøng hôïp 2.2. c ≥ a ≥ b > 0. Khi ñoù, ta coù Sc ≥ 0 .

+ Tröôøng hôïp 2.2.1. c ≥ a + b > 0.


a b c a+b b+c
Khi ñoù, xeùt haøm soá g (c) = + + − − − 1 vôùi c ≥ a + b
b c a b+c a+b
Ta coù
1 b 1 a+b
g / (c ) = − 2− +
a c a + b (b + c) 2
b b a+b
= − 2+
a ( a + b) c (b + c) 2
>0
⇒ g (c) laø haøm ñoàng bieán.
a b a + b a + b a + 2b
⇒ g (c ) ≥ g ( a + b ) = + + − − −1
b a+b a a + 2b a + b
a b 2a + 3b
= + −
b a a + 2b
Ta laïi coù
g ( a + b) > 0 (**)

Thaät vaäy
(**) ⇔ ( a 2 + b 2 )(a + 2b) − ab(2a + 3b) > 0
⇔ 2b3 − 2b 2 a + a 3 > 0 (ñuùng)

⇒ (**) ñuùng.

433
a b c a+b b+c
⇒ + + > + +1
b c a b+c a+b

+ Tröôøng hôïp 2.2.2. a + b ≥ c. Khi ñoù, ta coù S a ≥ 0.

Ta coù
2a + 4b 6
S a + Sb = −
abc (a + b)(b + c)
2c 6
≥ −
abc (a + b)(b + c)
2(( a + b)(b + c) − 3ab)
=
ab(a + b)(b + c)
2(ac + bc + b 2 − 2ab)
= ≥0
ab(a + b)(b + c)

Do c ≥ a ≥ b > 0 neân (b − c)2 ≥ (c − a)2 . Do ñoù

S a (b − c) 2 + Sb (c − a ) 2 + Sc ( a − b) 2 ≥ (c − a ) 2 ( S a + Sb ) ≥ 0

+ Tröôøng hôïp 2.3. b ≥ c ≥ a > 0. Khi ñoù, ta coù


b − c + 3a
Sa = ≥0
abc
c − a + 3b 6
Sb = −
abc (a + b)(b + c)
3 6
≥ −
ac ( a + a )(c + c)
3
=
2ac
>0

+ Tröôøng hôïp 2.3.1. c + a ≥ b > 0. Khi ñoù, ta coù Sb ≥ 0. Do ñoù

S a (b − c ) 2 + S b ( c − a ) 2 + S c ( a − b ) 2 ≥ 0

+ Tröôøng hôïp 2.3.2. b ≥ c + a.


a b c a+b b+c
Khi ñoù, xeùt haøm soá h(b) = + + − − − 1 vôùi b ≥ c + a .
b c a b+c a+b
Ta coù
434
b 2 − ac ⎛ 1 1 ⎞
h (b ) =
/
+ ( c − a ) ⎜ − ⎟≥0
b 2c ⎝ (b + a ) (b + c) 2 ⎠
2

⇒ h(b) laø haøm ñoàng bieán.


a a + c c 2a + c a + 2c
⇒ h(b) ≥ h(a + c) = + + − − −1
a+c c a a + 2c 2 a + c
a a c 2 a + c a + 2c
= + + − −
a + c c a a + 2c 2a + c
a ⎛a c ⎞ ⎛ 2 a + c a + 2c ⎞
= + ⎜ + − 2⎟ − ⎜ + − 2⎟
a+c ⎝c a ⎠ ⎝ a + 2c 2a + c ⎠
a (c − a ) 2 (c − a ) 2
= + −
a+c ca (a + 2c)(2a + c)
a ⎛ 1 1 ⎞
= + (c − a ) 2 ⎜ − ⎟≥0
a+c ⎝ ca (2 a + c )( a + 2 c ) ⎠

Toùm laïi, trong moïi tröôøng hôïp, ta luoân coù


a b c a+b b+c
+ + ≥ + +1
b c a b+c a+b
Ñaúng thöùc xaûy ra khi vaø chæ khi a = b = c.
+ Caùch 2.
Ta coù baát ñaúng thöùc caàn chöùng minh töông ñöông vôùi
a (a + b)(b + c) b(a + b)(b + c) c( a + b)(b + c)
+ + ≥
b c a
≥ (a + b) 2 + (b + c) 2 + (a + b)(b + c)
a 2c b 2 ( a + b) bc(b + c)
⇔ + a 2 + ab + ac + + b 2 + ab + c 2 + bc + ≥
b c a
≥ a 2 + ac + c 2 + 3b 2 + 3ab + 3bc
a 2c b 2 (a + b) bc(b + c)
⇔ + + ≥ ab + 2bc + 2b 2
b c a
AÙp duïng baát ñaúng thöùc AM-GM, ta coù

1 ⎛ a 2c b3 ⎞ 1 ⎛ a 2c bc 2 ⎞ 1 ⎛ b3 bc 2 ⎞ 2 ⎛ a c ⎞
.⎜ + ⎟ + .⎜ + ⎟ + .⎜ + ⎟+b ⎜ + ⎟≥
2 ⎝ b c ⎠ 2 ⎝ b a ⎠ 2 ⎝ c a ⎠ ⎝c a⎠

435
⎛ b 4c ⎞
≥ ab + ⎜ ac +
3
⎟ + 2b 2
⎜ a ⎟⎠

≥ ab + 2bc + 2b 2
a 2c b 2 ( a + b) bc(b + c)
⇒ + + ≥ ab + 2bc + 2b 2
b c a
⇒ ñpcm.

Ñaúng thöùc xaûy ra khi vaø chæ khi a = b = c.


Baøi toaùn 120.
Chöùng minh raèng vôùi moïi soá thöïc khoâng aâm a, b, c ta coù baát ñaúng thöùc

1 1 1 3
+ 2 + 2 ≥
a − ab + b
2 2
b − bc + c 2
c − ca + a 2
ab + bc + ca
Lôøi giaûi.
Khoâng maát tính toång quaùt, ta coù theå giaû söû c = min{a, b, c}. Khi ñoù, ta coù

⎧ 1 1
⎧⎪0 ≤ b − bc + c ≤ b
2 2 ⎪⎪ b − bc + c
2 2 2

b2
⎨ ⇒⎨
⎩⎪0 ≤ c − ca + a ≤ a
2 2 2 1 1
⎪ ≥ 2
⎪⎩ c − ca + a
2 2
a
Do ñoù
1 1 1 3
+ 2 + 2 − ≥
a − ab + b
2 2
b − bc + c 2
c − ca + a 2
ab + bc + ca
1 1 1 3
≥ 2 + 2+ 2−
a − ab + b 2
b c ab + bc + ca
1 1 1 3
≥ + + −
a 2 − ab + b 2 b 2 a 2 ab
( a − b) 4
= 2 2 2
a b (a − ab + b 2 )
≥0
⇒ ñpcm.
Ñaúng thöùc xaûy ra khi vaø chæ khi (a, b, c) = (t , t ,0) (t > 0).

436
Baøi toaùn 121.
Tìm k lôùn nhaát sao cho vôùi moïi soá khoâng aâm a, b, c (( a + b)(b + c)(c + a ) > 0) ta

coù baát ñaúng thöùc


a b c ⎛ 1 1 1 ⎞
+ 2 + 2 ≥ k⎜ + + ⎟
b +c c +a a +b ⎝a+b b+c c+a⎠
2 2 2 2

Lôøi giaûi.
4
Cho a = b = 1, c = 0 ta suy ra ñöôïc k ≤ . Ta chöùng minh ñaây laø giaù trò caàn tìm, töùc
5
laø chöùng minh
a b c 4 ⎛ 1 1 1 ⎞
+ 2 + 2 ≥ .⎜ + + ⎟
b +c
2 2
c +a 2
a +b 2
5 ⎝a+b b+c c+a⎠

+ Caùch 1.
Do 2 veá cuûa baát ñaúng thöùc treân ñoàng baäc neân khoâng maát tính toång quaùt, ta coù theå
1 1
giaû söû a + b + c = 1. Ñaët q = ab + bc + ca, r = abc ⇒ ≥ q > 0, ≥ r ≥ 0. Khi ñoù,
3 27
ta coù

a
∑ a(a 2 + b2 )(a 2 + c 2 )
∑ b2 + c 2 = (a 2cyc+ b2 )(b2 + c 2 )(c 2 + a 2 )
cyc

∑ a (a 2 (a 2 + b 2 + c 2 ) + b 2c 2 )
=
cyc

(a + b + c 2 )(a 2b 2 + b 2c 2 + c 2 a 2 ) − a 2b 2c 2
2 2

(a 3 + b3 + c3 )(a 2 + b 2 + c 2 ) + abc(ab + bc + ca )
=
(1 − 2q )(q 2 − 2r ) − r 2
(3r + 1 − 3q)(1 − 2q) + qr
= 2
−r − 2r (1 − 2q ) + q 2 (1 − 2q )
(3 − 5q )r + (1 − 2q )(1 − 3q )
= 2
−r − 2r (1 − 2q ) + q 2 (1 − 2q)

1
∑ (a + b)(a + c) ∑ (a(a + b + c) + bc) q +1
∑ a + b = (a + b)(b + c)(c + a) = (a + bcyc+ c)(ab + bc + ca) − abc = q − r
cyc

cyc

437
Do ñoù, baát ñaúng thöùc caàn chöùng minh trôû thaønh
(3 − 5q )r + (1 − 2q )(1 − 3q ) 4 1+ q
≥ .
− r − 2r (1 − 2q ) + q (1 − 2q ) 5 − r + q
2 2

⇔ f (r ) = (29q − 11)r 2 + (3 + 32q − 71q 2 ) r + q(1 − 2q)(5 + q )(1 − 4q ) ≥ 0

Ta coù
f / (r ) = 2(29q − 11)r + 3 + 32q − 71q 2
1
≥ 2(29q − 11). + 3 + 32q − 71q 2
27
59 922 1
= + .q − 71q 2 ≥ 0 (do 0 ≤ q ≤ )
27 27 3
⇒ f (r ) laø haøm ñoàng bieán.

+ Neáu 1 ≥ 4q thì ta coù f (r ) ≥ f (0) = q(1 − 2q)(5 + q)(1 − 4q) ≥ 0

4q − 1
+ Neáu 4q ≥ 1 thì theo baát ñaúng thöùc Schur, ta coù r ≥ ≥ 0. Do ñoù
9

⎛ 4q − 1 ⎞ 2(4q − 1)(81q + 103q − 95q + 19)


3 2
f (r ) ≥ f ⎜ ⎟ = ≥0
⎝ 9 ⎠ 81

Toùm laïi, trong moïi tröôøng hôïp, ta luoân coù f (r ) ≥ 0 ⇒ ñpcm.

4
Vaäy kmax = .
5
+ Caùch 2.
Ta coù baát ñaúng thöùc caàn chöùng minh töông ñöông vôùi
5a 4
∑ b2 + c2 ≥ ∑ b + c
cyc cyc

5a(a + b + c) 4( a + b + c)
⇔∑ ≥ ∑
cyc b2 + c2 cyc b+c

10a 2 + 10a (b + c) 8a
⇔∑ ≥ 24 + ∑
cyc b2 + c2 cyc b + c

438
⎛ a2 a ⎞ ⎛ 2a 2 + bc 9 ⎞
⇔ 8⎜ ∑ 2
⎜ cyc b + c 2 ∑ ⎟⎟ + ⎜⎜ ∑ 2
− − ⎟+
⎝ cyc b + c ⎠ ⎝ cyc b + c 2
2 ⎟⎠
⎛ a (b + c) − bc 3 ⎞ ⎛ a (b + c) ⎞
+⎜∑ − ⎟ + 9⎜ ∑ 2 − 2⎟ ≥ 0
⎜ cyc b + c
2 2
2 ⎟⎠ ⎜⎝ cyc b + c 2 ⎟
⎝ ⎠

Do ñoù, ñeå chöùng minh baát ñaúng thöùc ñaõ cho, ta chæ caàn chöùng minh
a2 a
∑ b2 + c2 ≥ ∑ b + c (1)
cyc cyc

2a 2 + bc 9
∑ b2 + c2 ≥ 2 (2)
cyc

a (b + c) − bc 3
∑ b2 + c2

2
(3)
cyc

a (b + c)
∑ b2 + c2 ≥2 (4)
cyc

* Chöùng minh (1).


Ta coù
a2 a
∑ b2 + c2 ≥ ∑ b + c
cyc cyc

⎛ a2 a ⎞
⇔ ∑⎜ 2 2 − ⎟≥0
cyc ⎝ b + c b + c ⎠
ab(a − b) − ca (c − a )
⇔∑ ≥0
cyc (b 2 + c 2 )(b + c)
ab(a − b) ca (c − a )
⇔∑ −∑ 2 2 ≥0
cyc (b + c )(b + c) cyc (b + c )(b + c)
2 2

ab(a − b) ab(a − b)
⇔∑ −∑ 2 2 ≥0
cyc (b + c )(b + c) cyc (a + c )(a + c)
2 2

ab(a − b) 2
⇔ (a 2 + b 2 + c 2 + ab + bc + ca ).∑ ≥ 0 (ñuùng)
cyc (a 2 + c 2 )(b 2 + c 2 )(a + c)(b + c)

* Chöùng minh (2).


Ta coù

439
2a 2 + bc 9
∑ b2 + c2 ≥ 2
cyc

⎛ 4q 2 + 2bc ⎞
⇔ ∑ ⎜ 2 2 − 3⎟ ≥ 0
cyc ⎝ b + c ⎠

4a 2 − 3(b 2 + c 2 ) + 2bc
⇔∑ ≥0
cyc b2 + c2
(2a + 3b − c)( a − b) − (2a − b + 3c)(c − a)
⇔∑ ≥0
cyc b2 + c2
(2a + 3b − c)( a − b) (2a − b + 3c)(c − a )
⇔∑ −∑ ≥0
cyc b +c
2 2
cyc b 2
+ c 2

(2a + 3b − c)( a − b) (3a + 2b − c)(a − b)


⇔∑ −∑ ≥0
cyc b +c
2 2
cyc a2 + c2
( a − b) 2 (2a 2 + 2b 2 − c 2 − c(a + b) + 3ab)
⇔∑ ≥0
cyc (b 2 + c 2 )(a 2 + c 2 )
⇔ ∑ (a − b) 2 (2a 2 + 2b 2 − c 2 − c(a + b) + 3ab)(a 2 + b 2 ) ≥ 0
cyc

⇔ ∑ (a − b) 2 (2a 2 + 2b 2 − 2c 2 + 2ab + (c 2 − c(a + b) + ab))(a 2 + b 2 ) ≥ 0


cyc

⇔ 2∑ ( a − b) 2 (a 2 + b 2 − c 2 )(a 2 + b 2 ) + 2∑ ab(a − b) 2 (a 2 + b 2 ) −
cyc cyc

− (a − b)(b − c)(c − a).∑ (a − b)(a 2 + b 2 ) ≥ 0


cyc

⇔ 2∑ ( a − b) 2 (a 2 + b 2 − c 2 )(a 2 + b 2 ) + 2∑ ab(a − b) 2 (a 2 + b 2 ) −
cyc cyc

− ( a − b) 2 (b − c) 2 (c − a ) 2 ≥ 0

Khoâng maát tính toång quaùt, ta coù theå giaû söû a ≥ b ≥ c ≥ 0. Khi ñoù, ta coù

∑ (a − b)2 (a 2 + b2 − c 2 )(a 2 + b2 ) ≥
cyc

≥ (b − c) 2 (b 2 + c 2 − a 2 )(b 2 + c 2 ) + (c − a) 2 (c 2 + a 2 − b 2 )(c 2 + a 2 )
≥ (b − c) 2 (b 2 + c 2 − a 2 )(b 2 + c 2 ) + (b − c) 2 (c 2 + a 2 − b 2 )(b 2 + c 2 )
= 2c 2 (b − c) 2 (b 2 + c 2 )
≥0

440
2∑ ab(a − b) 2 (a 2 + b 2 ) ≥ 2ab(a − b) 2 (a 2 + b 2 ) ≥ 4(a − b) 2 a 2b 2
cyc

≥ 4(a − b) 2 (b − c) 2 (c − a ) 2

⇒ 2∑ (a − b) 2 (a 2 + b 2 − c 2 )(a 2 + b 2 ) + 2∑ ab(a − b) 2 (a 2 + b 2 ) −
cyc cyc

− (a − b) 2 (b − c) 2 (c − a) 2 ≥ 0

* Chöùng minh (3).


Ta coù
a (b + c) − bc 3
∑ b2 + c2

2
cyc

⎛ 2a(b + c) − 2bc ⎞
⇔ ∑⎜ − 1⎟ ≥ 0
cyc ⎝ b2 + c2 ⎠
−(b 2 + c 2 ) + 2a (b + c) − 2bc
⇔∑ ≥0
cyc b2 + c 2
(b + c)(a − b) − (b + c)(c − a )
⇔∑ ≥0
cyc b2 + c2
(b + c)( a − b) (b + c)(c − a)
⇔∑ −∑ ≥0
cyc b +c
2 2
cyc b 2
+ c 2

(b + c)(a − b) (a + c)(a − b)
⇔∑ −∑ ≥0
cyc b +c
2 2
cyc a2 + c2
(a − b) 2 (−c 2 + c(a + b) + ab)
⇔∑ ≥0
cyc (b 2 + c 2 )(a 2 + c 2 )
⇔ ∑ (a − b) 2 (−c 2 + c(a + b) + ab)(a 2 + b 2 ) ≥ 0
cyc

⇔ ∑ (a − b) 2 ((−c 2 + c(a + b) − ab) + 2ab)(a 2 + b 2 ) ≥ 0


cyc

⇔ 2∑ ab(a − b) 2 (a 2 + b 2 ) + (a − b)(b − c)(c − a ).∑ (a − b)(a 2 + b 2 ) ≥ 0


cyc cyc

⇔ 2∑ ab(a − b) 2 (a 2 + b 2 ) + (a − b) 2 (b − c)2 (c − a ) 2 ≥ 0 (ñuùng)


cyc

* Chöùng minh (4).


Ta coù

441
a (b + c)
∑ ab(a − b)2 (a 2 + b2 + 2c 2 ) + 8a 2b2c 2
∑ b2 + c2 −2=
cyc

(a 2 + b 2 )(b 2 + c 2 )(c 2 + a 2 )
≥0
cyc

a (b + c)
⇒∑ ≥2
cyc b2 + c2

Vaäy (1), (2), (3) vaø (4) ñuùng. Töø ñaây, ta suy ra ñpcm.
Vaäy
4
kmax = .
5
* Caùch 3.
AÙp duïng baát ñaúng Bunhiacopxki, ta coù

a (a + b + c) 2
∑ b2 + c2 ≥
cyc ∑ ab(a + b)
cyc

Do ñoù, ñeå chöùng minh baát ñaúng thöùc ñaõ cho, ta chæ caàn chöùng minh

(a + b + c) 2 4 ⎛ 1 1 1 ⎞
≥ .⎜ + +
∑ ab(a + b) 5 ⎝ a + b b + c c + a ⎟⎠
cyc

( a + b + c) 2 4 a 2 + b 2 + c 2 + 3(ab + bc + ca )
⇔ ≥ .
∑ ab ( a + b ) 5 ∑ ab(a + b) + 2abc
cyc cyc

5( S + 2 P ) 4( S + 3P )
⇔ ≥
Q Q + 2abc
⇔ SQ + 10abcS + 20abcP ≥ 2 PQ

Trong ñoù S = a 2 + b 2 + c 2 , P = ab + bc + ca, Q = ∑ ab(a + b).


cyc

Deã thaáy
PQ = ∑ a 2b 2 (a + b) + 2abc( S + P)
cyc

SQ ≥ ∑ ab(a 2 + b 2 )(a + b) ≥ 2∑ a 2b 2 (a + b)
cyc cyc

Töø ñaây, ta coù ngay ñpcm.


442
Vaäy
4
kmax = .
5

Baøi toaùn 122. (Vasile Cirtoaje)


Chöùng minh raèng vôùi moïi soá khoâng a, b, c ta coù baát ñaúng thöùc

a3 b3 c3 1
+ + ≤
(2a + b )(2a + c ) (2b + c )(2b + a ) (2c + a )(2c + b ) a + b + c
2 2 2 2 2 2 2 2 2 2 2 2

Lôøi giaûi.
AÙp duïng baát ñaúng thöùc Bunhiacopxki, ta coù
(2a 2 + b 2 )(2a 2 + c 2 ) = (a 2 + a 2 + b 2 )(a 2 + c 2 + a 2 )
≥ ( a 2 + ac + ab) 2
= a 2 ( a + b + c) 2

a3 a
⇒ ≤
(2a + b )(2a + c ) (a + b + c) 2
2 2 2 2

Töông töï, ta coù


b3 b

(2b + c )(2b + a ) (a + b + c) 2
2 2 2 2

c3 c

(2c + a )(2c + b ) (a + b + c) 2
2 2 2 2

Do ñoù
a3 b3 c3 1
+ + ≤
(2a + b )(2a + c ) (2b + c )(2b + a ) (2c + a )(2c + b ) a + b + c
2 2 2 2 2 2 2 2 2 2 2 2

⇒ ñpcm.
Ñaúng thöùc xaûy ra khi vaø chæ khi a = b = c.

443
Baøi toaùn 123. (Phaïm Kim Huøng)
Chöùng minh raèng vôùi moïi daõy soá döông a1 , a2 ,..., an ta coù baát ñaúng thöùc

1 1 1 ⎛1 1 1⎞
+ + ... + ≤ 2 ⎜ + + ... + ⎟
a1 a1 + a2 a1 + a2 + ... + an ⎝ a1 a2 an ⎠

Lôøi giaûi.
Neáu n = 1 thì baát ñaúng thöùc treân hieån nhieân ñuùng.
Xeùt n ≥ 2 .
AÙp duïng baát ñaúng thöùc Bunhiacopxki, ta coù vôùi moïi soá döông x1 , x2 ,..., xn thì

⎛ x12 x22 xk2 ⎞


(a1 + a2 + ... + ak ) ⎜ + + ... + ⎟ ≥ ( x1 + x2 + ... + xk ) 2
⎝ a1 a2 ak ⎠
1 1 ⎛ x12 x22 xk2 ⎞
⇒ ≤ . ⎜ + + ... + ⎟
a1 + a2 + ... + ak ( x1 + x2 + ... + xk ) 2 ⎝ a1 a2 ak ⎠

Cho k chaïy töø 1 ñeán n roài coäng caùc baát ñaúng thöùc treân laïi, ta ñöôïc
1 1 1 c c c
+ + ... + ≤ 1 + 2 + ... + n
a1 a1 + a2 a1 + a2 + ... + an a1 a2 an

Trong ñoù
xk2 xk2 xk2
ck = + + ... + ∀k = 1, n
( x1 + x2 + ... + xk ) 2 ( x1 + x2 + ... + xk +1 ) 2 ( x1 + x2 + ... + xn ) 2
Ta coù theå choïn xk = k ∀k = 1, n. Khi ñoù ∀k ≥ 2 , ta coù

⎛ 1 1 1 ⎞
ck = k 2 ⎜ + + ... + 2 ⎟
⎝ (1 + 2 + ... + k ) (1 + 2 + ... + (k + 1)) (1 + 2 + ... + n) ⎠
2 2

⎛ 4 4 4 ⎞
= k2 ⎜ 2 + + ... + 2 2 ⎟
⎝ k (k + 1) (k + 1) (k + 2) n ( n + 1) ⎠
2 2 2

⎛ 4 4 4 ⎞
≤ k2 ⎜ 2 + + ... + ⎟
⎝ k (k + 1) k 2 (k + 2) 2 k 2 (n + 1) 2 ⎠
2

⎛ 1 1 1 ⎞
≤ 4⎜ + + ... + ⎟
⎝ (k + 1) (k + 2) (n + 1) 2 ⎠
2 2

444
⎛ 1 1 1 ⎞
≤ 4⎜ + + ... + ⎟
⎝ k (k + 1) (k + 1)( k + 2) n( n + 1) ⎠

⎛⎛ 1 1 ⎞ ⎛ 1 1 ⎞ ⎛1 1 ⎞⎞
= 4⎜ ⎜ − ⎟+⎜ − ⎟ + ... + ⎜ − ⎟⎟
⎝⎝ k k +1⎠ ⎝ k +1 k + 2 ⎠ ⎝ n n +1⎠⎠
⎛1 1 ⎞ 4
= 4⎜ − ⎟≤ ≤2
⎝ k n +1⎠ k
Ngoaøi ra
1 1
c1 = 1 + + ... +
(1 + 2) 2 (1 + 2 + ... + n) 2
4 4
= 1 + 2 2 + ... + 2
2 .3 n .( n + 1) 2
4 4
≤ 1 + 2 2 + ... + 2
2 .3 2 .(n + 1) 2
1 1
= 1 + 2 + ... +
3 (n + 1) 2
1 1
≤ 1+ + ... +
2.3 n(n + 1)
⎛1 1⎞ ⎛1 1 ⎞
= 1 + ⎜ − ⎟ + ... + ⎜ − ⎟
⎝ 2 3⎠ ⎝ n n +1⎠
1 1
= 1+ −
2 n +1
<2

Do ñoù

ck ≤ 2 ∀k = 1, n

Töø ñaây, ta suy ra ñieàu phaûi chöùng minh.


Baøi toaùn 124. (Phaïm Vaên Thuaän, Voõ Quoác Baù Caån)

Cho caùc soá khoâng aâm a, b, c thoûa a 2 + b 2 + c 2 = 1. Tìm giaù trò lôùn nhaát cuûa bieåu

thöùc
P = (a − b)(b − c)(c − a)(a + b + c)

Lôøi giaûi.

445
Tröôùc heát, ta xeùt tröôøng hôïp a = 0 . Khi ñoù, baøi toaùn chuyeån veà

“Caùc soá khoâng aâm b, c thoûa b 2 + c 2 = 1. Tìm giaù trò lôùn nhaát cuûa bieåu thöùc

Q = bc(c − b)(c + b) = bc(c 2 − b 2 ) .”


1
Khoâng maát tính, toång quaùt ta chæ caàn xeùt c ≥ b laø ñuû ⇒ c 2 ≥ .
2
Ta coù

Q 2 = b 2c 2 (c 2 − b2 )2 = c 2 (1 − c 2 )(2c 2 − 1)2 = m(1 − m)(2m − 1)2 = f (m)


1
Trong ñoù m = c 2 ≥ .
2
Ta coù
f / (m) = (1 − 2m)(8m 2 − 8m + 1)
2+ 2 1
f / ( m) = 0 ⇔ m = (do m ≥ )
4 2

2+ 2
Qua thì f / (m) ñoåi daáu töø döông sang aâm neân
4
⎛2+ 2 ⎞ 1 1
f ( m) ≤ f ⎜ ⎟= ∀m ≥
⎝ 4 ⎠ 16 2

1
⇒Q≤ .
4

2− 2 2+ 2
Ñaúng thöùc xaûy ra khi vaø chæ khi b = ,c = .
2 2
Vaäy
1
max Q = .
4
Trôû laïi baøi toaùn cuûa ta
Khoâng maát tính toång quaùt, ta coù theå giaû söû 0 ≤ a ≤ b ≤ c.
1
Ta seõ chöùng minh max P = , töùc laø chöùng minh
4

446
(a 2 + b 2 + c 2 )2
F (a, b, c) = ≥4
(a − b)(b − c)(c − a)(a + b + c)

Khi ñoù, vôùi moïi 0 ≤ t ≤ min{a, b, c} , ñaët x = a − t , y = b − t , z = c − t , ta coù

(( x + t ) 2 + ( y + t ) 2 + ( z + t ) 2 ) 2
F ( a , b, c ) =
( x − y )( y − z )( z − t )( x + y + z + 3t )
(3t 2 + 2( x + y + z )t + x 2 + y 2 + z 2 ) 2
=
( x − y )( y − z )( z − t )( x + y + z + 3t )
(2( x + y + z )t + x 2 + y 2 + z 2 ) 2

( x − y )( y − z )( z − t )( x + y + z + 3t )
4( x + y + z ) 2 t 2 + 4( x 2 + y 2 + z 2 )( x + y + z )t + ( x 2 + y 2 + z 2 ) 2
=
( x − y )( y − z )( z − t )( x + y + z + 3t )
4( x 2 + y 2 + z 2 )( x + y + z )t + ( x 2 + y 2 + z 2 ) 2

( x − y )( y − z )( z − t )( x + y + z + 3t )
4( x 2 + y 2 + z 2 ) 2

( x − y )( y − z )( z − t )( x + y + z )
= F ( x, y , z )
4( x 2 + y 2 + z 2 )( x + y + z )t + ( x 2 + y 2 + z 2 ) 2
(vì haøm soá g (t ) = laø haøm ñoàng bieán)
( x − y )( y − z )( z − t )( x + y + z + 3t )
AÙp duïng keát quaû naøy vôùi t = a, ta ñöôïc

(m 2 + n 2 )2
F (a, b, c) ≥ F (0, b − a, c − a ) = F (0, m, n) =
mn( n 2 − m 2 )

Vôùi n = c − a ≥ m = b − a ≥ 0
Do ñoù, ñeå chöùng minh F (a, b, c) ≥ 4 , ta chæ caàn chöùng minh
F (0, m, n) ≥ 4
⇔ (m 2 + n 2 ) 2 ≥ 4mn(n 2 − m 2 ) (*)

Khoâng maát tính toång quaùt, ta coù theå giaû söû m 2 + n 2 = 1 . Khi ñoù
1
(*) ⇔ nm(n 2 − m 2 ) ≤
4
Theo chöùng minh treân thì baát ñaúng thöùc naøy ñuùng, töø ñaây, ta suy ra ñpcm, töùc laø

447
1
P≤ .
4

2− 2 2+ 2
Ñaúng thöùc xaûy ra chaúng haïn khi a = 0, b = ,c = .
2 2
1
Vaäy max P = .
4

448
TÀI LIỆU THAM KHẢO

[1] T.An dreescu, V.Cirtoaje, G.Dospinescu, M.Lascu, Old and New Inequalities
[2] Hojoo Lee, Topics In Inequalities
[3] Pierre Bornsztein, Inégalités
[4] K.S.Kedlaya, A < B
[5] Thomas J.Mildorf, Olympiad Inequalities
[6] Kin – Yin Li, Using Tangent to Prove Inequalities, Mathematical Excalibur,
Vol.10, No. 05, Dec.05 − Jan.06
[7] Lau Chi Hin, Muirhead’ s Inequality, Mathematical Excalibur, Vol.11, No.01,
Feb.05 − Mar.06
[8] Crux Mathematicorum
[9] Phan Huy Khải, 10000 Bài Toán Sơ Cấp − Bất Đẳng Thức Kinh Điển,
NXB Hà Nội 2001
[10] Tạp chí Toán Học Và Tuổi Trẻ
[11] Phạm Kim Hùng, Sáng tạo bất đẳng thức, NXB Tri Thức 2006
[12] Các trang web toán học:

449

You might also like